Fundamentals of Linear Algebra - Mathematics & Statisticsmelbial2/classes/Linear Algebra... ·...

274
Fundamentals of Linear Algebra Marcel B. Finan Arkansas Tech University c All Rights Reserved October 9, 2001

Transcript of Fundamentals of Linear Algebra - Mathematics & Statisticsmelbial2/classes/Linear Algebra... ·...

Page 1: Fundamentals of Linear Algebra - Mathematics & Statisticsmelbial2/classes/Linear Algebra... · 2006-01-13 · 1.1 Systems of Linear Equations Many practical problems can be reduced

Fundamentals of Linear Algebra

Marcel B. FinanArkansas Tech University

c©All Rights Reserved

October 9, 2001

Page 2: Fundamentals of Linear Algebra - Mathematics & Statisticsmelbial2/classes/Linear Algebra... · 2006-01-13 · 1.1 Systems of Linear Equations Many practical problems can be reduced

2

PREFACE

Linear algebra has evolved as a branch of mathematics with wide range ofapplications to the natural sciences, to engineering, to computer sciences, tomanagement and social sciences, and more.

This book is addressed primarely to second and third your college studentswho have already had a course in calculus and analytic geometry. It is theresult of lecture notes given by the author at The University of North Texasand the University of Texas at Austin. It has been designed for use either as asupplement of standard textbooks or as a textbook for a formal course in linearalgebra.

This book is not a ”traditional” book in the sense that it does not includeany applications to the material discussed. Its aim is solely to learn the basictheory of linear algebra within a semester period. Instructors may wish to in-corporate material from various fields of applications into a course.

I have included as many problems as possible of varying degrees of difficulty.Most of the exercises are computational, others are routine and seek to fixsome ideas in the reader’s mind; yet others are of theoretical nature and havethe intention to enhance the reader’s mathematical reasoning. After all doingmathematics is the way to learn mathematics.

Marcecl B. FinanAustin, TexasMarch, 2001.

Page 3: Fundamentals of Linear Algebra - Mathematics & Statisticsmelbial2/classes/Linear Algebra... · 2006-01-13 · 1.1 Systems of Linear Equations Many practical problems can be reduced

Contents

1 Linear Systems 51.1 Systems of Linear Equations . . . . . . . . . . . . . . . . . . . . . 51.2 Geometric Meaning of Linear Systems . . . . . . . . . . . . . . . 81.3 Matrix Notation . . . . . . . . . . . . . . . . . . . . . . . . . . . 101.4 Elementary Row Operations . . . . . . . . . . . . . . . . . . . . 131.5 Solving Linear Systems Using Augmented Matrices . . . . . . . . 171.6 Echelon Form and Reduced Echelon Form . . . . . . . . . . . . . 211.7 Echelon Forms and Solutions to Linear Systems . . . . . . . . . . 281.8 Homogeneous Systems of Linear Equations . . . . . . . . . . . . 321.9 Review Problems . . . . . . . . . . . . . . . . . . . . . . . . . . . 37

2 Matrices 432.1 Matrices and Matrix Operations . . . . . . . . . . . . . . . . . . 432.2 Properties of Matrix Multiplication . . . . . . . . . . . . . . . . . 502.3 The Inverse of a Square Matrix . . . . . . . . . . . . . . . . . . . 552.4 Elementary Matrices . . . . . . . . . . . . . . . . . . . . . . . . . 592.5 An Algorithm for Finding A−1 . . . . . . . . . . . . . . . . . . . 622.6 Review Problems . . . . . . . . . . . . . . . . . . . . . . . . . . . 67

3 Determinants 753.1 Definition of the Determinant . . . . . . . . . . . . . . . . . . . . 753.2 Evaluating Determinants by Row Reduction . . . . . . . . . . . . 793.3 Properties of the Determinant . . . . . . . . . . . . . . . . . . . . 833.4 Finding A−1 Using Cofactor Expansions . . . . . . . . . . . . . . 863.5 Cramer’s Rule . . . . . . . . . . . . . . . . . . . . . . . . . . . . . 933.6 Review Problems . . . . . . . . . . . . . . . . . . . . . . . . . . . 96

4 The Theory of Vector Spaces 1014.1 Vectors in Two and Three Dimensional Spaces . . . . . . . . . . 1014.2 Vector Spaces, Subspaces, and Inner Product Spaces . . . . . . . 1084.3 Linear Independence . . . . . . . . . . . . . . . . . . . . . . . . . 1144.4 Basis and Dimension . . . . . . . . . . . . . . . . . . . . . . . . . 1204.5 Transition Matrices and Change of Basis . . . . . . . . . . . . . . 1284.6 The Rank of a matrix . . . . . . . . . . . . . . . . . . . . . . . . 133

3

Page 4: Fundamentals of Linear Algebra - Mathematics & Statisticsmelbial2/classes/Linear Algebra... · 2006-01-13 · 1.1 Systems of Linear Equations Many practical problems can be reduced

4 CONTENTS

4.7 Review Problems . . . . . . . . . . . . . . . . . . . . . . . . . . . 144

5 Eigenvalues and Diagonalization 1515.1 Eigenvalues and Eigenvectors of a Matrix . . . . . . . . . . . . . 1515.2 Diagonalization of a Matrix . . . . . . . . . . . . . . . . . . . . . 1645.3 Review Problems . . . . . . . . . . . . . . . . . . . . . . . . . . . 171

6 Linear Transformations 1756.1 Definition and Elementary Properties . . . . . . . . . . . . . . . 1756.2 Kernel and Range of a Linear Transformation . . . . . . . . . . . 1806.3 The Matrix Representation of a Linear Transformation . . . . . . 1896.4 Review Problems . . . . . . . . . . . . . . . . . . . . . . . . . . . 194

7 Solutions to Review Problems 197

Page 5: Fundamentals of Linear Algebra - Mathematics & Statisticsmelbial2/classes/Linear Algebra... · 2006-01-13 · 1.1 Systems of Linear Equations Many practical problems can be reduced

Chapter 1

Linear Systems

In this chapter we shall develop the theory of general systems of linear equations.The tool we will use to find the solutions is the row-echelon form of a matrix. Infact, the solutions can be read off from the row- echelon form of the augmentedmatrix of the system. The solution technique, known as elimination method,is developed in Section 1.4.

1.1 Systems of Linear Equations

Many practical problems can be reduced to solving systems of linear equations.The main purpose of linear algebra is to find systematic methods for solvingthese systems. So it is natural to start our discussion of linear algebra by study-ing linear equations.

A linear equation in n variables is an equation of the form

a1x1 + a2x2 + ... + anxn = b (1.1)

where x1, x2, ..., xn are the unknowns (i.e. quantities to be found) and a1, · · · , an

are the coefficients ( i.e. given numbers). Also given the number b known asthe constant term. Observe that a linear equation does not involve any prod-ucts, inverses, or roots of variables. All variables occur only to the first powerand do not appear as arguments for trigonometric, logarithmic, or exponentialfunctions.

Exercise 1Determine whether the given equations are linear or not:(a) 3x1 − 4x2 + 5x3 = 6.(b) 4x1 − 5x2 = x1x2.(c) x2 = 2

√x1 − 6.

(d) x1 + sin x2 + x3 = 1.(e) x1 − x2 + x3 = sin 3.

5

Page 6: Fundamentals of Linear Algebra - Mathematics & Statisticsmelbial2/classes/Linear Algebra... · 2006-01-13 · 1.1 Systems of Linear Equations Many practical problems can be reduced

6 CHAPTER 1. LINEAR SYSTEMS

Solution(a) The given equation is in the form given by (1.1) and therefore is linear.(b) The equation is not linear because the term on the right side of the equationinvolves a product of the variables x1 and x2.(c) A nonlinear equation because the term 2

√x1 involves a square root of the

variable x1.(d) Since x2 is an argument of a trigonometric function then the given equationis not linear.(e) The equation is linear according to (1.1)

A solution of a linear equation (1.1) in n unknowns is a finite ordered col-lection of numbers s1, s2, ..., sn which make (1.1) a true equality when x1 =s1, x2 = s2, · · · , xn = sn are substituted in (1.1). The collection of all solutionsof a linear equation is called the solution set or the general solution.

Exercise 2Show that (5 + 4s− 7t, s, t), where s, t ∈ IR, is a solution to the equation

x1 − 4x2 + 7x3 = 5.

Solutionx1 = 5 + 4s− 7t, x2 = s, and x3 = t is a solution to the given equation because

x1 − 4x2 + 7x3 = (5 + 4s− 7t)− 4s + 7t = 5.

A linear equation can have infinitely many solutions, exactly one solution or nosolutions at all (See Theorem 5 in Section 1.7).

Exercise 3Determine the number of solutions of each of the following equations:(a) 0x1 + 0x2 = 5.(b) 2x1 = 4.(c) x1 − 4x2 + 7x3 = 5.

Solution.(a) Since the left-hand side of the equation is 0 and the right-hand side is 5 thenthe given equation has no solution.(b) By dividing both sides of the equation by 2 we find that the given equationhas the unique solution x1 = 2.(c) To find the solution set of the given equation we assign arbitrary values sand t to x2 and x3 , respectively, and solve for x1, we obtain

x1 = 5 + 4s− 7tx2 = sx3 = t

Thus, the given equation has infinitely many solutions

Page 7: Fundamentals of Linear Algebra - Mathematics & Statisticsmelbial2/classes/Linear Algebra... · 2006-01-13 · 1.1 Systems of Linear Equations Many practical problems can be reduced

1.1. SYSTEMS OF LINEAR EQUATIONS 7

s and t of the previous exercise are referred to as parameters. The solu-tion in this case is said to be given in parametric form.

Many problems in the sciences lead to solving more than one linear equation.The general situation can be described by a linear system.A system of linear equations or simply a linear system is any finite col-lection of linear equations. A particular solution of a linear system is anycommon solution of these equations. A system is called consistent if it has asolution. Otherwise, it is called inconsistent. A general solution of a systemis a formula which gives all the solutions for different values of parameters (SeeExercise 3 (c) ).

A linear system of m equations in n variables has the form

a11x1 + a12x2 + ... + a1nxn = b1

a21x1 + a22x2 + ... + a2nxn = b2

........................ ....am1x1 + am2x2 + ... + amnxn = bm

As in the case of a single linear equation, a linear system can have infinitelymany solutions, exactly one solution or no solutions at all. We will provide aproof of this statement in Section 1.7 (See Theorem 5). An alternative proof ofthe fact that when a system has more than one solution then it must have aninfinite number of solutions will be given in Exercise 14.

Exercise 4Find the general solution of the linear system

{x1 + x2 = 72x1 + 4x2 = 18.

Solution.Multiply the first equation of the system by −2 and then add the resultingequation to the second equation to find 2x2 = 4. Solving for x2 we find x2 = 2.Plugging this value in one of the equations of the given system and then solvingfor x1 one finds x1 = 5

Exercise 5By letting x3 = t, find the general solution of the linear system

{x1 + x2 + x3 = 72x1 + 4x2 + x3 = 18.

Solution.By letting x3 = t the given system can be rewritten in the form

{x1 + x2 = 7− t2x1 + 4x2 = 18− t.

By multiplying the first equation by −2 and adding to the second equation onefinds x2 = 4+t

2 . Substituting this expression in one of the individual equationsof the system and then solving for x1 one finds x1 = 10−3t

2

Page 8: Fundamentals of Linear Algebra - Mathematics & Statisticsmelbial2/classes/Linear Algebra... · 2006-01-13 · 1.1 Systems of Linear Equations Many practical problems can be reduced

8 CHAPTER 1. LINEAR SYSTEMS

1.2 Geometric Meaning of Linear Systems

In the previous section we stated that a linear system can have exactly one solu-tion, infinitely many solutions or no solutions at all. In this section, we supportour claim using geometry. More precisely, we consider the plane since a linearequation in the plane is represented by a straight line.

Consider the x1x2 − plane and the set of points satisfying ax1 + bx2 = c. Ifa = b = 0 but c 6= 0 then the set of points satisfying the above equation isempty. If a = b = c = 0 then the set of points is the whole plane since theequation is satisfied for all (x1, x2) ∈ IR2.

Exercise 6Show that if a 6= 0 or b 6= 0 then the set of points satisfying ax1 + bx2 = c is astraight line.

Solution.If a 6= 0 but b = 0 then the equation x1 = c

a is a vertical line in the x1x2-plane.If a = 0 but b 6= 0 then x2 = c

b is a horizontal line in the plane. Finally, sup-pose that a 6= 0 and b 6= 0. Since x2 can be assigned arbitrary values then thegiven equation possesses infinitely many solutions. Let A(a1, a2), B(b1, b2), andC(c1, c2) be any three points in the plane with components satisfying the givenequation. The slope of the line AB is given by the expression mAB = b2−a2

b1−a1

whereas that of AC is given by mAC = c2−a2c1−a1

. From the equations aa1 +ba2 = c

and ab1 + bb2 = c one finds b2−a2b1−a1

= −ab . Similarly, c2−a2

c1−a1= −a

b . This showsthat the lines AB and AC are parallel. Since these lines have the point A incommon then A,B, and C are on the same straight line

The set of solutions of the system{

ax1 + bx2 = ca′x1 + b′x2 = c′

is the intersection of the set of solutions of the individual equations. Thus, if thesystem has exactly one solution then this solution is the point of intersection oftwo lines. If the system has infinitely many solutions then the two lines coincide.If the system has no solutions then the two lines are parallel.

Exercise 7Find the point of intersection of the lines x1 − 5x2 = 1 and 2x1 − 3x2 = 3.

Solution.To find the point of intersection we have to solve the system

{x1 − 5x2 = 12x1 − 3x2 = 3.

Using either elimination of unknowns or substitution one finds the solutionx1 = 12

7 , x2 = 17 .

Page 9: Fundamentals of Linear Algebra - Mathematics & Statisticsmelbial2/classes/Linear Algebra... · 2006-01-13 · 1.1 Systems of Linear Equations Many practical problems can be reduced

1.2. GEOMETRIC MEANING OF LINEAR SYSTEMS 9

Exercise 8Do the three lines 2x1 + 3x2 = −1, 6x1 + 5x2 = 0, and 2x1 − 5x2 = 7 have acommon point of intersection?

Solution.Solving the system {

2x1 + 3x2 = −16x1 + 5x2 = 0

we find the solution x1 = 58 , x2 = − 3

4 . Since 2x1 − 5x2 = 54 + 15

4 = 5 6= 7 thenthe three lines do not have a point in common

A similar geometrical interpretation holds for systems of equations in threeunknowns where in this case an equation is represented by a plane in IR3. Sincethere is no physical image of the graphs for linear equations in more than threeunknowns we will prove later by means of an algebraic argument(See Theorem5 of Section 1.7) that our statement concerning the number of solutions of alinear system is still valid.

Exercise 9Consider the system of equations

a1x1 + b1x2 = c1

a2x1 + b2x2 = c2

a3x1 + b3x2 = c3.

Discuss the relative positions of the above three lines when

(a) the system has no solutions,(b) the system has exactly one solution,(c) the system has infinitely many solutions.

Solution.(a) The lines have no point of intersection.(b) The lines intersect in exactly one point.(c) The three lines coincide

Exercise 10In the previous exercise, show that if c1 = c2 = c3 = 0 then the system hasalways a solution.

Solution.If c1 = c2 = c3 then the system has at least one solution, namely the trivialsolution x1 = x2 = 0

Page 10: Fundamentals of Linear Algebra - Mathematics & Statisticsmelbial2/classes/Linear Algebra... · 2006-01-13 · 1.1 Systems of Linear Equations Many practical problems can be reduced

10 CHAPTER 1. LINEAR SYSTEMS

1.3 Matrix Notation

Our next goal is to discuss some means for solving linear systems of equations.In Section 1.4 we will develop an algebraic method of solution to linear systems.But before we proceed any further with our discusion, we introduce a conceptthat simplifies the computations involved in the method.The essential information of a linear system can be recorded compactly in arectangular array called a matrix. A matrix of size m × n is a rectangulararray of the form

a11 a12 ... a1n

a21 a22 ... a2n

... ... ... ...am1 am2 ... amn

where the aij ’s are the entries of the matrix, m is the number of rows, and nis the number of columns. If n = m the matrix is called square matrix.We shall often use the notation A = (aij) for the matrix A, indicating that aij

is the (i, j) entry in the matrix A.An entry of the form aii is said to be on the main diagonal. An m×n matrixA with entries aij is called upper triangular (resp. lower triangular) if theentries below (resp. above) the main diagonal are all 0. That is, aij = 0 if i > j(resp. i < j). A is called a diagonal matrix if aij = 0 whenever i 6= j. By atriangular matrix we mean either an upper triangular, a lower triangular, or adiagonal matrix.Further definitions of matrices and related properties will be introduced in thenext chapter.

Now, let A be a matrix of size m × n and entries aij ; B is a matrix of sizen × p and entries bij . Then the product matrix is a matrix of size m × p andentries

cij = ai1b1j + ai2b2j + · · ·+ ainbnj

that is cij is obtained by multiplying componentwise the entries of the ith rowof A by the entries of the jth column of B. It is very important to keep in mindthat the number of columns of the first matrix must be equal to the number ofrows of the second matrix; otherwise the product is undefined.

Exercise 11Consider the matrices

A =(

1 2 42 6 0

), B =

4 1 4 30 −1 3 12 7 5 2

Compute, if possible, AB and BA.

Page 11: Fundamentals of Linear Algebra - Mathematics & Statisticsmelbial2/classes/Linear Algebra... · 2006-01-13 · 1.1 Systems of Linear Equations Many practical problems can be reduced

1.3. MATRIX NOTATION 11

Solution.We have

AB =(

1 2 42 6 0

)

4 1 4 30 −1 3 12 7 5 2

=(

4 + 8 1− 2 + 28 4 + 6 + 20 3 + 2 + 88 2− 6 8 + 18 6 + 6

)

=(

12 27 30 138 −4 26 12

).

BA is not defined since the number of columns of B is not equal to the numberof rows of A

Next, consider a system of linear equations

a11x1 + a12x2 + ... + a1nxn = b1

a21x1 + a22x2 + ... + a2nxn = b2

........................ ....am1x1 + am2x2 + ... + amnxn = bm

Then the matrix of the coefficients of the xi’s is called the coefficient matrix:

A =

a11 a12 ... a1n

a21 a22 ... a2n

... ... ... ...am1 am2 ... amn

The matrix of the coefficients of the xi’s and the right hand side coefficients iscalled the augmented matrix:

a11 a12 ... a1n b1

a21 a22 ... a2n b2

... ... ... ... ...am1 am2 ... amn bm

Finally, if we let

x =

x1

x2

...xn

and

b =

b1

b2

...bm

Page 12: Fundamentals of Linear Algebra - Mathematics & Statisticsmelbial2/classes/Linear Algebra... · 2006-01-13 · 1.1 Systems of Linear Equations Many practical problems can be reduced

12 CHAPTER 1. LINEAR SYSTEMS

then the above system can be represented in matrix notation as

Ax = b.

Exercise 12Consider the linear system

x1 − 2x2 + x3 = 02x2 − 8x3 = 8

−4x1 + 5x2 + 9x3 = − 9.

(a) Find the coefficient and augmented matrices of the linear system.(b) Find the matrix notation.

Solution.(a) The coefficient matrix of this system is

1 −2 10 2 −8−4 5 9

and the augmented matrix is

1 −2 1 00 2 −8 8−4 5 9 −9

(b) We can write the given system in matrix form as

1 −2 10 2 −8−4 5 9

x1

x2

x3

=

08−9

Exercise 13Write the linear system whose augmented matrix is given by

2 −1 0 −1−3 2 1 00 1 1 3

Solution.The given matrix is the augmented matrix of the linear system

2x1 − x2 = −1−3x1 + 2x2 + x3 = 0

x2 + x3 = 3

Page 13: Fundamentals of Linear Algebra - Mathematics & Statisticsmelbial2/classes/Linear Algebra... · 2006-01-13 · 1.1 Systems of Linear Equations Many practical problems can be reduced

1.4. ELEMENTARY ROW OPERATIONS 13

Exercise 14(a) Show that if A is an m × n matrix, x, y are n × 1 matrices and α, β arenumbers then A(αx + βy) = αAx + βAy.(b) Using the matrix notation of a linear system, prove that, if a linear systemhas more than one solution then it must have an infinite number of solutions.

Solution.Recall that a number is considered an 1× 1 matrix. Thus, using matrix multi-plication we find

A(αx + βy) =

a11 a12 · · · a1n

a21 a22 · · · a2n

......

...am1 am2 · · · amn

αx1 + βy1

αx2 + βy2

...αxn + βyn

=

α(a11x1 + a12x2 + · · ·+ a1nxn) + β(a11y1 + a12y2 + · · ·+ a1nyn)...

α(am1x1 + am2x2 + · · ·+ amnxn) + β(am1y1 + am2y2 + · · ·+ amnyn)

= α

a11x1 + a12x2 + · · ·+ a1nxn

...am1x1 + am2x2 + · · ·+ amnxn

+ β

a11y1 + a12y2 + · · ·+ a1nyn

...am1y1 + am2y2 + · · ·+ amnyn

= αAx + βAy.

(b) Let X1 and X2 be two solutions of Ax = b. For any t ∈ IR let Xt =tX1 +(1− t)X2. Then AXt = tAX1 +(1− t)AX2 = tb+(1− t)b = b. That is, Xt

is a solution to the linear system. Note that for s 6= t we have Xs 6= Xt. Sincet is arbitrary then there exists infinitely many Xt. In other words, the systemhas infinitely many solutions

1.4 Elementary Row Operations

In this section we introduce the concept of elementary row operations that willbe vital for our algebraic method of solving linear systems.We start with the following definition: Two linear systems are said to be equiv-alent if and only if they have the same set of solutions.

Exercise 15Show that the system {

x1 − 3x2 = −72x1 + x2 = 7

is equivalent to the system

8x1 − 3x2 = 73x1 − 2x2 = 010x1 − 2x2 = 14.

Page 14: Fundamentals of Linear Algebra - Mathematics & Statisticsmelbial2/classes/Linear Algebra... · 2006-01-13 · 1.1 Systems of Linear Equations Many practical problems can be reduced

14 CHAPTER 1. LINEAR SYSTEMS

Solution.Solving the first system one finds the solution x1 = 2, x2 = 3. Similarly, solvingthe second system one finds the solution x1 = 2 and x2 = 3. Hence, the twosystems are equivalent

Exercise 16Show that if x1 + kx2 = c and x1 + lx2 = d are equivalent then k = l and c = d.

Solution.For arbitrary t the ordered pair (c− kt, t) is a solution to the second equation.That is c− kt + lt = d for all t ∈ IR. In particular, if t = 0 we find c = d. Thus,kt = lt for all t ∈ IR. Letting t = 1 we find k = l

Our basic method for solving a linear system is known as the method of elim-ination. The method consists of reducing the original system to an equivalentsystem that is easier to solve. The reduced system has the shape of an upper(resp. lower) triangle. This new system can be solved by a technique calledbackward-substitution (resp. forward-substitution): The unknowns arefound starting from the bottom (resp. the top) of the system.The three basic operations in the above method, known as the elementaryrow operations, are summarized as follows.

(I) Multiply an equation by a non-zero number.(II) Replace an equation by the sum of this equation and another equation mul-tiplied by a number.(III) Interchange two equations.

To indicate which operation is being used in the process one can use the follow-ing shorthand notation. For example, r3 ← 1

2r3 represents the row operation oftype (I) where each entry of row 3 is being replaced by 1

2 that entry. Similarinterpretations for types (II) and (III) operations.The following theorem asserts that the system obtained from the original sys-tem by means of elementary row operations has the same set of solutions as theoriginal one.

Theorem 1Suppose that an elementary row operation is performed on a linear system. Thenthe resulting system is equivalent to the original system.

Proof.We prove the theorem only for operations of type (II). The cases of operations oftypes (I) and (III) are left as an exercise for the reader (See Exercise 20 below).Let

c1x1 + c2x2 + · · ·+ cnxn = d (1.2)

and

a1x1 + a2x2 + · · ·+ anxn = b (1.3)

Page 15: Fundamentals of Linear Algebra - Mathematics & Statisticsmelbial2/classes/Linear Algebra... · 2006-01-13 · 1.1 Systems of Linear Equations Many practical problems can be reduced

1.4. ELEMENTARY ROW OPERATIONS 15

denote two different equations of the original system. Suppose a new system isobtained by replacing (1.3) by (1.4)

(a1 + kc1)x1 + (a2 + kc2)x2 + · · ·+ (an + kcn)xn = b + kd (1.4)

obtained by adding k times equation (1.2) to equation (1.3). If s1, s2, · · · , sn isa solution to the original system then

c1s1 + c2s2 + · · ·+ cnsn = d

and

a1s1 + a2s2 + · · ·+ ansn = b.

By multiplication and addition, these give

(a1 + kc1)s1 + (a2 + kc2)s2 + · · ·+ (an + kcn)sn = b + kd.

Hence, s1, s2, · · · , sn is a solution to the new system. Conversely, suppose thats1, s2, · · · , sn is a solution to the new system. Then

a1s1 + a2s2 + · · ·+ ansn = b + kd− k(c1s1 + c2s2 + · · ·+ cnsn) = b + kd− kd = b.

That is, s1, s2, · · · , sn is a solution to the original system.

Exercise 17Use the elimination method described above to solve the system

x1 + x2 − x3 = 3x1 − 3x2 + 2x3 = 12x1 − 2x2 + x3 = 4.

Solution.Step 1: We eliminate x1 from the second and third equations by performing twooperations r2 ← r2 − r1 and r3 ← r3 − 2r1 obtaining

x1 + x2 − x3 = 3− 4x2 + 3x3 = −2− 4x2 + 3x3 = −2

Step 2: The operation r3 ← r3 − r2 leads to the system{

x1 + x2 − x3 = 3− 4x2 + 3x3 = −2

By assigning x3 an arbitrary value t we obtain the general solution x1 =t+10

4 , x2 = 2+3t4 , x3 = t. This means that the linear system has infinitely many

solutions. Every time we assign a value to t we obtain a different solution

Page 16: Fundamentals of Linear Algebra - Mathematics & Statisticsmelbial2/classes/Linear Algebra... · 2006-01-13 · 1.1 Systems of Linear Equations Many practical problems can be reduced

16 CHAPTER 1. LINEAR SYSTEMS

Exercise 18Determine if the following system is consistent or not

3x1 + 4x2 + x3 = 12x1 + 3x2 = 04x1 + 3x2 − x3 = −2.

Solution.Step 1: To eliminate the variable x1 from the second and third equations weperform the operations r2 ← 3r2−2r1 and r3 ← 3r3−4r1 obtaining the system

3x1 + 4x2 + x3 = 1x2 − 2x3 = −2

− 7x2 − 7x3 = − 10.

Step 2: Now, to eliminate the variable x3 from the third equation we apply theoperation r3 ← r3 + 7r2 to obtain

3x1 + 4x2 + x3 = 1x2 − 2x3 = −2

− 21x3 = − 24.

Solving the system by the method of backward substitution we find the uniquesolution x1 = − 3

7 , x2 = 27 , x3 = 8

7 . Hence the system is consistent

Exercise 19Determine whether the following system is consistent:

{x1 − 3x2 = 4

−3x1 + 9x2 = 8.

Solution.Multiplying the first equation by 3 and adding the resulting equation to thesecond equation we find 0 = 20 which is impossible. Hence, the given system isinconsistent

Exercise 20(a) Show that the linear system obtained by interchanging two equations is equiv-alent to the original system.(b) Show that the linear system obtained by multiplying a row by a scalar isequivalent to the original system.

Solution.(a) Interchanging two equations in a linear system does yield the same system.(b) Suppose now a new system is obtained by multiplying the ith row by α 6= 0.Then the ith equation of this system looks like

(αai1)x1 + (αai2)x2 + · · ·+ (αain)xn = αdi. (1.5)

Page 17: Fundamentals of Linear Algebra - Mathematics & Statisticsmelbial2/classes/Linear Algebra... · 2006-01-13 · 1.1 Systems of Linear Equations Many practical problems can be reduced

1.5. SOLVING LINEAR SYSTEMS USING AUGMENTED MATRICES 17

If s1, s2, · · · , sn is a solution to the original system then

ai1s1 + ai2s2 + · · ·+ ainsn = di.

Multiply both sides of this equation by α yields (1.5). That is, s1, s2, · · · , sn isa solution to the new system.Now if s1, s2, · · · , sn is a solution to (1.5) then by dividing through by α we findthat s1, s2, · · · , sn is a solution of the original system

1.5 Solving Linear Systems Using AugmentedMatrices

In this section we apply the elimination method described in the previous sec-tion to the augmented matrix corresponding to a given system rather than tothe individual equations. Thus, we obtain a triangular matrix which is rowequivalent to the original augmented matrix, a concept that we define next.We say that a matrix A is row equivalent to a matrix B if B can be obtainedby applying a finite number of elementary row operations to the matrix A.This definition combined with Theorem 1 lead to the following

Theorem 2 Let Ax = b be a linear system. If [C|d] is row equivalent to [A|b]then the system Cx = d is equivalent to Ax = b.

Proof.The system Cx = d is obtained from the system Ax = b by applying a finitenumber of elementary row operations. By Theorem 1, this system is equivalentto Ax = b

The above theorem provides us with a method for solving a linear system usingmatrices. It suffices to apply the elementary row operations on the augmentedmatrix and reduces it to an equivalent triangular matrix. Then the correspond-ing system is triangular as well. Next, use either the backward-substitution orthe forward-substitution technique to find the unknowns.

Exercise 21Solve the following linear system using elementary row operations on the aug-mented matrix:

x1 − 2x2 + x3 = 02x2 − 8x3 = 8

−4x1 + 5x2 + 9x3 = −9.

Solution.The augmented matrix for the system is

1 −2 1 00 2 −8 8−4 5 9 −9

Page 18: Fundamentals of Linear Algebra - Mathematics & Statisticsmelbial2/classes/Linear Algebra... · 2006-01-13 · 1.1 Systems of Linear Equations Many practical problems can be reduced

18 CHAPTER 1. LINEAR SYSTEMS

Step 1: The operations r2 ← 12r2 and r3 ← r3 + 4r1 give

1 −2 1 00 1 −4 40 −3 13 −9

Step 2: The operation r3 ← r3 + 3r2 gives

1 −2 1 00 1 −4 40 0 1 3

The corresponding system of equations is

x1 − 2x2 + x3 = 0x2 − 4x3 = 4

x3 = 3

Using back-substitution we find the unique solution x1 = 29, x2 = 16, x3 = 3

Exercise 22Solve the following linear system using the method described above.

x2 + 5x3 = −4x1 + 4x2 + 3x3 = −22x1 + 7x2 + x3 = −1.

Solution.The augmented matrix for the system is

0 1 5 −41 4 3 −22 7 1 −1

Step 1:The operation r2 ↔ r1 gives

1 4 3 −20 1 5 −42 7 1 −1

Step 2: The operation r3 ← r3 − 2r1 gives the system

1 4 3 −20 1 5 −40 −1 −5 3

Step 3: The operation r3 ← r3 + r2 gives

1 4 3 −20 1 5 −40 0 0 −1

Page 19: Fundamentals of Linear Algebra - Mathematics & Statisticsmelbial2/classes/Linear Algebra... · 2006-01-13 · 1.1 Systems of Linear Equations Many practical problems can be reduced

1.5. SOLVING LINEAR SYSTEMS USING AUGMENTED MATRICES 19

The corresponding system of equations is

x1 + 4x2 + 3x3 = −2x2 + 5x3 = −4

0 = −1

From the last equation we conclude that the system is inconsistent

Exercise 23Solve the following linear system using the elimination method of this section.

x1 + 2x2 = 0−x1 + 3x2 + 3x3 = −2

x2 + x3 = 0.

Solution.The augmented matrix for the system is

1 2 0 0−1 3 3 −20 1 1 0

Step 1: Applying the operation r2 ← r2 + r1 gives

1 2 0 00 5 3 −20 1 1 0

Step 2: The operation r2 ↔ r3 gives

1 2 0 00 1 1 00 5 3 −2

Step 3: Now performing the operation r3 ← r3 − 5r2 yields

1 2 0 00 1 1 00 0 −2 −2

The system of equations equivalent to the original system is

x1 + 2x2 = 0x2 + x3 = 0

− 2x3 = −2

Using back-substitution we find x1 = 2, x2 = −1, x3 = 1

Exercise 24Determine if the following system is consistent.

x2 − 4x3 = 82x1 − 3x2 + 2x3 = 15x1 − 8x2 + 7x3 = 1.

Page 20: Fundamentals of Linear Algebra - Mathematics & Statisticsmelbial2/classes/Linear Algebra... · 2006-01-13 · 1.1 Systems of Linear Equations Many practical problems can be reduced

20 CHAPTER 1. LINEAR SYSTEMS

Solution.The augmented matrix of the given system is

0 1 −4 82 −3 2 15 −8 7 1

Step 1: The operation r3 ← r3 − 2r2 gives

0 1 −4 82 −3 2 11 −2 3 −1

Step 2: The operation r3 ↔ r1 leads to

1 −2 3 −12 −3 2 10 1 −4 8

Step 3: Applying r2 ← r2 − 2r1 to obtain

1 −2 3 −10 1 −4 30 1 −4 8

Step 4: Finally, the operation r3 ← r3 − r2 gives

1 −2 3 −10 1 −4 30 0 0 5

Hence, the equivalent system is

x1 − 2x2 + 3x3 = 0x2 − 4x3 = 3

0 = 5

This last system has no solution ( the last equation requires x1, x2, and x3 tosatisfy the equation 0x1 + 0x2 + 0x3 = 5 and no such x1, x2, and x3 exist).Hence the original system is inconsistent

Pay close attention to the last row of the row equivalent augmented matrixof the previous exercise. This situation is typical of an inconsistent system.

Exercise 25Find an equation involving g, h, and k that makes the following augmented ma-trix corresponds to a consistent system.

2 5 −3 g4 7 −4 h−6 −3 1 k

Page 21: Fundamentals of Linear Algebra - Mathematics & Statisticsmelbial2/classes/Linear Algebra... · 2006-01-13 · 1.1 Systems of Linear Equations Many practical problems can be reduced

1.6. ECHELON FORM AND REDUCED ECHELON FORM 21

Solution.The augmented matrix for the given system is

2 5 −3 g4 7 −4 h−6 −3 1 k

Step 1: Applying the operations r2 ← r2 − 2r1 and r3 ← r3 + 3r1 give

2 5 −3 g0 −3 2 h− 2g0 12 −8 k + 3g

Step 2: Now, the operation r3 ← r3 + 4r2 gives

2 5 −3 g0 −3 2 h− 2g0 0 0 k + 4h− 5g

For the system , whose augmented matrix is the last matrix, to be consistent theunknowns x1, x2, and x3 must satisfy the property 0x1+0x2+0x3 = −5g+4h−k,that is −5g + 4h + k = 0

1.6 Echelon Form and Reduced Echelon Form

The elimination method introduced in the previous section reduces the aug-mented matrix to a ”nice” matrix ( meaning the corresponding equations areeasy to solve). Two of the ”nice” matrices discussed in this section are matricesin either row-echelon form or reduced row-echelon form, concepts that we dis-cuss next.By a leading entry of a row in a matrix we mean the leftmost non-zero entryin the row.A rectangular matrix is said to be in row-echelon form if it has the followingthree characterizations:

(1) All rows consisting entirely of zeros are at the bottom.(2) The leading entry in each non-zero row is 1 and is located in a column tothe right of the leading entry of the row above it.(3) All entries in a column below a leading entry are zero.

The matrix is said to be in reduced row-echelon form if in addition tothe above, the matrix has the following additional characterization:

(4) Each leading 1 is the only nonzero entry in its column.

RemarkFrom the definition above, note that a matrix in row-echelon form has zeros

Page 22: Fundamentals of Linear Algebra - Mathematics & Statisticsmelbial2/classes/Linear Algebra... · 2006-01-13 · 1.1 Systems of Linear Equations Many practical problems can be reduced

22 CHAPTER 1. LINEAR SYSTEMS

below each leading 1, whereas a matrix in reduced row-echelon form has zerosboth above and below each leading 1.

Exercise 26Determine which matrices are in row-echelon form (but not in reduced row-echelon form) and which are in reduced row-echelon form(a)

1 −3 2 10 1 −4 80 0 0 1

(b)

1 0 0 290 1 0 160 0 1 1

Solution.(a)The given matrix is in row-echelon form but not in reduced row-echelon formsince the (1, 2)−entry is not zero.(b) The given matrix satisfies the characterization of a reduced row-echelon form

The importance of the row-echelon matrices is indicated in the following theo-rem.

Theorem 3Every nonzero matrix can be brought to (reduced) row-echelon form by a finitenumber of elementary row operations.

Proof.The proof consists of the following steps:

Step 1. Find the first column from the left containing a nonzero entry (callit a), and move the row containing that entry to the top position.

Step 2. Multiply the row from Step 1 by 1a to create a leading 1.

Step 3. By subtracting multiples of that row from rows below it, make eachentry below the leading 1 zero.

Step 4. This completes the first row. Now repeat steps 1-3 on the matrixconsisting of the remaining rows.

The process stops when either no rows remain in step 4 or the remaining rowsconsist of zeros. The entire matrix is now in row-echelon form.To find the reduced row-echelon form we need the following additional step.

Page 23: Fundamentals of Linear Algebra - Mathematics & Statisticsmelbial2/classes/Linear Algebra... · 2006-01-13 · 1.1 Systems of Linear Equations Many practical problems can be reduced

1.6. ECHELON FORM AND REDUCED ECHELON FORM 23

Step 5. Beginning with the last nonzero row and working upward, add suitablemultiples of each row to the rows above to introduce zeros above the leading 1

The process of reducing a matrix to a row-echelon form discussed in Steps 1 -4 is known as Gaussian elimination. That of reducing a matrix to a reducedrow-echelon form, i.e. Steps 1 - 5, is known as Gauss-Jordan elimination.We illustrate the above algorithm in the following problems.

Exercise 27Use Gauss-Jordan elimination to transform the following matrix first into row-echelon form and then into reduced row-echelon form

0 −3 −6 4 9−1 −2 −1 3 1−2 −3 0 3 −11 4 5 −9 −7

Solution.The reduction of the given matrix to row-echelon form is as follows.

Step 1: r1 ↔ r4

1 4 5 −9 −7−1 −2 −1 3 1−2 −3 0 3 −10 −3 −6 4 9

Step 2: r2 ← r2 + r1 and r3 ← r3 + 2r1

1 4 5 −9 −70 2 4 −6 −60 5 10 −15 −150 −3 −6 4 9

Step 3: r2 ← 12r2 and r3 ← 1

5r3

1 4 5 −9 −70 1 2 −3 −30 1 2 −3 −30 −3 −6 4 9

Step 4: r3 ← r3 − r2 and r4 ← r4 + 3r2

1 4 5 −9 −70 1 2 −3 −30 0 0 0 00 0 0 −5 0

Step 5: r3 ↔ r4

1 4 5 −9 −70 1 2 −3 −30 0 0 −5 00 0 0 0 0

Page 24: Fundamentals of Linear Algebra - Mathematics & Statisticsmelbial2/classes/Linear Algebra... · 2006-01-13 · 1.1 Systems of Linear Equations Many practical problems can be reduced

24 CHAPTER 1. LINEAR SYSTEMS

Step 6: r5 ← − 15r5

1 4 5 −9 −70 1 2 −3 −30 0 0 1 00 0 0 0 0

Step 7: r1 ← r1 − 4r2

1 0 −3 3 50 1 2 −3 −30 0 0 1 00 0 0 0 0

Step 8: r1 ← r1 − 3r3 and r2 ← r2 + 3r3

1 0 −3 0 50 1 2 0 −30 0 0 1 00 0 0 0 0

Exercise 28Use Gauss-Jordan elimination to transform the following matrix first into row-echelon form and then into reduced row-echelon form

0 3 −6 6 4 −53 −7 8 −5 8 93 −9 12 −9 6 15

Solution.By following the steps in the Gauss-Jordan algorithm we find

Step 1: r3 ← 13r3

0 3 −6 6 4 −53 −7 8 −5 8 91 −3 4 −3 2 5

Step 2: r1 ↔ r3

1 −3 4 −3 2 53 −7 8 −5 8 90 3 −6 6 4 −5

Step 3: r2 ← r2 − 3r1

1 −3 4 −3 2 50 2 −4 4 2 −60 3 −6 6 4 −5

Step 4: r2 ← 12r2

1 −3 4 −3 2 50 1 −2 2 1 −30 3 −6 6 4 −5

Page 25: Fundamentals of Linear Algebra - Mathematics & Statisticsmelbial2/classes/Linear Algebra... · 2006-01-13 · 1.1 Systems of Linear Equations Many practical problems can be reduced

1.6. ECHELON FORM AND REDUCED ECHELON FORM 25

Step 5: r3 ← r3 − 3r2

1 −3 4 −3 2 50 1 −2 2 1 −30 0 0 0 1 4

Step 6: r1 ← r1 + 3r2

1 0 −2 3 5 −40 1 −2 2 1 −30 0 0 0 1 4

Step 7: r1 ← r1 − 5r3 and r2 ← r2 − r3

1 0 −2 3 0 −240 1 −2 2 0 −70 0 0 0 1 4

It can be shown that no matter how the elementary row operations are varied,one will always arrive at the same reduced row-echelon form; that is the reducedrow echelon form is unique (See Theorem 68). On the contrary row-echelon formis not unique. However, the number of leading 1’s of two different row-echelonforms is the same (this will be proved in Chapter 4). That is, two row-echelonmatrices have the same number of nonzero rows. This number is called therank of A and is denoted by rank(A). In Chapter 6, we will prove that if A isan m× n matrix then rank(A) ≤ n and rank(A) ≤ m.

Exercise 29Find the rank of each of the following matrices(a)

A =

2 1 43 2 50 −1 1

(b)

B =

3 1 0 1 −90 −2 12 −8 −62 −3 22 −14 −17

Solution.(a) We use Gaussian elimination to reduce the given matrix into row-echelonform as follows:

Step 1: r2 ← r2 − r1

2 1 41 1 10 −1 1

Page 26: Fundamentals of Linear Algebra - Mathematics & Statisticsmelbial2/classes/Linear Algebra... · 2006-01-13 · 1.1 Systems of Linear Equations Many practical problems can be reduced

26 CHAPTER 1. LINEAR SYSTEMS

Step 2: r1 ↔ r2

1 1 12 1 40 −1 1

Step 3: r2 ← r2 − 2r1

1 1 10 −1 20 −1 1

Step 4: r3 ← r3 − r2

1 1 10 −1 20 0 −1

Thus, rank(A) = 3.(b) As in (a), we reduce the matrix into row-echelon form as follows:

Step 1: r1 ← r1 − r3

1 4 −22 15 80 −2 12 − 8 − 62 −3 22 −14 −17

Step 2: r3 ← r3 − 2r1

1 4 −22 15 250 − 2 12 − 8 − 60 −11 −22 −44 −33

Step 3: r2 ← − 12r2

1 4 −22 15 80 1 − 6 4 30 −11 −22 −44 −33

Step 4: r3 ← r3 + 11r2

1 4 −22 15 80 1 − 6 4 30 0 −88 0 0

Step 5: r3 ← 18r3

1 4 −22 15 80 1 − 6 4 30 0 1 0 0

Hence, rank(B) = 3

Page 27: Fundamentals of Linear Algebra - Mathematics & Statisticsmelbial2/classes/Linear Algebra... · 2006-01-13 · 1.1 Systems of Linear Equations Many practical problems can be reduced

1.6. ECHELON FORM AND REDUCED ECHELON FORM 27

Exercise 30Consider the system {

ax + by = kcx + dy = l.

Show that if ad − bc 6= 0 then the reduced row-echelon form of the coefficientmatrix is the matrix (

1 00 1

)

Solution.The coefficient matrix is the matrix

(a bc d

)

Assume first that a 6= 0. Using Gaussian elimination we reduce the above matrixinto row-echelon form as follows:

Step 1: r2 ← ar2 − cr1 (a b0 ad− bc

)

Step 2: r2 ← 1ad−bcr2 (

a b0 1

)

Step 3: r1 ← r1 − br2 (a 00 1

)

Step 4: r1 ← 1ar1 (

1 00 1

)

Next, assume that a = 0. Then c 6= 0 and b 6= 0. Following the steps of Gauss-Jordan elimination algorithm we find

Step 1: r1 ↔ r2 (c d0 b

)

Step 2: r1 ← 1c r1 and r2 ← 1

b r2

(1 d

c0 1

)

Step 3: r1 ← r1 − dc r2 (

1 00 1

)

Page 28: Fundamentals of Linear Algebra - Mathematics & Statisticsmelbial2/classes/Linear Algebra... · 2006-01-13 · 1.1 Systems of Linear Equations Many practical problems can be reduced

28 CHAPTER 1. LINEAR SYSTEMS

1.7 Echelon Forms and Solutions to Linear Sys-tems

In this section we give a systematic procedure for solving systems of linearequations; it is based on the idea of reducing the augmented matrix to eitherthe row-echelon form or the reduced row-echelon form. The new system isequivalent to the original system as provided by the following

Theorem 4Let Ax = b be a system of linear equations. Let [C|d] be the (reduced) row-echelon form of the augmented matrix [A|b]. Then the system Cx = d is equiv-alent to the system Ax = b.

Proof.This follows from Theorem 2 and Theorem 3

Unknowns corresponding to leading entries in the echelon augmented matrixare called dependent or leading variables. If an unknown is not dependentthen it is called free or independent variable.

Exercise 31Find the dependent and independent variables of the following system

x1 + 3x2 − 2x3 + 2x5 = 02x1 + 6x2 − 5x3 − 2x4 + 4x5 − 3x6 = −1

5x3 + 10x4 + 15x6 = 52x1 + 6x2 + 8x4 + 4x5 + 18x6 = 6

Solution.The augmented matrix for the system is

1 3 −2 0 2 0 02 6 −5 −2 4 −3 −10 0 5 10 0 15 52 6 0 8 4 18 6

Using the Gaussian algorithm we bring the augmented matrix to row-echelonform as follows:

Step 1: r2 ← r2 − 2r1 and r4 ← r4 − 2r1

1 3 −2 0 2 0 00 0 −1 −2 0 −3 −10 0 5 10 0 15 50 0 4 8 0 18 6

Step 2: r2 ← −r2

1 3 −2 0 2 0 00 0 1 2 0 3 10 0 5 10 0 15 50 0 4 8 0 18 6

Page 29: Fundamentals of Linear Algebra - Mathematics & Statisticsmelbial2/classes/Linear Algebra... · 2006-01-13 · 1.1 Systems of Linear Equations Many practical problems can be reduced

1.7. ECHELON FORMS AND SOLUTIONS TO LINEAR SYSTEMS 29

Step 3: r3 ← r3 − 5r2 and r4 ← r4 − 4r2

1 3 −2 0 2 0 00 0 1 2 0 3 10 0 0 0 0 0 00 0 0 0 0 6 2

Step 4: r3 ↔ r4

1 3 −2 0 2 0 00 0 1 2 0 3 10 0 0 0 0 6 20 0 0 0 0 0 0

Step 5: r3 ← 16r3

1 3 −2 0 2 0 00 0 1 2 0 3 10 0 0 0 0 1 1

30 0 0 0 0 0 0

The leading variables are x1, x3, and x6. The free variables are x2, x4, and x5

It follows from Theorem 4 that one way to solve a linear system is to applythe elementary row operations to reduce the augmented matrix to a (reduced)row-echelon form. If the augmented matrix is in reduced row-echelon form thento obtain the general solution one just has to move all independent variables tothe right side of the equations and consider them as parameters. The dependentvariables are given in terms of these parameters.

Exercise 32Solve the following linear system.

x1 + 2x2 + x4 = 6x3 + 6x4 = 7

x5 = 1.

Solution.The augmented matrix is already in row-echelon form. The free variables are x2

and x4. So let x2 = s and x4 = t. Solving the system starting from the bottomwe find x1 = −2s− t + 6, x3 = 7− 6t, and x5 = 1

If the augmented matrix does not have the reduced row-echelon form but therow-echelon form then the general solution also can be easily found by using themethod of backward substitution.

Exercise 33Solve the following linear system

x1 − 3x2 + x3 − x4 = 2x2 + 2x3 − x4 = 3

x3 + x4 = 1.

Page 30: Fundamentals of Linear Algebra - Mathematics & Statisticsmelbial2/classes/Linear Algebra... · 2006-01-13 · 1.1 Systems of Linear Equations Many practical problems can be reduced

30 CHAPTER 1. LINEAR SYSTEMS

Solution.The augmented matrix is in row-echelon form. The free variable is x4 = t. Solv-ing for the leading variables we find, x1 = 11t + 4, x2 = 3t + 1, and x3 = 1− t

The questions of existence and uniqueness of solutions are fundamental questionsin linear algebra. The following theorem provides some relevant information.

Theorem 5A system of n linear equations in m unknowns can have exactly one solution,infinitely many solutions, or no solutions at all.

(1) If the reduced augmented matrix has a row of the form (0, 0, · · · , 0, b) whereb is a nonzero constant, then the system has no solutions.(2) If the reduced augmented matrix has indepedent variables and no rows of theform (0, 0, · · · , 0, b) with b 6= 0 then the system has infinitely many solutions.(3) If the reduced augmented matrix has no independent variables and no rowsof the form (0, 0, · · · , 0, b) with b 6= 0, then the system has exactly one solution.

ProofSuppose first that the reduced augmented matrix has a row of the form (0, · · · , 0, b)with b 6= 0. That is, 0x1 + 0x2 + ... + 0xm = b. Then the left side is 0 whereasthe right side is not. This cannot happen. Hence, the system has no solutions.This proves (1).Now suppose that the reduced augmented matrix has independent variablesand no rows of the form (0, 0, · · · , 0, b) for some b 6= 0. Then these variables aretreated as parameters and hence the system has infinitely many solutions. Thisproves (2).Finally, suppose that the reduced augmented matrix has no row of the form(0, 0, · · · , 0, b) where b 6= 0 and no indepedent variables then the system lookslike

x1 = c1

x2 = c2

x3 = c3

... . ...xm = cm

i.e. the system has a unique solution. Thus, (3) is established.

Exercise 34Find the general solution of the system whose augmented matrix is given by

1 2 −7−1 −1 12 1 5

Solution.We first reduce the system to row-echelon form as follows.

Page 31: Fundamentals of Linear Algebra - Mathematics & Statisticsmelbial2/classes/Linear Algebra... · 2006-01-13 · 1.1 Systems of Linear Equations Many practical problems can be reduced

1.7. ECHELON FORMS AND SOLUTIONS TO LINEAR SYSTEMS 31

Step 1: r2 ← r2 + r1 and r3 ← r3 − 2r1

1 2 −70 1 −60 −3 19

Step 2: r3 ← r3 + 3r2

1 2 −70 1 −60 0 1

The corresponding system is given by

x1 + 2x2 = −7x2 = −60 = 1

Because of the last equation the system is inconsistent

Exercise 35Find the general solution of the system whose augmented matrix is given by

1 −2 0 0 7 −30 1 0 0 −3 10 0 0 1 5 −40 0 0 0 0 0

Solution.By adding two times the second row to the first row we find the reduced row-echelon form of the augmented matrix.

1 0 0 0 1 −10 1 0 0 −3 10 0 0 1 5 −40 0 0 0 0 0

It follows that the free variables are x3 = s and x5 = t. Solving for the leadingvariables we find x1 = −1− t, x2 = 1 + 3t, and x4 = −4− 5t

Exercise 36Determine the value(s) of h such that the following matrix is the augmentedmatrix of a consistent linear system

(1 4 2−3 h −1

)

Solution.By adding three times the first row to the second row we find

(1 4 20 12 + h 5

)

Page 32: Fundamentals of Linear Algebra - Mathematics & Statisticsmelbial2/classes/Linear Algebra... · 2006-01-13 · 1.1 Systems of Linear Equations Many practical problems can be reduced

32 CHAPTER 1. LINEAR SYSTEMS

The system is consistent if and only if 12 + h 6= 0; that is, h 6= −12

From a computational point of view, most computer algorithms for solving sys-tems use the Gaussian elimination rather than the Gauss-Jordan elimination.Moreover, the algorithm is somehow varied so that to guarantee a reduction inroundoff error, to minimize storage, and to maximize speed. For instance, manyalgorithms do not normalize the leading entry in each row to be 1.

Exercise 37Find (if possible) conditions on the numbers a, b, and c such that the followingsystem is consistent

x1 + 3x2 + x3 = a−x1 − 2x2 + x3 = b3x1 + 7x2 − x3 = c

Solution.The augmented matrix of the system is

1 3 1 a−1 −2 1 b3 7 −1 c

Now apply Gaussian elimination as follows.

Step 1: r2 ← r2 + r1 and r3 ← r3 − 3r1

1 3 1 a0 1 2 b + a0 −2 −4 c− 3a

Step 2: r3 ← r3 + 2r2

1 3 1 a0 1 2 b + a0 0 0 c− a + 2b

The system has no solution if c − a + 2b 6= 0. The system has infinitely manysolutions if c− a + 2b = 0. In this case, the solution is given by x1 = 5t− (2a +3b), x2 = (a + b)− 2t, x3 = t

1.8 Homogeneous Systems of Linear Equations

So far we have been discussing practical and systematic procedures to solvelinear systems of equations. In this section, we will describe a theoretical methodfor solving systems. The idea consists of finding the general solution of thesystem with zeros on the right-hand side , call it (x1, x2, · · · , xn), and then

Page 33: Fundamentals of Linear Algebra - Mathematics & Statisticsmelbial2/classes/Linear Algebra... · 2006-01-13 · 1.1 Systems of Linear Equations Many practical problems can be reduced

1.8. HOMOGENEOUS SYSTEMS OF LINEAR EQUATIONS 33

find a particular solution to the given system, say (y1, y2, · · · , yn). The generalsolution of the original system is then the sum

(x1, x2, · · · , xn) + (y1, y2, · · · , yn) = (x1 + y1, x2 + y2, · · · , xn + yn).

A homogeneous linear system is any system of the form

a11x1 + a12x2 + · · ·+ a1nxn = 0a21x1 + a22x2 + · · ·+ a2nxn = 0

........................ ....am1x1 + am2x2 + · · ·+ amnxn = 0.

Every homogeneous system is consistent, since x1 = 0, x2 = 0, · · · , xn = 0is always a solution. This solution is called the trivial solution; any othersolution is called nontrivial. By Theorem 5, a homogeneous system has eithera unique solution (the trivial solution) or infinitely many solutions. That isa homogeneous system is always consistent. The following theorem is a casewhere a homogeneous system is assured to have a nontrivial solution.

Theorem 6Let Ax = 0 be a homogeneous system in m unknowns and n equations.(1) If rank(A) < m then the system has a nontrivial solution. Hence, by Exer-cise 14 the system has infinitely many solutions.(2) If the number of unknowns exceeds the number of equations, i.e. n < m,then the system has a nontrivial solution.

Proof.Applying the Gauss-Jordan elimination to the augmented matrix [A|0] we obtainthe matrix [B|0]. The number of nonzero rows of B is equals to rank(A). Supposefirst that rank(A) = r < m. In this case, the system Bx = 0 has r equations in munknowns. Thus, the system has m− r independent variables and consequentlythe system Bx = 0 has a nontrivial solution. By Theorem 4, the system Ax = 0has a nontrivial solution. This proves (1). To prove (2), suppose n < m. If thesystem has only the trivial solution then by (1) we must have rank(A) ≥ m.This implies that n < m ≤ rank(A) ≤ n, a contradiction.

Exercise 38Solve the following homogeneous system using Gauss-Jordan elimination.

2x1 + 2x2 − x3 + x5 = 0−x1 − x2 + 2x3 − 3x4 + x5 = 0x1 + x2 − 2x3 − x5 = 0

x3 + x4 + x5 = 0.

Solution.The reduction of the augmented matrix to reduced row-echelon form is outlinedbelow.

2 2 −1 0 1 0−1 −1 2 −3 1 01 1 −2 0 −1 00 0 1 1 1 0

Page 34: Fundamentals of Linear Algebra - Mathematics & Statisticsmelbial2/classes/Linear Algebra... · 2006-01-13 · 1.1 Systems of Linear Equations Many practical problems can be reduced

34 CHAPTER 1. LINEAR SYSTEMS

Step 1: r3 ← r3 + r2

2 2 −1 0 1 0−1 −1 2 −3 1 00 0 0 −3 0 00 0 1 1 1 0

Step 2: r3 ↔ r4 and r1 ↔ r2

−1 −1 2 −3 1 02 2 −1 0 1 00 0 1 1 1 00 0 0 −3 0 0

Step 3: r2 ← r2 + 2r1 and r4 ← − 13r4

−1 −1 2 −3 1 00 0 3 −6 3 00 0 1 1 1 00 0 0 1 0 0

Step 4: r1 ← −r1 and r2 ← 13r2

1 1 −2 3 −1 00 0 1 −2 1 00 0 1 1 1 00 0 0 1 0 0

Step 5: r3 ← r3 − r2

1 1 −2 3 −1 00 0 1 −2 1 00 0 0 3 0 00 0 0 1 0 0

Step 6: r4 ← r4 − 13r3 and r3 ← 1

3r3

1 1 −2 3 −1 00 0 1 −2 1 00 0 0 1 0 00 0 0 0 0 0

Step 7: r1 ← r1 − 3r3 and r2 ← r2 + 2r3

1 1 −2 0 −1 00 0 1 0 1 00 0 0 1 0 00 0 0 0 0 0

Page 35: Fundamentals of Linear Algebra - Mathematics & Statisticsmelbial2/classes/Linear Algebra... · 2006-01-13 · 1.1 Systems of Linear Equations Many practical problems can be reduced

1.8. HOMOGENEOUS SYSTEMS OF LINEAR EQUATIONS 35

Step 8: r1 ← r1 + 2r2

1 1 0 0 1 00 0 1 0 1 00 0 0 1 0 00 0 0 0 0 0

The corresponding system is

x1 + x2 + x5 = 0x3 + x5 = 0

x4 = 0

The free variables are x2 = s, x5 = t and the general solution is given by theformula: x1 = −s− t, x2 = s, x3 = −t, x4 = 0, x5 = t

Exercise 39Solve the following homogeneous system using Gaussian elimination.

x1 + 3x2 + 5x3 + x4 = 04x1 − 7x2 − 3x3 − x4 = 03x1 + 2x2 + 7x3 + 8x4 = 0

Solution.The augmented matrix for the system is

1 3 5 1 04 −7 −3 −1 03 2 7 8 0

We reduce this matrix into a row-echelon form as follows.Step 1: r2 ← r2 − r3

1 3 5 1 01 −9 −10 −9 03 2 7 8 0

Step 2: r2 ← r2 − r1 and r3 ← r3 − 3r1

1 3 5 1 00 −12 −15 −10 00 − 7 − 8 5 0

Step 3: r2 ← − 112r2

1 3 5 1 00 1 5

456 0

0 −7 −8 5 0

Step 4: r3 ← r3 + 7r2

1 3 5 1 00 1 5

456 0

0 0 34

656 0

Page 36: Fundamentals of Linear Algebra - Mathematics & Statisticsmelbial2/classes/Linear Algebra... · 2006-01-13 · 1.1 Systems of Linear Equations Many practical problems can be reduced

36 CHAPTER 1. LINEAR SYSTEMS

Step 5: r3 ← 43r3

1 3 5 1 00 1 5

456 0

0 0 1 1309 0

We see that x4 = t is the only free variable. Solving for the leading variablesusing back substitution we find x1 = 176

9 t, x2 = 1559 t, and x3 = − 130

9 t

A nonhomogeneous system is a homogeneous system together with a nonzeroright-hand side. Theorem 6 (2) applies only to homogeneous linear systems. Anonhomogeneous system with more unknowns than equations need not be con-sistent.

Exercise 40Show that the following system is inconsistent.

{x1 + x2 + x3 = 02x1 + 2x2 + 2x3 = 4.

Solution.Multiplying the first equation by −2 and adding the resulting equation to thesecond we obtain 0 = 4 which is impossible. So the system is inconsistent

The fundamental relationship between a nonhomogeneous system and its cor-responding homogeneous system is given by the following theorem.

Theorem 7Let Ax = b be a linear system of equations. If y is a particular solution of thenonhomogeneous system Ax = b and W is the solution set of the homogeneoussystem Ax = 0 then the general solution of Ax = b consists of elements of theform y + w where w ∈ W.

Proof.Let S be the solution set of Ax = b. We will show that S = {y + w : w ∈ W}.That is, every element of S can be written in the form y + w for some w ∈ W ,and conversely, any element of the form y + w, with w ∈ W, is a solution of thenonhomogeneous system, i.e. a member of S. So, let z be an element of S, i.e.Az = b. Write z = y + (z − y). Then A(z − y) = Az − Ay = b + 0 = b. Thus,z = y + w with w = z − y ∈ W. Conversely, let z = y + w, with w ∈ W. ThenAz = A(y + w) = Ay + Aw = b + 0 = b. That is z ∈ S.

We emphasize that the above theorem is of theoretical interest and does nothelp us to obtain explicit solutions of the system Ax = b. Solutions are obtainedby means of the methods discussed in this chapter, i.e. Gauss elimination,Gauss-Jordan elimination or by the methods of using determinants to be dis-cussed in Chapter 3.

Page 37: Fundamentals of Linear Algebra - Mathematics & Statisticsmelbial2/classes/Linear Algebra... · 2006-01-13 · 1.1 Systems of Linear Equations Many practical problems can be reduced

1.9. REVIEW PROBLEMS 37

Exercise 41Show that if a homogeneous system of linear equations in n unknowns has anontrivial solution then rank(A) < n, where A is the coefficient matrix.

Solution.Since rank(A) ≤ n then either rank(A) = n or rank(A) < n. If rank(A) < nthen we are done. So suppose that rank(A) = n. Then there is a matrix Bthat is row equivalent to A and that has n nonzero rows. Moreover, B has thefollowing form

1 a12 a13 · · · a1n 00 1 a23 · · · a2n 0...

......

......

0 0 0 · · · 1 0

The corresponding system is triangular and can be solved by back substitutionto obtain the solution x1 = x2 = · · · = xn = 0 which is a contradiction. Thuswe must have rank(A) < n

1.9 Review Problems

Exercise 42Which of the following equations are not linear and why:(a) x2

1 + 3x2 − 2x3 = 5.(b) x1 + x1x2 + 2x3 = 1.(c) x1 + 2

x2+ x3 = 5.

Exercise 43Show that (2s + 12t + 13, s,−s− 3t− 3, t) is a solution to the system

{2x1 + 5x2 + 9x3 + 3x4 = −1x1 + 2x2 + 4x3 = 1

Exercise 44Solve each of the following systems using the method of elimination:(a) {

4x1 − 3x2 = 02x1 + 3x2 = 18

(b) {4x1 − 6x2 = 106x1 − 9x2 = 15

(c) {2x1 + x2 = 32x1 + x2 = 1

Which of the above systems is consistent and which is inconsistent?

Page 38: Fundamentals of Linear Algebra - Mathematics & Statisticsmelbial2/classes/Linear Algebra... · 2006-01-13 · 1.1 Systems of Linear Equations Many practical problems can be reduced

38 CHAPTER 1. LINEAR SYSTEMS

Exercise 45Find the general solution of the linear system

{x1 − 2x2 + 3x3 + x4 = −3

2x1 − x2 + 3x3 − x4 = 0

Exercise 46Find a, b, and c so that the system

x1 + ax2 + cx3 = 0bx1 + cx2 − 3x3 = 1ax1 + 2x2 + bx3 = 5

has the solution x1 = 3, x2 = −1, x3 = 2.

Exercise 47Find a relationship between a,b,c so that the following system is consistent

x1 + x2 + 2x3 = ax1 + x3 = b2x1 + x2 + 3x3 = c

Exercise 48For which values of a will the following system have (a) no solutions? (b) exactlyone solution? (c) infinitely many solutions?

x1 + 2x2 − 3x3 = 43x1 − x2 + 5x3 = 24x1 + x2 + (a2 − 14)x3 = a + 2

Exercise 49Find the values of A,B,C in the following partial fraction

x2 − x + 3(x2 + 2)(2x− 1)

=Ax + B

x2 + 2+

C

2x− 1.

Exercise 50Find a quadratic equation of the form y = ax2 + bx + c that goes through thepoints (−2, 20), (1, 5), and (3, 25).

Exercise 51For which value(s) of the constant k does the following system have (a) nosolutions? (b) exactly one solution? (c) infinitely many solutions?

{x1 − x2 = 32x1 − 2x2 = k

Exercise 52Find a linear equation in the unknowns x1 and x2 that has a general solutionx1 = 5 + 2t, x2 = t.

Page 39: Fundamentals of Linear Algebra - Mathematics & Statisticsmelbial2/classes/Linear Algebra... · 2006-01-13 · 1.1 Systems of Linear Equations Many practical problems can be reduced

1.9. REVIEW PROBLEMS 39

Exercise 53Consider the linear system

2x1 + 3x2 − 4x3 + x4 = 5−2x1 + x3 = 73x1 + 2x2 − 4x3 = 3

(a) Find the coefficient and augmented matrices of the linear system.(b) Find the matrix notation.

Exercise 54Solve the following system using elementary row operations on the augmentedmatrix:

5x1 − 5x2 − 15x3 = 404x1 − 2x2 − 6x3 = 193x1 − 6x2 − 17x3 = 41

Exercise 55Solve the following system.

2x1 + x2 + x3 = −1x1 + 2x2 + x3 = 03x1 − 2x3 = 5

Exercise 56Which of the following matrices are not in reduced row-ehelon from and why?(a)

1 −2 0 00 0 0 00 0 1 00 0 0 1

(b)

1 0 0 30 2 0 −20 0 3 0

(c)

1 0 40 1 −20 0 0

Exercise 57Use Gaussian elimination to convert the following matrix into a row-echelonmatrix.

1 −3 1 −1 0 −1−1 3 0 3 1 32 −6 3 0 −1 2−1 3 1 5 1 6

Page 40: Fundamentals of Linear Algebra - Mathematics & Statisticsmelbial2/classes/Linear Algebra... · 2006-01-13 · 1.1 Systems of Linear Equations Many practical problems can be reduced

40 CHAPTER 1. LINEAR SYSTEMS

Exercise 58Use Gauss-Jordan elimination to convert the following matrix into reduced row-echelon form.

−2 1 1 156 −1 −2 −361 −1 −1 −11−5 −5 −5 −14

Exercise 59Solve the following system using Gauss-Jordan elimination.

3x1 + x2 + 7x3 + 2x4 = 132x1 − 4x2 + 14x3 − x4 = −105x1 + 11x2 − 7x3 + 8x4 = 592x1 + 5x2 − 4x3 − 3x4 = 39

Exercise 60Find the rank of each of the following matrices.(a)

−1 −1 0 00 0 2 34 0 −2 13 −1 0 4

(b)

1 −1 32 0 4−1 −3 1

Exercise 61Choose h and k such that the following system has (a) no solutions, (b) exactlyone solution, and (c) infinitely many solutions.

{x1 − 3x2 = 1

2x1 − hx2 = k

Exercise 62Solve the linear system whose augmented matrix is reduced to the following re-duced row-echelon form

1 0 0 −7 80 1 0 3 20 0 1 1 −5

Exercise 63Solve the linear system whose augmented matrix is reduced to the following row-echelon form

1 −3 7 10 1 4 00 0 0 1

Page 41: Fundamentals of Linear Algebra - Mathematics & Statisticsmelbial2/classes/Linear Algebra... · 2006-01-13 · 1.1 Systems of Linear Equations Many practical problems can be reduced

1.9. REVIEW PROBLEMS 41

Exercise 64Solve the linear system whose augmented matrix is given by

1 1 2 8−1 −2 3 13 −7 4 10

Exercise 65Find the value(s) of a for which the following system has a nontrivial solution.Find the general solution.

x1 + 2x2 + x3 = 0x1 + 3x2 + 6x3 = 02x1 + 3x2 + ax3 = 0

Exercise 66Solve the following homogeneous system.

x1 − x2 + 2x3 + x4 = 02x1 + 2x2 − x4 = 03x1 + x2 + 2x3 + x4 = 0

Exercise 67Let A be an m× n matrix.(a) Prove that if y and z are solutions to the homogeneous system Ax = 0 theny + z and cy are also solutions, where c is a number.(b) Give a counterexample to show that the above is false for nonhomogeneoussystems.

Exercise 68Show that the converse of Theorem 6 is false. That is, show the existence of anontrivial solution does not imply that the number of unknowns is greater thanthe number of equations.

Exercise 69 (Network Flow)The junction rule of a network says that at each junction in the network thetotal flow into the junction must equal the total flows out. To illustrate the useof this rule, consider the network shown in the accompanying diagram. Find thepossible flows in the network.

40 60

50

50

D

C

B

A

f_1 f_2

f_5f_4

f_3

Page 42: Fundamentals of Linear Algebra - Mathematics & Statisticsmelbial2/classes/Linear Algebra... · 2006-01-13 · 1.1 Systems of Linear Equations Many practical problems can be reduced

42 CHAPTER 1. LINEAR SYSTEMS

Page 43: Fundamentals of Linear Algebra - Mathematics & Statisticsmelbial2/classes/Linear Algebra... · 2006-01-13 · 1.1 Systems of Linear Equations Many practical problems can be reduced

Chapter 2

Matrices

Matrices are essential in the study of linear algebra. The concept of matriceshas become a tool in all branches of mathematics, the sciences, and engineering.They arise in many contexts other than as augmented matrices for systems oflinear equations. In this chapter we shall consider this concept as objects intheir own right and develop their properties for use in our later discussions.

2.1 Matrices and Matrix Operations

In this section, we discuss several types of matrices. We also examine four op-erations on matrices- addition, scalar multiplication, trace, and the transposeoperation- and give their basic properties. Also, we introduce symmetric, skew-symmetric matrices.

A matrix A of size m× n is a rectangular array of the form

A =

a11 a12 ... a1n

a21 a22 ... a2n

... ... ... ...am1 am2 ... amn

where the aij ’s are the entries of the matrix, m is the number of rows, n is thenumber of columns. The zero matrix 0 is the matrix whose entries are all 0.The n×n identity matrix In is a square matrix whose main diagonal consistsof 1′s and the off diagonal entries are all 0. A matrix A can be represented withthe following compact notation A = (aij). The ith row of the matrix A is

[ai1, ai2, ..., ain]

43

Page 44: Fundamentals of Linear Algebra - Mathematics & Statisticsmelbial2/classes/Linear Algebra... · 2006-01-13 · 1.1 Systems of Linear Equations Many practical problems can be reduced

44 CHAPTER 2. MATRICES

and the jth column is

a1j

a2j

...amj

In what follows we discuss the basic arithmetic of matrices.

Two matrices are said to be equal if they have the same size and their cor-responding entries are all equal. If the matrix A is not equal to the matrix Bwe write A 6= B.

Exercise 70Find x1, x2 and x3 such that

x1 + x2 + 2x3 0 12 3 2x1 + 4x2 − 3x3

4 3x1 + 6x2 − 5x3 5

=

9 0 12 3 14 0 5

Solution.Because corresponding entries must be equal, this gives the following linearsystem

x1 + x2 + 2x3 = 92x1 + 4x2 − 3x3 = 13x1 + 6x2 − 5x3 = 0

The augmented matrix of the system is

1 1 2 92 4 −3 13 6 −5 0

The reduction of this matrix to row-echelon form is

Step 1: r2 ← r2 − 2r1 and r3 ← r3 − 3r1

1 1 2 90 2 − 7 −170 3 −11 −27

Step 2: r2 ↔ r3

1 1 2 90 3 −11 −270 2 − 7 −17

Step 3: r2 ← r2 − r3

1 1 2 90 1 −4 −100 2 −7 −17

Page 45: Fundamentals of Linear Algebra - Mathematics & Statisticsmelbial2/classes/Linear Algebra... · 2006-01-13 · 1.1 Systems of Linear Equations Many practical problems can be reduced

2.1. MATRICES AND MATRIX OPERATIONS 45

Step 4: r3 ← r3 − 2r2

1 1 2 90 1 −4 −100 0 1 3

The corresponding system is

x1 + x2 + 2x3 = 9x2 − 4x3 = −10

x3 = 3

Using backward substitution we find: x1 = 1, x2 = 2, x3 = 3

Exercise 71Solve the following matrix equation for a, b, c, and d

(a− b b + c3d + c 2a− 4d

)=

(8 17 6

)

Solution.Equating corresponding entries we get the system

a − b = 8b + c = 1

c + 3d = 72a − 4d = 6

The augmented matrix is

1 −1 0 0 80 1 1 0 10 0 1 3 71 0 0 −4 6

We next apply Gaussian elimination as follows.

Step 1: r4 ← r4 − r1

1 −1 0 0 80 1 1 0 10 0 1 3 70 1 0 −4 −2

Step 2: r4 ← r4 − r2

1 −1 0 0 80 1 1 0 10 0 1 3 70 0 −1 −4 −3

Step 3: r4 ← r4 + r3

1 −1 0 0 80 1 1 0 10 0 1 3 70 0 0 −1 4

Page 46: Fundamentals of Linear Algebra - Mathematics & Statisticsmelbial2/classes/Linear Algebra... · 2006-01-13 · 1.1 Systems of Linear Equations Many practical problems can be reduced

46 CHAPTER 2. MATRICES

Using backward substitution to find: a = −10, b = −18, c = 19, d = −4

Next, we introduce the operation of addition of two matrices. If A and Bare two matrices of the same size, then the sum A + B is the matrix obtainedby adding together the corresponding entries in the two matrices. Matrices ofdifferent sizes cannot be added.

Exercise 72Consider the matrices

A =(

2 13 4

), B =

(2 13 5

), C =

(2 1 03 4 0

)

Compute, if possible, A + B, A + C and B + C.

Solution.We have

A + B =(

4 26 10

)

A + B and B + C are undefined since A and B are of different sizes as well asA and C

From now on, a constant number will be called a scalar. If A is a matrixand c is a scalar, then the product cA is the matrix obtained by multiplyingeach entry of A by c. Hence, −A = (−1)A. We define, A−B = A + (−B). Thematrix cIn is called a scalar matrix.

Exercise 73Consider the matrices

A =(

2 3 41 2 1

), B =

(0 2 71 −3 5

)

Compute A− 3B.

Solution.Using the above definitions we have

A− 3B =(

2 −3 −17−2 11 −14

)

Let Mmn be the collection of all m × n matrices. This set under the opera-tions of addition and scalar multiplication satisfies algebraic properties whichwill remind us of the system of real numbers. The proofs of these propertiesdepend on the properties of real numbers. Here we shall assume that the readeris familiar with the basic algebraic properties of IR. The following theorem listthe properties of matrix addition and multiplication of a matrix by a scalar.

Page 47: Fundamentals of Linear Algebra - Mathematics & Statisticsmelbial2/classes/Linear Algebra... · 2006-01-13 · 1.1 Systems of Linear Equations Many practical problems can be reduced

2.1. MATRICES AND MATRIX OPERATIONS 47

Theorem 8Let A,B, and C be m× n and let c, d be scalars. Then

(i) A + B = B + A,(ii) (A + B) + C = A + (B + C) = A + B + C,(iii) A + 0 = 0 + A = A,(iv) A + (−A) = 0,(v) c(A + B) = cA + cB,(vi) (c + d)A = cA + dA,(vii) (cd)A = c(dA),(viii) ImA = AIn = A.

Proof.(i) A+B = (aij)+ (bij) = (aij + bij) = (bij +aij) = (bij)+ (aij) = B +A, sinceaddition of scalars is commutative.(ii) Use the fact that addition of scalars is associative.(iii) A + 0 = (aij) + (0) = (aij + 0) = (aij) = A.We leave the proofs of the remaining properties to the reader

Exercise 74Solve the following matrix equation.

(3 2−1 1

)+

(a bc d

)=

(1 0−1 2

)

Solution.Adding and then equating corresponding entries we obtain a = −2, b = −2, c =0, and d = 1

If A is a square matrix then the sum of the entries on the main diagonal iscalled the trace of A and is denoted by tr(A).

Exercise 75Find the trace of the coefficient matrix of the system

− x2 + 3x3 = 1x1 + 2x3 = 2−3x1 − 2x2 = 4

Solution.If A is the coefficient matrix of the system then

A =

0 −1 31 0 2−3 −2 0

The trace of A is the number tr(A) = 0 + 0 + 0 = 0

Two useful properties of the trace of a matrix are given in the following theorem.

Page 48: Fundamentals of Linear Algebra - Mathematics & Statisticsmelbial2/classes/Linear Algebra... · 2006-01-13 · 1.1 Systems of Linear Equations Many practical problems can be reduced

48 CHAPTER 2. MATRICES

Theorem 9Let A = (aij) and B = (bij) be two n× n matrices and c be a scalar. Then(i) tr(A + B) = tr(A) + tr(B),(ii) tr(cA) = c tr(A).

Proof.(i) tr(A + B) =

∑ni=1(aii + bii) =

∑ni=1 aii +

∑ni=1 bii = tr(A) + tr(B).

(ii) tr(cA) =∑n

i=1 caii = c∑n

i=1 aii = c tr(A).

If A is an m × n matrix then the transpose of A, denoted by AT , is definedto be the n×m matrix obtained by interchanging the rows and columns of A,that is the first column of AT is the first row of A, the second column of AT isthe second row of A, etc. Note that, if A = (aij) then AT = (aji). Also, if A isa square matrix then the diagonal entries on both A and AT are the same.

Exercise 76Find the transpose of the matrix

A =(

2 3 41 2 1

),

Solution.The transpose of A is the matrix

AT =

2 13 24 1

The following result lists some of the properties of the transpose of a matrix.

Theorem 10Let A = (aij), and B = (bij) be two m × n matrices, C = (cij) be an n × nmatrix, and c a scalar. Then(i) (AT )T = A,(ii) (A + B)T = AT + BT ,(iii) (cA)T = cAT ,(iv) tr(CT ) = tr(C).

Proof.(i) (AT )T = (aji)T = (aij) = A.(ii) (A + B)T = (aij + bij)T = (aji + bji) = (aji) + (bji) = AT + BT .(iii) (cA)T = (caij)T = (caji) = c(aji) = cAT .(iv) tr(CT ) =

∑ni=1 cii = tr(C).

Exercise 77A square matrix A is called symmetric if AT = A. A square matrix A is called

Page 49: Fundamentals of Linear Algebra - Mathematics & Statisticsmelbial2/classes/Linear Algebra... · 2006-01-13 · 1.1 Systems of Linear Equations Many practical problems can be reduced

2.1. MATRICES AND MATRIX OPERATIONS 49

skew-symmetric if AT = −A.(a) Show that the matrix

A =

1 2 32 4 53 5 6

is a symmetric matrix.(b) Show that the matrix

A =

0 2 3−2 0 −4−3 4 0

is a skew-symmetric matrix.(c) Show that for any square matrix A the matrix S = 1

2 (A + AT ) is symmetricand the matrix K = 1

2 (A−AT ) is skew-symmetric.(d) Show that if A is a square matrix, then A = S + K, where S is symmetricand K is skew-symmetric.(e) Show that the representation in (d) is unique.

Solution.(a) A is symmetric since

AT =

0 2 3−2 0 −4−3 4 0

= A

(b) A is skew-symmetric since

AT =

0 −2 −32 0 43 −4 0

= −A

(c) Because ST = 12 (A + AT )T = 1

2 (A + AT ) then S is symmetric. Similarly,KT = 1

2 (A − AT )T = 12 (AT − A) = − 1

2 (A − AT ) = −K so that K is skew-symmetric.(d) S + K = 1

2 (A + AT ) + 12 (A−AT ) = A.

(e) Let S′ be a symmetric matrix and K ′ be skew-symmetric such that A =S′ + K ′. Then S + K = S′ + K ′ and this implies that S − S′ = K −K ′. Butthe matrix S−S′ is symmetric and the matrix K ′−K is skew-symmetric. Thisequality is true only when S − S′ is the zero matrix. That is S = S′. Hence,K = K ′

Exercise 78Let A be an n× n matrix.(a) Show that if A is symmetric then A and AT have the same main diagonal.(b) Show that if A is skew-symmetric then the entries on the main diagonal are0.(c) If A and B are symmetric then so is A + B.

Page 50: Fundamentals of Linear Algebra - Mathematics & Statisticsmelbial2/classes/Linear Algebra... · 2006-01-13 · 1.1 Systems of Linear Equations Many practical problems can be reduced

50 CHAPTER 2. MATRICES

Solution.(a) Let A = (aij) be symmetric. Let AT = (bij). Then bij = aji for all 1 ≤i, j ≤ n. In particular, when i = j we have bii = aii. That is, A and AT havethe same main diagonal.(b) Since A is skew-symmetric then aij = −aji. In particular, aii = −aii andthis implies that aii = 0.(c) Suppose A and B are symmetric. Then (A + B)T = AT + BT = A + B.That is, A + B is symmetric

Exercise 79Let A be an m×n matrix and α a real number. Show that if αA = 0 then eitherα = 0 or A = 0.

Solution.Let A = (aij). Then αA = (αaij). Suppose αA = 0. Then αaij = 0 for all0 ≤ i ≤ m and 0 ≤ j ≤ n. If α 6= 0 then aij = 0 for all indices i and j. In thiscase, A = 0

2.2 Properties of Matrix Multiplication

In the previous section we discussed some basic properties associated with ma-trix addition and scalar multiplication. Here we introduce another importantoperation involving matrices-the product.We have already introduced the concept of matrix multiplication in Section 1.3.For the sake of completeness we review this concept.Let A = (aij) be a matrix of size m×n and B = (bij) be a matrix of size n× p.Then the product matrix is a matrix of size m× p and entries

cij = ai1b1j + ai2b2j + ... + ainbnj ,

that is, cij is obtained by multiplying componentwise the entries of the ith rowof A by the entries of the jth column of B. It is very important to keep in mindthat the number of columns of the first matrix must be equal to the number ofrows of the second matrix; otherwise the product is undefined.An interesting question associated with matrix multiplication is the following:If A and B are square matrices then is it always true that AB = BA?The answer to this question is negative. In general, matrix multiplication is notcommutative, as the following exercise shows.

Exercise 80

A =(

1 23 2

), B =

(2 −1−3 4

)

Show that AB 6= BA. Hence, matrix multiplication is not commutative.

Page 51: Fundamentals of Linear Algebra - Mathematics & Statisticsmelbial2/classes/Linear Algebra... · 2006-01-13 · 1.1 Systems of Linear Equations Many practical problems can be reduced

2.2. PROPERTIES OF MATRIX MULTIPLICATION 51

Solution.Using the definition of matrix multiplication we find

AB =( −4 7

0 5

), BA =

( −1 29 2

)

Hence, AB 6= BA

Exercise 81Consider the matrices

A =(

2 13 4

), B =

(2 13 5

), C =

( −1 −211 4

)

(a) Compare A(BC) and (AB)C.(b) Compare A(B + C) and AB + AC.(c) Compute I2A and AI2, where I2 is the 2× 2 identity matrix.

Solution.(a)

A(BC) = (AB)C =(

70 14235 56

)

(b)

A(B + C) = AB + AC =(

16 759 33

)

(c) AI2 = I2A = A

Exercise 82Let A be an m× n matrix and B an n× p matrix.

(a) Show that the entries in the jth column of AB are the entries in the productABj where Bj is the jth column of B. Thus, AB = [AB1, AB2, · · · , ABp].(b) Show that the entries in the ith row of AB are the entries of the productAiB where Ai is the ith row of A. That is,

AB =

A1BA2B

...AmB

Solution.Let Cj be the jth column of AB then

Cj =

a11b1j + a12b2j + · · ·+ a1nbnj

a21b1j + a22b2j + · · ·+ a2nbnj

...an1b1j + an2b2j + · · ·+ amnbnj

Page 52: Fundamentals of Linear Algebra - Mathematics & Statisticsmelbial2/classes/Linear Algebra... · 2006-01-13 · 1.1 Systems of Linear Equations Many practical problems can be reduced

52 CHAPTER 2. MATRICES

On the other hand,

ABj =

a11 a12 · · · a1n

a21 a22 · · · a2n

......

...am1 am2 · · · amn

b1j

b2j

...bnj

=

a11b1j + a12b2j + · · ·+ a1nbnj

a21b1j + a22b2j + · · ·+ a2nbnj

...am1b1j + am2b2j + · · ·+ amnbnj

= Cj

(b) Let r1, r2, · · · , rm be the rows of AB. Then

rTi =

ai1b11 + ai2b21 + · · ·+ ainbn1

ai1b12 + ai2b22 + · · ·+ ainbn2

...ai1b1p + ai2b2p + · · ·+ ainbnp

If Ai is the ith row of A then

AiB =[

ai1 ai2 · · · ain

]

b11 b12 · · · b1p

b21 b22 · · · b2p

......

...bn1 bn2 · · · bnp

= ri

Exercise 83(a) If B has a column of zeros, show that the same is true of AB for any A.(b) If A has a row of zeros, show that the same is true of AB for any B.

Solution.(a) Follows from (a) of the previous exercise.(b) Follows from (b) of the previous exercise

Exercise 84Let A be an m × n matrix. Show that if Ax = 0 for all n × 1 matrix x thenA = 0.

Solution.Suppose the contrary. Then there exist indices i and j such that aij 6= 0. Let xbe the n× 1 matrix whose jth row is 1 and 0 elsewhere. Then the jth entry ofAx is 0 = aij 6= 0, a contradiction

As the reader has noticed so far, most of the basic rules of arithmetic of realnumbers also hold for matrices but a few do not. In Exercise 80 we have seenthat matrix multiplication is not commutative. The following exercise showsthat the cancellation law of numbers does not hold for matrix product.

Page 53: Fundamentals of Linear Algebra - Mathematics & Statisticsmelbial2/classes/Linear Algebra... · 2006-01-13 · 1.1 Systems of Linear Equations Many practical problems can be reduced

2.2. PROPERTIES OF MATRIX MULTIPLICATION 53

Exercise 85(a) Consider the matrices

A =(

1 00 0

), B =

(0 01 0

), C =

(0 00 1

)

Compare AB and AC. Is it true that B = C?(b) Find two square matrices A and B such that AB = 0 but A 6= 0 and B 6= 0.

Solution.(a) Note that B 6= C even though AB = AC = 0.(b) The given matrices satisfy AB = 0 with A 6= 0 and B 6= 0

Matrix multiplication shares many properties of the product of real numberswhich are listed in the following theorem

Theorem 11Let A be a matrix of size m× n. Then(a) A(BC) = (AB)C, where B is of size n× p, C of size p× q.(b) A(B + C) = AB + AC, where B and C are of size n× p.(c) (B + C)A = BA + CA, where B and C are of size l ×m.(d) c(AB) = (cA)B = A(cB), where c denotes a scalar.

Proof.Let A = (aij), B = (bij), C = (cij).(a) Let AB = (dij), BC = (eij), A(BC) = (fij), and (AB)C = (gij). Then fromthe definition of matrix multiplication we have

fij = (ith row of A)(jth column of BC)= ai1e1j + ai2e2j + · · ·+ ainenj

= ai1(b11c1j + b12c2j + · · ·+ b1pcpj)+ ai2(b21c1j + b22c2j + · · ·+ b2pcpj)+ · · ·+ ain(bn1c1j + bn2c2j + · · ·+ bnpcpj)= (ai1b11 + ai2b21 + · · ·+ ainbn1)c1j

+ (ai1b12 + ai2b22 + · · ·+ ainbn2)c2j

+ · · ·+ (ai1b1p + ai2b2p + · · ·+ ainbnp)cpj

= di1c1j + di2c2j + · · ·+ dipcpj

= (ith row of AB)(jth column of C)= gij .

(b) Let A(B + C) = (kij), AC = (hij) Then

kij = (ith row of A)(jth column of B + C)= ai1(b1j + c1j) + ai2(b2j + c2j) + · · ·+ ain(bnj + cnj)= (ai1b1j + ai2b2j + · · ·+ ainbnj)+ (ai1c1j + ai2c2j + · · ·+ aincnj)= dij + hij .

Page 54: Fundamentals of Linear Algebra - Mathematics & Statisticsmelbial2/classes/Linear Algebra... · 2006-01-13 · 1.1 Systems of Linear Equations Many practical problems can be reduced

54 CHAPTER 2. MATRICES

(c) Similar to (b).(d) The proof is left for the reader.

In the next theorem we establish a property about the transpose of a matrix.

Theorem 12Let A = (aij), B = (bij) be matrices of sizes m×n and n×m respectively. Then(AB)T = BT AT .

proofLet AB = (cij). Then (AB)T = (cji). Let BT AT = (c′ij).Then

c′ij = (ith row of BT )(jth column of AT )= aj1b1i + aj2b2i + · · ·+ ajnbni

= cji

This ends a proof of the theorem

Exercise 86Let A be any matrix. Show that AAT and AT A are symmetric matrices.

Solution.First note that for any matrix A the matrices AAT and AT A are well-defined.Since (AAT )T = (AT )T AT = AAT then AAT is symmetric. Similarly, (AT A)T =AT (AT )T = AT A

Finally, we discuss the powers of a square matrix. Let A be a square ma-trix of size n × n. Then the non-negative powers of A are defined as follows:A0 = In, A1 = A, and for k ≥ 2, Ak = (Ak−1)A.

Exercise 87suppose that

A =(

1 23 4

)

Compute A3.

Solution.Multiplying the matrix A by itself three times we obtain

A3 =(

37 5481 118

)

For the sake of completeness, we mention few words regarding the mathemati-cal proof by induction which will be used in the next theorem as well as in therest of these notes. Let S be a statement that depends on a non-negative inte-ger n ≥ n0. To prove that S is valid for all n ≥ n0 one has to show the following:

1. Show that S is valid for n0.2. Supposing that S is valid for n > n0 show that S is valid for n + 1.

Page 55: Fundamentals of Linear Algebra - Mathematics & Statisticsmelbial2/classes/Linear Algebra... · 2006-01-13 · 1.1 Systems of Linear Equations Many practical problems can be reduced

2.3. THE INVERSE OF A SQUARE MATRIX 55

Theorem 13For any non-negative integers s, t we have(a) As+t = AsAt

(b) (As)t = Ast.

Proof.(a) Fix s. Let t = 1. Then by definition above we have As+1 = AsA. Now, weprove by induction on t that As+t = AsAt. The equality holds for t = 1. As theinduction hypothesis, suppose that As+t = AsAt. Then As+(t+1) = A(s+t)+1 =As+tA = (AsAt)A = As(AtA) = AsAt+1.(b) The proof is similar to (a) and is left to the reader.

Exercise 88Let A and B be two n× n matrices.(a) Show that tr(AB) = tr(BA).(b) Show that AB −BA = In is impossible.

Solution.(a) Let A = (aij) and B = (bij). Thentr(AB) =

∑ni=1(

∑nk=1 aikbki) =

∑ni=1(

∑nk=1 bikaki) = tr(BA).

(b) If AB − BA = In then 0 = tr(AB) − tr(BA) = tr(AB − BA) = tr(In) =n ≥ 1, a contradiction

2.3 The Inverse of a Square Matrix

Most problems in practice reduces to a system with matrix notation Ax = b.Thus, in order to get x we must somehow be able to eliminate the coefficientmatrix A. One is tempted to try to divide by A. Unfortunately such an operationhas not been defined for matrices. In this section we introduce a special type ofsquare matrices and formulate the matrix analogue of numerical division. Recallthat the n×n identity square matrix is the matrix In whose main diagonal entriesare 1 and off diagonal entries are 0.A square matrix A of size n is called invertible or non-singular if there existsa square matrix B of the same size such that AB = BA = In. In this caseB is called the inverse of A. A square matrix that is not invertible is calledsingular.

Exercise 89Show that the matrix

B =( −2 1

32 − 1

2

)

is the inverse of the matrix

A =(

1 23 4

)

Solution.Using matrix multiplication one checks that AB = BA = I2

Page 56: Fundamentals of Linear Algebra - Mathematics & Statisticsmelbial2/classes/Linear Algebra... · 2006-01-13 · 1.1 Systems of Linear Equations Many practical problems can be reduced

56 CHAPTER 2. MATRICES

Exercise 90Show that the matrix

A =(

1 00 0

)

is singular.

Solution.Let B = (bij) be a 2 × 2 matrix. If BA = I2 then the (2, 2)-th entry of BAis zero while the (2, 2)−entry of I2 is 1, which is impossible. Thus, A is singular

In Section 2.5 (See Theorem 20), we will prove that for a square matrix tobe invertible it suffices to prove the existence of a matrix B such that eitherAB = In or BA = In. In other words, there is no need to check the doubleequality AB = BA = In.However, it is important to keep in mind that the concept of invertibility isdefined only for square matrices. In other words, it is possible to have a matrixA of size m× n and a matrix B of size n×m such that AB = Im. It would bewrong to conclude that A is invertible and B is its inverse.

Exercise 91Let

A =(

1 0 00 1 0

), B =

1 00 10 0

Show that AB = I2.

Solution.Simple matrix multiplication shows that AB = I2. However, this does not implythat B is the inverse of A since BA is undefnied so that the condition BA = I2

fails

Exercise 92Show that the identity matrix is invertible but the zero matrix is not.

Solution.Since InIn = In then In is nonsingular and its inverse is In. Now, for any n×nmatrix B we have B0 = 0 6= In so that the zero matrix is not invertible

Now if A is a nonsingular matrix then how many different inverses does itpossess? The answer to this question is provided by the following theorem.

Theorem 14The inverse of a matrix is unique.

Page 57: Fundamentals of Linear Algebra - Mathematics & Statisticsmelbial2/classes/Linear Algebra... · 2006-01-13 · 1.1 Systems of Linear Equations Many practical problems can be reduced

2.3. THE INVERSE OF A SQUARE MATRIX 57

Proof.Suppose A has two inverses B and C. We will show that B = C. Indeed,B = BIn = B(AC) = (BA)C = InC = C.

Since an invertible matrix A has a unique inverse then we will denote it fromnow on by A−1.For an invertible matrix A one can now define the negative power of a squarematrix as follows: For any positive integer n ≥ 1, we define A−n = (A−1)n.The next theorem lists some of the useful facts about inverse matrices.

Theorem 15Let A and B be two square matrices of the same size n× n.(a) If A and B are invertible matrices then AB is invertible and (AB)−1 =B−1A−1.(b) If A is invertible then A−1 is invertible and (A−1)−1 = A.(c) If A is invertible then AT is invertible and (AT )−1 = (A−1)T

Proof.(a) If A and B are invertible then AA−1 = A−1A = In and BB−1 = B−1B = In.In This case, (AB)(B−1A−1) = A[B(B−1A−1)] = A[(BB−1)A−1] = A(InA−1) =AA−1 = In. Similarly, (B−1A−1)(AB) = In. It follows that B−1A−1 is the in-verse of AB.(b) Since A−1A = AA−1 = In then A is the inverse of A−1, i.e. (A−1)−1 = A.(c) Since AA−1 = A−1A = In then by taking the transpose of both sides weget (A−1)T AT = AT (A−1)T = In. This shows that AT is invertible with inverse(A−1)T .

Exercise 93(a) Under what conditions a diagonal matrix is invertible?(b) Is the sum of two invertible matrices necessarily invertible?

Solution.(a) Let D = (dii) be a diagonal n×n matrix. Let B = (bij) be an n×n matrixsuch that DB = In and let DB = (cij). Then using matrix multiplication wefind cij =

∑nk=1 dikbkj . If i 6= j then cij = diibij = 0 and cii = diibii = 1.

If dii 6= 0 for all 1 ≤ i ≤ n then bij = 0 for i 6= j and bii = 1dii

. Thus, ifd11d22 · · · dnn 6= 0 then D is invertible and its inverse is the diagonal matrixD−1 = ( 1

dii).

(b) The following two matrices are invertible but their sum , which is the zeromatrix, is not.

A =(

1 00 −1

),

( −1 00 1

)

Exercise 94Consider the 2× 2 matrix

A =(

a bc d

).

Page 58: Fundamentals of Linear Algebra - Mathematics & Statisticsmelbial2/classes/Linear Algebra... · 2006-01-13 · 1.1 Systems of Linear Equations Many practical problems can be reduced

58 CHAPTER 2. MATRICES

Show that if ad− bc 6= 0 then A−1 exists and is given by

A−1 =1

ad− bc

(d −b−c a

).

Solution.Let

B =(

x yz w

)

be a matrix such that BA = I2. Then using matrix multiplication we find(

ax + cy bx + dyaz + cw bz + dw

)=

(1 00 1

)

Equating corresponding entries we obtain the following systems of linear equa-tions in the unknowns x, y, z and w.

{ax + cy = 1bx + dy = 0

and {az + cw = 0bz + dw = 0

In the first system, using elimination we find (ad−bc)y = −b and (ad−bc)x = d.Similarly, using the second system we find (ad− bc)z = −c and (ad− bc)w = a.If ad− bc 6= 0 then one can solve for x, y, z, and w and in this case B = A−1 asgiven in the statement of the problem

Finally, we mention here that matrix inverses can be used to solve systemsof linear equations as suggested by the following theorem.

Theorem 16If A is an n × n invertible matrix and b is a column matrix then the equationAx = b has a unique solution x = A−1b.

Proof.Since A(A−1b) = (AA−1)b = Inb = b then A−1b is a solution to the equationAx = b. Now, if y is another solution then y = Iny = (A−1A)y = A−1(Ay) =A−1b.

Exercise 95Let A and B be n× n matrices.(a) Verify that A(In +BA) = (In +AB)A and that (In +BA)B = B(In +AB).(b) If In+AB is invertible show that In+BA is also invertible and (In+BA)−1 =In −B(In + AB)−1A.

Page 59: Fundamentals of Linear Algebra - Mathematics & Statisticsmelbial2/classes/Linear Algebra... · 2006-01-13 · 1.1 Systems of Linear Equations Many practical problems can be reduced

2.4. ELEMENTARY MATRICES 59

Solution.(a) A(In + BA) = AIn + ABA = InA + ABA = (In + AB). Similar argumentfor the second equality.(b) Suppose In + AB is invertible. Then postmultiplying the second equalityin (a) by −(In + AB)−1A to obtain −(In + BA)B(In + AB)−1A = −BA. Nowadd In + BA to both sides to obtain

(In + BA)[In −B(In + AB)−1A] = In

This says that In +BA is invertible and (I +n+BA)−1 = In−B(In +AB)−1A

Exercise 96If A is invertible and k 6= 0 show that (kA)−1 = 1

kA−1.

Solution.Suppose that A is invertible and k 6= 0. Then (kA)A−1 = k(AA−1) = kIn. Thisimplies (kA)( 1

kA−1) = In. Thus, kA is invertible with inverse equals to 1kA−1

Exercise 97Prove that if R is an n×n matrix in reduced row-echelon form and with no zerorows then R = In.

Solution.Since R has no zero rows then RT has no zero rows as well. Since the first rowof R is nonzero then the leading 1 must be in the (1, 1) position; otherwise,the first column will be zero and this shows that RT has a zero row which is acontradiction. By the definition of reduced row-echelon form all entries belowthe leading entry in the first column must be 0 and all entries to the right ofthe leading entry in the first row are 0. Now, we repeat this argument to the(n− 1)× (n− 1) matrix obtained by deleting the first row and the first columnof R and so on

2.4 Elementary Matrices

In this section we introduce a special type of invertible matrices, the so-calledelementary matrices, and we discuss some of their properties. As we shall see,elementary matrices will be used in the next section to develop an algorithm forfinding the inverse of a square matrix.An n × n elementary matrix is a matrix obtained from the identity matrixby performing one single elementary row operation.

Exercise 98Show that the following matrices are elementary matrices(a)

1 0 00 1 00 0 1

,

Page 60: Fundamentals of Linear Algebra - Mathematics & Statisticsmelbial2/classes/Linear Algebra... · 2006-01-13 · 1.1 Systems of Linear Equations Many practical problems can be reduced

60 CHAPTER 2. MATRICES

(b)

1 0 00 0 10 1 0

,

(c)

1 0 30 1 00 0 1

Solution.We list the operations that produce the given elementary matrices.(a) r1 ← 1r1.(b) r2 ↔ r3.(c) r1 ← r1 + 3r3

Exercise 99Consider the matrix

A =

1 0 2 32 −1 3 61 4 4 0

(a) Find the row equivalent matrix to A obtained by adding 3 times the first rowof A to the third row. Call the equivalent matrix B.(b) Find the elementary matrix E corresponding to the above elementary rowoperation.(c) Compare EA and B.

Solution.(a)

B =

1 0 2 32 −1 3 64 4 10 9

(b)

E =

1 0 00 1 03 0 1

(c) EA = B

The conclusion of the above exercise holds for any matrix of size m× n.

Theorem 17If the elementary matrix E results from performing a certain row operations onIm and if A is an m × n matrix, then the product of EA is the matrix thatresults when this same row operation is performed on A.

Page 61: Fundamentals of Linear Algebra - Mathematics & Statisticsmelbial2/classes/Linear Algebra... · 2006-01-13 · 1.1 Systems of Linear Equations Many practical problems can be reduced

2.4. ELEMENTARY MATRICES 61

Proof.We prove the theorem only for type II operations. Let E be the elementarymatrix of size m × m obtained by adding k times row p of Im to row q. LetB be the matrix obtained from A by adding k times row p to row q. We mustshow that B = EA. Let A = (aij). Then B = (bij), where bij = aij for i 6= q,and bqj = kapj + aqj . Let EA = (dij). We will show that bij = dij . If i 6= qthen dij = (ith row of E)(jth column of A) = aij = bij . If i = q, thendqj = (qth row of E)(jth column of A) = kapj + aqj = bqj . It follows thatEA = B. Types I and III are left for the reader

It follows from the above theorem that a matrix A is row equivalent to a ma-trix B if and only if B = EkEk−1 · · ·E1A, where E1, E2, · · · , Ek are elementarymatrices.The above theorem is primarily of theoretical interest and will be used for de-velopping some results about matrices and systems of linear equations. From acomputational point of view, it is preferred to perform row operations directlyrather than multiply on the left by an elementary matrix. Also, this theoremsays that an elementary row operation on A can be achieved by premultiplyingA by the corresponding elementary matrix E.Given any elementary row operation, there is another row operation ( called itsinverse) that reverse the effect of the first operation. The inverses are describedin the following chart.

Type Operation Inverse operationI ri ← cri ri ← 1

c ri

II rj ← cri + rj rj ← −cri + rj

III ri ↔ rj ri ↔ rj

The following theorem gives an important property of elementary matrices.

Theorem 18Every elementary matrix is invertible, and the inverse is an elementary matrix.

Proof.Let A be any n×n matrix. Let E be an elementary matrix obtained by applyinga row elementary operation ρ on In. By Theorem 17, applying ρ on A producesa matrix EA. Applying the inverse operation ρ−1 to EA gives F (EA) where Fis the elementary matrix obtained from In by applying the operation ρ−1. Sinceinverse row operations cancel the effect of each other, it follows that FEA = A.Since A was arbitrary, we can choose A = In. Hence, FE = In. A similarargument shows that EF = In. Hence E is invertible and E−1 = F

Exercise 100Show the following(a) A is row equivalent to A.(b) If A is row equivalent to B then B is row equivalent to A.(c) If A is row equivalent to B and B is row equivalent to C then A is rowequivalent to C.

Page 62: Fundamentals of Linear Algebra - Mathematics & Statisticsmelbial2/classes/Linear Algebra... · 2006-01-13 · 1.1 Systems of Linear Equations Many practical problems can be reduced

62 CHAPTER 2. MATRICES

Solution.(a) Suppose that A is an m×n matrix. Then A = ImA. Since Im is an elemen-tary matrix then A ∼ A.(b) Suppose that A and B are two matrices of sizes m× n. If A ∼ B then B =EkEk−1 · · ·E1A where E1, E2, · · · , Ek are m × m elementary matrices. Sincethese matrices are invertible elementary matrices then A = E−1

1 E−12 · · ·E−1

k B.That is, B ∼ A.(c) Suppose that A ∼ B and B ∼ C. Then B = Ek · · ·E1A and C = FlFl−1 · · ·F1B.But then we have C = (F )lFl−1 · · ·F1EkEk−1 · · ·E1A. That is, A ∼ C

Exercise 101Write down the inverses of the following elementary matrices:

E1 =

0 1 01 0 00 0 1

, E2 =

1 0 00 1 00 0 9

, E3 =

1 0 50 1 00 0 1

Solution.(a) E−1

1 = E1.(b)

E−12 =

1 0 00 1 00 0 1

9

(c)

E−13 =

1 0 −50 1 00 0 1

Exercise 102If E is an elementary matrix show that ET is also an elementary matrix of thesame type.

Solution.Suppose that E is the elementary matrix obtained by interchanging rows i andj of In with i < j. This is equivalent to interchanging columns i and j of In. Butthen ET is obtained by interchanging rows i and j of In and so is an elementarymatrix. If E is obtained by multiplying the ith row of In by a nonzero constantk then this is the same thing as multiplying the ith column of In by k. Thus,ET is obtained by multiplying the ith row of In by k and so is an elementarymatrix. Finally, if E is obtained by adding k times the ith row of In to the jthrow then ET is obtained by adding k times the jth row of In to the ith row.Note that if E is of Type I or Type III then ET = E

2.5 An Algorithm for Finding A−1

Before we establish the main results of this section, we recall the reader of thefollowing method of mathematical proofs. To say that statements p1, p2, · · · , pn

Page 63: Fundamentals of Linear Algebra - Mathematics & Statisticsmelbial2/classes/Linear Algebra... · 2006-01-13 · 1.1 Systems of Linear Equations Many practical problems can be reduced

2.5. AN ALGORITHM FOR FINDING A−1 63

are all equivalent means that either they are all true or all false. To prove thatthey are equivalent, one assumes p1 to be true and proves that p2 is true, thenassumes p2 to be true and proves that p3 is true, continuing in this fashion,assume that pn−1 is true and prove that pn is true and finally, assume that pn istrue and prove that p1 is true. This is known as the proof by circular argument.Now, back to our discussion of inverses. The following result establishes re-lationships between square matrices and systems of linear equations. Theserelationships are very important and will be used many times in later sections.

Theorem 19If A is an n× n matrix then the following statements are equivalent.

(a) A is invertible.(b) Ax = 0 has only the trivial solution.(c) A is row equivalent to In.(d) rank(A) = n.

Proof.(a) ⇒ (b) : Suppose that A is invertible and x0 is a solution to Ax = 0. ThenAx0 = 0. Multiply both sides of this equation by A−1 to obtain A−1Ax0 =A−10, that is x0 = 0. Hence, the trivial solution is the only solution.

(b) ⇒ (c) : Suppose that Ax = 0 has only the trivial solution. Then thereduced row-echelon form of the augmented matrix has no rows of zeros or freevariables. Hence it must look like

1 0 0 . . . 0 00 1 0 . . . 0 00 0 1 . . . 0 0...

......

......

0 0 0... 1 0

If we disregard the last column of the previous matrix we can conclude that Acan be reduced to In by a sequence of elementary row operations, i.e. A is rowequivalent to In.

(c) ⇒ (d) : Suppose that A is row equivalent to In. Then rank(A) = rank(In) =n.

(d) ⇒ (a) : Suppose that rank(A) = n. Then A is row equivalent to In. That isIn is obtained by a finite sequence of elementary row operations performed onA. Then by Theorem 17, each of these operations can be accomplished by pre-multiplying on the left by an appropriate elementary matrix. Hence, obtaining

EkEk−1 . . . E2E1A = In,

where k is the necessary number of elementary row operations needed to reduceA to In. Now, by Theorem 18 each Ei is invertible. Hence, EkEk−1 . . . E2E1 is

Page 64: Fundamentals of Linear Algebra - Mathematics & Statisticsmelbial2/classes/Linear Algebra... · 2006-01-13 · 1.1 Systems of Linear Equations Many practical problems can be reduced

64 CHAPTER 2. MATRICES

invertible and A−1 = EkEk−1 . . . E2E1.

Using the definition, to show that an n × n matrix A is invertible we find amatrix B of the same size such that AB = In and BA = In. The next theoremshows that one of these equality is enough to assure invertibilty.

Theorem 20If A and B are two square matrices of size n×n such that AB = I then BA = In

and B−1 = A.

ProofSuppose that Bx = 0. Multiply both sides by A to obtain ABx = 0. Thatis, x = 0. This shows that the homogenenous system Bx = 0 has only thetrivial solution so by Theorem 9 we see that B is invertible, say with inverseC. Hence, C = InC = (AB)C = A(BC) = AIn = A so that B−1 = A. Thus,BA = BB−1 = In.

As an application of Theorem 19, we describe an algorithm for finding A−1.We perform elementary row operations on A until we get In; say that the prod-uct of the elementary matrices is EkEk−1 . . . E2E1. Then we have

(EkEk−1 . . . E2E1)[A|In] = [(EkEk−1 . . . E2E1)A|(EkEk−1 . . . E2E1)In]= [In|A−1]

We ask the reader to carry the above algorithm in solving the following problems.

Exercise 103Find the inverse of

A =

1 2 32 5 31 0 8

Solution.We first construct the matrix

1 2 3 | 1 0 02 5 3 | 0 1 01 0 8 | 0 0 1

Applying the above algorithm to obtain

Step 1: r2 ← r2 − 2r1 and r3 ← r3 − r1

1 2 3 | 1 0 00 1 −3 | −2 1 00 −2 5 | −1 0 1

Page 65: Fundamentals of Linear Algebra - Mathematics & Statisticsmelbial2/classes/Linear Algebra... · 2006-01-13 · 1.1 Systems of Linear Equations Many practical problems can be reduced

2.5. AN ALGORITHM FOR FINDING A−1 65

Step 2: r3 ← r3 + 2r2

1 2 3 | 1 0 00 1 −3 | −2 1 00 0 −1 | −5 2 1

Step 3: r1 ← r1 − 2r2

1 0 9 | 5 −2 00 1 −3 | −2 1 00 0 −1 | −5 2 1

Step 4: r2 ← r2 − 3r3 and r1 ← r1 + 9r3

1 0 0 | −40 16 90 1 0 | 13 −5 −30 0 −1 | − 5 2 1

Step 5: r3 ← −r3

1 0 0 | −40 16 90 1 0 | 13 −5 −30 0 1 | 5 −2 −1

It follows that

A−1 =

−40 16 913 −5 −35 −2 −1

Exercise 104Show that the following homogeneous system has only the trivial solution.

x1 + 2x2 + 3x3 = 02x1 + 5x2 + 3x3 = 0x1 + 8x3 = 0.

Solution.The coefficient matrix of the given system is invertible by the previous exercise.Thus, by Theorem 19 the system has only the trivial solution

The following result exhibit a criterion for checking the singularity of a squarematrix.

Theorem 21If A is a square matrix with a row consisting entirely of zeros then A is singular.

Proof.The reduced row-echelon form will have a row of zeros. So the rank of the co-efficient matrix of the homogeneous system Ax = 0 is less than n. By Theorem5, Ax = 0 has a nontrivial solution and as a result of Theorem 19 the matrix A

Page 66: Fundamentals of Linear Algebra - Mathematics & Statisticsmelbial2/classes/Linear Algebra... · 2006-01-13 · 1.1 Systems of Linear Equations Many practical problems can be reduced

66 CHAPTER 2. MATRICES

must be singular.

How can we tell when a square matrix A is singular? i.e., when does the algo-rithm of finding A−1 fail? The answer is provided by the following theorem

Theorem 22An n× n matrix A is singular if and only if A is row equivalent to a matrix Bthat has a row of zeros.

Proof.Suppose first that A is singular. Then by Theorem 19, A is not row equivalentto In. Thus, A is row equivalent to a matrix B 6= In which is in reduced echelonform. By Theorem 19, B must have a row of zeros.Conversely, suppose that A is row equivalent to matrix B with a row con-sisting entirely of zeros. Then B is singular by Theorem 251. Now, B =EkEk−1 . . . E2E1A. If A is nonsingular then B is nonsingular, a contradiction.Thus, A must be singular

The following theorem establishes a result of the solvability of linear systemsusing the concept of invertibility of matrices.

Theorem 23An n × n square matrix A is invertible if and only if the linear system Ax = bis consistent for every n× 1 matrix b.

Proof.Suppose first that A is invertible. Then for any n×1 matrix b the linear systemAx = b has a unique solution, namely x = A−1b.Conversely, suppose that the system Ax = b is solvable for any n× 1 matrix b.In particular, Axi = ei, 1 ≤ i ≤ n, has a solution, where ei is the ith column ofIn. Construct the matrix

C =(

x1 x2 · · · xn

)

Then

AC =(

Ax1 Ax2 · · · Axn

)=

(e1 e2 · · · en

)= In.

Hence, by Theorem 20, A is non-singular

Exercise 105Solve the following system by using the previous theorem

x1 + 2x2 + 3x3 = 52x1 + 5x2 + 3x3 = 3x1 + 8x3 = 17

Page 67: Fundamentals of Linear Algebra - Mathematics & Statisticsmelbial2/classes/Linear Algebra... · 2006-01-13 · 1.1 Systems of Linear Equations Many practical problems can be reduced

2.6. REVIEW PROBLEMS 67

Solution.Using Exercise 103 and Theorem 23 we have

x1

x2

x3

=

−40 16 913 −5 −35 −2 −1

5317

=

1−12

Exercise 106If P is an n×n matrix suxh that PT P = In then the matrix H = In− 2PPT iscalled the Householder matrix. Show that H is symmetric and HT H = In.

Solution.Taking the transpose of H we have HT = IT

n − 2(PT )T PT = H. That is, H issymmetric. On the other hand, HT H = H2 = (In − 2PPT )2 = In − 4PPT +4(PPT )2 = In − 4PPT + 4P (PT P )PT = In − 4PPT + 4PPT = In

Exercise 107Let A and B be two square matrices. Show that AB is nonsingular if and onlyif both A and B are nonsingular.

Solution.Suppose that AB is nonsingular. Then (AB)−1AB = AB(AB)−1 = In. Supposethat A is singular. Then A has a row consisting of 0. Then AB has a rowconsisting of 0 (Exercise 90) but then AB is singular according to Theorem 251,a contradiction. Since (AB)−1AB = In then by Theorem 20, B is nonsingular.The converse is just Theorem 15 (a)

2.6 Review Problems

Exercise 108Compute the matrix

3(

2 1−1 0

)T

− 2(

1 −12 3

)

Exercise 109Find w, x, y, and z.

1 2 w2 x 4y −4 z

=

1 2 −12 −3 40 −4 5

Page 68: Fundamentals of Linear Algebra - Mathematics & Statisticsmelbial2/classes/Linear Algebra... · 2006-01-13 · 1.1 Systems of Linear Equations Many practical problems can be reduced

68 CHAPTER 2. MATRICES

Exercise 110Determine two numbers s and t such that the following matrix is symmetric.

A =

2 s t2s 0 s + t3 3 t

Exercise 111Let A be a 2× 2 matrix. Show that

A = a

(1 00 0

)+ b

(0 10 0

)+ c

(0 01 0

)+ d

(0 00 1

)

Exercise 112Let A =

[1 1 −1

], B =

[0 1 2

], C =

[3 0 1

]. If rA+sB+ tC =

0 show that s = r = t = 0.

Exercise 113Show that the product of two diagonal matrices is again a diagonal matrix.

Exercise 114Let A be an arbitrary matrix. Under what conditions is the product AAT de-fined?

Exercise 115(a) Show that AB = BA if and only if (A−B)(A + B) = A2 −B2.(b) Show that AB = BA if and only if (A + B)2 = A2 + 2AB + B2.

Exercise 116Let A be a matrix of size m × n. Denote the columns of A by C1, C2, · · · , Cn.Let x be the n × 1 matrix with entries x1, x2, · · · , xn. Show that Ax = x1C1 +x2C2 + · · ·+ xnCn.

Exercise 117Let A be an m× n matrix. Show that if yA = 0 for all y ∈ IRm then A = 0.

Exercise 118An n× n matrix A is said to be idempotent if A2 = A.(a) Show that the matrix

A =12

(1 11 1

)

is idempotent(b) Show that if A is idempotent then the matrix (In −A) is also idempotent.

Page 69: Fundamentals of Linear Algebra - Mathematics & Statisticsmelbial2/classes/Linear Algebra... · 2006-01-13 · 1.1 Systems of Linear Equations Many practical problems can be reduced

2.6. REVIEW PROBLEMS 69

Exercise 119The purpose of this exercise is to show that the rule (ab)n = anbn does not holdwith matrix multiplication. Consider the matrices

A =(

2 −41 3

), B =

(3 2−1 5

)

Show that (AB)2 6= A2B2.

Exercise 120Show that AB = BA if and only if AT BT = BT AT .

Exercise 121Suppose AB = BA and n is a non-negative integer.(a) Use induction to show that ABn = BnA.(b) Use induction and (a) to show that (AB)n = AnBn.

Exercise 122Let A and B be symmetric matrices. Show that AB is symmetric if and only ifAB = BA.

Exercise 123Show that tr(AAT ) is the sum of the squares of all the entries of A.

Exercise 124(a) Find two 2× 2 singular matrices whose sum in nonsingular.(b) Find two 2× 2 nonsingular matrices whose sum is singular.

Exercise 125Show that the matrix

A =

1 4 02 5 03 6 0

is singular.

Exercise 126Let

A =(

1 21 3

)

Find A−3.

Exercise 127Let

A−1 =(

2 −13 5

)

Find A.

Page 70: Fundamentals of Linear Algebra - Mathematics & Statisticsmelbial2/classes/Linear Algebra... · 2006-01-13 · 1.1 Systems of Linear Equations Many practical problems can be reduced

70 CHAPTER 2. MATRICES

Exercise 128Let A and B be square matrices such that AB = 0. Show that if A is invertiblethen B is the zero matrix.

Exercise 129Find the inverse of the matrix

A =(

sin θ cos θ− cos θ sin θ

)

Exercise 130Which of the following are elementary matrices?(a)

1 1 00 0 10 1 0

(b) (1 0−5 1

)

(c)

1 0 00 1 90 0 1

(d)

2 0 0 20 1 0 00 0 1 00 0 0 1

Exercise 131Let A be a 4×3 matrix. Find the elementary matrix E, which as a premultiplierof A, that is, as EA, performs the following elementary row operations on A :(a) Multiplies the second row of A by -2.(b) Adds 3 times the third row of A to the fourth row of A.(c) Interchanges the first and third rows of A.

Exercise 132For each of the following elementary matrices, describe the corresponding ele-mentary row operation and write the inverse.(a)

E =

0 0 10 1 01 0 0

(b)

E =

1 0 0−2 1 00 0 1

Page 71: Fundamentals of Linear Algebra - Mathematics & Statisticsmelbial2/classes/Linear Algebra... · 2006-01-13 · 1.1 Systems of Linear Equations Many practical problems can be reduced

2.6. REVIEW PROBLEMS 71

(c)

E =

1 0 00 1 00 0 5

Exercise 133Consider the matrices

A =

3 4 12 −7 −18 1 5

, B =

8 1 52 −7 −13 4 1

, C =

3 4 12 −7 −12 −7 3

Find elementary matrices E1, E2, E3, and E4 such that(a)E1A = B, (b)E2B = A, (c)E3A = C, (d)E4C = A.

Exercise 134Determine if the following matrix is invertible.

1 6 42 4 −1−1 2 5

Exercise 135For what values of a does the following homogeneous system have a nontrivialsolution? {

(a− 1)x1 + 2x2 = 02x1 + (a− 1)x2 = 0

Exercise 136Find the inverse of the matrix

1 1 10 2 35 5 1

Exercise 137What conditions must b1, b2, b3 satisfy in order for the following system to beconsistent?

x1 + x2 + 2x3 = b1

x1 + x3 = b2

2x1 + x2 + 3x3 = b3

Exercise 138Prove that if A is symmetric and nonsingular than A−1 is symmetric.

Exercise 139If

D =

4 0 00 −2 00 0 3

find D−1.

Page 72: Fundamentals of Linear Algebra - Mathematics & Statisticsmelbial2/classes/Linear Algebra... · 2006-01-13 · 1.1 Systems of Linear Equations Many practical problems can be reduced

72 CHAPTER 2. MATRICES

Exercise 140Prove that a square matrix A is nonsingular if and only if A is a product ofelementary matrices.

Exercise 141Prove that two m× n matrices A and B are row equivalent if and only if thereexists a nonsingular matrix P such that B = PA.

Exercise 142Let A and B be two n × n matrices. Suppose A is row equivalent to B. Provethat A is nonsingular if and only if B is nonsingular.

Exercise 143Show that the product of two lower (resp. upper) triangular matrices is againlower (resp. upper) triangular.

Exercise 144Show that a 2× 2 lower triangular matrix is invertible if and only if a11a22 6= 0and in this case the inverse is also lower triangular.

Exercise 145Let A be an n × n matrix and suppose that the system Ax = 0 has only thetrivial solution. Show that Akx = 0 has only the trivial solution for any positiveinteger k.

Exercise 146Show that if A and B are two n× n matrices then A ∼ B.

Exercise 147Show that any n×n invertible matrix A satisfies the property (P): If AB = ACthen B = C.

Exercise 148Show that an n × n matrix A is invertible if and only if the equation yA = 0has only the trivial solution.

Exercise 149Let A be an n× n matrix such that An = 0. Show that In −A is invertible andfind its inverse.

Exercise 150Let A = (aij(t)) be an m × n matrix whose entries are differentiable functionsof the variable t. We define dA

dt = (daij

dt ). Show that if the entries in A and Bare differentiable functions of t and the sizes of the matrices are such that thestated operations can be performed, then(a) d

dt (kA) = k dAdt .

(b) ddt (A + B) = dA

dt + dBdt .

(c) ddt (AB) = dA

dt B + AdBdt .

Page 73: Fundamentals of Linear Algebra - Mathematics & Statisticsmelbial2/classes/Linear Algebra... · 2006-01-13 · 1.1 Systems of Linear Equations Many practical problems can be reduced

2.6. REVIEW PROBLEMS 73

Exercise 151Let A be an n×n invertible matrix with entries being differentiable functions oft. Find a formula for dA−1

dt .

Exercise 152 (Sherman-Morrison formula)Let A be an n × n invertible matrix. Let u, v be n × 1 matrices such thatvT A−1u + 1 6= 0. Show that(a) (A + uvT )(A−1 − A−1uvT A−1

1+vT A−1u) = In.

(b) Deduce from (a) that A is invertible.

Exercise 153Let x = (x1, x2, · · ·xm)T be an m × 1 matrix and y = (y1, y2, · · · , yn)T be ann× n matrix. Construct the m× n matrix xyT .

Exercise 154Show that a triangular matrix A with the property AT = A−1 is always diagonal.

Page 74: Fundamentals of Linear Algebra - Mathematics & Statisticsmelbial2/classes/Linear Algebra... · 2006-01-13 · 1.1 Systems of Linear Equations Many practical problems can be reduced

74 CHAPTER 2. MATRICES

Page 75: Fundamentals of Linear Algebra - Mathematics & Statisticsmelbial2/classes/Linear Algebra... · 2006-01-13 · 1.1 Systems of Linear Equations Many practical problems can be reduced

Chapter 3

Determinants

With each square matrix we can associate a real number called the determi-nant of the matrix. Determinants have important applications to the theory ofsystems of linear equations. More specifically, determinants give us a method(called Cramer’s method) for solving linear systems. Also, determinant tells uswhether or not a matrix is invertible.

3.1 Definition of the Determinant

Determinants can be used to find the inverse of a matrix and to determine ifa linear system has a unique solution. Throughout this chapter we use onlysquare matrices.

Let us first recall from probability theory the following important formula knownas the multiplication rule of counting: If a choice consists of k steps, of whichthe first can be made in n1 ways, for each of these the second step can be madein n2 ways, · · · , and for each of these the kth can be made in nk ways, then thechoice can be made in n1n2 · · ·nk ways.

Exercise 155If a new-car buyer has the choice of four body styles, three engines, and tencolors, in how many different ways can s/he order one of these cars?

Solution.By the multiplication rule of counting, there are (4)(3)(10) = 120 choices

A permutation of the set S = {1, 2, . . . , n} is an arrangement of the ele-ments of S in some order without omissions or repetitions. We write σ =(σ(1)σ(2) · · ·σ(n)).In terms of functions, a permutation is a function on S with range equals toS. Thus, any σ on S possesses an inverse σ−1, that is σ ◦ σ−1 = σ−1 ◦ σ = id

75

Page 76: Fundamentals of Linear Algebra - Mathematics & Statisticsmelbial2/classes/Linear Algebra... · 2006-01-13 · 1.1 Systems of Linear Equations Many practical problems can be reduced

76 CHAPTER 3. DETERMINANTS

where id = (123 · · ·n). So if σ = (613452) then σ−1 = (263451).(Show thatσ ◦ σ−1 = σ−1 ◦ σ = id).

Exercise 156Let σ = (24513) and τ = (41352) be permutations in S5. Find(a) σ ◦ τ and τ ◦ σ,(b) σ−1.

Solution.(a) Using the definition of composition of two functions we find, σ(τ(1)) =σ(4) = 1. A similar argument for σ(τ(2)), etc. Thus, σ ◦ τ = (12534) andτ ◦ σ = (15243).(b) Since σ(4) = 1 then σ−1(1) = 4. Similarly, σ−1(2) = 1, etc. Hence,σ−1 = (41523)

Let Sn denote the set of all permutations on S. How many permutations arethere in Sn? We have n positions to be filled by n numbers. For the first posi-tion, there are n possibilities. For the second there are n − 1 possibilities, etc.Thus, according to the multiplication rule of counting there are

n(n− 1)(n− 2) . . . 2.1 = n!

permutations.

Is there a way to list all the permutations of Sn? The answer is yes and one canfind the permutations by using a permutation tree which we describe in thefollowing exercise

Exercise 157List all the permutations of S = {1, 2, 3, 4}.Solution.

1↙ ↓ ↘2 3 4

↙↘ ↙↘ ↙↘3 4 2 4 2 3

2↙ ↓ ↘1 3 4

↙↘ ↙↘ ↙↘3 4 1 4 1 3

3↙ ↓ ↘1 2 4

↙↘ ↙↘ ↙↘2 4 1 4 1 2

4↙ ↓ ↘1 2 3

↙↘ ↙↘ ↙↘2 3 1 3 1 2

An inversion is said to occur whenever a larger integer precedes a smallerone. If the number of inversions is even (resp. odd) then the permutation issaid to be even (resp. odd). We define the sign of a permutation to be afunction sgn with domain Sn and range {−1, 1} such that sgn(σ) = −1 if σ isodd and sgn(σ) = +1 if σ is even.

Exercise 158Classify each of the permutations in S6 as even or odd.

Page 77: Fundamentals of Linear Algebra - Mathematics & Statisticsmelbial2/classes/Linear Algebra... · 2006-01-13 · 1.1 Systems of Linear Equations Many practical problems can be reduced

3.1. DEFINITION OF THE DETERMINANT 77

(i) (613452).(ii) (2413).(iii) (1234).(iv) id = (123456).

Solution.(i) sgn((613452)) = +1 since there are 8 inversions, namely, (61), (63), (64), (65),(62), (32), (42), and (52).(ii) sgn((2413)) = −1. The inversions are: (21), (41), (43).(iii) sgn((1234)) = +1. There are 0 inversions.(iv) sgn(id) = +1

The following two theorems are of theoretical interest.

Theorem 24Let τ and σ be two elements of Sn. Then sgn(τ ◦ σ) = sgn(τ)sgn(σ). Hence,the composition of two odd or even permutations is always even.

Proof.Define the polynomial in n indeterminates g(x1, x2, · · · , xn) = Πi<j(xi − xj).For any permutation σ ∈ Sn we define σ(g) = Πi<j(xσ(i) − xσ(j)). We willshow that for any σ ∈ Sn one has σ(g) = sgn(σ)g. Clearly, σ(g) = (Πi<j(xi −xj))(Πi>j(xi − xj)). But for i > j one can write xi − xj = −(xj − xi), with(xj−xi) being a factor of g. Let k be the number of factors of the form (xi−xj)with i > j. Then σ(g) = (−1)kg. It follows that if k is even then σ(g) = g andif k is odd σ(g) = −g.Now, let 1 ≤ i < j ≤ n be such that σ(i) > σ(j). Since Range(σ) = {1, 2, · · · , n}then there exit i∗, j∗ ∈ {1, 2, · · · , n} such that σ(i∗) = i and σ(j∗) = j. Sincei < j then σ(j∗) > σ(i∗). Thus for every pair (i, j) with i < j and σ(i) > σ(j)there is a factor of σ(g) of the form (xσ(j∗) − xσ(i∗)) with σ(j∗) > σ(i∗). Ifσ is even (resp. odd) then the number of (i, j) with the property i < j andσ(i) > σ(j) is even (resp. odd). This implies that k is even ( resp. odd). Thus,σ(g) = g if σ is even and σ(g) = −g if σ is odd.Finally, for any σ, τ ∈ Sn we have

sgn(σ ◦ τ)g = (σ ◦ τ)g = σ(τ(g)) = σ(sgn(τ)g) = (sgn(σ))(sgn(τ))g.

Since g was arbitrary then sgn(σ ◦ τ) = sgn(σ)sgn(τ). This concludes a proofof the theorem

Theorem 25For any σ ∈ Sn, we have sgn(σ) = sgn(σ−1).

proof.We have σ ◦ σ−1 = id. By Theorem 24 we have sgn(σ)sgn(σ−1) = 1 since id iseven. Hence, σ and σ−1 are both even or both odd

Let A be an n × n matrix. An elementary product from A is a productof n entries from A, no two of which come from the same row or same column.

Page 78: Fundamentals of Linear Algebra - Mathematics & Statisticsmelbial2/classes/Linear Algebra... · 2006-01-13 · 1.1 Systems of Linear Equations Many practical problems can be reduced

78 CHAPTER 3. DETERMINANTS

Exercise 159List all elementary products from the matrices(a)

(a11 a12

a21 a22

),

(b)

a11 a12 a13

a21 a22 a23

a31 a32 a33

Solution.(a) The only elementary products are a11a22, a12a21.(b) An elementary product has the form a1∗a2∗a3∗. Since no two factors comefrom the same column, the column numbers have no repetitions; consequentlythey must form a permutation of the set {1, 2, 3}. The 3! = 6 permutations yieldthe following elementary products:a11a22a33, a11a23a32, a12a23a31, a12a21a33, a13a21a32, a13a22a31

Let A be an n × n matrix. Consider an elementary product of entries of A.For the first factor, there are n possibilities for an entry from the first row.Once selected, there are n− 1 possibilities for an entry from the second row forthe second factor. Continuing, we find that there are n! elementary products.They are the products of the form a1σ(1)a2σ(2) . . . anσ(n), where σ is a permuta-tion of {1, 2, . . . , n}, i.e. a member of Sn.

Let A be an n × n matrix. Then we define the determinant of A to be thenumber

|A| =∑

sgn(σ)a1σ(1)a2σ(2) . . . anσ(n)

where the sum is over all permutations σ of {1, 2, . . . , n}.

Exercise 160Find |A| if(a)

A =(

a11 a12

a21 a22

),

(b)

A =

a11 a12 a13

a21 a22 a23

a31 a32 a33

Page 79: Fundamentals of Linear Algebra - Mathematics & Statisticsmelbial2/classes/Linear Algebra... · 2006-01-13 · 1.1 Systems of Linear Equations Many practical problems can be reduced

3.2. EVALUATING DETERMINANTS BY ROW REDUCTION 79

Solution.By using the definition of a determinant and Exercise 159 we obtain(a) |A| = a11a22 − a21a12.(b) |A| = a11a22a33−a11a23a32 +a12a23a31−a12a21a33 +a13a21a32−a13a22a31

Exercise 161Find all values of λ such that |A| = 0.(a)

A =(

λ− 1 −21 λ− 4

),

(b)

A =

λ− 6 0 00 λ −10 4 λ− 4

Solution.(a) Using Exercise 160 (a) we find |A| = (λ− 3)(λ− 2) = 0. Therefore, λ = 3 orλ = 2.(b) By Exercise 160 (b) we have |A| = (λ− 2)2(λ− 6) = 0 and this yields λ = 2or λ = 6

Exercise 162Prove that if a square matrix A has a row of zeros then |A| = 0.

Solution.Suppose that the ith row of A consists of 0. By the definition of the determinanteach elementary product will have one factor belonging to the ith row. Thus,each elementary product is zero and consequently |A| = 0

3.2 Evaluating Determinants by Row Reduction

In this section we provide a simple procedure for finding the determinant of amatrix. The idea is to reduce the matrix into row-echelon form which in thiscase is a triangular matrix. Recall that a matrix is said to be triangular if it isupper triangular, lower triangular or diagonal. The following theorem providesa formula for finding the determinant of a triangular matrix.

Theorem 26If A is an n× n triangular matrix then |A| = a11a22 . . . ann.

Proof.Assume that A is lower triangular. We will show that the only elementary

Page 80: Fundamentals of Linear Algebra - Mathematics & Statisticsmelbial2/classes/Linear Algebra... · 2006-01-13 · 1.1 Systems of Linear Equations Many practical problems can be reduced

80 CHAPTER 3. DETERMINANTS

product that appears in the formula of |A| is a11a22 · · · ann. To see this, considera typical elementary product

a1σ(1)a2σ(2) · · · anσ(n). (3.1)

Since a12 = a13 = · · · = a1n = 0 then for the above product to appear wemust have σ(1) = 1. But then σ(2) 6= 1 since no two factors come from thesame column. Hence, σ(2) ≥ 2. Further, since a23 = a24 = · · · = a2n = 0 wemust have σ(2) = 2 in order to have (3.1). Continuing in this fashion we obtainσ(3) = 3, · · · , σ(n) = n. That is, |A| = a11a22 · · · ann. The proof for an uppertriangular matrix is similar.

Exercise 163Compute |A|.(a)

A =

1 2 30 4 50 0 6

,

(b)

A =

1 0 02 3 04 5 6

,

Solution.(a) Since A is triangular then |A| = (1)(4)(6) = 24.(b) |A| = (1)(3)(6) = 18

Exercise 164Compute the determinant of the identity matrix In.

Solution.Since the identity matrix is triangular with entries equal to 1 on the main diag-onal then |In| = 1

The following theorem is of practical use. It provides a technique for evalu-ating determinants by greatly reducing the labor involved. We shall show thatthe determinant can be evaluated by reducing the matrix to row-echelon form.Before proving the next theorem, we make the following remark. Let τ be apermutation that interchanges only two numbers i and j with i < j. That is,τ(k) = k if k 6= i, j and τ(i) = j, τ(j) = i. We call such a permutation a trans-position. In this case, τ = (12 · · · j(i + 1)(i + 2) · · · (j− 1)i(j + 1) · · ·n). Hence,there are 2(j − i − 1) + 1 inversions, namely, (j, i + 1), · · · , (j, j − 1), (j, i), (i +1, i), (i+2, i), · · · , (j−1, i). This means that τ is odd. Now, for any permutationσ ∈ Sn we have sgn(τ ◦ σ) = sgn(τ)sgn(σ) = −sgn(σ).(See Theorem 24)

Page 81: Fundamentals of Linear Algebra - Mathematics & Statisticsmelbial2/classes/Linear Algebra... · 2006-01-13 · 1.1 Systems of Linear Equations Many practical problems can be reduced

3.2. EVALUATING DETERMINANTS BY ROW REDUCTION 81

Theorem 27Let A be an n× n matrix.(a) Let B be the matrix obtained from A by multiplying a row by a scalar c.Then |B| = c|A|.(b) Let B be the matrix obtained from A by interchanging two rows of A. Then|B| = −|A|.(c) If a square matrix has two identical rows then its determinant is zero.(d) Let B be the matrix obtained from A by adding c times a row to anotherrow. Then |B| = |A|.

Proof.(a) Multiply row r of A = (aij) by a scalar c. Let B = (bij) be the resultingmatrix. Then bij = aij for i 6= r and brj = carj . Using the definition of adeterminant we have

|B| =∑

σ∈Snsgn(σ)b1σ(1)b2σ(2) · · · brσ(r) · · · bnσ(n)

=∑

σ∈Snsgn(σ)a1σ(1)a2σ(2) · · · c(arσ(r)) · · · anσ(n)

= c∑

σ∈Snsgn(σ)a1σ(1)a2σ(2) · · · arσ(r) · · · anσ(n) = c|A|.

(b) Interchange rows r and s of A with r < s. Let B = (bij) be the resultingmatrix. Let τ be the transposition that interchanges the numbers r and s. Thenbij = aiτ(j). Using the definition of determinant we have

|B| =∑

σ∈Snsgn(σ)b1σ(1)b2σ(2) · · · bnσ(n)

=∑

σ∈Snsgn(σ)a1τ◦σ(1)a2τ◦σ(2) · · · anτ◦σ(n)

= −∑σ∈Sn

sgn(τ ◦ σ)a1τ◦σ(1)a2τ◦σ(2) · · · anτ◦σ(n)

= −∑δ∈Sn

sgn(δ)a1δ(1)a2δ(2) · · · anδ(n) = −|A|.

Note that as σ runs through all the elements of Sn, τ ◦ σ runs through all theelements of Sn as well. This ends a proof of (b).(c) Suppose that rows r and s of A are equal. Let B be the matrix obtainedfrom A by interchanging rows r and s. Then B = A so that |B| = |A|. But by(b), |B| = −|A|. Hence, |A| = −|A|, 2|A| = 0 and hence |A| = 0.(d) Let B = (bij) be obtained from A by adding to each element of the sth rowof A c times the corresponding element of the rth row of A. That is, bij = aij ifi 6= s and bsj = carj + asj . Then

|B| =∑

σ∈Snsgn(σ)b1σ(1)b2σ(2) · · · bsσ(s) · · · bnσ(n)

=∑

σ∈Snsgn(σ)a1σ(1)a2σ(2) · · · (carσ(s) + asσ(s)) · · · anσ(n)

=∑

σ∈Snsgn(σ)a1σ(1)a2σ(2) · · · arσ(r) · · · asσ(s) · · · anσ(n)

+ c(∑

σ∈Snsgn(σ)a1σ(1)a2σ(2) · · · arσ(r) · · · arσ(s) · · · anσ(n)).

The first term in the last expression is |A|. The second is c times the deter-minant of a matrix with two identical rows, namely, rows r and s with entries(ar1, ar2, · · · , arn). By (c), this quantity is zero. Hence, |B| = |A|.

Exercise 165

Page 82: Fundamentals of Linear Algebra - Mathematics & Statisticsmelbial2/classes/Linear Algebra... · 2006-01-13 · 1.1 Systems of Linear Equations Many practical problems can be reduced

82 CHAPTER 3. DETERMINANTS

Use Theorem 27 to evaluate the determinant of the following matrix

A =

0 1 53 −6 92 6 1

Solution.We use Gaussian elimination as follows.

Step 1: r1 ↔ r2 ∣∣∣∣∣∣

3 −6 90 1 52 6 1

∣∣∣∣∣∣= −|A|

Step 2: r1 ← r1 − r3 ∣∣∣∣∣∣

1 −12 80 1 52 6 1

∣∣∣∣∣∣= −|A|

Step 3: r3 ← r3 − 2r1 ∣∣∣∣∣∣

1 −12 80 1 50 30 −15

∣∣∣∣∣∣= −|A|

Step 4: r3 ← r3 − 30r2

∣∣∣∣∣∣

1 −12 80 1 50 0 −165

∣∣∣∣∣∣= −|A|

Thus,

|A| = −∣∣∣∣∣∣

1 −12 80 1 50 0 −165

∣∣∣∣∣∣= 165

Exercise 166Show that if a square matrix has two proportional rows then its determinant iszero.

Solution.Suppose that A is a square matrix such that row j is k times row i with k 6= 0.By adding − 1

k rj to ri then the ith row will consist of 0. By Thereom 27 (c),|A| = 0

Exercise 167Show that if A is an n× n matrix and c is a scalar then |cA| = cn|A|.

Solution.The matrix cA is obtained from the matrix A by multiplying the rows of A byc 6= 0. By mutliplying the first row of cA by 1

c we obtain |B| = 1c |cA| where B

Page 83: Fundamentals of Linear Algebra - Mathematics & Statisticsmelbial2/classes/Linear Algebra... · 2006-01-13 · 1.1 Systems of Linear Equations Many practical problems can be reduced

3.3. PROPERTIES OF THE DETERMINANT 83

is obtained from the matrix A by multiplying all the rows of A, except the firstone, by c. Now, divide the second row of B by 1

c to obtain |B′| = 1c |B|, where

B′ is the matrix obtained from A by multiplying all the rows of A, except thefirst and the second, by c. Thus, |B′| = 1

c2 |cA|. Repeating this process, we find|A| = 1

cn |cA| or |cA| = cn|A|Exercise 168(a) Let E1 be the elementary matrix corresponding to type I elementary rowoperation. Find |E1|.(b) Let E2 be the elementary matrix corresponding to type II elementary rowoperation. Find |E2|.(c) Let E3 be the elementary matrix corresponding to type III elementary rowoperation. Find |E3|.

Solution.(a) The matrix E1 is obtained from the identity matrix by multiplying a row ofIn by a nonzero scalar c. In this case,|E1| = c|In| = c.(b) E2 is obtained from In by adding a multiple of a row to another row. Thus,|E2| = |In| = 1.(c) The matrix E3 is obtained from the matrix In by interchanging two rows.In this case, |E3| = −|In| = −1

Exercise 169Find, by inspection, the determinant of the following matrix.

A =

3 −1 4 −26 −2 5 25 8 1 4−9 3 −12 6

Solution.Since the first and the fourth rows are proportional then the determinant is zeroby Exercise 166

3.3 Properties of the Determinant

In this section we shall derive some of the fundamental properties of the de-terminant. One of the immediate consequences of these properties will be animportant determinant test for the invertibility of a square matrix.

Theorem 28Let A = (aij) be an n× n square matrix. Then |AT | = |A|.Proof.Since A = (aij) then AT = (aji). Using the definition of determinant we have

|AT | =∑

σ∈Snsgn(σ)aσ(1),1aσ(2),2 · · · aσ(n),n

Page 84: Fundamentals of Linear Algebra - Mathematics & Statisticsmelbial2/classes/Linear Algebra... · 2006-01-13 · 1.1 Systems of Linear Equations Many practical problems can be reduced

84 CHAPTER 3. DETERMINANTS

Now, since σ is a permutation then aσ(1),1aσ(2),2 · · · aσ(n),n = a1k1a2k2 · · · ankn,

where σ(ki) = i, 1 ≤ i ≤ n. We claim that τ = (k1k2 · · · kn) = σ−1. Indeed, letτ(i) = ki for 1 ≤ i ≤ n. Then (σ ◦ τ)(i) = σ(τ(i)) = σ(ki) = i, i.e. σ ◦ τ = id.Hence, τ = σ−1.Next, by Theorem 25, sgn(τ) = sgn(σ) and therefore

|AT | =∑

σ∈Snsgn(τ)a1,τ(1)a2,τ(2) · · · an,τ(n)

=∑

τ∈Snsgn(τ)a1,τ(1)a2,τ(2) · · · an,τ(n) = |A|.

This ends a proof of the theorem

It is worth noting here that Theorem 28 says that every property about de-terminants that contains the word ”row” in its statement is also true when theword ”column” is substituted for ”row”.

Exercise 170Prove that if two columns of a square matrix are proportional then the matrixhas determinant equals to zero.

Solution.If A is a square matrix with two proportional columns then AT has two pro-portional rows. By Exercise 166 and Theorem 28 we have |A| = |AT | = 0

Exercise 171Show that if A is matrix with a column consisting of 0 then |A| = 0.

Solution.If A has a column consisting entirely of 0 then AT has a row consisitng entirelyof 0. By Theorem 28 and Exercise 162 we have |A| = |AT | = 0

Next, we prove that the determinant of the product of two matrices is theproduct of determinants and that A is invertible if and only if A has nonzerodeterminant. The next result relates the determinant of a product of a matrixwith an elementary matrix.

Theorem 29If E is an elementary matrix then |EA| = |E||A|.

Proof.If E is an elementary matrix of type I then |E| = c and by Theorem 27|EA| = c|A| = |E||A|. If E is an elementary matrix of type II, then |E| = 1and |EA| = |A| = |E||A|. Finally, if E is an elementary matrix of type III then|E| = −1 and |EA| = −|A| = |E||A|

By induction one can extend the above result.

Page 85: Fundamentals of Linear Algebra - Mathematics & Statisticsmelbial2/classes/Linear Algebra... · 2006-01-13 · 1.1 Systems of Linear Equations Many practical problems can be reduced

3.3. PROPERTIES OF THE DETERMINANT 85

Theorem 30If B = E1E2 · · ·EnA, where E1, E2, · · · , En are elementary matrices, then |B| =|E1||E2| · · · |En||A|.Proof.This follows from Theorem 29. Indeed, |B| = |E1(E2 · · ·EnA)| = |E1||E2(E3 · · ·EnA)| =· · · = |E1||E2| · · · |En||A|

Next we establish a condition for a matrix to be invertible.

Theorem 31If A is an n× n matrix then A is nonsingular if and only if |A| 6= 0.

ProofIf A is nonsingular then A is row equivalent to In (Theorem 19). That is,A = EkEk−1 · · ·E1In. By Theorem 30 we have |A| = |Ek| · · · |E1||In| 6= 0 sinceif E is of type I then |E| = c 6= 0; if E is of type II then |E| = 1, and if E is oftype III then |E| = −1.Now, suppose that |A| 6= 0. If A is singular then by Theorem 22, A is rowequivalent to a matrix B that has a row of zeros. Also, B = EkEk−1 · · ·E1A.By Theorem 30, 0 = |B| = |Ek||Ek−1| · · · |A| 6= 0, a contradiction. Hence, Amust be nonsingular.

The following result follows from Theorems 19 and 31.

Theorem 32The following statements are all equivalent:(i) A is nonsingular.(ii) |A| 6= 0.(iii) rank(A) = n.(iv) A is row equivalent to In.(v) The homogeneous systen Ax = 0 has only the trivial solution.

Exercise 172Prove that |A| = 0 if and only if Ax = 0 has a nontrivial solution.

Solution.If |A| = 0 then according to Theorem 32 the homogeneous system Ax = 0 musthave a nontrivial solution. Conversely, if the homogeneous system Ax = 0 hasa nontrivial solution then A must be singular by Theorem 32. By Theorem 32(a), |A| = 0

Theorem 33If A and B are n× n matrices then |AB| = |A||B|.Proof.If A is singular then AB is singular and therefore |AB| = 0 = |A||B| since |A| =

Page 86: Fundamentals of Linear Algebra - Mathematics & Statisticsmelbial2/classes/Linear Algebra... · 2006-01-13 · 1.1 Systems of Linear Equations Many practical problems can be reduced

86 CHAPTER 3. DETERMINANTS

0. So assume that A is nonsingular. Then by Theorem 32, A = EkEk−1 · · ·E1In.By Theorem 30 we have |A| = |Ek||Ek−1| · · · |E1|. Thus, |AB| = |EkEk−1 · · ·E1B| =|Ek||Ek−1| · · · |E1||B| = |A||B|.

Exercise 173Is it true that |A + B| = |A|+ |B|?

Solution.No. Consider the following matrices.

A =(

1 00 −1

), B =

( −1 00 1

)

Then |A + B| = |0| = 0 and |A|+ |B| = −2

Exercise 174Show that if A is invertible then |A−1| = 1

|A| .

Solution.If A is invertible then A−1A = In. Taking the determinant of both sides we find|A−1||A| = 1. That is, |A−1| = 1

|A| . Note that since A is invertible then |A| 6= 0

Exercise 175Let A and B be two similar matrices, i.e. there exists a nonsingular matrix Psuch that A = P−1BP. Show that |A| = |B|.

Solution.Using Theorem 33 and Exercise 174 we have, |A| = |P−1BP | = |P−1||B||P | =1|P | |B||P | = |B|. Note that since P is nonsingular then |P | 6= 0

3.4 Finding A−1 Using Cofactor Expansions

In Section 3.2 we discussed the row reduction method for computing the de-terminant of a matrix. This method is well suited for computer evaluation ofdeterminants because it is systematic and easily programmed. In this sectionwe introduce a method for evaluating determinants that is useful for hand com-putations and is important theoretically. Namely, we will obtain a formula forthe inverse of an invertible matrix as well as a formula for the solution of squaresystems of linear equations.Let A = (aij) be an n×n matrix. Let Mij be the (n−1)× (n−1) submatrix ofA obtained by deleting the ith row and the jth column of A. The determinantof Mij is called the minor of the entry aij . The number Cij = (−1)i+j |Mij |is called the cofactor of the entry aij .

Page 87: Fundamentals of Linear Algebra - Mathematics & Statisticsmelbial2/classes/Linear Algebra... · 2006-01-13 · 1.1 Systems of Linear Equations Many practical problems can be reduced

3.4. FINDING A−1 USING COFACTOR EXPANSIONS 87

Exercise 176Let

A =

3 1 −42 5 61 4 8

Find the minor and the cofactor of the entry a32 = 4.

Solution.The minor of the entry a32 is

|M32| =∣∣∣∣

3 −42 6

∣∣∣∣ = 26

and the cofactor is C32 = (−1)3+2|M32| = −26

Note that the cofactor and the minor of an entry differ only in sign. A quick wayfor determining whether to use the + or − is to use the following checkboardarray

+ − + − · · ·− + − + · · ·...

......

......

In order to prove the main result of this section, we first prove the following

Theorem 34Let A = (aij), A′ = (a′ij), and A′′ = (a′′ij) be n × n matrices such that aij =a′ij = a′′ij if i 6= r and a′′rj = arj + a′rj . Then

|A′′| = |A|+ |A′|.

The same result holds for columns.

Proof.Using the definition of determinant we have

|A′′| =∑

σ∈Snsgn(σ)a′′1σ(1)a

′′2σ(2) · · · a′′rσ(r) · · · a′′nσ(n)

=∑

σ∈Snsgn(σ)a′′1σ(1)a

′′2σ(2) · · · (arσ(r) + a′rσ(r) · · · a′′nσ(n)

=∑

σ∈Snsgn(σ)a′′1σ(1)a

′′2σ(2) · · · arσ(r) · · · a′′nσ(n)

+∑

σ∈Snsgn(σ)a′′1σ(1)a

′′2σ(2) · · · a′rσ(r) · · · a′′nσ(n)

=∑

σ∈Snsgn(σ)a1σ(1)a2σ(2) · · · arσ(r) · · · anσ(n)

+∑

σ∈Snsgn(σ)a′1σ(1)a

′2σ(2) · · · a′rσ(r) · · · a′nσ(n)

= |A|+ |A′|.

Since |AT | = |A| the same result holds for columns.

Page 88: Fundamentals of Linear Algebra - Mathematics & Statisticsmelbial2/classes/Linear Algebra... · 2006-01-13 · 1.1 Systems of Linear Equations Many practical problems can be reduced

88 CHAPTER 3. DETERMINANTS

Exercise 177Prove the following identity without evaluating the determinants .

∣∣∣∣∣∣

a1 b1 a1 + b1 + c1

a2 b2 a2 + b2 + c2

a3 b3 a3 + b3 + c3

∣∣∣∣∣∣=

∣∣∣∣∣∣

a1 b1 c1

a2 b2 c2

a3 b3 c3

∣∣∣∣∣∣Solution.By the previous theorem and Exercise 170 we have∣∣∣∣∣∣

a1 b1 a1 + b1 + c1

a2 b2 a2 + b2 + c2

a3 b3 a3 + b3 + c3

∣∣∣∣∣∣=

∣∣∣∣∣∣

a1 b1 a1

a2 b2 a2

a3 b3 a3

∣∣∣∣∣∣+

∣∣∣∣∣∣

a1 b1 b1

a2 b2 b2

a3 b3 b3

∣∣∣∣∣∣+

∣∣∣∣∣∣

a1 b1 c1

a2 b2 c2

a3 b3 c3

∣∣∣∣∣∣

=

∣∣∣∣∣∣

a1 b1 c1

a2 b2 c2

a3 b3 c3

∣∣∣∣∣∣The following result is known as the Laplace expansion of |A| along a row(respectively a column).

Theorem 35If aij denotes the ijth entry of A and Cij is the cofactor of the entry aij thenthe expansion along row i is

|A| = ai1Ci1 + ai2Ci2 + · · ·+ ainCin.

The expansion along column j is given by

|A| = a1jC1j + a2jC2j + · · ·+ anjCnj .

Proof.We prove the first formula. The second formula follows from the first becauseof the fact that |AT | = |A|. Let r1, r2, · · · , rn denote the rows of A. By Theorem34 we have

|A| =

∣∣∣∣∣∣∣∣∣∣∣∣∣∣

r1

r2

...∑nj=1 aijej

...rn

∣∣∣∣∣∣∣∣∣∣∣∣∣∣

=∑n

j=1 aij

∣∣∣∣∣∣∣∣∣∣∣∣∣∣

r1

r2

...ej

...rn

∣∣∣∣∣∣∣∣∣∣∣∣∣∣where ej is the 1× n matrix with 1 at the jth position and zero elsewhere. Wewill show that ∣∣∣∣∣∣∣∣∣∣∣∣∣∣

r1

r2

...ej

...rn

∣∣∣∣∣∣∣∣∣∣∣∣∣∣

Page 89: Fundamentals of Linear Algebra - Mathematics & Statisticsmelbial2/classes/Linear Algebra... · 2006-01-13 · 1.1 Systems of Linear Equations Many practical problems can be reduced

3.4. FINDING A−1 USING COFACTOR EXPANSIONS 89

is just the cofactor of the entry aij .By Theorem 27 (d) we have

∣∣∣∣∣∣∣∣∣∣∣∣∣∣

a11 a12 · · · a1j · · · a1n

a21 a22 · · · a2j · · · a2n

......

......

......

0 0 · · · 1 · · · 0...

......

......

...an1 an2 · · · anj · · · ann

∣∣∣∣∣∣∣∣∣∣∣∣∣∣

=

∣∣∣∣∣∣∣∣∣∣∣∣∣∣

a11 a12 · · · 0 · · · a1n

a21 a22 · · · 0 · · · a2n

......

......

......

0 0 · · · 1 · · · 0...

......

......

...an1 an2 · · · 0 · · · ann

∣∣∣∣∣∣∣∣∣∣∣∣∣∣

Now, move the ith row to the first row using (i−1) successive row interchanges.Then, move the jth column to the first using (j − 1) successive column inter-changes. Thus, we have

∣∣∣∣∣∣∣∣∣∣∣∣∣∣

r1

r2

...ej

...rn

∣∣∣∣∣∣∣∣∣∣∣∣∣∣

= (−1)i+j

∣∣∣∣1 01×(n−1)

0(n−1)×1 Mij

∣∣∣∣

where Mij is the (n− 1)× (n− 1) matrix obtained from A by deleting the ithrow and the jth column of A.Let E1, E2, · · · , Ek be (n−1)×(n−1) elementary matrices such that EkEk−1 · · ·E1Mij =Cij is an upper triangular matrix. Then the matrices E′

1, E′2, · · · , E′

k are n× nelementary matrices such that

∣∣∣∣E′kE′

k−1 · · ·E′1

(1 01×(n−1)

0(n−1)×1 Mij

)∣∣∣∣ =∣∣∣∣

1 01×(n−1)

0(n−1)×1 Cij

∣∣∣∣

where

E′i =

(1 01×(n−1)

0(n−1)×1 Ei

)

Therefore,∣∣∣∣

1 01×(n−1)

0(n−1)×1 Mij

∣∣∣∣ =|Cij |

|Ek||Ek−1 · · · |E1| = |Mij |.

Hence, ∣∣∣∣∣∣∣∣∣∣∣∣∣∣

r1

r2

...ej

...rn

∣∣∣∣∣∣∣∣∣∣∣∣∣∣

= (−1)i+j |Mij | = Cij

Page 90: Fundamentals of Linear Algebra - Mathematics & Statisticsmelbial2/classes/Linear Algebra... · 2006-01-13 · 1.1 Systems of Linear Equations Many practical problems can be reduced

90 CHAPTER 3. DETERMINANTS

This ends a proof of the theorem

RemarkIn general, the best strategy for evaluating a determinant by cofactor expansionis to expand along a row or a column having the largest number of zeros.

Exercise 178Find the determinant of each of the following matrices.(a)

A =

0 0 a13

0 a22 a23

a31 a32 a33

(b)

A =

0 0 0 a14

0 0 a23 a24

0 a32 a33 a34

a41 a42 a43 a44

Solution.(a) Expanding along the first column we find

|A| = −a31a22a13.

(b) Again, by expanding along the first column we obtain

|A| = a41a32a23a34

Exercise 179Use cofactor expansion along the first column to find |A| where

A =

3 5 −2 61 2 −1 12 4 1 53 7 5 3

Solution.Expanding along the first column we find

|A| = 3C11 + C21 + 2C31 + 3C41

= 3|M11| − |M21|+ 2|M31| − 3|M41|= 3(−54) + 78 + 2(60)− 3(18) = −18

If A is an n× n square matrix and Cij is the cofactor of the entry aij then thetranspose of the matrix

C11 C12 . . . C1n

C21 C22 . . . C2n

......

...Cn1 Cn2 . . . Cnn

is called the adjoint of A and is denoted by adj(A).

Page 91: Fundamentals of Linear Algebra - Mathematics & Statisticsmelbial2/classes/Linear Algebra... · 2006-01-13 · 1.1 Systems of Linear Equations Many practical problems can be reduced

3.4. FINDING A−1 USING COFACTOR EXPANSIONS 91

Exercise 180Let

A =

3 2 −11 6 32 −4 0

,

Find adj(A).

Solution.We first find the matrix of cofactors of A.

C11 C12 C13

C21 C22 C23

C31 C32 C33

=

12 6 −164 2 16

12 −10 16

The adjoint of A is the transpose of this cofactor matrix.

adj(A) =

12 4 126 2 −10

−16 16 16

Our next goal is to find another method for finding the inverse of a nonsingularsquare matrix. To this end, we need the following result.

Theorem 36For i 6= j we have

ai1Cj1 + ai2Cj2 + . . . + ainCjn = 0.

Proof.Let B be the matrix obtained by replacing the jth row of A by the ith rowof A. Then B has two identical rows and therefore |B| = 0(See Theorem 27(c)). Expand |B| along the jth row. The elements of the jth row of B areai1, ai2, . . . , ain. The cofactors are Cj1, Cj2, . . . , Cjn. Thus

0 = |B| = ai1Cj1 + ai2Cj2 + . . . + ainCjn

This concludes a proof of the theorem

The following theorem states that the product A · adj(A) is a scalar matrix.

Theorem 37 If A is an n× n matrix then A · adj(A) = |A|In.

Proof.The (i, j) entry of the matrix

A.adj(A) =

a11 a12 . . . a1n

a21 a22 . . . a2n

......

...an1 an2 . . . ann

C11 C21 . . . Cn1

C12 C22 . . . Cn2

......

...C1n C2n . . . Cnn

Page 92: Fundamentals of Linear Algebra - Mathematics & Statisticsmelbial2/classes/Linear Algebra... · 2006-01-13 · 1.1 Systems of Linear Equations Many practical problems can be reduced

92 CHAPTER 3. DETERMINANTS

is given by the sum

ai1Cj1 + ai2Cj2 + . . . + ainCjn = |A|

if i = j and 0 if i 6= j. Hence,

A.adj(A) =

|A| 0 . . . 00 |A| . . . 0...

......

0 0 . . . |A|

= |A|In.

This ends a proof of the theorem

The following theorem provides a way for finding the inverse of a matrix us-ing the notion of the adjoint.

Theorem 38If |A| 6= 0 then A is invertible and A−1 = adj(A)

|A| . Hence, adj(A) = A−1|A|.

Proof.By Theorem 37 we have that A(adj(A)) = |A|In. If |A| 6= 0 then A(adj(A)

|A| ) = In.

By Theorem 20, A is invertible with inverse A−1 = adj(A)|A| .

Exercise 181Let

A =

3 2 −11 6 32 −4 0

Use Theorem 38 to find A−1.

Solution.First we find the determinant of A given by |A| = 64. By Theorem 38

A−1 =1|A|adj(A) =

316

116

316

332

132 − 5

32− 1

414

14

In the next theorem we discuss three properties of the adjoint matrix.

Theorem 39Let A and B denote invertible n× n matrices. Then,

(a) adj(A−1) = (adj(A))−1.(b) adj(AT ) = (adj(A))T .(c) adj(AB) = adj(B)adj(A).

Page 93: Fundamentals of Linear Algebra - Mathematics & Statisticsmelbial2/classes/Linear Algebra... · 2006-01-13 · 1.1 Systems of Linear Equations Many practical problems can be reduced

3.5. CRAMER’S RULE 93

Proof.(a) Since A(adj(A)) = |A|In then adj(A) is invertible and (adj(A))−1 = A

|A| =(A−1)−1|A−1| = adj(A−1).

(b) adj(AT ) = (AT )−1|AT | = (A−1)T |A| = (adj(A))T .

(c) We have adj(AB) = (AB)−1|AB| = B−1A−1|A||B| = (B−1|B|)(A−1|A|) =adj(B)adj(A).

Exercise 182Show that if A is singular then A.adj(A) = 0.

Solution.If A is singular then |A| = 0. But then A · adj(A) = |A|In = 0

3.5 Cramer’s Rule

Cramer’s rule is another method for solving a linear system of n equations in nunknowns. This method is reasonable for inverting, for example, a 3× 3 matrixby hand; however, the inversion method discussed before is more efficient forlarger matrices.

Theorem 40Let Ax = b be a matrix equation with A = (aij), x = (xi), b = (bi). Then wehave the following matrix equation

|A|x1

|A|x2

...|A|xn

=

|A1||A2|

...|An|

where Ai is the matrix obtained from A by replacing its ith column by b. It fol-lows that

(1) If |A| 6= 0 then the above system has a unique solution given by

xi =|Ai||A| ,

where 1 ≤ i ≤ n.(2) If |A| = 0 and |Ai| 6= 0 for some i then the system has no solution.(3) If |A| = |A1| = . . . = |An| = 0 then the system has an infinite number ofsolutions.

Proof.We have the following chain of equalities

|A|x = |A|(Inx)

Page 94: Fundamentals of Linear Algebra - Mathematics & Statisticsmelbial2/classes/Linear Algebra... · 2006-01-13 · 1.1 Systems of Linear Equations Many practical problems can be reduced

94 CHAPTER 3. DETERMINANTS

= (|A|In)x= adj(A)Ax

= adj(A)b

The ith entry of the vector |A|x is given by

|A|xi = b1C1i + b2C2i + . . . + bnCni.

On the other hand by expanding |Ai| along the ith column we find that

|Ai| = C1ib1 + C2ib2 + . . . + Cnibn.

Hence|A|xi = |Ai|.

Now, (1), (2), and (3) follow easily. This ends a proof of the theorem

Exercise 183Use Cramer’s rule to solve

−2x1 + 3x2 − x3 = 1x1 + 2x2 − x3 = 4−2x1 − x2 + x3 = −3.

Solution.By Cramer’s rule we have

A =

−2 3 −11 2 −1−2 −1 1

, |A| = −2.

A1 =

1 3 −14 2 −1−3 −1 1

, |A1| = −4.

A2 =

−2 1 −11 4 −1−2 −3 1

, |A2| = −6.

A3 =

−2 3 11 2 4−2 −1 −3

, |A3| = −8.

Thus, x1 = |A1||A| = 2, x2 = |A2|

|A| = 3, x3 = |A3||A| = 4

Exercise 184Use Cramer’s rule to solve

5x1 − 3x2 − 10x3 = −92x1 + 2x2 − 3x3 = 4−3x1 − x2 + 5x3 = 1.

Page 95: Fundamentals of Linear Algebra - Mathematics & Statisticsmelbial2/classes/Linear Algebra... · 2006-01-13 · 1.1 Systems of Linear Equations Many practical problems can be reduced

3.5. CRAMER’S RULE 95

Solution.By Cramer’s rule we have

A =

5 −3 −102 2 − 3−3 −1 5

, |A| = −2.

A1 =

−9 −3 −104 2 − 31 −1 5

, |A1| = 66.

A2 =

5 −9 −102 4 − 3−3 1 5

, |A2| = −16.

A3 =

5 −3 −92 2 4−3 −1 1

, |A3| = 36.

Thus, x1 = |A1||A| = −33, x2 = |A2|

|A| = 8, x3 = |A3||A| = −18

Exercise 185Let ABC be a triangle such that dist(A,B) = c, dist(A, C) = b, dist(B,C) = a,the angle between AB and AC is α, between BA and BC is β, and that betweenCA and CB is γ.(a) Use trigonometry to show that

b cos γ + c cos β = ac cosα + a cos γ = ba cosβ + b cosα = c

(b) Use Cramer’s rule to express cosα, cosβ, and cos γ in terms of a, b, and c.

Solution.(a) Let A1 be the leg of the perpendicular to BC through A. Then the trianglesA1AB and A1AC are right triangles. In this case, we have cos β = dist(B,A1)

cor dist(B, A1) = c cos β. Simlarly, dist(C,A1) = b cos γ. But a = dist(B,A1) +dist(C,A1) = b cos γ + c cosβ. Similar argument for the remaining two equali-tites.(b)

A =

0 c bc 0 ab a 0

, |A| = 2abc.

A1 =

a c bb 0 ac a 0

, |A1| = a(c2 + b2)− a3.

Page 96: Fundamentals of Linear Algebra - Mathematics & Statisticsmelbial2/classes/Linear Algebra... · 2006-01-13 · 1.1 Systems of Linear Equations Many practical problems can be reduced

96 CHAPTER 3. DETERMINANTS

A2 =

0 a bc b ab c 0

, |A2| = b(a2 + c2)− b3.

A3 =

0 c ac 0 bb a c

, |A3| = c(a2 + b2)− c3.

Thus, cos α = |A1||A| = c2+b2−a2

2bc , cos β = |A2||A| = a2+c2−b2

2ac , cos γ = |A3||A| = a2+b2−c2

2ab

3.6 Review Problems

Exercise 186(a) Find the number of inversions in the permutation (41352).(b) Is this permutation even or odd?

Exercise 187Evaluate the determinant of each of the following matrices(a)

A =(

3 5−2 4

)

(b)

A =

−2 7 65 1 −23 8 4

Exercise 188Find all values of t for which the determinant of the following matrix is zero.

A =

t− 4 0 00 t 00 3 t− 1

Exercise 189Solve for x

∣∣∣∣x −13 1− x

∣∣∣∣ =

∣∣∣∣∣∣

1 0 −32 x −61 3 x− 5

∣∣∣∣∣∣

Exercise 190Evaluate the determinant of the following matrix

A =

1 2 34 5 60 0 0

Page 97: Fundamentals of Linear Algebra - Mathematics & Statisticsmelbial2/classes/Linear Algebra... · 2006-01-13 · 1.1 Systems of Linear Equations Many practical problems can be reduced

3.6. REVIEW PROBLEMS 97

Exercise 191Evaluate the determinant of the following matrix.

∣∣∣∣∣∣∣∣∣∣

2 7 −3 8 30 −3 7 5 10 0 6 7 60 0 0 9 80 0 0 0 4

∣∣∣∣∣∣∣∣∣∣Exercise 192Use the row reduction technique to find the determinant of the following matrix.

A =

2 5 −3 −2−2 −3 2 −51 3 −2 2−1 −6 4 3

Exercise 193Given that ∣∣∣∣∣∣

a b cd e fg h i

∣∣∣∣∣∣= −6,

find(a) ∣∣∣∣∣∣

d e fg h ia b c

∣∣∣∣∣∣,

(b) ∣∣∣∣∣∣

3a 3b 3c−d −e −f4g 4h 4i

∣∣∣∣∣∣(c) ∣∣∣∣∣∣

a + g b + h c + id e fg h i

∣∣∣∣∣∣(d) ∣∣∣∣∣∣

−3a −3b −3cd e f

g − 4d h− 4e i− 4f

∣∣∣∣∣∣Exercise 194Determine by inspection the determinant of the following matrix.

1 2 3 4 56 7 8 9 1011 12 13 14 1516 17 18 19 202 4 6 8 10

Page 98: Fundamentals of Linear Algebra - Mathematics & Statisticsmelbial2/classes/Linear Algebra... · 2006-01-13 · 1.1 Systems of Linear Equations Many practical problems can be reduced

98 CHAPTER 3. DETERMINANTS

Exercise 195Find the determinant of the 1× 1 matrix A = (3).

Exercise 196Let A be a 3× 3 matrix such that |2A| = 6. Find |A|.

Exercise 197Show that if n is any positive integer then |An| = |A|n.

Exercise 198Show that if A is an n× n skew-symmetric and n is odd then |A| = 0.

Exercise 199Show that if A is orthogonal, i.e. AT A = AAT = In then |A| = ±1. Note thatA−1 = AT .

Exercise 200If A is a nonsingular matrix such that A2 = A, what is |A|?

Exercise 201True or false: If

A =

1 0 01 2 03 1 0

then rank(A) = 3. Justify your answer.

Exercise 202Find out, without solving the system, whether the following system has a non-trivial solution

x1 − 2x2 + x3 = 02x1 + 3x2 + x3 = 03x1 + x2 + 2x3 = 0

Exercise 203For which values of c does the matrix

A =

1 0 −c−1 3 10 2c −4

have an inverse.

Exercise 204If |A| = 2 and |B| = 5, calculate |A3B−1AT B2|.

Exercise 205Show that |AB| = |BA|.

Page 99: Fundamentals of Linear Algebra - Mathematics & Statisticsmelbial2/classes/Linear Algebra... · 2006-01-13 · 1.1 Systems of Linear Equations Many practical problems can be reduced

3.6. REVIEW PROBLEMS 99

Exercise 206Show that |A + BT | = |AT + B| for any n× n matrices A and B.

Exercise 207Let A = (aij) be a triangular matrix. Show that |A| 6= 0 if and only if aii 6= 0,for 1 ≤ i ≤ n.

Exercise 208Express ∣∣∣∣

a1 + b1 c1 + d1

a2 + b2 c2 + d2

∣∣∣∣as a sum of four determinants whose entries contain no sums.

Exercise 209Let

A =

3 −2 15 6 21 0 −3

(a) Find adj(A).(b) Compute |A|.Exercise 210Find the determinant of the matrix

A =

3 0 0 05 1 2 02 6 0 −1−6 3 1 0

Exercise 211Find the determinant of the following Vandermonde matrix.

A =

1 1 1a b ca2 b2 c2

Exercise 212Let A be an n× n matrix. Show that |adj(A)| = |A|n−1.

Exercise 213If

A−1 =

3 0 10 2 33 1 −1

find adj(A).

Exercise 214If |A| = 2, find |A−1 + adj(A)|.

Page 100: Fundamentals of Linear Algebra - Mathematics & Statisticsmelbial2/classes/Linear Algebra... · 2006-01-13 · 1.1 Systems of Linear Equations Many practical problems can be reduced

100 CHAPTER 3. DETERMINANTS

Exercise 215Show that adj(αA) = αn−1adj(A), where n is a positive integer.

Exercise 216Consider the matrix

A =

1 2 32 3 41 5 7

(a) Find |A|.(b) Find adj(A).(c) Find A−1.

Exercise 217Prove that if A is symmetric then adj(A) is also symmetric.

Exercise 218Prove that if A is a nonsingular triangular matrix then A−1 is also triangular.

Exercise 219Let A be an n× n matrix.(a) Show that if A has integer entries and |A| = 1 then A−1 has integer entriesas well.(b) Let Ax = b. Show that if the entries of A and b are integers and |A| = 1then the entries of x are also integers.

Exercise 220Show that if Ak = 0 for some positive integer k then A is singular.

Exercise 221Use Cramer’s Rule to solve

x1 + 2x3 = 6−3x1 + 4x2 + 6x3 = 30−x1 − 2x2 + 3x3 = 8

Exercise 222Use Cramer’s Rule to solve

5x1 + x2 − x3 = 49x1 + x2 − x3 = 1x1 − x2 + 5x3 = 2

Page 101: Fundamentals of Linear Algebra - Mathematics & Statisticsmelbial2/classes/Linear Algebra... · 2006-01-13 · 1.1 Systems of Linear Equations Many practical problems can be reduced

Chapter 4

The Theory of VectorSpaces

In Chapter 2, we saw that the operations of addition and scalar multiplicationon the set Mmn of m×n matrices possess many of the same algebraic propertiesas addition and scalar multiplication on the set IR of real numbers. In fact, thereare many other sets with operations that share these same properties. Insteadof studying these sets individually, we study them as a class.In this chapter, we define vector spaces to be sets with algebraic operationshaving the properties similar to those of addition and scalar multiplication onIR and Mmn. We then establish many important results that apply to all vectorspaces, not just IR and Mmn.

4.1 Vectors in Two and Three Dimensional Spaces

Many Physical quantities are represented by vectors. For examples, the velocityof a moving object, displacement, force of gravitation, etc. In this section wediscuss the geometry of vectors in two and three dimensional spaces, discussthe arithmetic operations of vectors and study some of the properties of theseoperations. As we shall see later in this chapter, the present section will provideus with an example of a vector space, a concept that will be defined in the nextsection, that can be illustrated geometrically.A vector in space is a directed segment with a tail, called the initial point,and a tip, called the terminal point. A vector will be denoted by ~v and scalarsby greek letters α, β, · · · Finally, we point out that most of the results of thissection hold for vectors in IR2.

Two vectors ~v and ~w are said to be equivalent if they have the same lengthand the same direction. We write ~v = ~w.

Since we can always draw a vector with initial point O that is equivalent to

101

Page 102: Fundamentals of Linear Algebra - Mathematics & Statisticsmelbial2/classes/Linear Algebra... · 2006-01-13 · 1.1 Systems of Linear Equations Many practical problems can be reduced

102 CHAPTER 4. THE THEORY OF VECTOR SPACES

a given vector then from now on we assume that the vectors have all the sameinitial point O unless indicated otherwise. The coordinates (x, y, z) of the ter-minal point of a vector ~v are called the components of ~v and we write

~v = (x, y, z).

In terms of components, two vectors ~v = (x, y, z) and ~w = (x′, y′, z′) are equiv-alent if and only if x = x′, y = y′, and z = z′.Given two vectors ~v = (x1, y1, z1) and ~w = (x2, y2, z2), we define the sum ~v + ~wto be the vector

~v + ~w = (x1 + x2, y1 + y2, z1 + z2).

Geometrically, the ~v + w is the diagonal of the parallelogram with sides thevectors ~v and ~w.We define the negative vector −~v to be the vector

−~v = (−x1,−y1,−z1)

and we define

~v − ~w = ~v + (−~w) = (x1 − x2, y1 − y2, z1 − z2).

Assuming that ~v and ~w have the same initial point, the vector ~v − ~w is thevector with initial point the terminal point of ~w and its terminal point is theterminal point of ~v.If α is a scalar we define α~v to be the vector

α~v = (αx1, αy1, αz1).

With the above definitions, every vector ~u = (x, y, z) can be expressed as acombination of the three vectors ~i = (1, 0, 0),~j = (0, 1, 0), and ~k = (0, 0, 1).That is

~u = x~i + y~j + z~k.

Exercise 223Given two points P1(x1, y1, z1) and P2(x2, y2, z2), find the components of thevector −−−→P1P2.

Solution.The vector −−−→P1P2 is the difference of the vectors −−→OP1 and −−→OP2 so−−−→P1P2 = −−→

OP2 −−−→OP1 = (x2, y2, z2)− (x1, y1, z1) = (x2 − x1, y2 − y1, z2 − z1)

Exercise 224Let ~u = (1, 2, 3), ~v = (2,−3, 1), and ~w = (3, 2,−1).

(a) Find the components of the vector ~u− 3~v + 8~w.(b) Find scalars c1, c2, c3 such that

c1~u + c2~v + c3 ~w = (6, 14,−2).

Page 103: Fundamentals of Linear Algebra - Mathematics & Statisticsmelbial2/classes/Linear Algebra... · 2006-01-13 · 1.1 Systems of Linear Equations Many practical problems can be reduced

4.1. VECTORS IN TWO AND THREE DIMENSIONAL SPACES 103

Solution.(a) ~u − 3~v + 8~w = (1, 2, 3) − 3(2,−3, 1) + 8(3, 2,−1) = (1, 2, 3) − (6,−9, 3) +(24, 16,−8) = (19, 27,−8).(b) We have (c1, 2c1, 3c1)+(2c2,−3c2, c2)+(3c3, 2c3,−c3) = (c1+2c2+3c3, 2c1−3c2 + 2c3, 3c1 + c2 − c3) = (6, 14,−2). This yields the linear system

c1 + 2c2 + 3c3 = 62c1 − 3c2 + 2c3 = 143c1 + c2 − c3 = −2

The augmented matrix of the system is

1 2 3 62 −3 2 143 1 −1 −2

The reduction of this matrix to row-echelon form is

Step 1: r2 ← r2 − 2r1 and r3 ← r3 − 3r1

1 2 3 60 −7 − 4 20 −5 −10 −20

Step 2: r2 ← 2r2 − 3r3

1 2 3 60 1 22 640 −5 −10 −20

Step 3: r3 ← r3 + 5r2

1 2 3 60 1 22 640 0 100 300

Step 4: r3 ← 1100r3

1 2 3 60 1 22 640 0 1 3

The corresponding system is

c1 + 2c2 + 3c3 = 6c2 + 22c3 = 64

c3 = 3

Solving the above triangular system to obtain: c1 = 1, c2 = −2, c3 = 3

The basic properties of vector addition and scalar multiplication are collectedin the following theorem.

Page 104: Fundamentals of Linear Algebra - Mathematics & Statisticsmelbial2/classes/Linear Algebra... · 2006-01-13 · 1.1 Systems of Linear Equations Many practical problems can be reduced

104 CHAPTER 4. THE THEORY OF VECTOR SPACES

Theorem 41If ~u,~v, and ~w are vectors and α, β are scalars then the following properties hold.

(a) ~u + ~v = ~v + ~u.(b) (~u + ~v) + ~w = ~u + (~v + ~w).(c) ~u +~0 = ~u, where ~0 = (0, 0, 0).(d) ~u + (−~u) = ~0.(e) α(β~u) = (αβ)~u.(f) α(~u + ~v) = α~u + α~v.(g) (α + β)~u = α~u + β~u.(h) 1~u = ~u.

Proof.The prove of this theorem uses the properties of addition and scalar multipli-cation of real numbers. We prove (a). The remaining properties can be provedeasily. Suppose that ~u = (x1, y1, z1) and ~v = (x2, y2, z2). Then using the factthat the addition of real numbers is commutative we have

~u + ~v = (x1, y1, z1) + (x2, y2, z2)= (x1 + x2, y1 + y2, z1 + z2)= (x2 + x1, y2 + y1, z2 + z1)= (x2, y2, z2) + (x1, y1, z1)= ~v + ~u

This ends the proof of (a)

Exercise 225(a) Let ~u = (x1, y1, z1). Show that the length of ~u, known as the norm of ~u, isgiven by the expression

||~u|| =√

x21 + y2

1 + z21 .

(b) Given two points P1(x1, y1, z1) and P2(x2, y2, z2) show that the distance be-tween these points is given by the formula

||−−−→P1P2|| =√

(x2 − x1)2 + (y2 − y1)2 + (z2 − z1)2.

Solution.(a) Let ~u = −−→

OP. Let Q be the orthogonal projection of P onto the xy-plane.Then by the Pythagorean Theorem we have

||−−→OQ||2 + ||−−→PQ||2 = ||−−→OP ||2

But ||−−→OQ||2 = x21 + y2

1 and ||−−→PQ||2 = z21 . Thus,

||~u||2 = x21 + y2

1 + z21

Now take the square root of both sides.(b) By Exercise 223, −−−→P1P2 = (x2 − x1, y2 − y1, z2 − z1). Hence, by part (a) wehave

||−−−→P1P2|| =√

(x2 − x1)2 + (y2 − y1)2 + (z2 − z1)2

Page 105: Fundamentals of Linear Algebra - Mathematics & Statisticsmelbial2/classes/Linear Algebra... · 2006-01-13 · 1.1 Systems of Linear Equations Many practical problems can be reduced

4.1. VECTORS IN TWO AND THREE DIMENSIONAL SPACES 105

Exercise 226Show that if ~u is a non zero vector then the length of the vector ~u

||~u|| is 1.

Solution.If ~u = (x, y, z) then ~u

||~u|| = ( x||~u|| ,

y||~u|| ,

z||~u|| ). Hence,

|| ~u

||~u|| || =√

x2

||~u||2 +y2

||~u||2 +z2

||~u||2 = 1

Exercise 227Suppose that an xyz−coordinate system is translated to obtain a new systemx′y′z′ with origin the point O′(k, l, m). Let P be a point in the space. Supposethat (x, y, z) are the coordinates of P in xyz−system and (x′, y′, z′) are thecoordinates of P in the x′y′z′− system. Show that x′ = x − k, y′ = y − l, z′ =z −m.

Solution.By Exercise 223,

−−→O′P = (x−k, y− l, z−m) = (x′, y′, z′). Thus, x′ = x−k, y′ =

y − l, z′ = z −m

Next, we shall discuss a way for multiplying vectors. If ~u and ~v are two vectorsin the space and θ is the angle between them we define the inner product(sometimes called scalar product or dot product) of ~u and ~v to be the num-ber

< ~u,~v >= ||~u||||~v|| cos θ 0 ≤ θ ≤ π.

The following theorem lists the most important properties of the dot product.

Theorem 42Let ~u = (x1, y1, z1), ~v = (x2, y2, z2) and ~w = (x3, y3, z3) be vectors and α be ascalar. Then(a) < ~u,~v >= x1x2 + y1y2 + z1z2.(b) < ~u, ~u >= ||~u||2.(c) < ~u,~v >=< ~v, ~u > .(d) < ~u,~v + ~w >=< ~u,~v > + < ~u, ~w > .(e) α < ~u,~v >=< α~u,~v >=< ~u, α~v > .(f) < ~u, ~u >= 0 if and only if ~u = ~0.

Proof.(a) Let −−→OP = ~u = (x1, y1, z1) and −−→OQ = ~v = (x2, y2, z2). Let θ be the anglebetween them. Then −−→PQ = (x2 − x1, y2 − y1, z2 − z1). By the law of cosines wehave

||−−→PQ||2 = ||−−→OP ||2 + ||−−→OQ||2 − 2||−−→OP ||||−−→OQ|| cos θ= ||~u||2 + ||~v||2 − 2||~u||||~v|| cos θ.

Page 106: Fundamentals of Linear Algebra - Mathematics & Statisticsmelbial2/classes/Linear Algebra... · 2006-01-13 · 1.1 Systems of Linear Equations Many practical problems can be reduced

106 CHAPTER 4. THE THEORY OF VECTOR SPACES

But ||−−→PQ|| = ||~u− ~v||. Hence,

< ~u,~v > = ||~u||||~v|| cos θ= 1

2 (||~u||2 + ||~v||2 − ||~u− ~v||2)= 1

2 (x21 + y2

1 + z21 + x2

2 + y22 + z2

2 − (x2 − x1)2 − (y2 − y1)2 − (z2 − z1)2)= x1x2 + y1y2 + z1z2.

(b) < ~u, ~u >= ||~u||2 = x21 + y2

1 + z21 .

(c) < ~u,~v >= x1x2 + y1y2 + z1z2 = x2x1 + y2y1 + z2z1 =< ~v, ~u > .(d) < ~u,~v + ~w >= x1(x2 + x3) + y1(y2 + y3) + z1(z2 + z3) = (x1x2 + y1y2 +z1z2) + (x1x3 + y1y3 + z1z3) =< ~u,~v > + < ~u, ~w > .(e) α < ~u,~v >= α(x1x2 + y1y2 + z1z2) = (αx1)x2 + (αy1)y2 + (αz1)z2 =<α~u,~v > .(f) If < ~u, ~u >= 0 then x2

1 + y21 + z2

1 = 0 and this implies that x1 = y1 = z1 = 0.That is, ~u = ~0. The converse is straightforward.

Exercise 228Consider the vectors ~u = (2,−1, 1) and ~v = (1, 1, 2). Find < ~u,~v > and theangle θ between these two vectors.

Solution.Using the above theorem we find

< ~u,~v >= (2)(1) + (−1)(1) + (1)(2) = 3.

Also, ||~u|| = ||~v|| = √6. Hence,

cos θ =36

=12

This implies that θ = π3 radians.

Exercise 229Two vectors are said to be orthogonal if < ~u,~v >= 0. Show that the two vectors~u = (2,−1, 1) and ~v = (1, 1,−1) are orthogonal.

Solution.Indeed, < ~u,~v >= (2)(1) + (−1)(1) + (1)(−1) = 0

Next, we discuss the decomposition of a vector into a sum of two vectors. To bemore precise, let ~u and ~w be two vectors with the same initial point Q and with~w being horizontal. From the tip of ~u drop a perpendicular to the line through~w, and construct the vector ~u1 from Q to the foot of this perpendicular. Nextform the vector ~u2 = ~u− ~u1. Clearly, ~u = ~u1 + ~u2, where ~u1 is parallel to ~w and~u2 is perpendicular to ~w. We call ~u1 the orthogonal projection of ~u on ~wand we denote it by proj~w~u.The following theorem gives formulas for calculating ~u1 and ~u2.

Page 107: Fundamentals of Linear Algebra - Mathematics & Statisticsmelbial2/classes/Linear Algebra... · 2006-01-13 · 1.1 Systems of Linear Equations Many practical problems can be reduced

4.1. VECTORS IN TWO AND THREE DIMENSIONAL SPACES 107

Theorem 43For any two vectors ~u and ~w 6= 0 we have ~u1 = <~u,~w>

||~w||2 ~w and ~u2 = ~u− <~u,~w>||~w||2 ~w.

Proof.Since ~u1 is parallel to ~w then there is a scalar t such that ~u1 = t~w. Thus,< ~u, ~w >=< t~w + ~u2, ~w >= t||~w||2+ < ~u2, ~w > . But ~u2 and ~w are orthogonalso that the second term on the above equality is zero. Hence, t = <~u,~w>

||~w||2 . It

follows that ~u1 = t~w = <~u,~w>||~w||2 ~w

Exercise 230Let ~u = (2,−1, 3) and ~w = (4,−1, 2). Find ~u1 and ~u2.

Solution.We have: < ~u, ~w >= 15 and ||~w||2 = 21. Thus, ~u1 = ( 20

7 ,− 57 , 10

7 ) and ~u2 =~u− ~u1 = (− 6

7 ,− 27 , 11

7 )

Exercise 231Given a vector ~v = (a, b, c) in IR3. The angles α, β, γ between ~v and the unitvectors ~i,~j, and ~k, respectively, are called the direction angles of ~v and thenumbers cosα, cosβ, and cos γ are called the direction cosines of ~v.(a) Show that cos α = a

||~v|| .(b) Find cosβ and cos γ.(c) Show that ~v

||~v|| = (cos α, cos β, cos γ).(d) Show that cos2 α + cos2 β + cos2 γ = 1.

Solution.(a) cos α = <~v,~i>

||~v||||~i|| = a||~v|| .

(b) By repeating the arithmetic of part (a) one finds cos β = b||~v|| and cos γ =

c||~v|| .

(c) ~v||~v|| = ( a

||~v|| ,b||~v|| ,

c||~v|| ) = (cos α, cos β, cos γ).

(d) cosα2 + cos β2 + cos γ2 = || ~v||~v|| ||2 = 1

Exercise 232An interesting application of determinants and vectors is the construction of avector orthogonal to two given vectors.

(a) Given two vectors ~u = (x1, y1, z1) and ~v = (x2, y2, z2) then we define thecross product of ~u and ~v to be the vector

~u× ~v =

∣∣∣∣∣∣

~i ~j ~kx1 y1 z1

x2 y2 z2

∣∣∣∣∣∣

Find the components of the vector ~u× ~v.(b) Find ~u× ~v, where ~u = (1, 2,−2), ~v = (3, 0, 1).(c) Show that < ~u, ~u× ~v >= 0 and < ~v, ~u× ~v >= 0. Hence, ~u× ~v is orthogonalto both ~u and ~v

Page 108: Fundamentals of Linear Algebra - Mathematics & Statisticsmelbial2/classes/Linear Algebra... · 2006-01-13 · 1.1 Systems of Linear Equations Many practical problems can be reduced

108 CHAPTER 4. THE THEORY OF VECTOR SPACES

Solution.(a) Evaluating the determinant defining ~u×~v we find ~u×~v = (y1z2−y2z1, x2z1−x1z2, x1y2 − x2y1).(b) Substituting in (a) we find ~u× ~v = (2,−7,−6).(c) Indeed, < ~u, ~u×~v >= x1(y1z2−y2z1)+y1(x2z1−x1z2)+z1(x1y2−x2y1) = 0.Similarly, < ~v, ~u× ~v >= 0

4.2 Vector Spaces, Subspaces, and Inner Prod-uct Spaces

The properties listed in Theorem 41 of the previous section generalizes to IRn,the Euclidean space to be discussed below. Also, these properties hold for thecollection Mmn of all m× n matrices with the operation of addition and scalarmultiplication. In fact there are other sets with operations satisfying the condi-tions of Theorem 41. Thus, it make sense to study the group of sets with theseproperties. In this section, we define vector spaces to be sets with algebraicoperations having the properties similar to those of addition and scalar multi-plication on IRn and Mmn.

Let n be a positive integer. Let IRn be the collection of elements of the form(x1, x2, . . . , xn), where the xis are real numbers. Define the following operationson IRn :

(a) Addition: (x1, x2, . . . , xn) + (y1, y2, . . . , yn) = (x1 + y1, . . . , xn + yn)(b) Multiplication of a vector by a scalar:

α(x1, x2, . . . , xn) = (αx1, αx2, . . . , αxn).

The basic properties of addition and scalar multiplication of vectors in IRn arelisted in the following theorem.

Theorem 44The following properties hold, for u, v, w in IRn and α, β scalars:

(a) u + v = v + u(b) u + (v + w) = (u + v) + w(c) u + 0 = 0 + u = u where 0 = (0, 0, . . . , 0)(d) u + (−u) = 0(e) α(u + v) = αu + αv(f) (α + β)u = αu + βu(g) α(βu) = (αβ)u(h) 1u = u.

Proof.This is just a generalization of Theorem 41

Page 109: Fundamentals of Linear Algebra - Mathematics & Statisticsmelbial2/classes/Linear Algebra... · 2006-01-13 · 1.1 Systems of Linear Equations Many practical problems can be reduced

4.2. VECTOR SPACES, SUBSPACES, AND INNER PRODUCT SPACES109

The set IRn with the above operations and properties is called the Euclideanspace.

A vector space is a set V together with the following operations:(i) Addition: If u, v ∈ V then u + v ∈ V. We say that V is closed underaddition.(ii) Multiplication of an element by a scalar: If α ∈ IR and u ∈ V then αu ∈ V .That is, V is closed under scalar multiplication.(iii) These operations satisfy the properties (a) - (h) of Theorem 44.

Exercise 233Let F (IR) be the set of functions f : IR → IR. Define the operations

(f + g)(x) = f(x) + g(x)

and(αf)(x) = αf(x).

Show that F (IR) is a vector space under these operations.

Solution.The proof is based on the properties of the vector space IR.(a) (f + g)(x) = f(x) + g(x) = g(x) + f(x) = (g + f)(x) where we have usedthe fact that the addition of real numbers is commutative.(b) [(f + g)+h](x) = (f + g)(x)+h(x) = (f(x)+ g(x))+h(x) = f(x)+ (g(x)+h(x)) = f(x) + (g + h)(x) = [f + (g + h)](x).(c) Let 0 be the zero function. Then for any f ∈ F (IR) we have (f + 0)(x) =f(x) + 0(x) = f(x) = (0 + f)(x).(d) [f + (−f)](x) = f(x) + (−f(x)) = f(x)− f(x) = 0 = 0(x).(e) [α(f + g)](x) = α(f + g)(x) = αf(x) + αg(x) = (αf + αg)(x).(f) [(α + β)f ](x) = (α + β)f(x) = αf(x) + βf(x) = (αf + βf)(x).(g) [α(βf)](x) = α(βf)(x) = (αβ)f(x) = [(αβ)f ](x)(h)(1f)(x) = 1f(x) = f(x).Thus, F (IR) is a vector space

Exercise 234Let Mmn be the collection of all m × n matrices. Show that Mmn is a vectorspace using matrix addition and scalar multiplication.

Solution.This follows from Theorem 8

Exercise 235Let V = {(x, y) : x ≥ 0, y ≥ 0}. Show that the set V fails to be a vector spaceunder the standard operations on IR2.

Page 110: Fundamentals of Linear Algebra - Mathematics & Statisticsmelbial2/classes/Linear Algebra... · 2006-01-13 · 1.1 Systems of Linear Equations Many practical problems can be reduced

110 CHAPTER 4. THE THEORY OF VECTOR SPACES

Solution.For any (x, y) ∈ V with x, y > 0 we have −(x, y) 6∈ V. Thus, V is not a vectorspace

The following theorem exhibits some properties which follow directly from theaxioms of the definition of a vector space and therefore hold for every vectorspace.

Theorem 45Let V be a vector space, u a vector in V and α is a scalar. Then the followingproperties hold:

(a) 0u = 0.(b) α0 = 0(c) (−1)u = −u(d) If αu = 0 then α = 0 or u = 0.

Proof.(a) For any scalar α ∈ IR we have 0u = (α + (−α))u = αu + (−α)u =αu + (−(αu)) = 0.(b) Let u ∈ V. Then α0 = α(u + (−u)) = αu + α(−u) = αu + (−(αu)) = 0.(c) u + (−u) = u + (−1)u = 0. So that −u = (−1)u.(d) Suppose αu = 0. If α 6= 0 then α−1 exists and u = 1u = (α−1α)u =α−1(αu) = α−10 = 0.

Now, it is possible that a vector space in contained in a larger vector space.A subset W of a vector space V is called a subspace of V if the following twoproperties are satisfied:

(i) If u, v are in W then u + v is also in W.(ii) If α is a scalar and u is in W then αu is also in W.

Every vector space V has at least two subspaces:V itself and the subspaceconsisting of the zero vector of V. These are called the trivial subspaces of V.

Exercise 236Show that a subspace of a vector space is itself a vector space.

Solution.All the axioms of a vector space hold for the elements of a subspace

The following exercise provides a criterion for deciding whether a subset S of avector space V is a subspace of V.

Exercise 237Show that W is a subspace of V if and only if αu + v ∈ W for all u, v ∈ W andα ∈ IR.

Page 111: Fundamentals of Linear Algebra - Mathematics & Statisticsmelbial2/classes/Linear Algebra... · 2006-01-13 · 1.1 Systems of Linear Equations Many practical problems can be reduced

4.2. VECTOR SPACES, SUBSPACES, AND INNER PRODUCT SPACES111

Solution.Suppose that W is a subspace of V. If u, v ∈ W and α ∈ IR then αu ∈ W andtherefore αu + v ∈ W. Conversely, suppose that for all u, v ∈ W and α ∈ IRwe have αu + v ∈ IR. In particular, if α = 1 then u + v ∈ W. If v = 0 thenαu + v = αu ∈ W. Hence, W is a subspace

Exercise 238Let M22 be the collection of 2 × 2 matrices. Show that the set W of all 2 × 2matrices having zeros on the main diagonal is a subspace of M22.

Solution.The set W is the set

W ={(

0 ab 0

): a, b ∈ IR

}

Clearly, the 2× 2 zero matrix belongs to W. Also,

α

(0 ab 0

)+

(0 a′

b′ 0

)=

(0 αa + a′

αb + b′ 0

)∈ W

Thus, W is a subspace of M22

Exercise 239Let D([a, b]) be the collection of all differentiable functions on [a, b]. Show thatD([a, b]) is a subspace of the vector space of all functions defined on [a, b].

Solution.We know from calculus that if f, g are differentiable functions on [a, b] andα ∈ IR then αf + g is also differentiable on [a, b]. Hence, D([a, b]) is a subspaceof F ([a, b])

Exercise 240Let A be an m × n matrix. Show that the set S = {x ∈ IRn : Ax = 0} is asubspace of IRn.

Solution.See Exercise 67 (a)

Vector spaces are important in defining spaces with some kind of geometry.For example, the Euclidean space IRn. Such spaces are called inner productspaces, a notion that we discuss next.Recall that if ~u = (x1, y1, z1) and ~v = (x2, y2, z2) are two vectors in IR3 thenthe inner product of ~u and ~v is given by the formula

< ~u,~v >= x1x2 + y1y2 + z1z2.

We have seen that the inner product satisfies the following axioms:

Page 112: Fundamentals of Linear Algebra - Mathematics & Statisticsmelbial2/classes/Linear Algebra... · 2006-01-13 · 1.1 Systems of Linear Equations Many practical problems can be reduced

112 CHAPTER 4. THE THEORY OF VECTOR SPACES

(I1) < ~u,~v >=< ~v, ~u > (symmetry axiom)(I2) < ~u,~v + ~w >=< ~u,~v > + < ~u, ~w > (additivity axiom)(I3) < α~u,~v >= α < ~u,~v > (homogeneity axiom)(I4) < ~u, ~u >≥ 0 and < ~u, ~u >= 0 if and only if ~u = ~0.

We say that IR3 with the inner product operation is an inner product space. Ingeneral, a vector space with a binary operation that satisfies axioms (I1) - (I4)is called an inner product space.Some important properties of inner products are listed in the following theorem.

Theorem 46In an inner product space the following properties hold:

(a) < 0, u >=< u, 0 >= 0.(b) < u + v, w >=< u, w > + < v, w > .(c) < u,αv >= α < u, v > .

Proof.(a) < 0, u >=< u + (−u), u >=< u, u + (−u) >=< u, u > + < u,−u >=<u, u > + < −u, u >=< u, u > − < u, u >= 0. Similarly, < u, 0 >= 0.(b) < u + v, w >=< w, u + v >=< w, u > + < w, v >=< u,w > + < v, w > .(c) < u, αv >=< αv, u >= α < v, u >= α < u, v > .

We shall now prove a result that will enable us to give a definition for thecosine of an angle between two nonzero vectors in an inner product space. Thisresult has many applications in mathematics.

Theorem 47If u and v are vectors in an inner product space then

< u, v >2≤< u, u >< v, v > .

This is known as the Cauchy-Schwarz inequality.

Proof.If < u, u >= 0 or < v, v >= 0 then either u = 0 or v = 0. In this case,< u, v >= 0 and the inequality holds. So suppose that < u, u > 6= 0 and< v, v > 6= 0. Then < u

||u|| ,v||v|| >≤ 1. Since, || u

||u|| || = || v||v|| || = 1 then it suffices

to show that < u, v >≤ 1 for all u, v of norm 1.We have,

0 ≤ < u− v, u− v >= < u, u > + < v, v > −2 < u, v >

This implies that 2 < u, v >≤ ||u||2 + ||v||2 = 1+1 = 2. That is, < u, v >≤ 1.

Exercise 241 (Normed Vector Spaces)Let V be an inner product space. For u in V define ||u|| =< u, u >

12 . Show

that the norm function ||.|| satisfies the following axioms:

Page 113: Fundamentals of Linear Algebra - Mathematics & Statisticsmelbial2/classes/Linear Algebra... · 2006-01-13 · 1.1 Systems of Linear Equations Many practical problems can be reduced

4.2. VECTOR SPACES, SUBSPACES, AND INNER PRODUCT SPACES113

(a) ||u|| ≥ 0.(b) ||u|| = 0 if and only if u = 0.(c) ||αu|| = |α|||u||.(d) ||u + v|| ≤ ||u||+ ||v||. (Triangle Inequality)

Solution.(a) Follows from I4.(b) Follows from I4.(c) ||αu||2 =< αu, αu >= α2 < u, u > and therefore ||αu|| = |α| < u, u >

12 =

|α|||u||.(d) ||u + v||2 =< u + v, u + v >=< u, u > +2 < u, v > + < v, v >≤< u, u >

+2 < u, u >12 < v, v >

12 + < v, v >= ||u||2 + 2||u||||v|| + ||v||2 = (||u|| + ||v||)2.

Now take the square root of both sides

Exercise 242The purpose of this exercise is to provide a matrix formula for the dot productin IRn. Let

~u =

u1

u2

...un

, ~v =

v1

v2

...vn

be two vectors in IRn. Show that < ~u,~v >= ~vT ~u.

Solution.On one hand, we have < ~u,~v >= u1v1 + u2v2 + · · ·unvn. On the other hand,

~vT ~u =(

v1 v2 · · · vn

)

u1

u2

...un

= u1v1 + u2v2 + · · ·+ unvn

Exercise 243Show that < A, B >= tr(ABT ), where tr is the trace function, is an innerproduct in Mmn.

Solution.(a) < A,A >= sum of the squares of the entries of A and therefore is non-negative.(b) < A, A >= 0 if and only if

∑ni=1

∑nk=1 a2

ik = 0 and this is equivalent toA = 0.(c) < A,B >= tr(ABT ) = tr((ABT )T ) = tr(BAT ) =< B,A > .(d) < A,B+C >= tr(A(B+C)T ) = tr(ABT +ACT ) = tr(ABT )+tr(ACT ) =<A,B > + < A,C > .(e) < αA,B >= tr(αABT ) = αtr(ABT ) = α < A, B > . Thus, Mmn is an innerproduct space

Page 114: Fundamentals of Linear Algebra - Mathematics & Statisticsmelbial2/classes/Linear Algebra... · 2006-01-13 · 1.1 Systems of Linear Equations Many practical problems can be reduced

114 CHAPTER 4. THE THEORY OF VECTOR SPACES

Exercise 244Show that < p, q >= a0b0 + a1b1 + · · · + anbn defines an inner product in Pn,the vector space of all polynomials of degree n.

Solution.(a) < p, p >= a2

0 + a21 + · · ·+ a2

n ≥ 0.(b) < p, p >= 0 if and only if a2

0 + a21 + · · · + a2

n = 0 and this is equivalent toa0 = a1 = · · · = an = 0.(c) < p, q >= a0b0 + a1b1 + · · ·+ anbn = b0a0 + b1a1 + · · ·+ bnan =< q, p > .(d) < αp, q >= αa0b0 + αa1b1 + · · · + αanbn = α(a0b0 + a1b1 + · · · + anbn) =α < p, q > .(e) < p, q + r >= a0(b0 + c0) + a1(b1 + c1) + · · ·+ an(bn + cn) = a0b0 + a0c0 +a1b1 + a1c1 + · · ·+ anbn + ancn =< p, q > + < p, r > .Therefore, Pn is an inner product space

4.3 Linear Independence

The concepts of linear combination, spanning set, and basis for a vector spaceplay a major role in the investigation of the structure of any vector space. Inthis section we introduce and discuss the first two concepts and the third onewill be treated in the next section.The concept of linear combination will allow us to generate vector spaces froma given set of vectors in a vector space .Let V be a vector space and v1, v2, · · · , vn be vectors in V. A vector w ∈ V iscalled a linear combination of the vectors v1, v2, . . . , vn if it can be written inthe form

w = α1v1 + α2v2 + . . . + αnvn

for some scalars α1, α2, . . . , αn.

Exercise 245Show that the vector ~w = (9, 2, 7) is a linear combination of the vectors ~u =(1, 2,−1) and ~v = (6, 4, 2) whereas the vector ~w′ = (4,−1, 8) is not.

Solution.We must find numbers s and t such that

(9, 2, 7) = s(1, 2,−1) + t(6, 4, 2)

This leads to the system

s + 6t = 92s + 4t = 2−s + 2t = 7

Solving the first two equations one finds s = −3 and t = 2 both values satisfythe third equation.

Page 115: Fundamentals of Linear Algebra - Mathematics & Statisticsmelbial2/classes/Linear Algebra... · 2006-01-13 · 1.1 Systems of Linear Equations Many practical problems can be reduced

4.3. LINEAR INDEPENDENCE 115

Turning to (4,−1, 8), the question is whether s and t can be found such that(4,−1, 8) = s(1, 2,−1) + t(6, 4, 2). Equating components gives

s + 6t = 42s + 4t = −1−s + 2t = 8

Solving the first two equations one finds s = − 114 and t = 9

8 and these valuesdo not satisfy the third equation. That is the system is inconsistent

The process of forming linear combinations leads to a method of constructingsubspaces, as follows.

Theorem 48Let W = {v1, v2, . . . , vn} be a subset of a vector space V. Let span(W ) be the col-lection of all linear combinations of elements of W. Then span(W ) is a subspaceof V.

Proof.Let u, v ∈ W. Then there exist scalar α1, α2, · · · , αn and β1, β2, · · · , βn suchthat u = α1v1 + α2v2 + · · · + αnvn and v = β1v1 + β2v2 + · · ·βnvn. Thus,u + v = (α1 + β1)v1 + (α2 + β2)v2 + · · · + (αn + βn)vn ∈ span(W ) and αu =(αα1)v1 + (αα2)v2 + · · · + (ααn)vn ∈ Span(W ). Thus, span(W ) is a subspaceof V.

Exercise 246Show that Pn = span{1, x, x2, · · · , xn}.

Solution.If p(x) ∈ Pn then there are scalars a0, a1 · · · , an such that p(x) = a0 + a1x +· · ·+ anxn ∈ span{1, x, · · · , xn}

Exercise 247Show that IRn = span{e1, e2, · · · , en} where ei is the vector with 1 in the ithcomponent and 0 otherwise.

Solution.We must show that if u ∈ IRn then u is a linear combination of the e′is. Indeed,if u = (x1, x2, · · · , xn) ∈ IRn then

u = x1e1 + x2e2 + · · ·+ xnen

Hence u lies in span{e1, e2, · · · , en}

If every element of V can be written as a linear combination of elements ofW then we have V = span(W ) and in this case we say that W is a span of Vor W generates V.

Page 116: Fundamentals of Linear Algebra - Mathematics & Statisticsmelbial2/classes/Linear Algebra... · 2006-01-13 · 1.1 Systems of Linear Equations Many practical problems can be reduced

116 CHAPTER 4. THE THEORY OF VECTOR SPACES

Exercise 248(a)Determine whether ~v1 = (1, 1, 2), ~v2 = (1, 0, 1) and ~v3 = (2, 1, 3) span IR3.

(b) Show that the vectors ~i = (1, 0, 0),~j = (0, 1, 0), and ~k = (0, 0, 1) span IR3.

Solution.(a) We must show that an arbitrary vector ~v = (a, b, c) in IR3 is a linear com-bination of the vectors ~v1, ~v2, and ~v3. That is ~v = s~v1 + t ~v2 + w~v3. Expressingthis equation in terms of components gives

s + t + 2w = as + + w = b2s + t + 3w = c

The problem is reduced of showing that the above system is consistent. Thissystem will be consistent if and only if the coefficient matrix A

A =

1 1 21 0 12 1 3

is invertible. Since |A| = 0 then the system is inconsistent and thereforeIR3 6= span{~v1, ~v2, ~v3}.(b) See Exercise 247

As you have noticed by now, to show that n vectors in IRn span IRn it suf-fices to show that the matrix whose columns are the given vectors is invertible.

Exercise 249Show that P, the vector space of all polynomials, cannot be spanned by a finiteset of polynomials.

Solution.Suppose that P = span{1, x, x2, · · · , xn} for some positive integer n. But thenthe polynomial p(x) = 1 + x + x2 + · · · + xn + xn+1 is in P but not inspan{1, x, x2, · · · , xn}, a contradiction. Thus, P cannot be spanned by a finiteset of polynomials

Exercise 250Show that span{0, v1, v2, · · · , vn} = span{v1, v2, · · · , vn}.

Solution.If v ∈ span{0, v1, v2, · · · , vn} then v = α00 + α1v1 + · · · + αnvn = α1v1 +α2v2 + · · ·+αnvn ∈ span{v1, v2, · · · , vn}. Conversely, if v ∈ span{v1, v2, · · · , vn}then v = α1v1 + α2v2 + · · · + αnvn = 1(0) + α1v1 + α2v2 + · · · + αnvn ∈span{0, v1, v2, · · · , vn}

Next, we introduce a concept which guarantees that any vector in the spanof a set S has only one representation as a linear combination of vectors in S.

Page 117: Fundamentals of Linear Algebra - Mathematics & Statisticsmelbial2/classes/Linear Algebra... · 2006-01-13 · 1.1 Systems of Linear Equations Many practical problems can be reduced

4.3. LINEAR INDEPENDENCE 117

Spanning sets with this property play a fundamental role in the study of vectorspaces as we shall see in the next section.If v1, v2, . . . , vn are vectors in a vector space with the property that

α1v1 + α2v2 + . . . + αnvn = 0

holds only for α1 = α2 = . . . = αn = 0 then the vectors are said to be linearlyindependent. If there are scalars not all 0 such that the above equation holdsthen the vectors are called linearly dependent.

Exercise 251Show that the set S = {1, x, x2, · · · , xn} is a linearly independent set in Pn.

Solution.Suppose that a0+a1x+a2x

2+· · ·+anxn = 0 for all x ∈ IR. By the FundamentalTheorem of Algebra, a polynomial of degree n has at most n roots. But by theabove equation, every real number is a root of the equation. This forces thenumbers a0, a1, · · · , an to be 0

Exercise 252Let u be a nonzero vector. Show that {u} is linearly independent.

Solution.Suppose that αu = 0. If α 6= 0 then we can multiply both sides by α−1 andobtain u = 0. But this contradicts the fact that u is a nonzero vector

Exercise 253(a) Show that the vectors ~v1 = (1, 0, 1, 2), ~v2 = (0, 1, 1, 2), and ~v3 = (1, 1, 1, 3)are linearly independent.(b) Show that the vectors ~v1 = (1, 2,−1), ~v2 = (1, 2,−1), and ~v3 = (1,−2, 1) arelinearly dependent.

Solution.(a) Suppose that s, t, and w are real numbers such that s~v1 = t ~v2 + w~v3 = 0.Then equating components gives

s + w = 0t + w = 0

s + t + w = 02s + 2t + 3w = 0

The second and third equation leads to s = 0. The first equation gives w = 0 andthe second equation gives t = 0. Thus, the given vectors are linearly independent.(b) These vectors are linearly dependent since ~v1 + ~v2 − 2~v3 = 0

Exercise 254Show that the polynomials p1(x) = 1 − x, p2(x) = 5 + 3x − 2x2, and p3(x) =1 + 3x− x2 are linearly dependent vectors in P2.

Page 118: Fundamentals of Linear Algebra - Mathematics & Statisticsmelbial2/classes/Linear Algebra... · 2006-01-13 · 1.1 Systems of Linear Equations Many practical problems can be reduced

118 CHAPTER 4. THE THEORY OF VECTOR SPACES

Solution.Indeed, 3p1(x)− p2(x) + 2p3(x) = 0

Exercise 255Show that the unit vectors e1, e2, · · · , en in IRn are linearly independent.

Solution.Suppose that x1e1 + x2e2 + · · · + xnen = (0, 0, · · · , 0). Then (x1, x2, · · · , xn) =(0, 0, · · · , 0) and this leads to x1 = x2 = · · · = xn = 0. Hence the vectorse1, e2, · · · , en are linearly independent

The following theorem provides us with a criterion for deciding whether a setof vectors is linearly dependent.

Theorem 49The vectors v1, v2, . . . , vn are linearly dependent if and only if there is at leastone vector that is a linear combination of the remaining vectors.

Proof.Suppose that v1, v2, · · · , vn are linearly dependent then there exist scalars α1, α2,· · · , αn not all zeros such that α1v1+α2v2+· · ·+αnvn = 0. Suppose that αi 6= 0.Dividing through by this scalar we get vi = −α1

αiv1−· · ·− αi−1

αivi−1− αi+1

αivi+1−

αn

αivn.

Conversely, suppose that one of the vectors v1, v2, · · · , vn is a linear combinationof the remaining vectors. Say, vi = α1v1+ · · ·+αi−1vi−1+αi+1vi+1+ · · ·+αnvn.Then α1v1 + · · ·+ αi−1vi−1 + (−1)vi + · · ·+ αnvn = 0 with the coefficient of vi

being −1. Thus, the vectors v1, v2, · · · , vn are linearly dependent. This ends aproof of the theorem

Exercise 256Let S = {v1, v2, . . . , vn} be a set of vectors in a vector space V. Show that if oneof the vectors is zero, then the set is linearly dependent.

Solution.Suppose for the sake of argument that v1 = 0. Then 1v1 + 0v2 + · · ·+ 0vn = 0with coefficients of v1, v2, · · · , vn not all 0. Thus, the set {v1, v2, · · · , vn} is lin-early dependent

A criterion for linear independence is established in the following theorem.

Theorem 50Nonzero vectors v1, v2, · · · , vn are linearly independent if and only if one of them,say vi, is a linear combination of the preceding vectors:

vi = α1v1 + α2v2 + · · ·+ αi−1vi−1. (4.1)

Page 119: Fundamentals of Linear Algebra - Mathematics & Statisticsmelbial2/classes/Linear Algebra... · 2006-01-13 · 1.1 Systems of Linear Equations Many practical problems can be reduced

4.3. LINEAR INDEPENDENCE 119

Proof.Suppose that (4.1) holds. Then α1v1 +α2v2 + · · ·+αi−1vi−1 +(−1)vi +0vi+1 +· · ·+ 0vn = 0. Thus, v1, v2, · · · , vn are linearly dependent.Conversely, suppose that v1, v2, · · · , vn are linearly dependent. Then there ex-ist scalars not all zero such that α1v1 + α2v2 + · · · + αnvn = 0. Let i be thelargest index such that αi 6= 0. Then, α1v1 + α2v2 + · · · + αivi = 0. Thus,vi = −∑i−1

k=1αk

αivk

We conclude this section with a theorem which will found to be very usefulin later sections.

Theorem 51Let S1 and S2 be finite subsets of a vector space V such that S1 is a subset of S2.

(a) If S1 is linearly dependent then so is S2.(b) If S2 is linearly independent then so is S1.

Proof.Without loss of generality we may assume that S1 = {v1, v2, · · · , vn} and S2 ={v1, v2, · · · , vn, vn+1}.(a) Suppose that S1 is a linearly dependent set. We want to show that S2 is alsolinearly dependent. By the assumption on S1 there exist scalars α1, α2, · · · , αn

not all 0 such thatα1v1 + α2v2 + · · ·+ αnvn = 0.

But this implies that

α1v1 + α2v2 + · · ·+ αnvn + 0vn+1 = 0

with αi 6= 0 for some i. That is, S2 is linearly dependent.(b) Now, suppose that S2 is a linearly independent set. We want to show thatS1 is also linearly independent. Indeed, if

α1v1 + α2v2 + · · ·+ αnvn+ = 0

thenα1v1 + α2v2 + · · ·+ αnvn + 0vn+1 = 0.

Since S2 is linearly independent then α1 = α2 = · · · = αn = 0. But this showsthat S1 is linearly independent.

Exercise 257Let A be an n×m matrix in reduced row-echelon form. Prove that the nonzerorows of A, viewed as vectors in IRm, form a linearly independent set of vectors.

Solution.Let S1 = {v1, v2, · · · , vk} be the set of nonzero rows of a reduced row-echelonn × m matrix A. Then S1 ⊂ {e1, e2 · · · , em}. Since the set {e1, e2, · · · , em} islinearly independent then by Theorem 51 (b), S1 is linearly independent

Page 120: Fundamentals of Linear Algebra - Mathematics & Statisticsmelbial2/classes/Linear Algebra... · 2006-01-13 · 1.1 Systems of Linear Equations Many practical problems can be reduced

120 CHAPTER 4. THE THEORY OF VECTOR SPACES

4.4 Basis and Dimension

In this section we continue the study of the structure of a vector space by de-termining a set of vectors that completely describes the vector space.Let S = {v1, v2, . . . , vn} be a subset of a vector space V . We say that S is abasis for V if

(i) S is linearly independent set.(ii) V = span(S).

Exercise 258Let ei be the vector of IRn whose ith component is 1 and zero otherwise. Showthat the set S = {e1, e2, . . . , en} is a basis for IRn. This is called the standardbasis of IRn.

Solution.By Exercise 247, we have IRn = span{e1, e2, · · · , en}. By Exercise 255, thevectors e1, e2, · · · , en are linearly independent. Thus {e1, e2, · · · , en} is a basisof IR3

Exercise 259Show that {1, x, x2, · · · , xn} is a basis of Pn.

Solution.By Exercise 246, Pn = span{1, x, x2, · · · , xn} and by Exercise 251, the setS = {1, x, x2, · · · , xn} is linearly independent. Thus, S is a basis of Pn

If S = {v1, v2, . . . , vn} is a basis for V then we say that V is a finite dimen-sional space of dimension n. We write dim(V ) = n. A vector space which is notfinite dimensional is said to be infinite dimensional vector space. We definethe zero vector space to have dimension zero. The vector spaces Mmn, IRn, andPn are finite-dimensional spaces whereas the space P of all polynomials and thevector space of all real-valued functions defined on IR are inifinite dimensionalvector spaces.Unless otherwise specified, the term vector space shall always mean a finite-dimensional vector space.

Exercise 260Determine a basis and the dimension for the solution space of the homogeneoussystem

2x1 + 2x2 − x3 + + x5 = 0−x1 − x2 + 2x3 − 3x4 + x5 = 0x1 + x2 − 2x3 − x5 = 0

x3 + x4 + x5 = 0

Page 121: Fundamentals of Linear Algebra - Mathematics & Statisticsmelbial2/classes/Linear Algebra... · 2006-01-13 · 1.1 Systems of Linear Equations Many practical problems can be reduced

4.4. BASIS AND DIMENSION 121

Solution.By Exercise 38, we found that x1 = −s − t, x2 = s, x3 = −t, x4 = 0, x5 = t.So if S is the vector space of the solutions to the given system then S ={(−s−t, s,−t, 0, t) : s, t ∈ IR} = {s(−1, 1, 0, 0, 0)+t(−1, 0,−1, 0, 1) : s, t ∈ IR} =span{(−1, 1, 0, 0, 0), (−1, 0,−1, 0, 1)}. Moreover, if s(−1, 1, 0, 0, 0)+t(−1, 0,−1, 0, 1) =(0, 0, 0, 0, 0) then s = t = 0. Thus the set {(−1, 1, 0, 0, 0), (−1, 0,−1, 0, 1)} is abasis for the solution space of the homogeneous system

The following theorem will indicate the importance of the concept of a basisin investigating the structure of vector spaces. In fact, a basis for a vector spaceV determines the representation of each vector in V in terms of the vectors inthat basis.

Theorem 52If S = {v1, v2, . . . , vn} is a basis for V then any element of V can be written inone and only one way as a linear combination of the vectors in S.

Proof.Suppose v ∈ V has the following two representations v = α1v1+α2v2+· · ·+αnvn

and v = β1v1 + β2v2 + · · ·+ βnvn. We want to show that αi = βi for 1 ≤ i ≤ n.But this follows from (α1−β1)v1 + (α2−β2)v2 + · · ·+ (αn−βn)vn = 0 and thefact that S is a linearly independent set.

We wish to emphasize that if a spanning set for a vector space is not a ba-sis then a vector may have different representations in terms of the vectors inthe spanning set. For example let

S ={(

12

),

(21

),

(13

)}

then the vector (11

)

has the following two different representations(

11

)=

13

(12

)+

13

(21

)+ 0

(13

)

and (11

)= −4

3

(12

)+

23

(21

)+

(13

)

The following theorem indicates how large a linearly independent set can be ina finite-dimensional vector space.

Theorem 53If S = {v1, v2, . . . , vn} is a basis for a vector space V then every set with morethan n vectors of V is linearly dependent.

Page 122: Fundamentals of Linear Algebra - Mathematics & Statisticsmelbial2/classes/Linear Algebra... · 2006-01-13 · 1.1 Systems of Linear Equations Many practical problems can be reduced

122 CHAPTER 4. THE THEORY OF VECTOR SPACES

Proof.Let S′ = {w1, w2, . . . , wm} be any subset of V with m > n vectors. Since S isa basis then we can write

wi = ai1v1 + ai2v2 + . . . + ainvn,

where 1 ≤ i ≤ m. Now, suppose that

α1w1 + α2w2 + . . . + αmwm = 0.

Then this implies

(α1a11 + α2a21 + · · ·+ αmam1)v1 + (α1a12 + α2a22 + · · ·+ αmam2)v2

+ · · ·+ (α1a1n + α2a2n + . . . + αmamn)vn = 0.

Since S is linearly independent then we have

α1a11 + α2a21 + . . . + αmam1 = 0α1a12 + α2a22 + . . . + αmam2 = 0

...α1a1n + α2a2n + . . . + αmamn = 0

But the above homogeneous system has more unknowns then equations andtherefore it has nontrivial solutions (Theorem 6). Thus, S′ is linearly depen-dent

An immediate consequence of the above theorem is the following

Theorem 54Suppose that V is a vector space of dimension n. If S is a subset of V of mlinearly independent vectors then m ≤ n.

Proof.If not, i.e. m > n then by Theorem 53 S is linearly dependent which is a con-tradiction.

We wish to emphasize here that a basis (when it exists) needs not be unique.

Exercise 261Show that

S ={(

1−1

),

(32

)}

is a basis of IR2.

Solution.Suppose that

s

(1−1

)+ t

(32

)=

(00

)

Page 123: Fundamentals of Linear Algebra - Mathematics & Statisticsmelbial2/classes/Linear Algebra... · 2006-01-13 · 1.1 Systems of Linear Equations Many practical problems can be reduced

4.4. BASIS AND DIMENSION 123

Then this leads to the equations s+3t = 0 and −s+2t = 0. Solving this systemwe find s = t = 0. Thus, we have shown that S is linearly independent. Next, weshow that every vector in IR2 lies in the span of S. Indeed, if v = (a, b)T ∈ IR2

we must show that there exist scalars s and t such that

s

(1−1

)+ t

(32

)=

(ab

)

That is the coefficient matrix of this system

A =(

1 3−1 2

)

is invertible. Since |A| = 5 then the given system is consistent. This shows thatIR2 = spanS. Hence, S is a basis of IR2. Thus, we have found a basis for IR2

which is different than the standard basis

A vector space can have different bases; however, all of them have the samenumber of elements as indicated by the following theorem.

Theorem 55Let V be a finite dimensionsl space. If S = {v1, v2, · · · , vn} and S′ = {w1, · · · , wm}are two bases of V then n = m. That is, any two bases for a finite dimensionalvector space have the same number of vectors.

Proof.Since S is a basis of V and S′ is a linearly independent susbet of V then byTheorem 53 we must have m ≤ n. Now interchange the roles of S and S′ toobtain n ≤ m. Hence, m = n

Next, we consider the following problem: If S is a span of a finite dimensionalvector space then is it possible to find a subset of S that is a basis of V ? Theanswer to this problem is given by the following theorem.

Theorem 56If S = {v1, v2, · · · , vn} spans a vector space V then S contains a basis of V anddim(V ) ≤ n.

Proof.Either S is linearly independent or linearly dependent. If S is linearly indepen-dent then we are finished. So assume that S is linearly dependent. Then there isa vector vi which is a linear combination of the preceding vectors in S (Theorem50). Thus, V = spanS = spanS1 where S1 = {v1, v2, · · · , vi−1, vi+1, · · · , vn}.Now we repeat the argument. If S1 is linearly independent then S1 is a basis ofV. Otherwise delete a vector of S1 which is a linear combination of the precedingvectors and obtain a subset S2 of S1 that spans V. Continue this process. If abasis is encountered at some stage, we are finished. If not, we ultimately reachV = span{vl} for some l. Since {vl} is also linearly independent then {vl} is abasis of V.

Page 124: Fundamentals of Linear Algebra - Mathematics & Statisticsmelbial2/classes/Linear Algebra... · 2006-01-13 · 1.1 Systems of Linear Equations Many practical problems can be reduced

124 CHAPTER 4. THE THEORY OF VECTOR SPACES

Exercise 262Let S = {(1, 2), (2, 4), (2, 1), (3, 3), (4, 5)}

(a) Show that IR2 = spanS.(b) Find a subset S′ of S such that S′ is a basis of IR2.

Solution.(a) Note that (2, 4) = (1, 2) and (3, 3) = (2, 1)+(1, 2) and (4, 5) = (2, 1)+2(1, 2).Thus, spanS = span{(1, 2), (2, 1)}. Now, for any vector (x, y) ∈ IR2 we mustshow the existence of two numbers a and b such that (x, y) = a(1, 2) + b(2, 1).This is equivalent to showing that the system

{a + 2b = x2a + b = y

is consistent. The coefficient matrix of the system is

A =(

1 22 1

)

This matrix is invertible since |A| = −3. Thus the system is consistent andtherefore IR2 = spanS.(b) S′ = {(1, 2), (2, 1)}

We now prove a theorem that we shall have occasion to use several times inconstructing a basis containing a given set of linearly independent vectors.

Theorem 57If S = {v1, v2, . . . , vr} is a set of r linearly independent vectors in a n-dimensionalvector space V and r < n then there exist n − r vectors vr+1, · · · , vn such thatthe enlarged set S′ = S ∪ {vr+1, · · · , vn} a basis for V.

Proof.Since V is of dimension n and S is a linearly independent set then r ≤ n by The-orem 57. If span(S) = V then S is a basis of V and by Theorem 55, r = n. Sosuppose span(S) 6= V, i.e. r < n. Then,, there is a vector vr+1 ∈ V which is nota linear combination of the vectors of S. In this case, the set {v1, v2, · · · , vr, vr+1}is linearly independent. As long as span{v1, v2, · · · , vr, vr+1} 6= V we continuethe expansion as above. But the number of linearly independent vectors in theexpansion of S can not exceed n according to Theorem 58. That is, the processwill terminate in a finite number of steps

It follows from the above theorem that a finite-dimensional nonzero vector spacealways has a finite basis.

Exercise 263Construct a basis for IR3 that contains the vector (1, 2, 3).

Page 125: Fundamentals of Linear Algebra - Mathematics & Statisticsmelbial2/classes/Linear Algebra... · 2006-01-13 · 1.1 Systems of Linear Equations Many practical problems can be reduced

4.4. BASIS AND DIMENSION 125

Solution.Since the vector (1, 0, 0) is not a multiple of the vector (1, 2, 3) then the set{(1, 2, 3), (1, 0, 0)} is linearly independent. Since dim(IR3) = 3 then this set isnot a basis of IR3 and therefore cannot span IR3. Next, we see that the vector(0, 1, 0) is not a linear combination of {(1, 2, 3), (1, 0, 0)}. That is, (0, 1, 0) =s(1, 2, 3) + t(1, 0, 0) for some scalars s, t ∈ IR. This leads to the system

s + t = 02s = 13s = 0

Because of the last two equations this system is inconsistent. Thus the setS′ = {(1, 2, 3), (1, 0, 0), (0, 1, 0)} is linearly independent. By Theorem 58 below,S′ is a basis of IR3

In general, to show that a set of vectors S is a basis for a vector space Vwe must show that S spans V and S is linearly independent. However, if wehappen to know that V has dimension equals to the number of elements in Sthen it suffices to check that either linear independence or spanning- the re-maining condition will hold automatically. This is the content of the followingtwo theorems.

Theorem 58If S = {v1, v2, . . . , vn} is a set of n linearly independent vectors in a n-dimensionalvector space V, then S is a basis for V.

Proof.Since S is a linearly independent set then by Theorem 57 we can extend S to abasis W of V. By Theorem 55, W must have n vectors. Thus, S = W and so Sis a basis of V.

Theorem 59If S = {v1, v2, . . . , vn} is a set of n vectors that spans an n-dimensional vectorspace V, then S is a basis for V.

Proof.Suppose that V is of dimension n and S is a span of V. If S is linearly independentthen S is a basis and we are done. So, suppose that S is linearly dependent.Then by Theorem 49 there is a vector in S which can be written as a linearcombination of the remaining vectors. By rearrangement, we can assume thatvn is a linear combination of v1, v2, · · · , vn−1. Hence, span{v1, v2, · · · , vn} =span{v1, v2, · · · , vn−1}. Now, we repeat this process until the spanning set islinearly independent. But in this case the basis will have a number of elements< n and this contradicts Theorem 55

Exercise 264Show that if V is a finite dimensional vector space and W is a subspace of Vthen dim(W ) ≤ dim(V ).

Page 126: Fundamentals of Linear Algebra - Mathematics & Statisticsmelbial2/classes/Linear Algebra... · 2006-01-13 · 1.1 Systems of Linear Equations Many practical problems can be reduced

126 CHAPTER 4. THE THEORY OF VECTOR SPACES

Solution.If W = {0} then dim(W ) = 0 ≤ dim(V ). So suppose that W 6= {0} and let 0 6=u1 ∈ W. If W = span{u1} then dim(W ) = 1 If W 6= span{u1} then there existsa vector u2 ∈ W and u 6∈ span{u1}. In this case, {u1, u2} is linearly independent.If W = span{u1, u2} then dim(W ) = 2. If not, repeat the process to find u3 ∈ Wand u3 6∈ span{u1, u2}. Then {u1, u2, u3} is linearly independent. Continue inthis way. The process must terminate because the vector space cannot havemore than dim(V ) linearly independent vectors. Hence W has a basis of atmost dim(V ) vectors

Exercise 265Show that if W is a subspace of a finite dimensional vector space V and dim(W ) =dim(V ) then W = V.

Solution.Suppose dim(W ) = dim(V ) = m. Then any basis {u1, w2, · · · , um} of W is anindependent set of m vectors in V and so is a basis of V by Theorem 59. Inparticular V = span{u1, u2, · · · , um} = W

Next, we discuss an example of an infinite dimensional vector space. We havestated previously that the vector space F (IR) is an infinite dimensional vectorspace. We are now in a position to prove this statement. We must show thatevery finite subset of F (IR) fails to be a spanning set of F (IR). We prove thisby contradiction.Suppose that S = {f1, f2, · · · , fn} spans F (IR). Without loss of generality wemay assume that 0 6∈ S. By Theorem 56 there exists a subset S′ of S such thatS′ is a basis of F (IR). Hence, dim(S′) ≤ n. The set {1, x, · · · , xn} is containedin F (IR) and is linearly independent and contains n + 1 elements. This violatesTheorem 53.

In the remainder of this section we consider inner product spaces.A set of vectors {v1, v2, . . . , vn} of an inner product space is said to be or-thonormal if the following two conditions are met

(i) < vi, vj >= 0, for i 6= j.(ii) < vi, vi >= 1 for 1 ≤ i ≤ n.

Exercise 266(a) Show that the vectors ~v1 = (0, 1, 0), ~v2 = ( 1√

2, 0, 1√

2), ~v3 = ( 1√

2, 0,− 1√

2) form

an orthonormal set in IR3 with the Euclidean inner product.(b) If S = {v1, v2, . . . , vn} is a basis of an inner product space and S is orthonor-mal then S is called an orthonormal basis. Show that if S is an orthonormalbasis and u is any vector then

u =< u, v1 > v1+ < u, v2 > v2 + . . . + < u, vn > vn.

(c) Show that if S = {v1, v2, . . . , vn} is an orthogonal set of nonzero vectors inan inner product space then S is linearly independent.

Page 127: Fundamentals of Linear Algebra - Mathematics & Statisticsmelbial2/classes/Linear Algebra... · 2006-01-13 · 1.1 Systems of Linear Equations Many practical problems can be reduced

4.4. BASIS AND DIMENSION 127

(d) Use part (b) to show that if S = {v1, v2, · · · , vn} is an orthonormal basis ofan inner product space V then every vector u can be written in the form

u = w1 + w2

where w1 belongs to the span of S and w2 is orthogonal to the span of S. w1

is called the orthogonal projection of u on the span of S and is denoted byprojspan(S)u = w1.

Solution.(a) It is easy to check that < ~v1, ~v1 >=< ~v2, ~v2 >=< ~v3, ~v3 >= 0 and <~v1, ~v2 >=< ~v1, ~v3 >=< ~v2, ~v3 >= 0. Thus, {~v1, ~v2, ~v3} is an orthonormal set.(b) Since S is a basis of an inner product V. For u ∈ V we have u = α1v1 +α2v2 + · · ·+ αnvn. For 1 ≤ i ≤ n we have < u, vi >= αi since S is orthonormal.(c) We must show that if α1v1+α2v2+· · ·+αnvn = 0 then α1 = α2 = · · · = αn =0. Then for 1 ≤ i ≤ n we have 0 =< vi, 0 >=< vi, α1v1 + α2v2 + · · ·+ αnvn >=α1 < v1, vi > + · · · + αi−1 < vi, vi−1 > +αi < vi, vi > + · · · + αn < vi, vn >=αi||vi||2. Since vi 6= 0 then ||vi|| > 0. Hence. αi = 0.(d) Indeed, if u ∈ V then u = w1 + w2 where w1 =< u, v1 > v1+ < u, v2 >v2 + · · ·+ < u, vn > vn ∈ span(S) and w2 = u − w1. Moreover, < w2, vi >=<u − w1, vi >=< u, vi > − < w1, vi >=< u, vi > − < u, vi >= 0, 1 ≤ i ≤ n.Thus w2 is orthogonal to span(S)

Exercise 267 (Gram-Schmidt)The purpose of this exercise is to show that every nonzero finite dimensionalinner product space with basis {u1, u2, . . . , un} has an orthonormal basis.

(a) Let v1 = u1||u1|| . Show that ||v1|| = 1.

(b) Let W1 = span{v1}. Recall that projW1u2 =< u2, v1 > v1. Let v2 =u2−<u2,v1>v1||u2−<u2,v1>v1|| . Show that ||v2|| = 1 and < v1, v2 >= 0.

(c) Let W2 = span{v1, v2} and projW2u3 =< u3, v1 > v1+ < u3, v2 > v2.Let

v3 =u3− < u3, v1 > v1− < u3, v2 > v2

||u3− < u3, v1 > v1− < u3, v2 > v2|| .

Show that ||v3|| = 1 and < v1, v3 >=< v2, v3 >= 0.

Solution.(a) || u1

||u1|| ||2 =< u1||u1|| ,

u1||u1|| >= <u1,u1>

||u1||2 = ||u1||2||u1||2 = 1.

(b) The proof that ||v2|| = 1 is similar to (a). On the other hand, < v1, v2 >=<<

u2, v1 > v1,u2−<u2,v1>v1||u2−<u2,v1>v1|| >= <u2,v1>2

u2−<u2,v1>v1− <u2,v1>2

u2−<u2,v1>v1= 0.

(c) Similar to (c)

Continuing the process of the above exercise we obtain an orthonormal set of

Page 128: Fundamentals of Linear Algebra - Mathematics & Statisticsmelbial2/classes/Linear Algebra... · 2006-01-13 · 1.1 Systems of Linear Equations Many practical problems can be reduced

128 CHAPTER 4. THE THEORY OF VECTOR SPACES

vectors {v1, v2, . . . , vn}. By the previous exercise this set is linearly independent.By Theorem 58, {v1, v2, . . . , vn} is a basis of V and consequently an othonormalbasis.

4.5 Transition Matrices and Change of Basis

In this section we introduce the concept of coordinate vector in a given basisand then find the relationship between two coordinate vectors of the same vectorwith respect to two different bases.From Theorem 52, it follows that if S = {v1, v2, . . . , vn} is a basis for a vectorspace V and u is a vector in V then u can be written uniquely in the form

u = αv1 + α2v2 + . . . + αnvn.

The scalars α1, α2, . . . , αn are called the coordinates of u relative to thebasis S and we call the matrix

[u]S =

α1

α2

...αn

the coordinate matrix of u relative to S.

Exercise 268Let u and v be two vectors in V and S = {v1, v2, · · · , vn} be a basis of V. Showthat(a) [u + v]S = [u]S + [v]S .(b) [αu]S = α[u]S , where α is a scalar.

Solution.(a) Let u = α1v1 + α2v2 + · · ·+ αnvn and v = β1v1 + β2v2 + · · ·+ βnvn. Thenu + v = (α1 + β1)v1 + (α2 + β2)v2 + · · ·+ (αn + βn)vn. It follows that

[u + v]S =

α1 + β1

α2 + β2

...αn + βn

=

α1

α2

...αn

+

β1

β2

...βn

= [u]S + [v]S

(b) Since αu = αα1v1 + αα2v2 + · · ·+ ααnvn then

[αu]S =

αα1

αα2

...ααn

= α

α1

α2

...αn

= α[u]S

Exercise 269Let S = {v1, v2, · · · , vn} be a basis of V. Define the function T : V −→ IRn given

Page 129: Fundamentals of Linear Algebra - Mathematics & Statisticsmelbial2/classes/Linear Algebra... · 2006-01-13 · 1.1 Systems of Linear Equations Many practical problems can be reduced

4.5. TRANSITION MATRICES AND CHANGE OF BASIS 129

by Tv = [v]S .(a) Show that T (αu + v) = αTu + Tv.(b) Show that if Tu = Tv then u = v.(c) Show that for any w ∈ IRn there exists a v ∈ V such that Tv = w.

Solution.(a) T (αu + v) = [αu + v]S = [αu]S + [v]S = α[u]S + [v]S = αTu + Tv.(b) Suppose that u = α1v1 +α2v2 + · · ·+αnvn and v = β1v1 +β2v2 + · · ·+βnvn.If Tu = Tv then αi = βi for 1 ≤ i ≤ n. Hence, u = v.(c) If w ∈ IRn then w = (α1, α2, · · · , αn). Let u = α1v1 + α2v2 + · · ·+ αnvn ∈ Vand Tu = w

If S′ = {u1, u2, . . . , un} is another basis for V then we can write

u1 = c11v1 + c12v2 + . . . c1nvn

u2 = c21v1 + c22v2 + . . . c2nvn

......

un = cn1v1 + cn2v2 + . . . cnnvn.

The matrix

Q =

c11 c21 . . . cn1

c12 c22 . . . cn2

......

c1n c2n . . . cnn

is called the transition matrix from S′ to S. Note that the jth column of Q isjust [uj ]S .

Exercise 270Let ~v1 = (0, 1, 0)T , ~v2 = (− 4

5 , 0, 35 )T , ~v3 = ( 3

5 , 0, 45 )T .

(a) Show that S = {~v1, ~v2, ~v3} is a basis of IR3.(b) Find the coordinate matrix of the vector ~u = (1, 1, 1)T relative to the basisS.

Solution.By Theorem 58, it suffices to show that S is linearly independent. This canbe accomplished by showing that the determinant of the matrix with columns~v1, ~v2, ~v3 is invertible. Indeed,

∣∣∣∣∣∣

0 − 45

35

1 0 00 3

545

∣∣∣∣∣∣= 1

(b) We have ~u = ~v1 − 15 ~v2 + 7

5 ~v3. Therefore,

[~u]S =

1− 1

575

Page 130: Fundamentals of Linear Algebra - Mathematics & Statisticsmelbial2/classes/Linear Algebra... · 2006-01-13 · 1.1 Systems of Linear Equations Many practical problems can be reduced

130 CHAPTER 4. THE THEORY OF VECTOR SPACES

Exercise 271Consider the following two bases of P2: S = {x− 1, x + 1} and S′ = {x, x− 2}.Find the transition matrix from S to S′.

Solution.Writing the elements of S in terms of the elements of S′ we find x − 1 =x2 + 1

2 (x − 2) and x + 1 = 32x − 1

2 (x − 2). Hence, the transition matrix from Sto S′ is the matrix

P =(

12

12

32 − 1

2

)

The next theorem provides a relationship between the coordinate matrices of avector with respect to two different bases.

Theorem 60With the above notation we have [u]S = Q[u]S′

Proof.Suppose

[u]S′ =

β1

β2

...βn

That is, u = β1u1 +β2u2 + · · ·+βnun. Now, writting the ui in terms of the vi wefind that u = (β1c11 +β2c21 + · · ·+βncn1)v1 +(β1c12 +β2c22 + · · ·+βncn2)v2 +· · ·+ (β1c1n + β2c2n + · · · cnn)vn. That is, [u]S = Q[u]S′ .

Exercise 272Consider the vectors ~v1 = (1, 0)T , ~v2 = (0, 1)T , ~u1 = (1, 1)T , and ~u2 = (2, 1)T .(a) Show that S = {~v1, ~v2} and S′ = { ~u1, ~u2} are bases of IR2.(b) Find the transition matrix Q from S′ to S.(c) Find [~v]S given that

[~v]S′ =( −3

5

)

Solution.(a) Since dim(IR2) = 2 then it suffices to show that S and S′ are linearlyindependent. Indeed, finding the determinant of the matrix with columns ~v1

and ~v2 we get ∣∣∣∣1 00 1

∣∣∣∣ = 1

Thus, S is linearly independent. Similar proof for S′.(b) It is easy to check that ~u1 = ~v1 + ~v2 and ~u2 = 2~v1 + ~v2. Thus, the transitionmatrix Q from S′ to S is the matrix

Q =(

1 12 1

)

Page 131: Fundamentals of Linear Algebra - Mathematics & Statisticsmelbial2/classes/Linear Algebra... · 2006-01-13 · 1.1 Systems of Linear Equations Many practical problems can be reduced

4.5. TRANSITION MATRICES AND CHANGE OF BASIS 131

(c) By Theorem 60 we have [~v]S = Q[~v]S′ =(

1 12 1

)( −35

)=

(72

)

We next show that Q is nonsingular and its inverse is the transition P ma-trix from S to S′.

Theorem 61The matrix Q is invertible and its inverse is the transition matrix from S to S′.

Proof.Let P be the transition matrix from S to S′. Then by Theorem 60 we have[u]S′ = P [u]S . Since [u]S = Q[u]S′ then [u]S = QP [u]S . Let x ∈ IRn. Thenx = [u]S for some vector u ∈ V. Thus, QPx = x, i.e. (QP − In)x = 0. Since thisis true for arbitrary x then by Exercise 84 we must have QP = In. By Theorem20, Q is invertible and Q−1 = P.

Exercise 273

Consider the two bases S = {~v1, ~v2} and S′ = { ~u1, ~u2} where ~v1 =(

10

), ~v2 =

(01

), ~u1 =

(11

), and ~u2 =

(21

).

(a) Find the transition matrix P from S to S′.(b) Find the transition matrix Q from S′ to S.

Solution.(a) Expressing the vectors of S in terms of the vectors of S′ we find ~v1 = − ~u1+ ~u2

and ~v2 = 2 ~u1 − ~u2. Thus the transition matrix from S to S′ is the matrix

P =( −1 2

2 −1

)

(b) We have already found the matrix Q in the previous exercise

The next theorem shows that the transition matrix from one orthonormal basisto another has the property PT = P−1. That is, PPT = PT P = In. In thiscase we call P an orthogonal matrix.

Theorem 62If P is the transition matrix from one orthonormal basis S to another orthonor-mal basis S′ for an inner product space V then P−1 = PT .

Proof.Suppose P = (bij) is the transition matrix from the orthonormal basis S ={u1, u2, · · · , un} to the orthonormal basis S′ = {v1, v2, · · · , vn}. We will showthat PPT = In. Indeed, for 1 ≤ i ≤ n we have

vi = bi1u1 + bi2u2 + · · ·+ binun.

Page 132: Fundamentals of Linear Algebra - Mathematics & Statisticsmelbial2/classes/Linear Algebra... · 2006-01-13 · 1.1 Systems of Linear Equations Many practical problems can be reduced

132 CHAPTER 4. THE THEORY OF VECTOR SPACES

Thus, < vi, vj >=< bi1u1 + bi2u2 + · · ·+ binun, bj1u1 + bj2u2 + · · ·+ bjnun >=bi1bj1 + bi2bj2 + · · ·+ binbjn since S is an orthonormal set. Thus, < vi, vj >=<ci, cj > where c1, c2, · · · , cn are the columns of P. On the other hand, if PPT =(dij) then dij =< vi, vj > . Since v1, v2, · · · , vn are orthonormal then PPT = In.That is P is orthogonal

The following result provides a tool for proving that a matrix is orthogonal.

Theorem 63Let A be an n× n matrix. Then the following are equivalent:(a) A is orthogonal.(b) The rows of A form an orthonormal set.(c) The columns of A form an orthonormal set.

Proof.Let {r1, r2, · · · , rn}, {c1, c2, · · · , cn} denote the rows ond columns of A respec-tively.(a) ⇔ (b): We will show that if AAT = (bij) then bij =< ri, rj > . Indeed, fromthe definition of matrix multiplication we have

bij = (ith row of A)(jth column of AT )= ai1aj1 + ai2aj2 + · · ·+ ainajn

= < ri, rj >

AAT = In if and only if bij = 0 if i 6= j and bii = 1. That is, AAT = In if andonly if < ri, rj >= 0 for i 6= j and < ri, ri >= 1.(a) ⇔ (c): If AT A = (dij) then one can easily show that dij =< cj , ci > . Thus,AT A = In if and only if dij = 0 for i 6= j and dii = 1. Hence, AT A = In if andonly if < cj , ci >= 0 for i 6= j and < ci, ci >= 1.

Exercise 274Show that the matrix

A =1√2

(1 −11 1

)

is orthogonal.

Solution.We will show that this matrix is orthogonal by using the definition. Indeed,using matrix multiplication we find

AAT =

(1√2

− 1√2

1√2

1√2

)(1√2

− 1√2

1√2

1√2

)= I2

Exercise 275(a) Show that if A is orthogonal then AT is also orthogonal.(b) Show that if A is orthogonal then |A| = ±1.

Page 133: Fundamentals of Linear Algebra - Mathematics & Statisticsmelbial2/classes/Linear Algebra... · 2006-01-13 · 1.1 Systems of Linear Equations Many practical problems can be reduced

4.6. THE RANK OF A MATRIX 133

Solution.(a)Suppose that A is orthogonal. Then AT = A−1. Taking the transpose ofboth sides of this equality we find (AT )T = (A−1)T = (AT )−1. That is, AT isorthogonal.(b) Since A is orthogonal then AAT = In. Taking the determinant of both sidesof this equality to obtain |AAT | = |A||AT | = |A|2 = 1 since |AT | = |A| and|In| = 1. Hence, |A| = ±1

4.6 The Rank of a matrix

In this section we study certain vector spaces associated with matrices. We shallalso attach a unique number to a matrix that gives us information about thesolvability of linear systems and the invertibility of matrices.Let A = (aij) be an m× n matrix. For 1 ≤ i ≤ m, the ith row of A is the 1× nmatrix

ri = (ai1, ai2, . . . , ain)

and for 1 ≤ j ≤ n the jth column of A is the m× 1 matrix

cj =

a1j

a2j

...amj

The subspace span{r1, r2, . . . , rm} of IRn is called the row space of A and thesubspace span{c1, c2, . . . , cn} of IRm is called the column space of A.The following theorem will lead to techniques for computing bases for the rowand column spaces of a matrix.

Theorem 64Elementary row operations do not change the row space of a matrix. That is,the row spaces of two equivalent matrices are equal.

Proof.It suffices to show that if B is the matrix obtained from A by applying a rowoperation then

span{r1, r2, . . . , rm} = span{r′1, r′2, . . . , r′m}

where r′1, r′2, . . . , r

′m are the rows of B.

If the row operation is a row interchange then A and B have the same rows andconsequently the above equality holds.If the row operation is multiplication of a row by a nonzero scalar, for in-stance, r′i = αri then we get r′j = rj for j 6= i. Hence, r′j is an element ofspan{r1, r2, . . . , rm} for j 6= i. . Moreover, r′i = 0r1 + . . . 0ri−1 + αri + 0ri+1 +. . .+0rm is also an element of span{r1, r2, . . . , rm}. Hence, span{r′1, r′2, . . . , r′m}

Page 134: Fundamentals of Linear Algebra - Mathematics & Statisticsmelbial2/classes/Linear Algebra... · 2006-01-13 · 1.1 Systems of Linear Equations Many practical problems can be reduced

134 CHAPTER 4. THE THEORY OF VECTOR SPACES

is a subset of span{r1, r2, . . . , rm}. On the other hand, for j 6= i we have rj = r′jand therefore rj ∈ span{r′1, r′2, . . . , r′m}. But ri = 0r′1 + 0r′2 + . . . + 0r′i−1 +1αr′i + 0r′i+1 + . . . + 0r′m which is an element of span{r′1, r′2, . . . , r′m}. Hence,span{r1, r2, . . . , rm} is contained in span{r′1, r′2, . . . , r′m}.Similar argument holds when the elementary row operation is an addition of amultiple of one row to another

It follows from the above theorem that the row space of a matrix A is notchanged by reducing the matrix to a row-echelon form B. It turns out thatthe nonzero rows of B form a basis of the row space of A as indicated by thefollowing theorem.

Theorem 65The nonzero row vectors in a row-echelon form of a matrix form a basis for therow space of A.

Proof.If B is the matrix in echelon form that is equivalent to A. Then by Theorem 64,span{r1, r2, · · · , rm} = span{r′1, r′2, · · · , r′p} where r′1, r

′2, · · · , r′p are the nonzero

rows of B with p ≤ m. We will show that S = {r′1, r′2, · · · , r′p} is a linearlyindependent set. Suppose not, i.e. S = {r′p, r′p−1, · · · , r′1} is linearly dependent.Then by Theorem 50, one of the rows, say r′i is a linear combination of thepreceding rows:

r′i = αi+1r′i+1 + αi+2r

′i+2 + · · ·+ α1r

′p. (4.2)

Suppose that the leading 1 of r′i occurs at the jth column. Since the matrixis in echelon form, the jth component of r′i+1r

′i+2, · · · , r′i are all 0 and so the

jth component of (4.2) is αi+10 + αi+20 + · · · + αp0 = 0. But this contradictsthe assumption that the jth component of r′i is 1. Thus, S must be linearlyindependent

Exercise 276Find a basis for the space spanned by the vectors v1 = (1,−2, 0, 0, 3), v2 =(2,−5,−3,−2, 6), v3 = (0, 5, 15, 10, 0), and v4 = (2, 6, 18, 8, 6).

Solution.The space spanned by the given vectors is the row space of the matrix

1 −2 0 0 32 −5 −3 −2 60 5 15 10 02 6 18 8 6

The reduction of this matrix to row-echelon form is as follows.

Step 1: r2 ← r2 − 2r1 and r4 ← r4 − 2r1

1 −2 0 0 30 −1 −3 −2 00 5 15 10 00 10 18 8 0

Page 135: Fundamentals of Linear Algebra - Mathematics & Statisticsmelbial2/classes/Linear Algebra... · 2006-01-13 · 1.1 Systems of Linear Equations Many practical problems can be reduced

4.6. THE RANK OF A MATRIX 135

Step 2: r3 ← r3 + 5r2 and r4 ← r4 + 10r2

1 −2 0 0 30 −1 − 3 − 2 00 0 0 0 00 0 −12 −12 0

Step 3: r2 ← −r2 and r3 ↔ − 112r4

1 −2 0 0 30 1 3 2 00 0 1 1 00 0 0 0 0

Thus, the vectors (1,−2, 0, 0, 3), (0, 1, 3, 2, 0), and (0, 0, 1, 1, 0) form a basis ofthe vector space spanned by the given vectors

Exercise 277Find a basis for the row space of the matrix

A =

1 2 0 0 0 2−1 −2 1 3 0 −12 4 1 3 1 91 2 1 3 0 3

Solution.The reduction of this matrix to row-echelon form is as follows.

Step 1: r2 ← r2 + r1, r3 ← r3 − 2r1 and r4 ← r4 − r1

A =

1 2 0 0 0 20 0 1 3 0 10 0 1 3 1 50 0 1 3 0 1

Step 2: r3 ← r3 − r2 and r4 ← r4 − r2

A =

1 2 0 0 0 20 0 1 3 0 10 0 0 0 1 40 0 0 0 0 0

Thus a basis for the vector spanned by the rows of the given matrix consists ofthe vectors (1, 2, 0, 0, 0, 2), (0, 0, 1, 3, 0, 1), and (0, 0, 0, 0, 1, 4)

In the next theorem we list some of the properties of the row and columnspaces.

Page 136: Fundamentals of Linear Algebra - Mathematics & Statisticsmelbial2/classes/Linear Algebra... · 2006-01-13 · 1.1 Systems of Linear Equations Many practical problems can be reduced

136 CHAPTER 4. THE THEORY OF VECTOR SPACES

Theorem 66Let A be an m× n matrix, U a p×m matrix, and V an n× q matrix. Then(a) the row space of UA is a subset of the row space of A. The two spaces areequal whenever U is nonsingular.(b) The column space of AV is contained in the solumn space of A. Equalityholds when V is nonsingular.

Proof.(a) Let Ui = (ui1, ui2, · · · , uim) denote the ith row of U. If r1, r2, · · · , rm denotethe rows of A then the ith row of UA is

UiA = (ui1, ui2, · · · , uim)

r1

r2

· · ·rm

= ui1r1 + ui2r2 + · · ·+ uimrm

This shows that each row of UA is in the row space of A. Hence, the row spaceof UA is contained in the row space of A. If U is invertible then the row spaceof A is equal to the row space of U−1(UA) which in turn is a subset of the rowspace of UA. This ends a proof of (a).(b) Similar to (a)

Next, we will show that a matrix A is row equivalent to a unique reducedrow-echelon matrix. First, we prove the following

Theorem 67If A and B are two reduced row-echelon matrices with the same row space thenthe leading 1 of A and B occur in the same position.

Proof.Suppose that the leading 1 of the first row of A occurs in column j1 and thatof B in column k1. We will show that j1 = k1. Suppose j1 < k1. Then thej1th column of B is zero. Since the first row of A is in the span of the rowsof B then the (1, j1)th entry of A is a linear combination of the entries ofthe j1th column of B. Hence, 1 = α1b1j1 + α2b2j1 + · · · + αmbmj1 = 0 sinceb1j1 = b2j2 = · · · = bmj1 = 0. (Remember that the j1th column of B is zero).But this is a contradiction. Hence, j1 ≥ k1. Interchanging the roles of A and Bto obtain k1 ≥ j1. Hence, j1 = k1.Next, let A′ be the matrix obtained by deleting the first row of A and B′ thematrix obtained by deleting the first row of B. Then clearly A′ and B′ are inreduced echelon form. We will show that A′ and B′ have the same row space.Let R1, R2, · · · , Rm denote the rows of B. Let r = (a1, a2, · · · , an) be any rowof A′. Since r is also a row of A then we can find scalars d1, d2, · · · , dm suchthat r = d1R1 + d2R2 + · · · + dmRm. Since A is in reduced row echelon formand r is not the first row of A then aj1 = ak1 = 0. Furthermore, since B is inreduced row echelon form all the entries of the k1th column of B are zero exceptb1k1 = 1. Thus,

0 = ak1 = d1b1k1 + d2b2k1 + · · ·+ dmbmk1 .

Page 137: Fundamentals of Linear Algebra - Mathematics & Statisticsmelbial2/classes/Linear Algebra... · 2006-01-13 · 1.1 Systems of Linear Equations Many practical problems can be reduced

4.6. THE RANK OF A MATRIX 137

That is, d1 = 0. Hence, r is in the row space of B′. Since r was arbitrary thenthe row space of A′ is contained in the row space of B′. Interchanging the rolesof A′ and B′ to obtain that the row space of B′ is contained in the row spaceof A′. Hence, A′ and B′ have the same row space. Repeating the argumentearlier, we see that the leading 1 in the first row of A′ and the leading 1 in thefirst row of B′ have the same position. Now, the theorem follows by induction

Theorem 68Reduced row-echelon matrices have the same row space if and only if they havethe same nonzero rows. Hence, every matrix is row equivalent to a uniquereduced row-echelon matrix.

Proof.Let A = (aij) and B = (bij) be two reduced row-echelon matrices. If A and Bhave the same nonzero rows then they have the same row space.Conversely, suppose that A and B have the same row space. Let r1, r2, · · · , rs

be the nonzero rows of B. Let Ri be the ith nonzero row of A. Then there existscalars α1, α2, · · · , αs such that

Ri = α1r1 + α2r2 + · · ·+ αsrs. (4.3)

We will show that Ri = ri or equivalently αi = 1 and αk = 0 for k 6= i. Supposethat the leading 1 of Ri occurs at the ji column. Then by (4.3) we have

1 = α1b1ji + α2b2ji + · · ·+ αsbsji . (4.4)

By Theorem 67, biji = 1 and bkji = 0 for k 6= i. Hence, αi = 1.It remains to show that αk = 0 for k 6= i. So suppose that k 6= i. Suppose theleading 1 of Rk occurs in the jkth column. By (4.3) we have

aijk= α1b1jk

+ α2b2jk+ · · ·+ αsbsjk

. (4.5)

Since B is in reduced row-echelon form then bkjk= 1 and bijk

= 0 for i 6= k.Hence, αk = aijk

. Since, the leading 1 of Rk occurs at the jk column then byTheorem 67 we have akjk

= 1 and aijk= 0 for i 6= k. Hence, αk = 0, for k 6= i.

RemarkA procedure for finding a basis for the column space of a matrix A is describedas follows: From A we construct the matrix AT . Since the rows of AT are thecolumns of A, then the row space of AT and the column space of A are identical.Thus, a basis for the row space of AT will yield a basis for the column space ofA. We transform AT into a row-reduced echelon matrix B. The nonzero rowsof B form a basis for the row space of AT . Finally, the nonzero columns of BT

form a basis for the column space of A.

Page 138: Fundamentals of Linear Algebra - Mathematics & Statisticsmelbial2/classes/Linear Algebra... · 2006-01-13 · 1.1 Systems of Linear Equations Many practical problems can be reduced

138 CHAPTER 4. THE THEORY OF VECTOR SPACES

Exercise 278Find a basis for the column space of the matrix

A =

1 2 0 0 0 2−1 −2 1 3 0 −12 4 1 3 1 91 2 1 3 0 3

Solution.The columns of A are the rows of the transpose matrix.

1 −1 2 12 −2 4 20 1 1 10 3 3 30 0 1 02 −1 9 3

The reduction of this matrix to row-echelon form is as follows.

Step 1: r2 ← r2 − 2r1 and r6 ← r6 − 2r1

1 −1 2 10 0 0 00 1 1 10 3 3 30 0 1 00 1 5 1

Step 2: r2 ↔ r6

1 −1 2 10 1 5 10 1 1 10 3 3 30 0 1 00 0 0 0

Step 3: r3 ← r3 − r2 and r4 ← r4 − 3r2

1 −1 2 10 1 5 10 0 − 4 00 0 −12 00 0 1 00 0 0 0

Page 139: Fundamentals of Linear Algebra - Mathematics & Statisticsmelbial2/classes/Linear Algebra... · 2006-01-13 · 1.1 Systems of Linear Equations Many practical problems can be reduced

4.6. THE RANK OF A MATRIX 139

Step 4: r4 ← r4 + 12r5 and r3 ← − 14

1 −1 2 10 1 5 10 0 1 00 0 0 00 0 1 00 0 0 0

Step 5: r5 ← r5 − r3

1 −1 2 10 1 5 10 0 1 00 0 0 00 0 0 00 0 0 0

thusS {(1,−1, 2, 1), (0, 1, 5, 1), (0, 0, 1, 0)} is a basis for the row space of AT , and

1−121

,

0151

,

0010

is a basis for the column space of A

Next, we shall establish the relationship between the dimensions of the rowspace and the column space of a matrix.

Theorem 69If A is any m × n matrix, then the row space and the column space of A havethe same dimension.

Proof.Let r1, r2, . . . , rm be the row vectors of A where

ri = (ai1, ai2, . . . , ain).

Let r be the dimension of the row space. Suppose that {v1, v2, . . . , vr} is a basisfor the row space of A where

vi = (bi1, bi2, . . . , bin).

By definition of a basis we have

r1 = c11v1 + c12v2 + . . . c1rvr

r2 = c21v1 + c22v2 + . . . c2rvr

......

rm = cm1v1 + cm2v2 + . . . cmrvr.

Page 140: Fundamentals of Linear Algebra - Mathematics & Statisticsmelbial2/classes/Linear Algebra... · 2006-01-13 · 1.1 Systems of Linear Equations Many practical problems can be reduced

140 CHAPTER 4. THE THEORY OF VECTOR SPACES

Equating entries of these vector equations to obtain

a1j = c11b1j + c12b2j + . . . c1rbrj

a2j = c21bij + c22b2j + . . . c2rbrj

......

amj = cm1b1j + cm2b2j + . . . cmrbrj .

or equivalently

a1j

a2j

...amj

= b1j

c11

c21

...cm1

+ b2j

c12

c22

...cm2

+ . . . + brj

c1r

c2r

...cmr

where 1 ≤ j ≤ n. It follows that the columns of A are linear combinations ofr vectors and consequently the dimension of the column space is less than orequal to the dimension of the row space. Now interchanging row and column weget that the dimension of the row space is less than or equal to the dimensionof the column space.

The dimension of the row space or the column space of a matrix A is calledthe rank of A and is denoted by rank(A). It follows from Theorem 69 thatrank(A) = rank(AT ). Also, by Theorem 65 the rank of A is equal to the num-ber of nonzero rows in the row-echelon matrix equivalent to A.

RemarkRecall that in Section 1.6 it was asserted that no matter how a matrix is reduced(by row operations) to a matrix R in row-echelon form, the number of nonzerorows of R is always the same (and was called the rank of A.) Theorem 69 showsthat this assertion is true and that the two notions of rank agree.

Exercise 279Find the rank of the matrix

A =

1 0 1 13 2 5 10 4 4 −4

Solution.The transpose of the matrix A is the matrix

1 3 00 2 41 5 41 1 −4

The reduction of this matrix to row-echelon form is as follows.

Page 141: Fundamentals of Linear Algebra - Mathematics & Statisticsmelbial2/classes/Linear Algebra... · 2006-01-13 · 1.1 Systems of Linear Equations Many practical problems can be reduced

4.6. THE RANK OF A MATRIX 141

Step 1: r3 ← r3 − r1 and r4 ← r4 − r1

1 3 00 2 40 2 40 −2 −4

Step 2: r4 ← r4 + r2 and r3 ← r3 − r2

1 3 00 2 40 0 00 0 0

Step 3: r2 ← 12r2

1 3 00 1 20 0 00 0 0

Thus {(1, 3, 0), (0, 1, 2)} form a basis for the row space of AT or equivalently

130

,

012

form a basis for the column space of A and hence rank(A) = 2

Theorem 70If A is an m× n matrix U and V invertible matrices of size m×m and n× nrespectively. Then rank(A) = rank(UA) = rank(AV ).

Proof.Follows from the definition of rank and Theorem 66

The following theorem shades information about the invertibility of a squarematrix as well as the question of existence of solutions to linear systems.

Theorem 71If A is an n× n matrix then the following statements are equivalent

(a) A is row equivalent to In.(b) rank(A) = n.(c) The rows of A are linearly independent.(d) The columns of A are linearly independent.

Proof.(a) ⇒ (b) : Since A is row equivalent to In and In is a matrix in reduced echelon

Page 142: Fundamentals of Linear Algebra - Mathematics & Statisticsmelbial2/classes/Linear Algebra... · 2006-01-13 · 1.1 Systems of Linear Equations Many practical problems can be reduced

142 CHAPTER 4. THE THEORY OF VECTOR SPACES

form then the rows of In are linearly independent . Since the row space of Ais the span of n vectors then the row space is n-dimensional vector space andconsequently rank(A) = n.

(b) ⇒ (c) : Since rank(A) = n then the dimension of the row space is n.Hence, by Theorem 58 the rows of A are linearly independent.

(c) ⇒ (d) : Assume that the row vectors of A are linearly independent thenthe dimension of row space is n and consequently the dimension of the columnspace is n. By Theorem 58, the columns of A are linearly independent.

(d) ⇒ (a) : Assume the columns of A are linearly independent. Then thedimension of the row space of A is n. Thus, the reduced row echelon form ofA has n non-zero rows. Since A is a square matrix the reduced echelon matrixmust be the identity matrix. Hence A is row equivalent to In.

Now consider a system of linear equations Ax = b or equivalently

a11 a12 ... a1m

a21 a22 ... a2m

... ... ... ...an1 an2 ... anm

x1

x2

...xm

=

c1

c2

...cn

This implies

a11x1 + a12x2 + . . . + a1mxm

a21x1 + a22x2 + . . . + a2mxm

...an1x1 + an2x2 + . . . + anmxm

=

b1

b2

...bn

or

x1

a11

a21

...an1

+ x2

a12

a22

...an2

+ . . . + xm

a1m

a2m

...anm

=

b1

b2

...bn

Exercise 280Show that a system of linear equations Ax = b is consistent if and only if b isin the column space of A.

Solution.If b is in the column space of A then there exist scalars x1, x2, · · ·xn such that

x1

a11

a21

...an1

+ x2

a12

a22

...an2

+ . . . + xm

a1m

a2m

...anm

=

b1

b2

...bn

Page 143: Fundamentals of Linear Algebra - Mathematics & Statisticsmelbial2/classes/Linear Algebra... · 2006-01-13 · 1.1 Systems of Linear Equations Many practical problems can be reduced

4.6. THE RANK OF A MATRIX 143

But this is equivalent to Ax = b. Hence, the system Ax = b is consistent. Theconverse is similar

Let A be an n× n matrix. The set ker(A) = {x ∈ IRn : Ax = 0} is a subspaceof IRn called the nullspace. The dimension of this vector space is called thenullity of A.

Exercise 281Find a basis for the nullspace of

A =

2 2 −1 0 1−1 −1 2 −3 11 1 −2 0 −10 0 1 1 1

Solution.By Exercise 38 we found that

ker(A) = span{(−1, 1, 0, 0, 0), (−1, 0,−1, 0, 1)}

Thus, nullity(A) = 2

The following theorem establishes an important relationship between the rankand nullity of a matrix.

Theorem 72For any m× n matrix we have

rank(A) + nullity(A) = n. (4.6)

Proof.Since A has n columns then the system Ax = 0 has n unknowns. These fallinto two categories: the leading (or dependent) variables and the independentvariables. Thus,

[number of leading variables] + [number of independent variables] = n.

But the number of leading variables is the same as the number of nonzero rowsof the reduced row-echelon form of A, and this is just the rank of A. On theother hand, the number of independent variables is the same as the number ofparameters in the general solution which is the same as the dimension of ker(A).Now (4.6) follows

Exercise 282Find the number of parameters in the solution set of the system Ax = 0 if A isa 5× 7 matrix and rank(A) = 3.

Page 144: Fundamentals of Linear Algebra - Mathematics & Statisticsmelbial2/classes/Linear Algebra... · 2006-01-13 · 1.1 Systems of Linear Equations Many practical problems can be reduced

144 CHAPTER 4. THE THEORY OF VECTOR SPACES

Solution.The proof of the foregoing theorem asserts that rank(A) gives the number ofleading variables that occur in solving the system AX = 0 whereas nullity(A)gives the number of free variables. In our case, rank(A) = 3 so by the previoustheorem nullity(A) = 7− rank(A) = 4

The following theorem ties together all the major topics we have studied sofar.

Theorem 73If A is an n× n matrix, then all the following are equivalent.

(a) A is invertible.(b) |A| 6= 0.(c) Ax = 0 has only the trivial solution.(d) nullity(A) = 0.(e) A is row equivalent to In.(f) Ax = b is consistent for all n× 1 matrix b.(g) rank(A) = n.(h) The rows of A are linearly independent.(i) The columns of A are linearly independent.

4.7 Review Problems

Exercise 283Show that the midpoint of P (x1, y1, z1) and Q(x1, y2, z2) is the point

M(x1 + x2

2,y1 + y2

2,z1 + z2

2).

Exercise 284(a) Let ~n = (a, b, c) be a vector orthogonal to a plane P. Suppose P0(x0, y0, z0)is a point in the plane. Write the equation of the plane.(b) Find an equation of the plane passing through the point (3,−1, 7) and per-pendicular to the vector ~n = (4, 2,−5).

Exercise 285Find the equation of the plane through the points P1(1, 2,−1), P2(2, 3, 1) andP3(3,−1, 2).

Exercise 286Find the parametric equations of the line passing through a point P0(x0, y0, z0)and parallel to a vector ~v = (a, b, c).

Exercise 287Compute < ~u,~v > when ~u = (2,−1, 3) and ~v = (1, 4,−1).

Page 145: Fundamentals of Linear Algebra - Mathematics & Statisticsmelbial2/classes/Linear Algebra... · 2006-01-13 · 1.1 Systems of Linear Equations Many practical problems can be reduced

4.7. REVIEW PROBLEMS 145

Exercise 288Compute the angle between the vectors ~u = (−1, 1, 2) and ~v = (2, 1,−1).

Exercise 289For any vectors ~u and ~v we have

||~u× ~v||2 = ||~u||2||~v||2− < ~u,~v >2 .

Exercise 290Show that ||~u× ~v|| is the area of the parallelogram with sides ~u and ~v.

Exercise 291Let P be the collection of polynomials in the indeterminate x. Let p(x) = a0 +ax +a2x

2 + · · · and q(x) = b0 +b1x+b2x2 +c . . . be two polynomials in P. Define

the operations:(a) Addition: p(x) + q(x) = a0 + b0 + (a1 + b1)x + (a2 + b2)x2 + · · ·(b) Multiplication by a scalar: αp(x) = αa0 + (αa1)x + (αa2)x2 + · · · .Show that P is a vector space.

Exercise 292Define on IR2 the following operations:(i) (x, y) + (x′, y′) = (x + x′, y + y, );(ii) α(x, y) = (αy, αx).Show that IR2 with the above operations is not a vector space.

Exercise 293Let U = {p(x) ∈ P : p(3) = 0}. Show that U is a subspace of P.

Exercise 294Let Pn denote the collection of all polynomials of degree n. Show that Pn is asubspace of P.

Exercise 295Show that < f, g >=

∫ b

af(x)g(x)dx is an inner product on the space C([a, b])

of continuous functions.

Exercise 296Show that if u and v are two orthogonal vectors of an inner product space, i.e.< u, v >= 0, then

||u + v||2 = ||u||2 + ||v||2.Exercise 297(a) Prove that a line through the origin in IR3 is a subspace of IR3 under thestandard operations.(b) Prove that a line not through the origin in IR3 in not a subspace of IR3.

Exercise 298Show that the set S = {(x, y) : x ≤ 0} is not a vector space of IR2 under theusual operations of IR2.

Page 146: Fundamentals of Linear Algebra - Mathematics & Statisticsmelbial2/classes/Linear Algebra... · 2006-01-13 · 1.1 Systems of Linear Equations Many practical problems can be reduced

146 CHAPTER 4. THE THEORY OF VECTOR SPACES

Exercise 299Show that the collection C([a, b]) of all continuous functions on [a, b] with theoperations:

(f + g)(x) = f(x) + g(x)(αf)(x) = αf(x)

is a vector space.

Exercise 300Let S = {(a, b, a + b) : a, b ∈ IR}. Show that S is a subspace of IR3 under theusual operations.

Exercise 301Let V be a vector space. Show that if u, v, w ∈ V are such that u + v = u + wthen v = w.

Exercise 302Let H and K be subspaces of a vector space V.(a) The intersection of H and K, denoted by H ∩K, is the subset of V thatconsists of elements that belong to both H and K. Show that H∩V is a subspaceof V.(b) The union of H and K, denoted by H ∪K, is the susbet of V that consistsof all elements that belong to either H or K. Give, an example of two subspacesof V such that H ∪K is not a subspace.(c) Show that if H ⊂ K or K ⊂ H then H ∪K is a subspace of V.

Exercise 303Let u and v be vectors in an inner product vector space. Show that

||u + v||2 + ||u− v||2 = 2(||u||2 + ||v||2).

Exercise 304Let u and v be vectors in an inner product vector space. Show that

||u + v||2 − ||u− v||2 = 4 < u, v > .

Exercise 305(a) Use Cauchy -Schwarz’s inequality to show that if u = (x1, x2, · · · , xn) ∈ IRn

and v = (y1, y2, · · · , yn) ∈ IRn then

(x1y1 + x2y2 + · · ·+ xnyn)2 ≤ (x21 + x2

2 + · · ·+ x2n)(y2

1 + y22 + · · ·+ y2

n).

(b) Use (a) to show

n2 ≤ (a1 + a2 + · · ·+ an)(1a1

+1a2

+ · · ·+ 1an

)

where ai > 0 for 1 ≤ i ≤ n.

Page 147: Fundamentals of Linear Algebra - Mathematics & Statisticsmelbial2/classes/Linear Algebra... · 2006-01-13 · 1.1 Systems of Linear Equations Many practical problems can be reduced

4.7. REVIEW PROBLEMS 147

Exercise 306Let C([0, 1]) be the vector space of continuous functions on [0, 1]. Define

< f, g >=∫ 1

0

f(x)g(x)dx.

(a) Show that < ., . > is an inner product on C([0, 1]).(b) Show that

[∫ 1

0

f(x)g(x)dx

]2

≤[∫ 1

0

f(x)2dx

] [∫ 1

0

g(x)2dx

].

This inequality is known as Holder’s inequality.(c) Show that

[∫ 1

0

(f(x) + g(x))2dx

] 12

≤[∫ 1

0

f(x)2dx

] 12

+[∫ 1

0

g(x)2dx

] 12

.

This inequality is known as Minkowski’s inequality.

Exercise 307Let W = span{v1, v2, · · · , vn}, where v1, v2, · · · , vn are vectors in V. Show thatany subspace U of V containing the vectors v1, v2, · · · , vn must contain W, i.e.W ⊂ U. That is, W is the smallest subspace of V containing v1, v2, · · · , vn.

Exercise 308Express the vector ~u = (−9,−7,−15) as a linear combination of the vectors~v1 = (2, 1, 4), ~v2 = (1,−1, 3), ~v3 = (3, 2, 5).

Exercise 309(a) Show that the vectors ~v1 = (2, 2, 2), ~v2 = (0, 0, 3), and ~v3 = (0, 1, 1) span IR3.(b) Show that the vectors ~v1 = (2,−1, 3), ~v2 = (4, 1, 2), and ~v3 = (8,−1, 8) donot span IR3.

Exercise 310Show that

M22 = span{(

1 00 0

),

(0 10 0

),

(0 01 0

),

(0 00 1

)}

Exercise 311(a) Show that the vectors ~v1 = (2,−1, 0, 3), ~v2 = (1, 2, 5,−1), and ~v3 = (7,−1, 5, 8)are linearly dependent.(b) Show that the vectors ~v1 = (4,−1, 2) and ~v2 = (−4, 10, 2) are linearly inde-pendent.

Exercise 312Show that the {u, v} is linearly dependent if and only if one is a scalar multipleof the other.

Page 148: Fundamentals of Linear Algebra - Mathematics & Statisticsmelbial2/classes/Linear Algebra... · 2006-01-13 · 1.1 Systems of Linear Equations Many practical problems can be reduced

148 CHAPTER 4. THE THEORY OF VECTOR SPACES

Exercise 313Let V be the vector of all real-valued functions with domain IR. If f, g, h aretwice differentiable functions then we define w(x) by the determinant

w(x) =

∣∣∣∣∣∣

f(x) g(x) h(x)f ′(x) g′(x) h′(x)f ′′(x) g′′(x) h′′(x)

∣∣∣∣∣∣

We call w(x) the Wronskian of f, g, and h. Prove that f, g, and h are linearlyindependent if and only if w(x) 6= 0.

Exercise 314Use the Wronskian to show that the functions ex, xex, x2ex are linearly indepen-dent.

Exercise 315Show that {1 + x, 3x + x2, 2 + x− x2} is linearly independent in P2.

Exercise 316Show that {1 + x, 3x + x2, 2 + x− x2} is a basis for P2.

Exercise 317Find a basis of P3 containing the linearly independent set {1 + x, 1 + x2}.

Exercise 318Let ~v1 = (1, 2, 1), ~v2 = (2, 9, 0), ~v3 = (3, 3, 4). Show that S = {~v1, ~v2, ~v3} is abasis for IR3.

Exercise 319Let S be a subset of IRn with n+1 vectors. Is S linearly independent or linearlydependent?

Exercise 320Find a basis for the vector space M22 of 2× 2 matrices.

Exercise 321(a) Let U,W be subspaces of a vector space V. Show that the set U + W ={u + w : u ∈ U and w ∈ W} is a subspace of V.(b) Let M22 be the collection of 2×2 matrices. Let U be the collection of matricesin M22 whose second row is zero, and W be the collection of matrices in M22

whose second column is zero. Find U + W.

Exercise 322Let S = {(1, 1)T , (2, 3)T } and S′ = {(1, 2)T , (0, 1)T } be two bases of IR2. Let~u = (1, 5)T and ~v = (5, 4)T .

(a) Find the coordinate vectors of ~u and ~v with respect to the basis S.

Page 149: Fundamentals of Linear Algebra - Mathematics & Statisticsmelbial2/classes/Linear Algebra... · 2006-01-13 · 1.1 Systems of Linear Equations Many practical problems can be reduced

4.7. REVIEW PROBLEMS 149

(b) What is the transition matrix P from S to S′?(c) Find the coordinate vectors of ~u and ~v with respect to S′ using P.(d) Find the coordinate vectors of ~u and ~v with respect to S′ directly.(e) What is the transition matrix Q from S′ to S?(f) Find the coordinate vectors of ~u and ~v with respect to S using Q.

Exercise 323Suppose that S′ = { ~u1, ~u2, ~u3} is a basis of IR3, where ~u1 = (1, 0, 1), ~u2 =(1, 1, 0), ~u3 = (0, 0, 1). Let S = {~v1, ~v2, ~v3}. Suppose that the transition matrixfrom S to S′ is

1 1 22 1 1−1 −1 1

Determine S.

Exercise 324Show that if A is not a square matrix then either the row vectors of A or thecolumn vectors of A are linearly dependent.

Exercise 325Prove that the row vectors of an n×n invertible matrix A form a basis for IRn.

Exercise 326Compute the rank of the matrix

A =

1 2 2 −13 6 5 01 2 1 2

and find a basis for the row space of A.

Exercise 327Let U and W be subspaces of a vector space V. We say that V is the direct sumof U and W if and only if V = U +W and U ∩W = {0}. We write V = U ⊕W.Show that V is the direct sum of U and W if and only if every vector v in Vcan be written uniquely in the form v = u + w.

Exercise 328Let V be an inner product space and W is a subspace of V. Let W⊥ = {u ∈V :< u, w >= 0, ∀w ∈ W}.(a) Show that W⊥ is a subspace of V.(b) Show that V = W ⊕W⊥.

Exercise 329Let U and W be subspaces of a vector space V . Show that dim(U + W ) =dim(U) + dim(W )− dim(U ∩W ).

Page 150: Fundamentals of Linear Algebra - Mathematics & Statisticsmelbial2/classes/Linear Algebra... · 2006-01-13 · 1.1 Systems of Linear Equations Many practical problems can be reduced

150 CHAPTER 4. THE THEORY OF VECTOR SPACES

Exercise 330Show that cos 2x ∈ span{cos2 x, sin2 x}.

Exercise 331If X and Y are subsets of a vector space V and if X ⊂ Y , show that spanX ⊂spanY.

Exercise 332Show that the vector space F (IR) of all real-valued functions defined on IR is aninfinite-dimensional vector space.

Exercise 333Show that {sin x, cos x} is linearly independent in the vector space F ([0, 2π]).

Exercise 334Find a basis for the vector space V of all 2× 2 symmetric matrices.

Exercise 335Find the dimension of the vector space Mmn of all m× n matrices.

Exercise 336Let A be an n × n matrix. Show that there exist scalars a0, a1, a2, · · · , an2 notall 0 such that

a0In + a1A + a2A2 + · · ·+ an2An2

= 0. (4.7)

Exercise 337Show that if A is an n × n invertible skew-symmetric matrix then n must beeven.

Page 151: Fundamentals of Linear Algebra - Mathematics & Statisticsmelbial2/classes/Linear Algebra... · 2006-01-13 · 1.1 Systems of Linear Equations Many practical problems can be reduced

Chapter 5

Eigenvalues andDiagonalization

Eigenvalues and eigenvectors arise in many physical applications such as thestudy of vibrations, electrical systems, genetics, chemical reactions, quantummechanics, economics, etc. In this chapter we introduce these two conceptsand we show how to find them. Eigenvalues and eigenvectors are used in adiagonalization process of a square matrix that we discuss in Section 5.2

5.1 Eigenvalues and Eigenvectors of a Matrix

If A is an n × n matrix and x is a nonzero vector in IRn such that Ax = λxfor some real number λ then we call x an eigenvector corresponding to theeigenvalue λ.

Exercise 338

Show that x =(

12

)is an eigenvector of the matrix

A =(

3 08 −1

)

corresponding to the eigenvalue λ = 3.

Solution.The value λ = 4 is an eigenvalue of A with eigenvector x since

Ax =(

3 08 −1

) (12

)=

(36

)= 3x

Eigenvalues can be either real numbers or complex numbers. In this book aneigenvalue is always assumed to be a real number.

151

Page 152: Fundamentals of Linear Algebra - Mathematics & Statisticsmelbial2/classes/Linear Algebra... · 2006-01-13 · 1.1 Systems of Linear Equations Many practical problems can be reduced

152 CHAPTER 5. EIGENVALUES AND DIAGONALIZATION

To find the eigenvalues of a square matrix A we rewrite the equation Ax = λxas

Ax = λInx

or equivalently(λIn −A)x = 0.

For λ to be an eigenvalue, there must be a nonzero solution to the above homo-geneous system. But, the above system has a nontrivial solution if and only ifthe coefficient matrix (λIn − A) is singular (Exercise 172), that is , if and onlyif

|λIn −A| = 0.

This equation is called the characteristic equation of A.

Exercise 339Find the characteristic equation of the matrix

A =

0 1 00 0 14 −17 8

Solution.The characteristic equation of A is the equation

∣∣∣∣∣∣

λ −1 00 λ −1−4 17 λ− 8

∣∣∣∣∣∣= 0

That is, the equation: λ3 − 8λ2 + 17λ− 4 = 0

Theorem 74p(λ) = |λIn−A| is a polynomial function in λ of degree n and leading coefficient1. This is called the characteristic polynomial of A.

Proof.One of the elementary product will contain all the entries on the main diago-nal,i.e. will be the product (λ−a11)((λ−a22) · · · (λ−ann). This is a polynomialof degree n and leading coefficient 1. Any other elementary product will containat most n− 2 factors of the form λ− aii. Thus,

p(λ) = λn − (a11 + a22 + · · ·+ ann)λn−1 + terms of lower degree (5.1)

This ends a proof of the theorem

Exercise 340Find the characteristic polynomial of the matrix

A =

5 8 164 1 8−4 −4 −11

Page 153: Fundamentals of Linear Algebra - Mathematics & Statisticsmelbial2/classes/Linear Algebra... · 2006-01-13 · 1.1 Systems of Linear Equations Many practical problems can be reduced

5.1. EIGENVALUES AND EIGENVECTORS OF A MATRIX 153

Solution.The characteristic polynomial of A is

p(λ) =

∣∣∣∣∣∣

λ− 5 −8 −16−4 λ− 1 −84 4 λ + 11

∣∣∣∣∣∣

Expanding this determinant we obtain p(λ) = (λ+3)(λ2+2λ−3) = (λ+3)2(λ−1)

Exercise 341Show that the coefficient of λn−1 is the negative of the trace of A.

Solution.This follows from (5.1)

Exercise 342Show that the constant term in the characteristic polynomial of a matrix A is(−1)n|A|.

Solution.The constant term of the polynomial p(λ) corresponds to p(0). It follows thatp(0) = constant term = | −A| = (−1)n|A|

Exercise 343Find the eigenvalues of the matrices

(a)

A =(

3 2−1 0

)

(b)

B =( −2 −1

5 2

)

Solution.(a) The characteristic equation of A is given by

∣∣∣∣λ− 3 −2

1 λ

∣∣∣∣ = 0

Expanding the determinant and simplifying, we obtain

λ2 − 3λ + 2 = 0

or(λ− 1)(λ− 2) = 0.

Page 154: Fundamentals of Linear Algebra - Mathematics & Statisticsmelbial2/classes/Linear Algebra... · 2006-01-13 · 1.1 Systems of Linear Equations Many practical problems can be reduced

154 CHAPTER 5. EIGENVALUES AND DIAGONALIZATION

Thus, the eigenvalues of A are λ = 2 and λ = 1.

(b) The characteristic equation of the matrix B is∣∣∣∣

λ + 2 1−5 λ− 2

∣∣∣∣ = 0

Expanding the determinant and simplifying, we obtain

λ2 − 9 = 0

and the eigenvalues are λ = ±3

Exercise 344Find the eigenvalues of the matrix

A =

0 1 00 0 14 −17 8

Solution.According to Exercise 339 the characteristic equation of A is λ3−8λ2+17λ−4 =0. Using the rational root test we find that λ = 4 is a solution to this equation.Using synthetic division of polynomials we find

(λ− 4)(λ2 − 4λ + 1) = 0.

The eigenvalues of the matrix A are the solutions to this equation, namely,λ = 4, λ = 2 +

√3, and λ = 2−√3

Next, we turn to the problem of finding the eigenvectors of a square ma-trix. Recall that an eigenvector is a nontrivial solution to the matrix equation(λIn −A)x = 0.

Theorem 75The set of eigenvectors of a matrix corresponding to an eigenvalue λ, togetherwith the zero vector, is a subspace of IRn. This subspace is called the eigenspaceof A corresponding to λ and will be denoted by Vλ.

Proof.Let Vλ = {x ∈ IRn : Ax = λx}. We will show that Vλ ∪{0} is a subspace of IRn.Let v, w ∈ Vλ and α ∈ IR. Then A(v + w) = Av + Aw = λv + λw = λ(v + w).That is v + w ∈ Vλ. Also, A(αv) = αAv = λ(αv) so αv ∈ Vλ. Hence, Vλ is asubspace of IRn.

By the above theorem, determining the eigenspaces of a square matrix is re-duced to two problems: First find the eigenvalues of the matrix, and then findthe corresponding eigenvectors which are solutions to linear homogeneous sys-tems.

Page 155: Fundamentals of Linear Algebra - Mathematics & Statisticsmelbial2/classes/Linear Algebra... · 2006-01-13 · 1.1 Systems of Linear Equations Many practical problems can be reduced

5.1. EIGENVALUES AND EIGENVECTORS OF A MATRIX 155

Exercise 345Find the eigenspaces of the matrix

A =

3 −2 0−2 3 00 0 5

Solution.The characteristic equation of the matrix A is

λ− 3 2 02 λ− 3 00 0 λ− 5

Expanding the determinant and simplifying we obtain

(λ− 5)2(λ− 1) = 0.

The eigenvalues of A are λ = 5 and λ = 1.A vector x = (x1, x2, x3)T is an eigenvector corresponding to an eigenvalue λ ifand only if x is a solution to the homogeneous system

(λ− 3)x1 + 2x2 = 02x1 + (λ− 3)x2 = 0

(λ− 5)x3 = 0(5.2)

If λ = 1, then (5.2) becomes−2x1 + 2x2 = 02x1 − 2x2 = 0

− 4x3 = 0(5.3)

Solving this system yields

x1 = s, x2 = s, x3 = 0

The eigenspace corresponding to λ = 1 is

V1 =

ss0

: s ∈ IR

= span

110

If λ = 5, then (5.2) becomes

2x1 + 2x2 = 02x1 + 2x2 = 0

0x3 = 0(5.4)

Solving this system yields

x1 = −t, x2 = t, x3 = s

Page 156: Fundamentals of Linear Algebra - Mathematics & Statisticsmelbial2/classes/Linear Algebra... · 2006-01-13 · 1.1 Systems of Linear Equations Many practical problems can be reduced

156 CHAPTER 5. EIGENVALUES AND DIAGONALIZATION

The eigenspace corresponding to λ = 5 is

V5 =

−tts

: s ∈ IR

=

t

−110

+ s

001

: s, t ∈ IR

= span

−110

,

001

Exercise 346Find bases for the eigenspaces of the matrix

A =

0 0 −21 2 11 0 3

Solution.The characteristic equation of the matrix A is

λ 0 2−1 λ− 2 −1−1 0 λ− 3

Expanding the determinant and simplifying we obtain

(λ− 2)2(λ− 1) = 0.

The eigenvalues of A are λ = 2 and λ = 1.A vector x = (x1, x2, x3)T is an eigenvector corresponding to an eigenvalue λ ifand only if x is a solution to the homogeneous system

λx1 + 2x3 = 0−x1 + (λ− 2)x2 − x3 = 0−x1 + (λ− 3)x3 = 0

(5.5)

If λ = 1, then (5.5) becomes

x1 + 2x3 = 0−x1 − x2 − x3 = 0−x1 − 2x3 = 0

(5.6)

Solving this system yields

x1 = −2s, x2 = s, x3 = s

The eigenspace corresponding to λ = 1 is

V1 =

−2sss

: s ∈ IR

= span

−211

Page 157: Fundamentals of Linear Algebra - Mathematics & Statisticsmelbial2/classes/Linear Algebra... · 2006-01-13 · 1.1 Systems of Linear Equations Many practical problems can be reduced

5.1. EIGENVALUES AND EIGENVECTORS OF A MATRIX 157

and (−2, 1, 1)T is a basis for V1.

If λ = 2, then (5.5) becomes

2x1 + 2x3 = 0−x1 − x3 = 0−x1 − x3 = 0

(5.7)

Solving this system yields

x1 = −s, x2 = t, x3 = s

The eigenspace corresponding to λ = 2 is

V2 =

−sts

: s ∈ IR

=

s

−101

+ t

010

: s, t ∈ IR

= span

−101

,

010

One can easily check that the vectors (−1, 0, 1)T and (0, 1, 0)T are linearly in-dependent and therefore these vectors form a basis for V2

Exercise 347Show that λ = 0 is an eigenvalue of a matrix A if and only if A is singular.

Solution.If λ = 0 is an eigenvalue of A then it must satisfy |0In −A| = | −A| = 0. Thatis |A| = 0 and this implies that A is singular. Conversely, if A is singular then0 = |A| = |0In −A| and therefore 0 is an eigenvalue of A

Exercise 348(a) Show that the eigenvalues of a triangular matrix are the entries on the maindiagonal.(b) Find the eigenvalues of the matrix

A =

12 0 0−1 2

3 05 −8 − 1

4

Solution.(a) Suppose that A is upper triangular n× n matrix. Then the matrix λIn −Ais also upper triangular with entries on the main diagonal are λ − a11, λ −

Page 158: Fundamentals of Linear Algebra - Mathematics & Statisticsmelbial2/classes/Linear Algebra... · 2006-01-13 · 1.1 Systems of Linear Equations Many practical problems can be reduced

158 CHAPTER 5. EIGENVALUES AND DIAGONALIZATION

a22, · · · , λann. Since the determinant of a triangular matrix is just the productof the entries of the main diaginal then the characteristic equation of A is

(λ− a11)(λ− a22) · · · (λ− ann) = 0.

Hence, the eigenvalues of A are a11, a22, · · · , ann.(b) Using (a), the eigenvalues of A are λ = 1

2 , λ = 23 , and λ = − 1

4

Exercise 349Show that A and AT have the same characteristic polynomial and hence thesame eigenvalues.

Solution.We use the fact that a matrix and its transpose have the same determinant(Theorem 28). Hence,

|λIn −AT | = |(λIn −A)T | = |λIn −A|.Thus, A and AT have the same characteristic equation and therefore the sameeigenvalues

The algebraic multiplicity of an eigenvalue λ of a matrix A is the multi-plicity of λ as a root of the characteristic polynomial, and the dimension of theeigenspace corresponding to λ is called the geometric multiplicity of λ.

Exercise 350Find the algebraic and the geometric multiplicity of the eigenvalues of the matrix

A =

2 1 00 2 02 3 1

Solution.The characteristic equation of the matrix A is

λ− 2 −1 00 λ− 2 0−2 −3 λ− 1

Expanding the determinant and simplifying we obtain

(λ− 2)2(λ− 1) = 0.

The eigenvalues of A are λ = 2 (of algebraic multiplicity 2) and λ = 1 (ofalgebraic multiplicity 1).A vector x = (x1, x2, x3)T is an eigenvector corresponding to an eigenvalue λ ifand only if x is a solution to the homogeneous system

(λ− 2)x1 − x2 = 0(λ− 2)x2 = 0

−2x1 − 3x2 + (λ− 1)x3 = 0(5.8)

Page 159: Fundamentals of Linear Algebra - Mathematics & Statisticsmelbial2/classes/Linear Algebra... · 2006-01-13 · 1.1 Systems of Linear Equations Many practical problems can be reduced

5.1. EIGENVALUES AND EIGENVECTORS OF A MATRIX 159

If λ = 1, then (5.8) becomes

−x1 − x2 = 0− x2 = 0

−2x1 − 3x2 = 0(5.9)

Solving this system yields

x1 = 0, x2 = 0, x3 = s

The eigenspace corresponding to λ = 1 is

V1 =

00s

: s ∈ IR

= span

001

and (0, 0, 1)T is a basis for V1. The geometric multiplicity of λ = 1 is 1.

If λ = 2, then (5.8) becomes{ − x2 = 0−2x1 − 3x2 + x3 = 0 (5.10)

Solving this system yields

x1 =12s, x2 = 0, x3 = s

The eigenspace corresponding to λ = 2 is

V2 =

12s0s

: s ∈ IR

= span

1201

and the vector ( 12 , 0, 1)T is a basis for V2 so that the geometric multiplicity of

λ = 2 is 1

There are many matrices with real entries but with no real eigenvalues. Anexample is given in the next exercise.

Exercise 351Show that the following matrix has no real eigenvalues.

A =(

0 1−1 0

)

Page 160: Fundamentals of Linear Algebra - Mathematics & Statisticsmelbial2/classes/Linear Algebra... · 2006-01-13 · 1.1 Systems of Linear Equations Many practical problems can be reduced

160 CHAPTER 5. EIGENVALUES AND DIAGONALIZATION

Solution.The characteristic equation of the matrix A is

(λ −11 λ

)

Expanding the determinant we obtain

λ2 + 1 = 0.

The solutions to this equation are the imaginary complex numbers λ = i andλ = −i, and since we are assuming in this chapter that all our scalars are realnumbers, A has no real eigenvalues

Symmetric matrices with real entries have always real eigenvalues. In orderto prove this statement we recall the reader of the following definition. A com-plex number is any number of the form z = a + bi where i =

√−1 is theimaginary root. The number a is called the real part of z and b is called theimaginary part. Also, recall that a + bi = a′ + b′i if and only if a = a′ andb = b′.

Theorem 76If λ = a + bi is an eigenvalue of a real symmetric n × n matrix A then b = 0,that is λ is real.

Proof.Since λ = a + bi is an eigenvalue of A then λ satisfies the equation

[(a + bi)In −A](u + vi) = 0 + 0i,

where u, v, are vectors in IRn not both equal to zero and 0 is the zero vector ofIRn. The above equation can be rewritten in the following form

(aInu−Au− bInv) + (aInv + bInu−Av)i = 0 + 0i

Setting the real and imaginary parts equal to zero, we have

aInu−Au− bInv = 0

andaInv + bInu−Av = 0.

These equations yield the following equalities

< v, aInu−Au− bInv >=< v, 0 >= 0

and< aInv + bInu−Av, u >=< 0, u >= 0

Page 161: Fundamentals of Linear Algebra - Mathematics & Statisticsmelbial2/classes/Linear Algebra... · 2006-01-13 · 1.1 Systems of Linear Equations Many practical problems can be reduced

5.1. EIGENVALUES AND EIGENVECTORS OF A MATRIX 161

or equivalently

a < v, Inu > − < v, Au > −b < v, Inv > = 0 (5.11)a < Inv, u > − < Av, u > +b < Inu, u > = 0. (5.12)

But < Inv, u >= uT (Inv) = (Inu)T v =< v, Inu > and < Av, u >= uT (Av) =uT (AT v) = (uT At)v = (Au)T v =< v,Au >, where < ., . > is the inner productinIRn. Subtracting the two equations in (5.11), we now get

−b < v, Inv > −b < Inu, u >= 0

or−b[< u, u > + < v, v >] = 0.

Since u and v not both the zero vector, then either < u, u >> 0 or < v, v >> 0.From the previous equation we conclude that b = 0.

We next introduce a concept for square matrices that will be fundamental inthe next section. We say that two n× n matrices A and B are similar if thereexists a nonsingular matrix P such that B = P−1AP. We write A ∼ B. Thematrix P is not unique. For example, if A = B = In then any invertible matrixP will satisfy the definition.

Exercise 352Show that ∼ is an equivalence relation on the collection Mnn of square matrices.That is, show the following:(a) A ∼ A (∼ is reflexive).(b) If A ∼ B then B ∼ A (∼ is symmetric).(c) If A ∼ B and B ∼ C then A ∼ C (∼ is transitive).

Solution.(a) Since A = I−1

n AIn then A ∼ A.(b) Suppose that A ∼ B. Then there is an invertible matrix P such thatB = P−1AP. By premultiplying by P and postmultiplying by P−1 we findA = PBP−1 = Q−1BQ, where Q = P−1. Hence, B ∼ A.(c) Suppose that A ∼ B and B ∼ C then there exist invertible matric P andQ such that B = P−1AP and C = Q−1BQ. It follows that C = Q−1BQ =Q−1P−1APQ = (PQ)−1A(PQ) = R−1AR, with R = PQ. This says thatA ∼ C

Exercise 353Let A and B be similar matrices. Show the following:(a) |A| = |B|.(b) tr(A) = tr(B).(c) rank(A) = rank(B).(d) |λIn −A| = |λIn −B|.

Page 162: Fundamentals of Linear Algebra - Mathematics & Statisticsmelbial2/classes/Linear Algebra... · 2006-01-13 · 1.1 Systems of Linear Equations Many practical problems can be reduced

162 CHAPTER 5. EIGENVALUES AND DIAGONALIZATION

Solution.Since A ∼ B then there exists an invertible matrix P such that B = P−1AP.

(a) |B| = |P−1AP | = |P−1||A||P | = |A| since |P−1| = |P |−1.(b) tr(B) = tr(P−1AP ) = tr(PP−1A) = tr(A) (See Exercise 88 (a)).(c) By Theorem 70, we have rank(B) = rank(PA) = rank(A).(d) Indeed, |λIn −B| = |λIn − P−1AP | = |P−1(λIn)P | = |λIn − A|. It followsthat two similar matrices have the same eigenvalues

Exercise 354Show that the following matrices are not similar.

A =(

1 22 1

), B =

(1 11 1

)

Solution.The eigenvalues of A are λ = 3 and λ = −1. The eigenvalues of B are λ = 0 andλ = 2. According to Exercise 353 (d), these two matrices cannot be similar

Exercise 355Let A be an n × n matrix with eigenvalues λ1, λ2, · · · , λn including repetitions.Show the following.

(a) tr(A) = λ1 + λ2 + · · ·+ λn.(b) |A| = λ1λ2 · · ·λn.

Solution.Factoring the characteristic polynomial of A we find

p(λ) = (λ− λ1)(λ− λ2) · · · (λ− λn)= λn − (λ1 + λ2 + · · ·+ λn)λn−1 + · · ·+ λ1λ2 · · ·λn

(a) By Exercise 341, tr(A) = λ1 + λ2 + · · ·+ λn.(b) |A| = p(0) = λ1λ2 · · ·λn

We end this section with the following important result of linear algebra.

Theorem 77 (Cayley-Hamilton)Every square matrix is the zero of its characteristic polynomial.

Proof.Let p(λ) = λn+an−1λ

n−1+· · ·+a1λ+a0 be the characteristic polynomial corre-sponding to A. We will show that p(A) = 0. The cofactors of A are polynomialsin λ of degree at most n− 1. Thus,

adj(λIn −A) = Bn−1λn−1 + · · ·+ B1λ + B0

where the Bi are n× n matrices with entries independent of λ. Hence,

(λIn −A)adj(λIn −A) = |λIn −A|In.

Page 163: Fundamentals of Linear Algebra - Mathematics & Statisticsmelbial2/classes/Linear Algebra... · 2006-01-13 · 1.1 Systems of Linear Equations Many practical problems can be reduced

5.1. EIGENVALUES AND EIGENVECTORS OF A MATRIX 163

That is

Bn−1λn + (Bn−2 −ABn−1)λn−1 + (Bn−3

−ABn−2)λn−2 + · · ·+ (B0 −AB1)λ−AB0 =Inλn−1 + an−1Inλn−1 + an−2Inλn−2 + · · · a1Inλ + a0In.

Equating coefficients of corresponding powers of λ,

Bn−1 = In

Bn−2 −ABn−1 = an−1In

Bn−3 −ABn−2 = an−2In

.............................B0 −AB1 = a1In

−AB0 = a0In

Multiplying the above matrix equations by An, An−1, · · · , A, In respectively andadding the resulting matrix equations to obtain

0 = An + an−1An−1 + · · ·+ a1A + a0In.

In other words, p(A) = 0

Exercise 356Let A be an n × n matrix whose characteristic polynomial is p(λ) = λn +an−1λ

n−1 + · · · + a1λ + a0. Show that if A is invertible then its inverse canbe found by means of the formula

A−1 = − 1a0

(An−1 + an−1An−2 + · · ·+ a1In)

Solution.By the Cayley-Hamilton theorem p(A) = 0. that is

An + an−1An−1 + · · ·+ a1A + a0In = 0.

Multiplying both sides by A−1 to obtain

a0A−1 = −An−1 − an−1A

n−2 − · · · − a1In.

Since A is invertible then |A| = a0 6= 0. Divide both sides of the last equalityby a0 to obtain

A−1 = − 1a0

(An−1 + an−1An−2 + · · ·+ a1In)

Page 164: Fundamentals of Linear Algebra - Mathematics & Statisticsmelbial2/classes/Linear Algebra... · 2006-01-13 · 1.1 Systems of Linear Equations Many practical problems can be reduced

164 CHAPTER 5. EIGENVALUES AND DIAGONALIZATION

5.2 Diagonalization of a Matrix

In this section we shall discuss a method for finding a basis of IRn consistingof eigenvectors of a given n × n matrix A. It turns out that this is equivalentto finding an invertible matrix P such that P−1AP is a diagonal matrix. Thelatter statement suggests the following terminology.A square matrix A is called diagonalizable if A is similar to a diagonal matrix.That is, there exists an invertible matrix P such that P−1AP = D is a diagonalmatrix. The next theorem gives a characterization of diagonalizable matricesand tells how to construct a suitable characterization. In fact, it supports ourstatement mentioned at the begining of this section that the problem of findinga basis of IRn consisting of eigenvectors of A is equivalent to diagonalizing A.

Theorem 78If A is an n×n square matrix, then the following statements are all equivalent.(a) A is diagonalizable.(b) A has n linearly independent eigenvectors.

Proof.(a) ⇒ (b) : Suppose A is diagonalizable. Then there are an invertible matrix Pand a diagonal matrix D such that A = PDP−1. That is

AP = PD =

p11 p12 . . . p1n

p21 p22 . . . p2n

......

pn1 pn2 . . . pnn

λ1 0 0 . . . 00 λ2 0 . . . 0...

...0 0 0 . . . λn

For 1 ≤ i ≤ n, let

pi =

p1i

p2i

...pni

Then the columns of AP are λ1p1, λ2p2, . . . , λnpn. But AP = [Ap1, Ap2, . . . , Apn].Hence, Api = λipi, for 1 ≤ i ≤ n. Since P is invertible its column vectors arelinearly independent and hence are all nonzero vectors (Theorem 71). Thusλ1, λ2, . . . , λn are eigenvalues of A and p1, p2, . . . , pn are the corresponding eigen-vectors. Hence, A has n linearly independent eigenvectors.

(b) ⇒ (a) : Suppose that A has n linearly independent eigenvectors p1, p2, . . . , pn

with corresponding eigenvalues λ1, λ2, . . . , λn. Let

P =

p11 p12 . . . p1n

p21 p22 . . . p2n

......

pn1 pn2 . . . pnn

Page 165: Fundamentals of Linear Algebra - Mathematics & Statisticsmelbial2/classes/Linear Algebra... · 2006-01-13 · 1.1 Systems of Linear Equations Many practical problems can be reduced

5.2. DIAGONALIZATION OF A MATRIX 165

Then the columns of AP are Ap1, Ap2, . . . , Apn. But Ap1 = λ1p1, Ap2 = λ2p2, . . . , Apn =λnpn. Hence

AP =

λ1p11 λ2p12 . . . λnp1n

λ1p21 λ2p22 . . . λnp2n

......

λ1pn1 λ2pn2 . . . λnpnn

=

p11 p12 . . . p1n

p21 p22 . . . p2n

......

pn1 pn2 . . . pnn

λ1 0 0 . . . 00 λ2 0 . . . 0...

...0 0 0 . . . λn

= PD

Since the column vectors of P ar linearly independent, P is invertible. HenceA = PDP−1, that is A is similar to a diagonal matrix

From the above proof we obtain the following procedure for diagonalizing adiagonalizable matrix.

Step 1. Find n linearly independent eigenvectors of A, say p1, p2, · · · , pn.Step 2. Form the matrix P having p1, p2, · · · , pn as its column vectors.Step 3. The matrix P−1AP will then be diagonal with λ1, λ2, . . . , λn as itsdiagonal entries, where λi is the eigenvalue corresponding to pi, 1 ≤ i ≤ n.

Exercise 357Find a matrix P that diagonalizes

A =

3 −2 0−2 3 00 0 5

Solution.From Exercise 345 of the previous section the eigenspaces corresponding to theeigenvalues λ = 1 and λ = 5 are

V1 =

ss0

: s ∈ IR

= span

110

and

V5 =

−tts

: s ∈ IR

=

t

−110

+ s

001

: s, t ∈ IR

= span

−110

,

001

Page 166: Fundamentals of Linear Algebra - Mathematics & Statisticsmelbial2/classes/Linear Algebra... · 2006-01-13 · 1.1 Systems of Linear Equations Many practical problems can be reduced

166 CHAPTER 5. EIGENVALUES AND DIAGONALIZATION

Let ~v1 = (1, 1, 0)T , ~v2 = (−1, 1, 0), and ~v3 = (0, 0, 1)T . It is easy to verify thatthese vectors are linearly independent. The matrices

P =

1 −1 01 1 00 0 1

and

D =

1 0 00 5 00 0 5

satisfy AP = PD or D = P−1AP

Exercise 358Show that the matrix

A =( −3 2−2 1

)

is not diagonalizable.

Solution.The characteristic equation of the matrix A is

(λ + 3 −2

2 λ− 1

)

Expanding the determinant and simplifying we obtain

(λ + 1)2 = 0.

The only eigenvalue of A is λ = −1.A vector x = (x1, x2)T is an eigenvector corresponding to an eigenvalue λ if andonly if x is a solution to the homogeneous system

{(λ + 3)x1 − 2x2 = 0

2x1 + (λ− 1)x2 = 0 (5.13)

If λ = −1, then (5.13) becomes{

2x1 − 2x2 = 02x1 − 2x2 = 0 (5.14)

Solving this system yields x1 = s, x2 = s. Hence the eigenspace correspondingto λ = −1 is

V−1 ={(

ss

): s ∈ IR

}= span

{(11

)}

Since dim(V−1) = 1, A does not have two linearly independent eigenvectors andis therefore not diagonalizable

In many applications one is concerned only with knowing whether a matrixis diagonalizable without the need of finding the matrix P. In order to establishthis result we first need the following theorem.

Page 167: Fundamentals of Linear Algebra - Mathematics & Statisticsmelbial2/classes/Linear Algebra... · 2006-01-13 · 1.1 Systems of Linear Equations Many practical problems can be reduced

5.2. DIAGONALIZATION OF A MATRIX 167

Theorem 79If v1, v2, . . . , vn are nonzero eigenvectors that correspond to distinct eigenvaluesλ1, λ2, . . . , λn then the set {v1, v2, . . . , vn} is linearly indepedent.

Proof.The proof is by induction on n. If n = 1 then {v1} is linearly independent (Exer-cise 252). So assume that the vectors {v1, v2, · · · , vn−1} are linearly independent.Suppose that

α1v1 + α2v2 + · · ·αnvn = 0. (5.15)

Apply A to both sides of (5.15) and using the fact that Avi = λivi to obtain

α1λ1v1 + α2λ2v2 + · · ·αnλnvn = 0. (5.16)

Now, multiplying (5.15) by λn and subtracting the resulting equation from(5.16) we obtain

α1(λ1 − λn)v1 + α2(λ2 − λn)v2 + · · ·αn−1(λn−1 − λn)vn−1 = 0. (5.17)

By the induction hypothesis, all the coefficients must be zero. Since the λi aredistinct, i.e. λi − λn 6= 0 for i 6= n then, α1 = α2 = · · · = αn−1 = 0. Substi-tuting this into (5.15) to obtain αnvn = 0, and hence αn = 0. This shows that{v1, v2, · · · , vn} is linearly indepedent.

As a consquence of the above theorem we have the following useful result.

Theorem 80If A is an n× n matrix with n distinct eigenvalues then A is diagonalizable.

Proof.If v1, v2, · · · , vn are nonzero eigenvectors corresponding to the distinct eigenval-ues λ1, λ2, · · ·λn then by Theorem 79, v1, v2, · · · , vn are linearly independentvectors. Thus, A is diagonalizable by Theorem 78

Exercise 359Show that the following matrix is diagonalizable.

A =

1 0 01 2 −31 −1 0

Solution.The characteristic equation of the matrix A is

λ− 1 0 0−1 λ− 2 3−1 1 λ

Page 168: Fundamentals of Linear Algebra - Mathematics & Statisticsmelbial2/classes/Linear Algebra... · 2006-01-13 · 1.1 Systems of Linear Equations Many practical problems can be reduced

168 CHAPTER 5. EIGENVALUES AND DIAGONALIZATION

Expanding the determinant and simplifying we obtain

(λ− 1)(λ− 3)(λ + 1) = 0

The eigenvalues are 1, 3 and −1, so A is diagonalizable by Theorem 78

The converse of Theorem 80 is false. That is, an n × n matrix A may bediagonalizable even if it does not have n distinct eigenvalues.

Exercise 360Show that the matrix

A =(

3 00 3

)

is diagonalizable with only one eigenvalue.

Solution.The characteristic equation of the matrix A is

(λ− 3 0

0 λ− 3

)

Expanding the determinant and simplifying we obtain

(λ− 3)2 = 0.

The only eigenvalue of A is λ = 3. By letting P = In and D = A we see thatD = P−1AP, i.e. A is diagonalizable

Next we turn our attention to the study of symmetric matrices since theyarise in many applications. The interesting fact about symmetric matrices isthat they are always diagonalizable. This follows from Theorem 76 and Theo-rem 80.Moreover, the diagonalizing matrix P has some noteworthy properties,namely, PT = P−1, i.e. P is orthogonal. More properties are listed in thenext theorem.

Theorem 81The following are all equivalent:(a) A is diagonalizable by an orthogonal matrix.(b) A has an orthonormal set of n eigenvectors.(c) A is symmetric.

Proof.(a) ⇒ (b): Since A is diagonalizable by an orthogonal matrix P then there existan orthogonal matrix P and a diagonal matrix D such that P−1AP = D. Asshown in the proof of Theorem 78, the column vectors of P are the n eigen-vectors of A. Since P is orthogonal then these vectors form an orthonormal setaccording to Theorem 63.

Page 169: Fundamentals of Linear Algebra - Mathematics & Statisticsmelbial2/classes/Linear Algebra... · 2006-01-13 · 1.1 Systems of Linear Equations Many practical problems can be reduced

5.2. DIAGONALIZATION OF A MATRIX 169

(b) ⇒ (c): Suppose that A has an orthonormal set of n eigenvectors of A. Asshown in the proof of Theorem 78 the matrix P with columns the eigenvectorsof A diagonalizes A and P−1AP = D. Since the columns of P are orthonor-mal then PPT = In. By Theorem 20 P−1 = PT . Thus, AT = (PDP−1)T =(P−1)T DT PT = (PT )−1DT PT = PDP−1 = A. This says that A is symmetric.(c)⇒ (a): Suppose that A is symmetric. We proceed by induction on n. If n = 1,A is already doagonal. Suppose that (c) implies (a) for all (n − 1) × (n − 1)symmetric matrices. Let λ1 be a real eigenvalue of A and let x1 be a corre-sponding eigenvector with ||x1|| = 1. Using the Gram-schmidt algorithm we canextend x1 to a orthonormal basis {x1, x2, · · · , xn} basis of IRn. Let P1 be thematrix whose columns are x1, x2, · · · , xn respectively. Then P1 is orthogonal byTheorem 63. Moreover

PT1 AP1 =

(λ1 X0 A1

)

Since A is symmetric then PT1 AP is also symmetric so that X = 0 and A1 is an

(n− 1)× (n− 1) symmetric matrix. By the induction hypothesis, there exist anorthogonal matrix Q and a diagonal matrix D1 such that QT A1Q = D1. Hence,the matrix

P2 =(

1 00 Q

)

is orthogonal and

(P2P1)T A(P2P1) = PT2 (P1AP1)P2

=(

1 00 QT

)(λ1 00 A1

)(1 00 Q

)

=(

λ1 00 D1

)

is diagonal. Let T = P1P2. Then one can easily check that P1P2 is orthogonal.This ends a proof of the theorem

Exercise 361Let A be an m× n matrix. Show that the matrix AT A has an orthonormal setof n eigenvectors.

Solution.The matrix AT A is of size n × n and is symmetric. By Theorem 81, AT A hasan orthonormal set of n eigenvectors

We now turn to the problem of finding an orthogonal matrix P to diagonal-ize a symmetric matrix. The key is the following theorem.

Theorem 82If A is a symmetric matrix then the eigenvectors corresponding to distinct eigen-values are orthogonal.

Page 170: Fundamentals of Linear Algebra - Mathematics & Statisticsmelbial2/classes/Linear Algebra... · 2006-01-13 · 1.1 Systems of Linear Equations Many practical problems can be reduced

170 CHAPTER 5. EIGENVALUES AND DIAGONALIZATION

Proof.Let A be an n × n symmetric matrix and α, β be two distinct eigenvaluesof A. Let v = (v1, v2, · · · , vn) and w = (w1, w2, · · · , wn) be two eigenvectorscorresponding to α and β. We want to show that < v, w >= 0. That is,v1w1 + v2w2 + · · ·+ vnwn = 0. But this is the same thing as the matrix multi-plication vT w = 0. Since v is an eigenvector corresponding to α then Av = αv.Taking transpose of both sides to obtain vT AT = αvT . Since A is symmetric,i.e. AT = A then vT A = αvT . Multiply this equation from the right by w toobtian vT Aw = αvT w. Hence, βvT w = αvT w. That is, (α − β)vT w = 0. ByExercise 79 we conclude that vT w = 0.

As a consequence of this theorem we obtain the following procedure for di-agonalizing a matrix by an orthogonal matrix P.

Step 1. Find a basis for each eigenspace of A.Step 2. Apply the Gram-Schmidt process to each of these bases to obtain anorthonormal basis for each eigenspace.Step 3. Form the matrix P whose columns are the basis vectors constructed inStep 2. This matrix is orthogonal.

Exercise 362Consider the matrix

A =

1 0 −10 1 2−1 2 5

.

(a) Show that A is symmetric.(b) Find an orthogonal matrix P and a diagonal matrix D such that P−1AP =D.

Solution.(a) One can either check that AT = A or by noticing that mirror images ofentries across the main diagonal are equal.(b) The characteristic equation of the matrix A is

λ− 1 0 10 λ− 1 −21 −2 λ− 5

Expanding the determinant and simplifying we obtain

λ(λ− 1)(λ− 6) = 0

The eigenvalues are 0, 1 and 6. One can eaily check the following

V0 = span

1−21

Page 171: Fundamentals of Linear Algebra - Mathematics & Statisticsmelbial2/classes/Linear Algebra... · 2006-01-13 · 1.1 Systems of Linear Equations Many practical problems can be reduced

5.3. REVIEW PROBLEMS 171

V1 = span

210

V9 = span

−125

Let ~v1 = 1√6(1,−2, 1)T , ~v2 = 1√

5(2, 1, 0)T , and ~v3 = 1√

30(−1, 2, 5)T . One can

easily check that these vectors form an orthonormal set so that the matrix Pwith columns the vectors ~v1, ~v2, ~v3 is orthogonal and

P−1AP =

0 0 00 1 00 0 6

= D

5.3 Review Problems

Exercise 363Show that λ = −3 is an eigenvalue of the matrix

A =

5 8 164 1 8−4 −4 −11

and then find the corresponding eigenspace V−3.

Exercise 364Find the eigenspaces of the matrix

A =(

3 08 −1

)

Exercise 365Find the characteristic polynomial, eigenvalues, and eigenspaces of the matrix

A =

2 1 12 1 −2−1 0 −2

Exercise 366Find the bases of the eigenspaces of the matrix

A =

−2 0 1−6 −2 019 5 −4

Page 172: Fundamentals of Linear Algebra - Mathematics & Statisticsmelbial2/classes/Linear Algebra... · 2006-01-13 · 1.1 Systems of Linear Equations Many practical problems can be reduced

172 CHAPTER 5. EIGENVALUES AND DIAGONALIZATION

Exercise 367Show that if λ is a nonzero eigenvalue of an invertible matrix A then 1

λ is aneigenvalue of A−1.

Exercise 368Show that if λ is an eigenvalue of a matrix A then λm is an eigenvalue of Am

for any positive integer m.

Exercise 369(a) Show that if D is a diagonal matrix then Dk, where k is a positive integer,is a diagonal matrix whose entries are the entries of D raised to the power k.(b) Show that if A is similar to a diagonal matrix D then Ak is similar to Dk.

Exercise 370Show that the identity matrix In has exactly one eigenvalue. Find the corre-sponding eigenspace.

Exercise 371Show that if A ∼ B then(a) AT ∼ BT .(b) A−1 ∼ B−1.

Exercise 372If A is invertible show that AB ∼ BA for all B.

Exercise 373Let A be an n× n nilpotent matrix, i.e. Ak = 0 for some positive ineteger k.

(a) Show that λ = 0 is the only eigenvalue of A.(b) Show that p(λ) = λn.

Exercise 374Suppose that A and B are n×n similar matrices and B = P−1AP. Show that ifλ is an eigenvalue of A with corresponding eigenvector x then λ is an eigenvalueof B with corresponding eigenvector P−1x.

Exercise 375Let A be an n×n matrix with n odd. Show that A has at least one real eigenvalue.

Exercise 376Consider the following n× n matrix

A =

0 1 0 00 0 1 00 0 0 1−a0 −a1 −a2 −a3

Show that the characterisitc polynomial of A is given by p(λ) = λ4 + a3λ3 +

a2λ2 + a1λ + a0. Hence, every monic polynomial (i.e. the coefficient of the

highest power of λ is 1) is the characteristic polynomial of some matrix. A iscalled the companion matrix of p(λ).

Page 173: Fundamentals of Linear Algebra - Mathematics & Statisticsmelbial2/classes/Linear Algebra... · 2006-01-13 · 1.1 Systems of Linear Equations Many practical problems can be reduced

5.3. REVIEW PROBLEMS 173

Exercise 377Find a matrix P that diagonalizes

A =

0 0 −21 2 11 0 3

Exercise 378Show that the matrix A is diagonalizable.

A =

0 1 00 0 14 −17 8

Exercise 379Show that the matrix A is not diagonalizable.

A =

2 1 12 1 −2−1 0 −2

Exercise 380Show that if A is diagonalizable then the rank of A is the number of nonzeroeigenvalues of A.

Exercise 381Show that A is diagonalizable if and only if AT is diagonalizable.

Exercise 382Show that if A and B are similar then A is diagonalizable if and only if B isdiagonalizable.

Exercise 383Give an example of two diagonalizable matrices A and B such that A+B is notdiagonalizable.

Exercise 384Show that the following are equivalent for a symmetric matrix A.(a) A is orthogonal.(b) A2 = In.(c) All eigenvalues of A are ±1.

Exercise 385A matrix that we obtain from the identity matrix by writing its rows in a differentorder is called permutation matrix. Show that every permutation matrix isorthogonal.

Exercise 386Let A be an n× n skew symmetric matrix. Show that(a) In + A is nonsingular.(b) P = (In −A)(In + A)−1 is orthogonal.

Page 174: Fundamentals of Linear Algebra - Mathematics & Statisticsmelbial2/classes/Linear Algebra... · 2006-01-13 · 1.1 Systems of Linear Equations Many practical problems can be reduced

174 CHAPTER 5. EIGENVALUES AND DIAGONALIZATION

Exercise 387We call square matrix E a projection matrix if E2 = E = ET .

(a) If E is a projection matrix, show that P = In − 2E is orthogonal andsymmetric.(b) If P is orthogonal and symmetric, show that E = 1

2 (In − P ) is a projectionmatrix.

Page 175: Fundamentals of Linear Algebra - Mathematics & Statisticsmelbial2/classes/Linear Algebra... · 2006-01-13 · 1.1 Systems of Linear Equations Many practical problems can be reduced

Chapter 6

Linear Transformations

In this chapter we shall discuss a special class of functions whose domains andranges are vector spaces. Such functions are referred to as linear transforma-tions, a concept to be defined in Section 6.1. Linear transformations play animportant role in many areas of mathematics, the physical and social sciences,engineering, and economics.

6.1 Definition and Elementary Properties

A linear transformation T from a vector space V to a vector space W is afunction T : V → W that satisfies the following two conditions

(i) T (u + v) = T (u) + T (v), for all u, v in V.(ii) T (αu) = αT (u) for all u in V and scalar α.

If W = IR then we call T a linear functional on V.It is important to keep in mind that the addition in u + v refers to the additionoperation in V whereas that in T (u) + T (v) refers to the addition operation inW. Similar remark for the scalar multiplication.

Exercise 388Show that T : IR2 → IR3 defined by

T (x, y) = (x, x + y, x− y)

is a linear transformation.

Solution.We verify the two conditions of the definition. Given (x1, y1) and (x2, y2) in

175

Page 176: Fundamentals of Linear Algebra - Mathematics & Statisticsmelbial2/classes/Linear Algebra... · 2006-01-13 · 1.1 Systems of Linear Equations Many practical problems can be reduced

176 CHAPTER 6. LINEAR TRANSFORMATIONS

IR2, compute

T ((x1, y1) + (x2, y2)) = T (x1 + x2, y1 + y2)= (x1 + x2, x1 + x2 + y1 + y2, x1 + x2 − y1 − y2)= (x1, x1 + y1, x1 − y1) + (x2, x2 + y2, x2 − y2)= T (x1, y1) + T (x2, y2)

This proves the first condition. For the second condition, we let α ∈ IR andcompute

T (α(x, y)) = T (αx, αy) = (αx, αx+αy, αx−αy) = α(x, x+y, x−y) = αT (x, y)

Hence T is a linear transformation

Exercise 389Let T : IR2 → IR3 be given by T (x, y) = (x, y, 1). Show that T is not linear.

Solution.We show that the first condition of the definition is violated. Indeed, for anytwo vectors (x1, y1) and (x2, y2) we have

T ((x1, y1) + (x2, y2)) = T (x1 + x2, y1 + y2) = (x1 + x2, y1 + y2, 1)6= (x1, y1, 1) + (x2, y2, 1) = T (x1, y1) + T (x2, y2)

Hence the given transformation is not linear

Exercise 390Show that an m× n matrix defines a linear transforamtion from IRn to IRm.

Solution.Given x and y in IRm and α ∈ IR, matrix arithmetic yields T (x+y) = A(x+y) =Ax + Ay = Tx + Ty and T (αx) = A(αx) = αAx = αTx. Thus, T is linear

Exercise 391Consider the matrices

E =(

0 11 0

), F =

(α 00 1

), G =

(1 10 1

)

(a) Show that the transformation TE(x, y) = (y, x) is linear. This transforma-tion is a reflection in the line y = x.(b) Show that TF (x, y) = (αx, y) is linear. Such a transformation is called anexpansion if α > 1 and a compression if α < 1.(c) Show that TG(x, y) = (x + y, y) is linear. This transformation is called ashear

Solution.(a) Given (x1, y1) and (x2, y2) is IR2 and α ∈ IR we find

TE((x1, y1) + (x2, y2)) = TE(x1 + x2, y1 + y2) = (y1 + y2, x1 + x2)= (y1, x1) + (y2, x2) = TE(x1, y1) + TE(x2, y2)

Page 177: Fundamentals of Linear Algebra - Mathematics & Statisticsmelbial2/classes/Linear Algebra... · 2006-01-13 · 1.1 Systems of Linear Equations Many practical problems can be reduced

6.1. DEFINITION AND ELEMENTARY PROPERTIES 177

andTE(α(x, y)) = TE(αx, αy) = (αy, αx) = α(y, x) = αTE(x, y)

Hence, TE is linear.(b) Given (x1, y1) and (x2, y2) is IR2 and β ∈ IR we find

TF ((x1, y1) + (x2, y2)) = TF (x1 + x2, y1 + y2) = (α(x1 + x2), y1 + y2)= (αx1, y1) + (αx2, y2) = TF (x1, y1) + TF (x2, y2)

andTF (β(x, y)) = TF (βx, βy) = (βαx, βy) = β(αx, y) = βTF (x, y)

Hence, TF is linear.(c) Given (x1, y1) and (x2, y2) is IR2 and α ∈ IR we find

TG((x1, y1) + (x2, y2)) = TG(x1 + x2, y1 + y2) = (x1 + x2 + y1 + y2, y1 + y2)= (x1 + y1, y1) + (x2 + y2, 22) = TG(x1, y1) + TG(x2, y2)

and

TG(α(x, y)) = TG(αx, αy) = (α(x + y), αy) = α(x + y, y) = αTG(x, y)

Hence, TG is linear.

Exercise 392(a) Show that the identity transformation defined by I(v) = v for all v ∈ V is alinear transformation.(b) Show that the zero transformation is linear.

Solution.(a) For all u, v ∈ V and α ∈ IR we have I(u + v) = u + v = Iu + Iv andI(αu) = αu = αIu. So I is linear.(b) For all u, v ∈ V and α ∈ IR we have 0(u + v) = 0 = 0u + 0v and0(αu) = 0 = α0u. So 0 is linear

The next theorem collects four useful properties of all linear transformations.

Theorem 83If T : V → W is a linear transformation then

(a) T (0) = 0(b) T (−u) = −T (u)(c) T (u− w) = T (u)− T (w)(d) T (α1u1 + α2u2 + . . . + αnun) = α1T (u1) + α2T (u2) + . . . + αnT (un).

Proof.(a) T (0) = T (2× 0) = 2T (0). Thus, T (0) = 0.(b) T (−u) = T ((−1)u) = (−1)T (u) = −T (u).(c) T (u−w) = T (u+(−w)) = T (u)+T (−w) = T (u)+(−T (w)) = T (u)−T (w).

Page 178: Fundamentals of Linear Algebra - Mathematics & Statisticsmelbial2/classes/Linear Algebra... · 2006-01-13 · 1.1 Systems of Linear Equations Many practical problems can be reduced

178 CHAPTER 6. LINEAR TRANSFORMATIONS

(d) Use induction

The following theorem provides a criterion for showing that a transformation islinear.

Theorem 84A function T : V → W is linear if and only if T (αu + v) = αT (u) + T (v) forall u, v ∈ V and α ∈ IR.

Proof.Suppose first that T is linear. Let u, v ∈ V and α ∈ IR. Then αu ∈ V. Since Tis linear we have T (αu + v) = T (αu) + T (v) = αT (u) + T (v).Conversely, suppose that T (αu+v) = αT (u)+T (v) for all u, v ∈ V and α ∈ IR.In particular, letting α = 1 we see that T (u + v) = T (u) + T (v) for all u, v ∈ V.Now, letting v = 0 we see that T (αu) = αT (u). Thus, T is linear

Exercise 393Let Mmn denote the vector space of all m× n matrices.(a) Show that T : Mmn → Mnm defined by T (A) = AT is a linear transforma-tion.(b) Show that T : Mnn → IR defined by T (A) = tr(A) is a linear functional.

Solution.(a) For any A,B ∈ Mmn and α ∈ IR we find T (αA + B) = (αA + B)T =αAT + BT = αT (A) + T (B). Hence, T is a linear transformation.(b) For any A,B ∈ Mnn and α ∈ IR we have T (αA + b) = tr(αA + B) =αtr(A) + tr(B) = αT (A) + T (B) so T is a linear functional

Exercise 394Let V be an inner product space and v0 be any fixed vector in V. Let T : V → IRbe the transformation T (v) =< v, v0 > . Show that T is linear functional.

SolutionIndeed, for u, v ∈ V and α ∈ IR we find T (αu+ v) =< αu+ v, v0 >=< αu, v0 >+ < v, v0 >= α < u, v0 > + < v, v0 >= αT (u) + T (v). Hence, T is a linearfunctional

Exercise 395Let {v1, v2, . . . , vn} be a basis for a vector space V and let T : V → W be alinear transformation. Show that if T (v1) = T (v2) = . . . = T (vn) = 0 thenT (v) = 0 for any vector v in V.

Solution.Let v ∈ V. Then there exist scalars α1, α2, · · · , αn such that v = α1v1 + α2v2 +· · ·+ αnvn. Since T is linear then T (v) = αTv1 + αTv2 + · · ·+ αnTvn = 0

Page 179: Fundamentals of Linear Algebra - Mathematics & Statisticsmelbial2/classes/Linear Algebra... · 2006-01-13 · 1.1 Systems of Linear Equations Many practical problems can be reduced

6.1. DEFINITION AND ELEMENTARY PROPERTIES 179

Exercise 396Let S : V → W and T : V → W be two linear transformations. Show thefollowing:(a) S + T and S − T are linear transformations.(b) αT is a linear transformation where α denotes a scalar.

Solution.(a) Let u, v ∈ V and α ∈ IR then

(S ± T )(αu + v) = S(αu + v)± T (αu + v)= αS(u) + S(v)± (αT (u) + T (v))= α(S(u)± T (u)) + (S(v)± T (v))= α(S ± T )(u) + (S ± T )(v)

(b) Let u, v ∈ V and β ∈ IR then

(αT )(βu + v) = (αT )(βu) + (αT )(v)= αβT (u) + αT (v)= β(αT (u)) + αT (v)= β(αT )(u) + (αT )(v)

Hence, αT is a linear transformation

The following theorem shows that two linear transformations defined on V areequal whenever they have the same effect on a basis of the vector space V.

Theorem 85Let V = span{v1, v2, · · · , vn}. If T and S are two linear transformations fromV into a vector space W such that T (vi) = S(vi) for each i then T = S.

Proof.Let v = α1v1 + α2v2 + · · ·+ αnvn ∈ V. Then

T (v) = T (α1v1 + α2v2 + · · ·+ αnvn)= α1T (v1) + α2T (v2) + · · ·+ αnT (vn)= α1S(v1) + α2S(v2) + · · ·+ αnS(vn)= S(α1v1 + α2v2 + · · ·+ αnvn) = S(v)

Since this is true for any v ∈ V then T = S.

The following very useful theorem tells us that once we say what a linear trans-formation does to a basis for V, then we have completely specified T.

Theorem 86Let V be an n-dimensional vector space with basis {v1, v2, · · · , vn}. If T : V −→W is a linear transformation then for any v ∈ V , Tv is completely determinedby {Tv1, T v2, · · · , T vn}.

Page 180: Fundamentals of Linear Algebra - Mathematics & Statisticsmelbial2/classes/Linear Algebra... · 2006-01-13 · 1.1 Systems of Linear Equations Many practical problems can be reduced

180 CHAPTER 6. LINEAR TRANSFORMATIONS

Proof.For v ∈ V we can find scalars α1, α2, · · · , αn such that v = α1v1 + α2v2 + · · ·+αnvn. Since T is linear then Tv = α1Tv1 +α2Tv2 + · · ·+αnTvn. This says thatTv is completely determined by the vectors Tv1, T v2, · · · , T vn

Theorem 87Let V and W be two vector spaces and {e1, e2, · · · , en} be a basis of V. Givenany vectors w1, w2, · · · , wn in W , there exists a unique linear transformationT : V −→ W such that T (ei) = wi for each i.

Proof.Let v ∈ V . Then v can be represented uniquely in the form v = α1e1 + α2e2 +· · ·+ αnen. Define T : V −→ W by T (v) = α1w1 + α2w2 + · · ·+ αnwn. Clearly,T (ei) = wi for each i. One can easily show that T is linear. Now the uniquenessof T follows from Theorem 85

Exercise 397Let T : IRn → IRm be a linear transformation. Write vectors in IRn as columns.Show that there exists an m× n matrix A such that T (x) = Ax for all x ∈ IRn.The matrix A is called the standard matrix of T.

Solution.Consider the standard basis of IRn, {e1, e2, · · · , en}. Let x = (x1, x2, · · · , xn)T ∈IRn. Then x = x1e1 + x2e2 + · · ·+ xnen. Thus,

T (x) = x1T (e1) + x2T (e2) + · · ·+ xnT (en) = Ax

where A = [ T (e1) T (e2) · · · T (en) ]

Exercise 398Find the standard matrix of T : IR3 → IR2 defined by

T

xyz

=

(x− 2y + z

x− z

)

Solution.Indeed, by simple inspection one finds that

T

xyz

=

(1 −2 11 0 −1

)

xyz

6.2 Kernel and Range of a Linear Transforma-tion

In this section we discuss two important subspaces associated with a lineartransformation T, namely the kernel of T and the range of T. Also, we discuss

Page 181: Fundamentals of Linear Algebra - Mathematics & Statisticsmelbial2/classes/Linear Algebra... · 2006-01-13 · 1.1 Systems of Linear Equations Many practical problems can be reduced

6.2. KERNEL AND RANGE OF A LINEAR TRANSFORMATION 181

some further properties of T as a function such as, the concepts of one-one, ontoand the inverse of T. Let T : V → W be a linear transformation. The kernel ofT (denoted by ker(T )) and the range of T (denoted by R(T )) are defined by

ker(T ) = {x ∈ V : T (v) = 0}R(T ) = {w ∈ W : T (v) = w, v ∈ V }

The following theorem asserts that ker(T ) and R(T ) are subspaces.

Theorem 88Let T : V → W be a linear transformation. Then(a) ker(T ) is a subspace of V .(b) R(T ) is a subspace of W.

Proof.(a) Let v1, v2 ∈ ker(T ) and α ∈ IR. Then T (αv1 + v2) = αTv1 + Tv2 = 0. Thatis, αv1 + v2 ∈ ker(T ). This proves that ker(T ) is a subspace of V.(b) Let w1, w2 ∈ R(T ). Then there exist v1, v2 ∈ V such that Tv1 = w1 andTv2 = w2. Let α ∈ IR. Then T (αv1 + v2) = αTv1 + Tv2 = αw1 + w2. Hence,αw1 + w2 ∈ R(T ). This shows that R(T ) is a subspace of W

Exercise 399If T : IR3 → IR3 is defined by T (x, y, z) = (x − y, z, y − x), find ker(T ) andR(T ).

Solution.If (x, y, z) ∈ ker(T ) then (0, 0, 0) = T (x, y, z) = (x − y, z, y − x). This leads tothe system

x − y = 0−x + y = 0

z = 0

The general solution is given by (s, s, 0) and therefore ker(T ) = span{(1, 1, 0)}.Now, let (u, v, w) ∈ R(T ) be given. Then there is a vector (x, y, z) ∈ IR3 suchthat T (x, y, z) = (u, v, w). This yields the following system

x − y = u−x + y = w

z = v

and the solution is given by (u, v,−u). Hence,

R(T ) = span{(1, 0,−1), (0, 1, 0)}

Exercise 400Let T : IRn → IRm be given by Tx = Ax. Find ker(T ) and R(T ).

Page 182: Fundamentals of Linear Algebra - Mathematics & Statisticsmelbial2/classes/Linear Algebra... · 2006-01-13 · 1.1 Systems of Linear Equations Many practical problems can be reduced

182 CHAPTER 6. LINEAR TRANSFORMATIONS

Solution.We have

ker(T ) = {x ∈ IRn : Ax = 0} = null(A)

andR(T ) = {Ax : x ∈ IRn}

Exercise 401Let V be any vector space and α be a scalar. Let T : V → V be the transforma-tion defined by T (v) = αv.

(a) Show that T is linear.(b) What is the kernel of T?(c) What is the range of T?

Solution.(a) Let u, v ∈ V and β ∈ IR. Then T (βu + v) = α(βu + v) = αβu + αv =βT (u) + T (v). Hence, T is linear(b) If v ∈ ker(T ) then 0 = T (v) = αv. If α = 0 then ker(T ) = V. If α 6= 0 thenker(T ) = {0}.(c) If α = 0 then R(T ) = {0}. If α 6= 0 then R(T ) = V since T ( 1

αv) = v for allv ∈ V

Since the kernel and the range of a linear transformation are subspaces of givenvector spaces, we may speak of their dimensions. The dimension of the kernelis called the nullity of T (denoted nullity(T )) and the dimension of the rangeof T is called the rank of T (denoted rank(T )).

Exercise 402Let T : IR2 −→ IR3 be given by T (x, y) = (x, x + y, y).

(a) Show that T is linear.(b) Find nullity(T ) and rank(T ).

Solution.(a) Let (x1, y1) and (x2, y2) be two vectors in IR2. Then for any α ∈ IR we have

T (α(x1, y1) + (x2, y2)) = T (αx1 + x2, αy1 + y2)= (αx1 + x2, αx1 + x2 + αy1 + y2, αy1 + y2)= (αx1, α(x1 + y1), αy1) + (x2, x2 + y2, y2)= αT (x1, y1) + T (x2, y2)

(b) Let (x, y) ∈ ker(T ). Then (0, 0, 0) = T (x, y) = (x, x + y, y) and this leadsto ker(T ) = {(0, 0)}. Hence, nullity(T ) = 0. Now, let (u, v, w) ∈ R(T ). Thenthere exists (x, y) ∈ IR2 such that (x, x + y, y) = T (x, y) = (u, v, w). Hence,R(T ) = {(x, x + y, y) : x, y ∈ IR} = span{(1, 1, 0), (0, 1, 1)}. Thus, rank(T ) = 2

Page 183: Fundamentals of Linear Algebra - Mathematics & Statisticsmelbial2/classes/Linear Algebra... · 2006-01-13 · 1.1 Systems of Linear Equations Many practical problems can be reduced

6.2. KERNEL AND RANGE OF A LINEAR TRANSFORMATION 183

Since linear transformations are functions then it makes sense to talk aboutone-one and onto functions. We say that a linear transformation T : V → Wis one-one if Tv = Tw implies v = w. We say that T is onto if R(T ) = W. IfT is both one-one and onto we say that T is an isomorphism and the vectorspaces V and W are said to be isomorphic and we write V ∼= W. The identitytransformation is an isomorphism of any vector space onto itself. That is, if Vis a vector space then V ∼= V.

The following theorem is used as a criterion for proving that a linear trans-formation is one-one.

Theorem 89Let T : V → W be a linear transformation. Then T is one-one if and only ifker(T ) = {0}.Proof.Suppose first that T is one-one. Let v ∈ ker(T ). Then Tv = 0 = T0. Since T isone-one then v = 0. Hence, ker(T ) = {0}.Conversely, suppose that ker(T ) = {0}. Let u, v ∈ V be such that Tu = Tv,i.e T (u− v) = 0. This says that u− v ∈ ker(T ), which implies that u− v = 0.Thus, T is one-one

Another criterion of showing that a linear transformation is one-one is providedby the following theorem.

Theorem 90Let T : V → W be a linear transformation. Then the following are equivalent:(a) T is one-one.(b) If S is linearly independent set of vectors then T (S) is also linearly inde-pendent.

Proof.(a) ⇒ (b): Follows from Exercise 405 below.(b) ⇒ (a): Suppose that T (S) is linearly independent for any linearly indepen-dent set S. Let v be a nonzero vector of V. Since {v} is linearly independentthen {Tv} is linearly independent. That is, Tv 6= 0. Hence, T is one-one

Exercise 403consider the transformation T : IR3 → IR2 given by T (x, y, z) = (x + y, x− y)(a) Show that T is linear.(b) Show that T is onto but not one-one.

Solution.(a) Let (x1, y1, z1) and (x2, y2, z2) be two vectors in IR3 and α ∈ IR. Then

T (α(x1, y1, z1) + (x2, y2, z2)) = T (αx1 + x2, αy1 + y2, αz1 + z2)= (αx1 + x2 + αy1 + y2, αx1 + x2 − αy1 − y2)= (α(x1 + y1), α(x1 − y1)) + (x2 + y2, x2 − y2)= αT (x1, y1, z1) + T (x2, y2, z2)

Page 184: Fundamentals of Linear Algebra - Mathematics & Statisticsmelbial2/classes/Linear Algebra... · 2006-01-13 · 1.1 Systems of Linear Equations Many practical problems can be reduced

184 CHAPTER 6. LINEAR TRANSFORMATIONS

(b) Since (0, 0, 1) ∈ ker(T ) then by Theorem 89 T is not one-one. Now, let(u, v, w) ∈ IR3 be such that T (u, v, w) = (x, y). In this case, x = 1

2 (u + v) andy = 1

2 (u− v). Hence, R(T ) = IR3 so that T is onto

Exercise 404Consider the transformation T : IR2 → IR3 given by T (x, y) = (x + y, x− y, x)(a) Show that T is linear.(b) Show that T is one-one but not onto.

Solution.(a) Let (x1, y1) and (x2, y2) be two vectors in IR2. Then for any α ∈ IR we have

T (α(x1, y1) + (x2, y2)) = T (αx1 + x2, αy1 + y2)= (αx1 + x2 + αy1 + y2, αx1 + x2 − αy1 − y2, αx1 + x2)= (α(x1 + y1), α(x1 − y1), αx1) + (x2 + y2, x2 − y2, x2)= αT (x1, y1) + T (x2, y2)

Hence, T is linear.(b) If (x, y) ∈ ker(T ) then (0, 0, 0) = T (x, y) = (x+y, x−y, x) and this leads to(x, y) = (0, 0). Hence, ker(T ) = {(0, 0)} so that T is one-one. To show that T isnot onto, take the vector (0, 0, 1) ∈ IR3. Suppose that (x, y) ∈ IR2 is such thatT (x, y) = (0, 0, 1). This leads to x = 1 and x = 0 which is impossible. Thus, Tis not onto

Exercise 405Let T : V → W be a one-one linear transformation. Show that if {v1, v2, . . . , vn}is a basis for V then {T (v1), T (v2), . . . , T (vn)} is a basis for R(T ).

Solution.The fact that {T (v1), T (v2), · · · , T (vn)} is linearly independent follows fromTheorem 90. It remains to show that R(T ) = span{T (v1), T (v2), · · · , T (vn)}.Indeed, let w ∈ R(T ). Then there exists v ∈ V such that T (v) = w. Since{v1, v2, · · · , vn} is a basis of V then v can be written uniquely in the formv = αv1 + α2v2 + · · · + αnvn. Hence, w = T (v) = α1T (v1) + α2T (v2) +· · · + αnT (vn). That is, w ∈ span{T (v1), T (v2), · · · , T (vn)}. We conclude that{T (v1), T (v2), · · · , T (vn)} is a basis of R(T )

Exercise 406Show that if T : V → W is a linear transformation such that dim(ker(T )) =dimV then ker(T ) = V.

Solution.This follows from Exercise 265

The following important result is called the dimension theorem.

Theorem 91If T : V → W is a linear transformation with dim(V ) = n, then

nullity(T ) + rank(T ) = n.

Page 185: Fundamentals of Linear Algebra - Mathematics & Statisticsmelbial2/classes/Linear Algebra... · 2006-01-13 · 1.1 Systems of Linear Equations Many practical problems can be reduced

6.2. KERNEL AND RANGE OF A LINEAR TRANSFORMATION 185

Proof.Let k = dim(ker(T )). If k = 0 then ker(T ) = {0} and consequently ker(T ) hasno basis. Let {v1, v2, . . . , vn} be a basis for V. Then {T (v1), T (v2), . . . , T (vn)}is a basis for R(T ) (Exercise 405). Thus the conclusion holds. Now, if k =n then ker(T ) = V (Exercise 406) and consequently T (v) = 0 for all v inV. Hence, R(T ) = {0} and the conclusion holds. So we assume that 0 <k < n. Let {v1, v2, . . . , vk} be a basis for ker(T ). Extend this basis to a basis{v1, . . . , vk, vk+1, . . . , vn} for V. We show that {T (vk+1), T (vk+2), . . . , T (vn)} isa basis for R(T ).Clearly, span{T (vk+1), T (vk+2), . . . , T (vn)} is contained in R(T ). On the otherhand, let w be an element of R(T ). Then w = T (v) for some v in V . But thenv = α1v1 + α2v2 + . . . + αnvn and consequently

w = α1v1 + α2v2 + . . . + αnvn

= αk+1T (vk+1) + αk+2T (vk+2) + . . . αn + T (vn)

since T (v1) = T (v2) = . . . = T (vk) = 0. It follows that

R(T ) = span{T (vk+1), T (vk+2), . . . , T (vn)}.

Now, suppose that

αk+1T (vk+1) + αk+2T (vk+2) + . . . αnT (vn) = 0

thenT (αk+1vk+1 + αk+2vk+2 + . . . + αnvn) = 0.

This implies that the vector αk+1vk+1 + αk+2vk+2 + . . . + αnvn is in ker(T ).Consequently,

αk+1vk+1 + αk+2vk+2 + . . . + αnvn = β1v1 + β2v2 + . . . + βkvk.

Hence, β1 = β2 = . . . = βk = αk+1 = . . . = αn = 0. It follows that the vectorsT (vk+1), T (vk+2), . . . , T (vn) are linearly independent and form a basis for R(T ).This shows that the conclusion of the theorem holds

RemarkAccording to the above theorem, if T : V → W is a linear transformation and{v1, v2, · · · , vk, vk+1, · · · , vn} is a basis of V such that {vk+1, · · · , vn} is a basisof ker(T ) then {T (v1), · · · , T (vk)} is a basis of R(T ).

We have seen that a linear transformation T : V → W can be one-one andonto, one-one but not onto, and onto but not one-one. The foregoing theoremshows that each of these properties implies the other if the vector spaces V andW have the same dimension.

Theorem 92Let T : V → W be a linear transformation such that dim(V ) = dim(W ) = n.

Page 186: Fundamentals of Linear Algebra - Mathematics & Statisticsmelbial2/classes/Linear Algebra... · 2006-01-13 · 1.1 Systems of Linear Equations Many practical problems can be reduced

186 CHAPTER 6. LINEAR TRANSFORMATIONS

Then(a) if T is one - one, then T is onto;(b) if T is onto, then T is one-one.

Proof.(a) If T is one-one then ker(T ) = {0}. Thus, dim(ker(T )) = 0. By Theorem 91we have dim(R(T )) = n. Hence, R(T ) = W. That is, T is onto.(b) If T is onto then dim(R(T )) = n. By Theorem 91, dim(ker(T )) = 0. Hence,ker(T ) = {0}, i.e. T is one-one

The following theorem says that every linear transformation from IRn to IRm isa multiplication by an m× n matrix.

Theorem 93Let T : IRn → IRm. Then there exists an m× n matrix A such that

Tx = Ax

for all x in IRn.

Proof.We have

x = Inx = [e1, e2, . . . , en]x= x1e1 + x2e2 + . . . + xnen

Now using the linearity of T to obtain

Tx = T (x1e1 + x2e2 + . . . + xnen)= x1T (e1) + x2T (e2) + . . . + xnT (en)= [T (e1), T (e2), . . . , T (en)]x= Ax.

This ends a proof of the theorem

The matrix A is called the standard matrix for the linear transforma-tion T.

Exercise 407Find the standard matrix A for the linear transformation T : IRn → IRn definedby T (x) = 3x.

Solution.Since T (ei) = 3ei then the standard matrix is the scalar matrix 3In

Theorem 94Let T : IRn → IRm be defined by T (x) = Ax, where A is an m×n matrix. Then(a) ker(T ) is the nullspace of A.(b) R(T ) is the column space of A.

Page 187: Fundamentals of Linear Algebra - Mathematics & Statisticsmelbial2/classes/Linear Algebra... · 2006-01-13 · 1.1 Systems of Linear Equations Many practical problems can be reduced

6.2. KERNEL AND RANGE OF A LINEAR TRANSFORMATION 187

Proof.(a) ker(T ) = {x ∈ IRn : Ax = 0} = null(A).(b) If b ∈ R(T ) then there exists x ∈ IRn such that Ax = b. This is equivalent tob = x1c1 + x2c2 + · · ·+ xncn, where x1, x2, · · · , xn are the components of x andc1, c2, · · · , cn are the columns of A. Hence, b ∈ span{c1, c2, · · · , cn} =columnspace of A. Hence, R(T ) is a subset of the column space of A. The converse issimilar

It follows from the above theorem that rank(T ) = rank(A). Hence, by Theorem91, dim(ker(A)) = n − rank(A). Thus, if rank(A) < n then the homogeneoussystem Ax = 0 has a nontrivial solution and if rank(A) ≥ n then the systemhas only the trivial solution.

A linear transformation T : V → W is said to be invertible if and only ifthere exists a unique function T−1 : W → V such that T ◦ T−1 = idW andT−1 ◦ T = idV .

Theorem 95Let T : V → W be an invertible linear transformation. Then(a) T−1 is linear.(b) (T−1)−1 = T.

Proof.(a) Suppose T−1(w1) = v1, T

−1(w2) = v2 and α ∈ IR. Then αw1 + w2 =αT (v1)+T (v2) = T (αv1+v2. That is, T−1(αw1+w2) = αv1+v2 = αT−1(w1)+T−1(w2).(b) Follows from the definition of invertible functions

The following theorem provides us with an important link between invertiblematrices and one-one linear transformations.

Theorem 96A linear transformation T : V → W is invertible if and only if ker(T ) = {0}and R(T ) = W.

Proof.Suppose that T is such that ker(T ) = {0} and R(T ) = W. That is, T is one-one and onto. Define, S : W → V by S(w) = v where T (v) = w. Since Tis one-one and onto then S is well-defined. Moreover, if w ∈ W then (T ◦S)(w) = T (S(w)) = T (v) = w, i.e. T ◦ S = idW . Similarly, one shows thatS ◦ T = idV . It remains to show that S is unique. Indeed, if S′ is a lineartransformation from W into V such that S′ ◦ T = idV and T ◦ S′ = idW thenS′(w) = S′(T (S(w))) = (S′ ◦ T )(S(w)) = S(w) for all w ∈ W. Hence, S′ = Sand so T is invertible.Conversely, suppose that T is invertible. If T (v1) = T (v2) then T−1(T (v1)) =T−1(T (v2)). This implies that v1 = v2, i.e. T is one-one. Now, if w ∈ W thenT (T−1(w)) = w. Let v = T−1(w) to obtain T (v) = w. That is T is onto.

Page 188: Fundamentals of Linear Algebra - Mathematics & Statisticsmelbial2/classes/Linear Algebra... · 2006-01-13 · 1.1 Systems of Linear Equations Many practical problems can be reduced

188 CHAPTER 6. LINEAR TRANSFORMATIONS

Exercise 408Let T : IR3 → IR3 be given by T (x) = Ax where A is the matrix

A =

1 1 10 1 21 2 2

(a) Prove that T is invertible.(b) What is T−1(x)?

Solution.(a) We must show that T is one-one and onto. Let x = (x1, y1, z1) ∈ ker(T ).Then Tx = Ax = (0, 0, 0). Since |A| = −1 6= 0 then A is invertible and thereforex = (0, 0, 0). Hence, ker(T ) = {(0, 0, 0)}. Now since A is invertible the systemAx = b is always solvable. This shows that R(T ) = IR3. Hence, by the abovetheorem, T is invertible.(b) T−1x = A−1x

The following theorem gives us a very useful characterization of isomorphism:They are linear transformations that preserve bases.

Theorem 97Let T : V → W be a linear transformation with V 6= {0}. Then the followingare all equivalent:(a) T is an isomorphism.(b) If {v1, v2, · · · , vn} is a basis of V then {T (v1, T (v2), · · · , T (vn)} is a basis ofW.(c) There exists a basis {v1, v2, · · · , vn} of V such that {T (v1), T (v2), · · · , T (vn)}is a basis of W.

Proof.(a) ⇒ (b): Suppose that T is an isomorphism. Let {v1, v2, · · · , vn} be a basis ofV. If α1T (v1)+α2T (v2)+ · · ·+αnT (vn) = 0 then T (α1v1 +α2v2 + · · ·+αnvn) =0. Since T is one-one then α1v1 + α2v2 + · · · + αnvn = 0. But the vectorsv1, v2, · · · , vn are linearly independent. Hence, α1 = α2 = · · · = αn = 0.Next, we show that span{T (v1), T (v2), · · · , T (vn)} = W. Indeed, if w ∈ Wthen there is a v ∈ V such that T (v) = w (recall that T is onto). But thenw = α1v1 +α2v2 + · · ·+αnvn. Thus, T (w) = α1T (v1)+α2T (v2)+ · · ·+αnT (vn).That is w ∈ span{T (v1), T (v2), · · · , T (vn)}.(b) ⇒ (c): Since V 6= {0} then V has a basis, say {v1, v2, · · · , vn}. By (b),{T (v1), T (v2), · · · , T (vn)} is a basis of W.(c)⇒ (a): Suppose that T (v) = 0. Since v ∈ V then v = α1v1+α2v2+· · ·+αnvn.Hence, α1T (v1) + α2T (v2) + · · · + αnT (vn) = 0. Since T (v1), T (v2), · · · , T (vn)are linearly independent then α1 = α2 = · · · = αn = 0. This implies that v = 0.Hence ker(T ) = {0} and T is one-one. To show that T is onto, let w ∈ W. Thenw = α1T (v1) + α2T (v2) + · · ·+ αnT (vn) = T (α1v1 + α2v2 + · · ·+ αnvn) = T (v)where v = α1v1 + α2v2 + · · ·+ αnvn ∈ V.

Page 189: Fundamentals of Linear Algebra - Mathematics & Statisticsmelbial2/classes/Linear Algebra... · 2006-01-13 · 1.1 Systems of Linear Equations Many practical problems can be reduced

6.3. THE MATRIX REPRESENTATION OF A LINEAR TRANSFORMATION189

Finally, we end this section with the following theorem.

Theorem 98Two finite dimensional vector spaces V and W are isomorphic if and only ifdim(V ) = dim(W ).

Proof.Suppose that V and W are isomorphic. Then there is an isomorphism T : V →W. Let {v1, v2, · · · , vn} be a basis of V. Then {T (v1), T (v2), · · · , T (vn)} is a basisof W (Theorem 97). Thus, dim(W ) = n = dim(V ).Conversely, suppose that dim(V ) = dim(W ). Let {v1, v2, · · · , vn} be a basisof V and {w1, w2, · · · , wn} be a basis of W. Then there exists a unique lineartransformation T : V → W such that T (vi) = wi ( Theorem 87). Hence,{T (v1), T (v2), · · · , T (vn)} is a basis of W. By Theorem 97, T is an isomorphism

6.3 The Matrix Representation of a Linear Trans-formation

Let V and W be two vector spaces such that dim(V ) = n and dim(W ) = m.Let T : V → W be a linear transformation. The purpose of this section is torepresent T as a matrix multiplication. The basic idea is to work with coordinatematrices rather than the vectors themselves.

Theorem 99Let V and W be as above. Let S = {v1, v2, · · · , vn} and S′ = {u1, u2, · · · , um}be ordered bases for V and W respectively. Let A be the m × n matrix whosejth column is the coordinate vector of T (vj) with respect to the basis S′. ThenA is the only matrix with the property that if w = T (v) for some v ∈ V then[w]S′ = A[v]S .

Proof.Since T (vj) ∈ W and S′ is a basis of W then there exist unique scalarsa1j , a2j , · · · , amj such that T (vj) = a1ju1 + a2ju2 + · · · + amjum. Hence, thecoordinate of T (vi) with respect to S′ is the vector

[T (vj)]S′ =

a1j

a2j

...amj

Let A be the m×n matrix whose jth column is [T (vj)]S′ . We next show that Asatisfies the property stated in the theorem. Indeed, let v ∈ V and w = T (v).

Page 190: Fundamentals of Linear Algebra - Mathematics & Statisticsmelbial2/classes/Linear Algebra... · 2006-01-13 · 1.1 Systems of Linear Equations Many practical problems can be reduced

190 CHAPTER 6. LINEAR TRANSFORMATIONS

Then v = α1v1 + α2v2 + · · ·+ αnvn. Take T of both sides to obtain

T (v) = α1T (v1) + α2T (v2) + · · ·+ αnT (vn)= α1(a11u1 + a21u2 + · · ·+ am1um)+ α2(a12u1 + a22u2 + · · ·+ am2um)+ · · ·+ αn(a1nu1 + a2nu2 + · · ·+ amnum)= (a11α1 + a12α2 + · · ·+ a1nαn)u1

+ (a21α1 + a22α2 + · · ·+ a2nαn)u2

+ · · ·+ (am1α1 + am2α2 + · · ·+ amnαn)um

Now, since w ∈ W then w = β1u1 + β2u2 + · · · + βmum. This and the aboveequalities lead to the following system of m equations in n unknowns:

a11α1 + a12α2 + · · · + a1nαn = β1

a21α1 + a22α2 + · · · + a2nαn = β2

...am1α1 + am2α2 + · · · + amnαn = βm

In matrix notation, we have A[v]S = [w]S′ .It remains to show that A defined as above is the only m × n matrix with theproperty A[v]S = [w]S′ . Let B be another matrix with the property B[v]S =[w]S′ and B 6= A. Write A = (aij) and B = (bij). Since A and B are assumedto be different then the kth columns of these two matrices are unequal. Thecoordinate vector of vk is the vector

[vk]S =

00...10...0

where the 1 is in the kth row of [vk]S . Thus,

[T (vk)]S′ = A[vk]S =

a1k

a2k

...amk

The right-hand side is just the kth column of A. On the other hand,

[T (vk)]S′ = B[vk]S =

b1k

b2k

...bmk

Page 191: Fundamentals of Linear Algebra - Mathematics & Statisticsmelbial2/classes/Linear Algebra... · 2006-01-13 · 1.1 Systems of Linear Equations Many practical problems can be reduced

6.3. THE MATRIX REPRESENTATION OF A LINEAR TRANSFORMATION191

which is the kth column of B. What we have shown here is that T (vk) has twodifferent coordinate vectors with respect to the same basis S′, which is impos-sible. Hence, A = B.

The matrix A is called the representation of T with respect to the or-dered bases S and S′. In case V = W then S = S′ and we call S the matrixrepresentation of T with respect to S.

Exercise 409Let T : IR2 → IR2 be defined by the formula

T

(xy

)=

(x + 2y2x− y

).

(a) Let S = {~e1, ~e2} be the standard basis of IR2. Find the matrix representationof T with respect to S.(b) Let

S′ = {( −1

2

),

(20

)}.

Find the matrix representation of T with respect to S and S′.

Solution.(a) We have the following computation

T

(10

)=

(12

)

T

(01

)=

(2−1

)

Thus, the matrix representation of T with respect to S is(

1 22 −1

)

(b) Let A be the matrix representation of T with respect to S and S′. Then thecolumns of A are determined as follows.

T

(10

)=

(12

)

=( −1

2

)+

(20

)

T

(01

)=

(2−1

)

= − 12

( −12

)+ 3

4

(20

)

Thus,

A =(

1 − 12

1 34

)

Page 192: Fundamentals of Linear Algebra - Mathematics & Statisticsmelbial2/classes/Linear Algebra... · 2006-01-13 · 1.1 Systems of Linear Equations Many practical problems can be reduced

192 CHAPTER 6. LINEAR TRANSFORMATIONS

Exercise 410Let T : IR3 → IR3 be defined by

T

100

=

110

, T

010

=

201

, T

001

=

101

(a) Find the matrix representation of T with respect to the standard basis S ofIR3.(b) Find

T

123

using the definition of T and then the matrix obtained in (a).

Solution.(a) The matrix representation of T with respect to the standard basis S of IR3

is

A =

1 2 11 0 00 1 1

(b) Using the definition of T we have

T

123

= T

100

+ 2T

010

+ 3T

001

=

110

+

402

+

303

=

815

Using (a) we find

T

123

=

1 211 0 00 1 1

123

=

815

Exercise 411Let T : V → V be a linear transformation defined by T (v) = αv, where α isa fixed scalar. Prove that the matrix representation of T with respect to anyordered basis for V is a scalar matrix, i.e. a diagonal matrix whose diagonalentries are equal.

Page 193: Fundamentals of Linear Algebra - Mathematics & Statisticsmelbial2/classes/Linear Algebra... · 2006-01-13 · 1.1 Systems of Linear Equations Many practical problems can be reduced

6.3. THE MATRIX REPRESENTATION OF A LINEAR TRANSFORMATION193

Solution.If {v1, v2, · · · , vn} is a basis for V then T (vi) = αvi for all 1 ≤ i ≤ n. It followsthat the matrix representation of T with respect to any basis is the scalar matrixαIn

Exercise 412Let V be an n-dimensional vector space. Prove that the matrix representationof the identity transformation on V is the identity matrix In.

Solution.Since id(v) = v for all v ∈ V then in particular id(v) = v if v is a basis element.It follows that the matrix representation of id is the identity matrix In

Exercise 413Let V and W be vector spaces such that dim(V ) = n and dim(W ) = m. Showthat the set L(V, W ) of all linear transformations from V into W is a vectorspace under the operations of additions of functions and scalar multiplication.

Solution.L(V, W ) is closed under addition and scalar multiplication according to Exercise396. It is left for the reader to check the axioms of a vector space

Theorem 100The vector space L(V,W ) is isomorphic to the vector space Mmn of all m × nmatrices.

Proof.Let S = {v1, v2, · · · , vn} and S′ = {u1, u2, · · · , um} be ordered bases for V andW respectively. Given T ∈ L(V, W ) there exists a unique matrix A ∈ Mmn

such that [w]S′ = A[v]S . Thus, the function f : L(V, W ) → Mmn given byf(T ) = A is well-defined linear transformation (See Exercise 414). We nextshow that f is one-one. Indeed, let T1, T2 ∈ L(V,W ) be such that T1 6= T2. ThenT1(vk) 6= T2(vk) for some 1 ≤ k ≤ n. This implies that [T1(vk)]S′ 6= [T2(vk)]S′which in turn implies that the matrix representation of T1 with respect to Sand S′ is different from the matrix representation of T2. Hence, f(T1) 6= f(T2).It remains ro show that f is onto. Let A = (aij) ∈ Mmn. Define a functionT : V → W by T (vk) = a1ku1 +a2ku2 + · · ·+amkum and T (α1v1 +α2v2 + · · ·+αnvn) = α1T (v1)+α2T (v2)+ · · ·+αnT (vn). Then T is a linear transformation,i,e T ∈ L(V,W ) and the matrix representing T with respect to S and S′ is A.(See Exercise 415)

Exercise 414Show that the function f defined in the previous theorem is a linear transforma-tion from L(V, W ) into Mmn.

Solution.Let S, T ∈ L(V,W ) and α ∈ IR. Then there exist matrices A,B ∈ Mmn such

Page 194: Fundamentals of Linear Algebra - Mathematics & Statisticsmelbial2/classes/Linear Algebra... · 2006-01-13 · 1.1 Systems of Linear Equations Many practical problems can be reduced

194 CHAPTER 6. LINEAR TRANSFORMATIONS

that f(T ) = A and f(S) = B. Since αT + S is a linear transformation thenthere exists a matric C ∈ Mmn such that f(αT + S) = C. If vi is a basis vectorof V then (αT (vi) + S(vi)) is the ith column of C. That is, the matrix C lookslike [ αT (v1) + S(v1) αT (v2) + S(v2) · · · αT (vn) + S(vn) ]. But this is thesame as αA + B. Hence, f(αT + S) = αf(T ) + f(S)

Exercise 415Show that the function T defined in Theorem 100 is a linear transforamtionfrom V into W with matrix representation with respect to S and S′ equals to A.

Solution.Let v = α1v1+α2v2+· · ·+αnvn and w = β1v1+β2v2+· · ·+βnvn be two vectorsin V and α ∈ IR. Then T (αv + w) = T ((αα1 + β1)v1 + (αα2 + β2)v2 + · · · +(ααn +βn)vn) = (αα1 +β1)T (v1)+ (αα2 +β2)T (v2)+ · · ·+(ααn +βn)T (vn) =αT (v) + T (w). That is, T is linear. The fact that the matrix representation ofT with respect to S and S′ is clear

Theorem 100 implies that dim(L(V, W )) = dim(Mmn) = mn. Also, it meansthat when dealing with finite-dimensional vector spaces, we can always replaceall linear transformations by their matrix representations and work only withmatrices.

6.4 Review Problems

Exercise 416Show that the function T : IR2 → IR3 defined by T (x, y) = (x + y, x− 2y, 3x) isa linear transformation.

Exercise 417Let Pn be the vector space of all polynomials of degree at most n.(a) Show that D : Pn −→ Pn−1 given by D(p) = p′ is a linear transformation.(b) Show that I : Pn −→ Pn+1 given by I(p) =

∫ x

0p(t)dt is a linear transforma-

tion.

Exercise 418If T : IR3 −→ IR is a linear transformation with T (3,−1, 2) = 5 and T (1, 0, 1) =2. Find T (−1, 1, 0).

Exercise 419Let T : IR3 −→ IR2 be the transformation T (x, y, z) = (x, y). Show that T islinear. This transformation is called a projection.

Exercise 420Let θ be a given angle. Define T : IR2 −→ IR2 by

T

(xy

)=

(cos θ − sin θsin θ cos θ

)(xy

)

Page 195: Fundamentals of Linear Algebra - Mathematics & Statisticsmelbial2/classes/Linear Algebra... · 2006-01-13 · 1.1 Systems of Linear Equations Many practical problems can be reduced

6.4. REVIEW PROBLEMS 195

Show that T is a linear transformation. Geometrically, Tv is the vector thatresults if v is rotated counterclockwise by an angle θ. We call this transformationthe rotation of IR2 through the angle θ

Exercise 421Show that the following transformations are not linear.

(a) T : Mnn −→ IR given by T (A) = |A|.(b) T : Mmn −→ IR given by T (A) = rank(A).

Exercise 422If T1 : U −→ V and T2 : V −→ W are linear transformations, then T2 ◦ T1 :U −→ W is also a linear transformation.

Exercise 423Let {v1, v2, · · · , vn} be a basis for a vector space V, and let T : V −→ V be alinear transformation. Show that if T (vi) = vi, for 1 ≤ i ≤ n then T = idV , i.e.T is the identity transformation on V.

Exercise 424Let T be a linear transformation on a vector space V such that T (v − 3v1) = wand T (2v − v1) = w1. Find T (v) and T (v1) in terms of w and w1.

Exercise 425Let T : Mmn → Mmn be given by T (X) = AX for all X ∈ Mmn, where A is anm×m invertible matrix. Show that T is both one-one and onto.

Exercise 426Consider the transformation T : IRn −→ IRm defined by T (x) = Ax, whereA ∈ Mmn.

(a) Show that R(T ) = span{c1, c2, · · · , cn}, where c1, c2, · · · , cn are the columnsof A.(b) Show that T is onto if and only if rank(A) = m(i.e. the rows of A arelinearly independent).(c) Show that T is one-one if and only if rank(A) = n (i.e. the columns of Aare linearly independent).

Exercise 427Let T : V −→ W be a linear transformation. Show that if the vectors

T (v1), T (v2), · · · , T (vn)

are linearly independent then the vectors v1, v2, · · · , vn are also linearly indepen-dent.

Exercise 428Show that the projection transformation T : IR3 → IR2 defined by T (x, y, z) =(x, y) is not one-one.

Page 196: Fundamentals of Linear Algebra - Mathematics & Statisticsmelbial2/classes/Linear Algebra... · 2006-01-13 · 1.1 Systems of Linear Equations Many practical problems can be reduced

196 CHAPTER 6. LINEAR TRANSFORMATIONS

Exercise 429Let Mnn be the vector space of all n×n matrices. Let T : Mnn → Mnn be givenby T (A) = A−AT .(a) Show that T is linear.(b) Find ker(T ) and R(T ).

Exercise 430Let T : V → W. Prove that T is one-one if and only if dim(R(T )) = dim(V ).

Exercise 431Show that the linear transformation T : Mnn → Mnn given by T (A) = AT is anisomorphism.

Exercise 432Let T : P2 → P1 be the linear transformation Tp = p′. Consider the standardordered bases S = {1, x, x2} and S′ = {1, x}. Find the matrix representation ofT with respect to the basis S and S′.

Exercise 433Let T : IR2 → IR2 be defined by

T

(xy

)=

(x−y

)

(a) Find the matrix representation of T with respect to the standard basis S ofIR2.(b) Let

S′ = {(

11

),

( −11

)}

Find the matrix representation of T with repspect to the bases S and S′.

Exercise 434Let V be the vector space of continuous functions on IR with the ordered basisS = {sin t, cos t}. Find the matrix representation of the linear transformationT : V → V defined by T (f) = f ′ with respect to S.

Exercise 435Let T : IR3 → IR3 be the linear transformation whose matrix representation withthe respect to the standard basis of IR3 is given by

A =

1 3 11 2 00 1 1

Find

T

123

.

Page 197: Fundamentals of Linear Algebra - Mathematics & Statisticsmelbial2/classes/Linear Algebra... · 2006-01-13 · 1.1 Systems of Linear Equations Many practical problems can be reduced

Chapter 7

Solutions to ReviewProblems

Chapter 1

Exercise 42Which of the following equations are not linear and why:(a) x2

1 + 3x2 − 2x3 = 5.(b) x1 + x1x2 + 2x3 = 1.(c) x1 + 2

x2+ x3 = 5.

Solution.(a) The given equation is linear by (1.1).(b) The equation is not linear because of the term x1x2.(c) The equation is nonlinear because x2 has a negative power

Exercise 43Show that (2s + 12t + 13, s,−s− 3t− 3, t) is a solution to the system

{2x1 + 5x2 + 9x3 + 3x4 = −1x1 + 2x2 + 4x3 = 1

Solution.Substituting these values for x1, x2, x3, and x4 in each equation.

2x1 + 5x2 + 9x3 + 3x4 = 2(2s + 12t + 13) + 5s + 9(−s− 3t− 3) + 3t = −1x1 + 2x2 + 4x3 = (2s + 12t + 13) + 2s + 4(−s− 3t− 3) = 1.

Since both equations are satisfied, then it is a solution for all s and t

197

Page 198: Fundamentals of Linear Algebra - Mathematics & Statisticsmelbial2/classes/Linear Algebra... · 2006-01-13 · 1.1 Systems of Linear Equations Many practical problems can be reduced

198 CHAPTER 7. SOLUTIONS TO REVIEW PROBLEMS

Exercise 44Solve each of the following systems using the method of elimination:(a) {

4x1 − 3x2 = 02x1 + 3x2 = 18

(b) {4x1 − 6x2 = 106x1 − 9x2 = 15

(c) {2x1 + x2 = 32x1 + x2 = 1

Which of the above systems is consistent and which is inconsistent?

Solution.(a) Adding the two equations to obtain 6x1 = 18 or x1 = 3. Substituting thisvalue for x1 in one of the given equations and then solving for x2 we find x2 = 4.So system is consistent.

(b) The augmented matrix of the given system is(

4 −6 106 −9 15

)

Divide the first row by 4 to obtain(

1 − 32

52

6 −9 15

)

Now, add to the second row −6 times the first row to obtain(

1 − 32

52

0 0 0

)

Hence, x2 = s is a free variable. Solving for x1 we find x1 = 5+3t2 .The system is

consistent.

(c) Note that according to the given equation 1 = 3 which is impossible. So thegiven system is inconsistent

Exercise 45Find the general solution of the linear system

{x1 − 2x2 + 3x3 + x4 = −3

2x1 − x2 + 3x3 − x4 = 0

Page 199: Fundamentals of Linear Algebra - Mathematics & Statisticsmelbial2/classes/Linear Algebra... · 2006-01-13 · 1.1 Systems of Linear Equations Many practical problems can be reduced

199

Solution.The augmented matrix of the given system is

(1 −2 3 1 −32 −1 3 −1 0

)

A corresponding row-echelon matrix is obtained by adding negative two timesthe first row to the second row.

(1 −2 3 1 −30 3 −3 −3 6

)

Thus x3 = s and x4 = t are free variables. Solving for the leading variables onefinds x1 = 1− s + t and x2 = 2 + s + t

Exercise 46Find a, b, and c so that the system

x1 + ax2 + cx3 = 0bx1 + cx2 − 3x3 = 1ax1 + 2x2 + bx3 = 5

has the solution x1 = 3, x2 = −1, x3 = 2.

Solution.Simply substitute these values into the given system to obtain

−a + 2c = −3

3b − c = 73a + 2b = 7

The augmented matrix of the system is−1 0 2 −30 3 −1 73 2 0 7

A row-echelon form of this matrix is obtained as follows.Step 1: r1 ← −r1

1 0 −2 30 3 −1 73 2 0 7

Step 2: r3 ← r3 − 3r1

1 0 −2 30 3 −1 70 2 6 −2

Step 3: r2 ← r2 − r3

1 0 −2 30 1 −7 90 2 6 −2

Page 200: Fundamentals of Linear Algebra - Mathematics & Statisticsmelbial2/classes/Linear Algebra... · 2006-01-13 · 1.1 Systems of Linear Equations Many practical problems can be reduced

200 CHAPTER 7. SOLUTIONS TO REVIEW PROBLEMS

Step 4: r3 ← r3 − 2r2

1 0 −2 30 1 −7 90 0 20 −20

Step 5: r3 ← 120r3

1 0 −2 30 1 −7 90 0 1 − 1

The corresponding system is

a − 2c = 3b − 7c = 9

c = −1

Using back substitution we find the solution a = 1, b = 2, c = −1

Exercise 47Find a relationship between a,b,c so that the following system is consistent

x1 + x2 + 2x3 = ax1 + x3 = b2x1 + x2 + 3x3 = c

Solution.The augmented matrix of the system is

1 1 2 a1 0 1 b2 1 3 c

We reduce this matrix into row-echelon form as follows.

Step 1: r2 ← r2 − r1 and r3 ← r3 − 2r1

1 1 2 a0 −1 −1 b− a0 −1 −1 c− 2a

Step 2: r2 ← −r2

1 1 2 a0 1 1 a− b0 −1 −1 c− 2a

Step 3: r3 ← r3 + r2

1 1 2 a0 1 1 a− b0 0 0 c− a− b

The system is consistent provided that a + b− c = 0

Page 201: Fundamentals of Linear Algebra - Mathematics & Statisticsmelbial2/classes/Linear Algebra... · 2006-01-13 · 1.1 Systems of Linear Equations Many practical problems can be reduced

201

Exercise 48For which values of a will the following system have (a) no solutions? (b) exactlyone solution? (c) infinitely many solutions?

x1 + 2x2 − 3x3 = 43x1 − x2 + 5x3 = 24x1 + x2 + (a2 − 14)x3 = a + 2

Solution.The augmented matrix is

1 2 −3 43 −1 5 24 1 a2 − 14 a + 2

The reduction of this matrix to row-echelon form is outlined below.

Step 1: r2 ← r2 − 3r1 and r3 ← r3 − 4r1

1 2 −3 40 −7 14 −100 −7 a2 − 2 a− 14

Step 2: r3 ← r3 − r2

1 2 −3 40 −7 14 −100 0 a2 − 16 a− 4

The corresponding system is

x1 + 2x2 − 3x3 = 4− 7x2 + 14x3 = −10

(a2 − 16)x3 = a− 4

(a) If a = −4 then the last equation becomes 0 = −8 which is impossible.Therefore, the system is inconsistent.(b) If a 6= ±4 then the system has exactly one solution, namely, x1 = 8a+15

7(a+4) , x2 =10a+547(a+4) , x3 = 1

a+4 .

(c) If a = 4 then the system has infinitely many solutions. In this case, x3 = tis the free variable and x1 = 8−7t

7 and x2 = 10+14t7

Exercise 49Find the values of A,B,C in the following partial fraction

x2 − x + 3(x2 + 2)(2x− 1)

=Ax + B

x2 + 2+

C

2x− 1.

Page 202: Fundamentals of Linear Algebra - Mathematics & Statisticsmelbial2/classes/Linear Algebra... · 2006-01-13 · 1.1 Systems of Linear Equations Many practical problems can be reduced

202 CHAPTER 7. SOLUTIONS TO REVIEW PROBLEMS

Solution.By multiplying both sides of the equation by (x2 +2)(2x−1) and then equatingcoefficients of like powers of x we obtain the following system

2A + C = 1−A + 2B = −1

− B + 2C = 3

Replace r3 by 2r3 + r2 to obtain

2A + C = 1−A + 2B = −1−A + 4C = 5

Next, replace r3 by 2r3 + r1 to obtain

2A + C = 1−A + 2B = −1

9C = 11

Solving backward, we find A = − 19 , B = − 5

9 , and C = 119

Exercise 50Find a quadratic equation of the form y = ax2 + bx + c that goes through thepoints (−2, 20), (1, 5), and (3, 25).

Solution.The components of these points satisfy the given quadratic equation. This leadsto the following system

4a − 2b + c = 20a + b + c = 59a + 3b + c = 25

Apply Gauss algorithm as follows.

Step1. r1 ↔ r2

a + b + c = 54a − 2b + c = 209a + 3b + c = 25

Step 2. r2 ← r2 − 4r1 and r3 ← r3 − 9r1

a + b + c = 5− 6b − 3c = 0− 6b − 8c = −20

Page 203: Fundamentals of Linear Algebra - Mathematics & Statisticsmelbial2/classes/Linear Algebra... · 2006-01-13 · 1.1 Systems of Linear Equations Many practical problems can be reduced

203

Step 3. r3 ← r3 − r2

a + b + c = 5− 6b − 3c = 0

− 5c = −20

Solving by the method of backward substitution we find a = 3, b = −2, andc = 4

Exercise 51For which value(s) of the constant k does the following system have (a) nosolutions? (b) exactly one solution? (c) infinitely many solutions?

{x1 − x2 = 32x1 − 2x2 = k

Solution.(a) The system has no solutions if k

2 6= 3, i.e. k 6= 6.(b) The system has no unique solution for any value of k.(c) The system has infinitely many solution if k = 6. The general solution isgiven by x1 = 3 + t, x2 = t

Exercise 52Find a linear equation in the unknowns x1 and x2 that has a general solutionx1 = 5 + 2t, x2 = t.

Solution.Since x2 = t then x1 = 5 + 2x2 that is x1 − 2x2 = 5

Exercise 53Consider the linear system

2x1 + 3x2 − 4x3 + x4 = 5−2x1 + x3 = 73x1 + 2x2 − 4x3 = 3

(a) Find the coefficient and augmented matrices of the linear system.(b) Find the matrix notation.

Solution.(a) If A is the coefficient matrix and B is the augmented matrix then

A =

2 3 −4 1−2 0 1 03 2 0 −4

, B =

2 3 −4 1 5−2 0 1 0 73 2 0 −4 3

(b) The given system can be written in matrix form as follows

2 3 −4 1−2 0 1 03 2 0 −4

x1

x2

x3

=

08−9

Page 204: Fundamentals of Linear Algebra - Mathematics & Statisticsmelbial2/classes/Linear Algebra... · 2006-01-13 · 1.1 Systems of Linear Equations Many practical problems can be reduced

204 CHAPTER 7. SOLUTIONS TO REVIEW PROBLEMS

Exercise 54Solve the following system using elementary row operations on the augmentedmatrix:

5x1 − 5x2 − 15x3 = 404x1 − 2x2 − 6x3 = 193x1 − 6x2 − 17x3 = 41

Solution.The augmented matrix of the system is

5 −5 −15 404 −2 − 6 193 −6 −17 41

The reduction of this matrix to row-echelon form is

Step 1: r1 ← 15r1

1 −1 − 3 84 −2 − 6 193 −6 −17 41

Step 2: r2 ← r2 − 4r1 and r3 ← r3 − 3r1

1 −1 −3 80 2 6 −130 −3 −8 17

Step 3: r2 ← r2 + r3

1 −1 −3 80 −1 −2 40 −3 −8 17

Step 4: r3 ← r3 − 3r2

1 −1 −3 80 −1 −2 40 0 −2 5

Step 5: r2 ← −r2 and r3 ← − 12r3

1 −1 −3 80 1 2 −40 0 1 − 5

2

It follows that x3 = − 52 , x2 = −6, and x1 = − 11

2

Exercise 55Solve the following system.

2x1 + x2 + x3 = −1x1 + 2x2 + x3 = 03x1 − 2x3 = 5

Page 205: Fundamentals of Linear Algebra - Mathematics & Statisticsmelbial2/classes/Linear Algebra... · 2006-01-13 · 1.1 Systems of Linear Equations Many practical problems can be reduced

205

Solution.The augmented matrix is given by

2 1 1 −11 2 1 03 0 −2 5

The reduction of the augmented matrix to row-echelon form is as follows.

Step 1: r1 ↔ r2

1 2 1 02 1 1 −13 0 −2 5

Step 2: r2 ← r2 − 2r1 and r3 ← r3 − 3r1

1 2 1 00 −3 −1 −10 −6 −5 5

Step 3: r3 ← r3 − 2r2

1 2 1 00 −3 −1 −10 0 −3 7

Step 4: r2 ← − 13r2 and r3 ← − 1

3r3

1 2 1 00 1 1

313

0 0 1 − 73

The solution is given by: x1 = 19 , x2 = 10

9 , x3 = − 73

Exercise 56Which of the following matrices are not in reduced row-ehelon from and why?(a)

1 −2 0 00 0 0 00 0 1 00 0 0 1

(b)

1 0 0 30 2 0 −20 0 3 0

(c)

1 0 40 1 −20 0 0

Page 206: Fundamentals of Linear Algebra - Mathematics & Statisticsmelbial2/classes/Linear Algebra... · 2006-01-13 · 1.1 Systems of Linear Equations Many practical problems can be reduced

206 CHAPTER 7. SOLUTIONS TO REVIEW PROBLEMS

Solution.(a) No, because the matrix fails condition 1 of the definition. Rows of zerosmust be at the bottom of the matrix.(b) No, because the matrix fails condition 2 of the definition. Leading entry inrow 2 must be 1 and not 2.(c) Yes. The given matrix satisfies conditions 1 - 4

Exercise 57Use Gaussian elimination to convert the following matrix into a row-echelonmatrix.

1 −3 1 −1 0 −1−1 3 0 3 1 32 −6 3 0 −1 2−1 3 1 5 1 6

Solution.We follow the steps in Gauss-Jordan algorithm.

Step 1: r2 ← r2 + r1, r3 ← r3 − 2r1, and r4 ← r4 + r1

1 −3 1 −1 0 −10 0 1 2 1 20 0 1 2 −1 40 0 2 4 1 5

Step 2: r3 ← r3 − r2 and r4 ← r4 = 2r2

1 −3 1 −1 0 −10 0 1 2 1 20 0 0 0 −2 20 0 0 0 −1 1

Step 3: r3 ← − 12r3

1 −3 1 −1 0 −10 0 1 2 1 20 0 0 0 1 −10 0 0 0 −1 1

Step 4: r4 ← r4 + r3

1 −3 1 −1 0 −10 0 1 2 1 20 0 0 0 1 −10 0 0 0 0 0

Exercise 58Use Gauss-Jordan elimination to convert the following matrix into reduced row-

Page 207: Fundamentals of Linear Algebra - Mathematics & Statisticsmelbial2/classes/Linear Algebra... · 2006-01-13 · 1.1 Systems of Linear Equations Many practical problems can be reduced

207

echelon form.

−2 1 1 156 −1 −2 −361 −1 −1 −11−5 −5 −5 −14

Solution.Using the Gauss-Jordan to bring the given matrix into reduced row-echelon formas follows.

Step 1: r1 ↔ r3

1 −1 −1 −116 −1 −2 −36−2 1 1 15−5 −5 −5 −14

Step 2: r2 ← r2 − 6r1, r3 ← r3 + 2r1, and r4 ← r4 + 5r1

1 − 1 − 1 −110 5 4 300 − 1 − 1 − 70 −10 −10 −69

Step 3: r2 ↔ −r3

1 − 1 − 1 −110 1 1 70 5 4 300 −10 −10 −69

Step 4: r3 ← r3 − 5r2 and r4 ← r4 + 10r2

1 −1 −1 −110 1 1 60 0 −1 − 50 0 0 − 9

Step 5: r3 ← −r3 and r4 ← − 19r4

1 −1 −1 −110 1 1 60 0 1 50 0 0 1

Step 6: r1 ← r1 + r2

1 0 0 −50 1 1 60 0 1 50 0 0 1

Page 208: Fundamentals of Linear Algebra - Mathematics & Statisticsmelbial2/classes/Linear Algebra... · 2006-01-13 · 1.1 Systems of Linear Equations Many practical problems can be reduced

208 CHAPTER 7. SOLUTIONS TO REVIEW PROBLEMS

Step 7: r2 ← r2 − r3

1 0 0 −50 1 0 10 0 1 00 0 0 1

Step 8: r1 ← r1 + 5r4 and r2 ← r2 − 6r4

1 0 0 00 1 0 10 0 1 00 0 0 1

Exercise 59Solve the following system using Gauss-Jordan elimination.

3x1 + x2 + 7x3 + 2x4 = 132x1 − 4x2 + 14x3 − x4 = −105x1 + 11x2 − 7x3 + 8x4 = 592x1 + 5x2 − 4x3 − 3x4 = 39

Solution.The augmented matrix of the system is

3 1 7 2 132 −4 14 −1 −105 11 −7 8 592 5 −4 −3 39

The reduction of the augmented matrix into reduced row-echelon form isStep 1: r1 ← r1 − r2

1 5 −7 3 232 −4 14 −1 −105 11 −7 8 592 5 −4 −3 39

Step 2: r2 ← r2 − 2r1, r3 ← r3 − 5r1, and r4 ← r4 − 2r1

1 5 −7 3 230 −14 28 −7 −560 −14 28 −7 − 560 − 5 10 −9 − 7

Step 3: r3 ← r3 − r2

1 5 −7 3 230 −14 28 −7 −560 0 0 0 00 − 5 10 −9 − 7

Page 209: Fundamentals of Linear Algebra - Mathematics & Statisticsmelbial2/classes/Linear Algebra... · 2006-01-13 · 1.1 Systems of Linear Equations Many practical problems can be reduced

209

Step 4: r4 ↔ r3 and r2 ← − 114r2

1 5 −7 3 230 1 −2 1

2 40 −5 10 −9 −70 0 0 0 0

Step 5: r1 ← r1 − 5r2 and r3 ← r3 + 5r2

1 0 3 .5 30 1 −2 .5 40 0 0 −6.5 130 0 0 0 0

Step 6: r3 ← − 213r3

1 0 3 .5 30 1 −2 .5 40 0 0 1 −20 0 0 0 0

Step 7: r1 ← r1 − .5r3 and r2 ← r2 − .5rr

1 0 3 0 40 1 −2 0 50 0 0 1 −20 0 0 0 0

The corresponding system is given by

x1 + 3x3 = 4x2 − 2x3 = 5

x4 = −2

The general solution is: x1 = 4− 3t, x2 = 5 + 2t, x3 = t, x4 = −2

Exercise 60Find the rank of each of the following matrices.(a)

−1 −1 0 00 0 2 34 0 −2 13 −1 0 4

(b)

1 −1 32 0 4−1 −3 1

Page 210: Fundamentals of Linear Algebra - Mathematics & Statisticsmelbial2/classes/Linear Algebra... · 2006-01-13 · 1.1 Systems of Linear Equations Many practical problems can be reduced

210 CHAPTER 7. SOLUTIONS TO REVIEW PROBLEMS

Solution.(a) We reduce the given matrix to row-echelon form.

Step 1: r3 ← r3 + 4r1 and r4 ← r4 + 3r1

−1 −1 0 00 0 2 30 −4 −2 10 −4 0 4

Step 2: r4 ← r4 − r3

−1 −1 0 00 0 2 30 −4 −2 10 0 2 3

Step 3: r1 ← −r1 and r2 ↔ r3

1 1 0 00 −4 −2 10 0 2 30 0 2 3

Step 4: r4 ← r3 − r4

1 1 0 00 −4 −2 10 0 2 30 0 0 0

Step 5: r2 ← − 14r2 and r3 ← 1

2r3

1 1 0 00 1 .5 −.250 0 1 1.50 0 0 0

Thus, the rank of the given matrix is 3.(b) Apply the Gauss algorithm as follows.

Step 1: r2 ← r2 − 2r1 and r3 ← r3 + r1

1 −1 30 2 −20 −4 4

Step 2: r3 ← r3 + 2r2

1 −1 30 2 −20 0 0

Page 211: Fundamentals of Linear Algebra - Mathematics & Statisticsmelbial2/classes/Linear Algebra... · 2006-01-13 · 1.1 Systems of Linear Equations Many practical problems can be reduced

211

Step 3: r2 ← 12r2

1 −1 30 1 −10 0 0

Hence, the rank is 2

Exercise 61Choose h and k such that the following system has (a) no solutions, (b) exactlyone solution, and (c) infinitely many solutions.

{x1 − 3x2 = 1

2x1 − hx2 = k

Solution.The augmented matrix of the system is

(1 −3 12 −h k

)

By performing the operation r2 ← r2 − 2r1 we find(

1 −3 10 6− h k − 2

)

(a) The system is inconsistent if h = 6 and k 6= 2.(b) The system has exactly one solution if h 6= 6 and for any k. The solution isgiven by the formula:x1 = 3k−h

6−h , x2 = k−26−h .

(c) The system has infinitely many solutions if h = 6 and k = 2. The parametricform of the solution is: x1 = 1 + 3s, x2 = s

Exercise 62Solve the linear system whose augmented matrix is reduced to the following re-duced row-echelon form

1 0 0 −7 80 1 0 3 20 0 1 1 −5

Solution.The free variable is x4 = s. Solving by back-substitution one finds x1 = 8 +7s, x2 = 2− 3s, and x3 = −5− s

Exercise 63Solve the linear system whose augmented matrix is reduced to the following row-echelon form

1 −3 7 10 1 4 00 0 0 1

Page 212: Fundamentals of Linear Algebra - Mathematics & Statisticsmelbial2/classes/Linear Algebra... · 2006-01-13 · 1.1 Systems of Linear Equations Many practical problems can be reduced

212 CHAPTER 7. SOLUTIONS TO REVIEW PROBLEMS

Solution.Because of the last row the system is inconsistent

Exercise 64Solve the linear system whose augmented matrix is given by

1 1 2 8−1 −2 3 13 −7 4 10

Solution.The reduction of the augmented matrix to row-echelon form is

Step 1: r2 ← r2 + r1 and r3 ← r3 − 3r1

1 1 2 80 − 1 5 90 −10 −2 −14

Step 2: r2 ← −r2

1 1 2 80 1 −5 − 90 −10 −2 −14

Step 3: r3 ← r3 + 10r2

1 1 2 80 1 − 5 − 90 0 −52 −104

Step 4: r3 ← − 152r3

1 1 2 80 1 −5 −90 0 1 2

Using backward substitution we find the solution: x1 = 3, x2 = 1, x3 = 2

Exercise 65Find the value(s) of a for which the following system has a nontrivial solution.Find the general solution.

x1 + 2x2 + x3 = 0x1 + 3x2 + 6x3 = 02x1 + 3x2 + ax3 = 0

Solution.The augmented matrix of the system is

1 2 1 01 3 6 02 3 a 0

Page 213: Fundamentals of Linear Algebra - Mathematics & Statisticsmelbial2/classes/Linear Algebra... · 2006-01-13 · 1.1 Systems of Linear Equations Many practical problems can be reduced

213

Reducing this matrix to row-echelon form as follows.

Step 1: r2 ← r2 − r1 and r3 ← r3 − 2r1

1 2 1 00 1 5 00 −1 a− 2 0

Step 2: r3 ← r3 + r2

1 2 1 00 1 5 00 0 a + 3 0

If a = −3 then the rank of the coefficient matrix is less than the number ofunknowns. Therefore, by Theorem 6 the system has a nontrivial solution andconsequently infinitely many solutions. By letting x3 = s we find x1 = 9s andx2 = −5s

Exercise 66Solve the following homogeneous system.

x1 − x2 + 2x3 + x4 = 02x1 + 2x2 − x4 = 03x1 + x2 + 2x3 + x4 = 0

Solution.The augmented matrix of the system is

1 −1 2 1 02 2 0 −1 03 1 2 1 0

Reducing the augmented matrix to row-echelon form.

Step 1: r2 ← r2 − 2r1 and r3 ← r3 − 3r1

1 −1 2 1 00 4 −4 −3 00 4 −4 −2 0

Step 2: r3 ← r3 − r2

1 −1 2 1 00 4 −4 −3 00 0 0 1 0

Step 3: r2 ← 14r2

1 −1 2 1 00 1 −1 − 3

4 00 0 0 1 0

It follows that the rank of the coefficient matrix is less than the number ofunknowns so the system has infinitely many solutions given by the formulax1 = −s, x2 = s, x3 = s, and x4 = 0

Page 214: Fundamentals of Linear Algebra - Mathematics & Statisticsmelbial2/classes/Linear Algebra... · 2006-01-13 · 1.1 Systems of Linear Equations Many practical problems can be reduced

214 CHAPTER 7. SOLUTIONS TO REVIEW PROBLEMS

Exercise 67Let A be an m× n matrix.(a) Prove that if y and z are solutions to the homogeneous system Ax = 0 theny + z and cy are also solutions, where c is a number.(b) Give a counterexample to show that the above is false for nonhomogeneoussystems.

Solution.(a) Suppose that Ax = Ay = 0 and c ∈ IR then A(x + y) = Ax = Ay = 0 andA(cx) = cAx = 0.(b) The ordered pair (1, 0) satisfies the nonhomogeneous system

{x1 + x2 = 1x1 − x2 = 1

However, 2(1, 0) is not a solution

Exercise 68Show that the converse of Theorem 6 is false. That is, show the existence of anontrivial solution does not imply that the number of unknowns is greater thanthe number of equations.

Solution.Consider the system

x1 + x2 = 02x1 + 2x2 = 03x1 + 3x2 = 0

The system consists of three lines that coincide

Exercise 69 (Network Flow)The junction rule of a network says that at each junction in the network thetotal flow into the junction must equal the total flows out. To illustrate the useof this rule, consider the network shown in the accompanying diagram. Find thepossible flows in the network.

40 60

50

50

D

C

B

A

f_1 f_2

f_5f_4

f_3

Page 215: Fundamentals of Linear Algebra - Mathematics & Statisticsmelbial2/classes/Linear Algebra... · 2006-01-13 · 1.1 Systems of Linear Equations Many practical problems can be reduced

215

Solution.Equating the flow in with the flow out at each intersection, we obtain the fol-lowing system of four equations in the unknowns f1, f2, f3, f4, and f5.

f1 − f3 + f4 = 40(IntersectionA)f1 + f2 = 50(IntersectionB)

f2 + f3 + f5 = 60(IntersectionC)f4 + f5 = 50(IntersectionD)

The augmented matrix of the system is

1 0 −1 1 0 401 1 0 0 0 500 1 1 0 1 600 0 0 1 1 50

The reduction of this system to row-echelon form is carried out as follows.

Step 1: r2 ← r2 − r1

1 0 −1 1 0 400 1 1 −1 0 100 1 1 0 1 600 0 0 1 1 50

Step 2: r3 ← r3 − r2

1 0 −1 1 0 400 1 1 −1 0 100 0 0 1 1 500 0 0 1 1 50

Step 3: r4 ← r4 − r3

1 0 −1 1 0 400 1 1 −1 0 100 0 0 1 1 500 0 0 0 0 0

This shows that f3 and f5 play the role of parameters. Now, solving for theremaining variables we find the general solution of the system

f1 = f3 + f5 − 10f2 = −f3 − f5 + 60f4 = 50− f5

which gives all the possible flows

Chapter 2

Page 216: Fundamentals of Linear Algebra - Mathematics & Statisticsmelbial2/classes/Linear Algebra... · 2006-01-13 · 1.1 Systems of Linear Equations Many practical problems can be reduced

216 CHAPTER 7. SOLUTIONS TO REVIEW PROBLEMS

Exercise 108Compute the matrix

3(

2 1−1 0

)T

− 2(

1 −12 3

)

Solution.

3(

2 1−1 0

)T

− 2(

1 −12 3

)=

(4 −1−1 −6

)

Exercise 109Find w, x, y, and z.

1 2 w2 x 4y −4 z

=

1 2 −12 −3 40 −4 5

Solution.Equating the corresponding entries we find w = −1, x = −3, y = 0, and z = 5

Exercise 110Determine two numbers s and t such that the following matrix is symmetric.

A =

2 s t2s 0 s + t3 3 t

Solution.Since A is symmetric then AT = A; that is,

AT =

2 2s 3s 0 3t s + t t

=

2 s t2s 0 s + t3 3 t

Equating corresponding entries we find that s = 0 and t = 3

Exercise 111Let A be a 2× 2 matrix. Show that

A = a

(1 00 0

)+ b

(0 10 0

)+ c

(0 01 0

)+ d

(0 00 1

)

Solution.Let A be the matrix

A =(

a bc d

)

Page 217: Fundamentals of Linear Algebra - Mathematics & Statisticsmelbial2/classes/Linear Algebra... · 2006-01-13 · 1.1 Systems of Linear Equations Many practical problems can be reduced

217

Then

a

(1 00 0

)+ b

(0 10 0

)+ c

(0 01 0

)+ d

(0 00 1

)=

(a 00 0

)+

(0 b0 0

)+

(0 0c 0

)+

(0 00 d

)=

(a bc d

)= A

Exercise 112Let A =

[1 1 −1

], B =

[0 1 2

], C =

[3 0 1

]. If rA+sB+ tC =

0 show that s = r = t = 0.

Solution.A simple arithmetic yields the matrix

rA + sB + tC =[

r + 3t r + s −r + 2s + t]

The condition rA + sB + tC = 0 yields the system

r + 3t = 0r + s = 0−r + 2s + t = 0

The augmented matrix is

1 0 3 01 1 0 0−1 2 1 0

Step 1: r2 ← r2 − r1 and r3 ← r3 + r1

1 0 3 00 1 −3 00 2 4 0

Step 2: r3 ← r3 − 2r2

1 0 3 00 1 −3 00 0 10 0

Solving the corresponding system we find r = s = t = 0

Exercise 113Show that the product of two diagonal matrices is again a diagonal matrix.

Solution.Let D = (dij) be a diagonal matrix of size m × n and D′ = (d′ij) be an n × p

diagonal matrix. Let DD′ = (aij). Then aij =∑n

k=1 dikd′kj . If i = j thenaii = diid

′ii since for i 6= k we have dik = d′ki = 0. Now, if i 6= j then dikd′kj = 0

for all possible values of k. Hence, DD′ is diagonal

Page 218: Fundamentals of Linear Algebra - Mathematics & Statisticsmelbial2/classes/Linear Algebra... · 2006-01-13 · 1.1 Systems of Linear Equations Many practical problems can be reduced

218 CHAPTER 7. SOLUTIONS TO REVIEW PROBLEMS

Exercise 114Let A be an arbitrary matrix. Under what conditions is the product AAT de-fined?

Solution.Suppose A is an m×n matrix. Then AT is an n×m matrix. Since the numberof columns of A is equal to the number of rows of AT then AAT is always defined

Exercise 115(a) Show that AB = BA if and only if (A−B)(A + B) = A2 −B2.(b) Show that AB = BA if and only if (A + B)2 = A2 + 2AB + B2.

Solution.(a) Suppose that AB = BA then (A−B)(A+B) = A2+AB−BA−B2 = A2−B2

since AB −BA = 0. Conversely, suppose that (A−B)(A + B) = A2 −B2 thenby expanding the left-hand side we obtain A2−AB +BA−B2 = A2−B2. Thisimplies that AB = BA.(b) Suppose that AB = BA then (A + B)2 = A2 + AB + BA + B2 = A2 +2AB +B2. Conversely, suppose that (A+B)2 = A2 +2AB +B2. By expandingthe left-hand side we obtain A2 + AB + BA + B2 = A2 + 2AB + B2 and thisleads to AB = BA

Exercise 116Let A be a matrix of size m × n. Denote the columns of A by C1, C2, · · · , Cn.Let x be the n × 1 matrix with entries x1, x2, · · · , xn. Show that Ax = x1C1 +x2C2 + · · ·+ xnCn.

Solution.Suppose that A = (aij). Then

Ax =

a11 a12 · · · a1n

a21 a22 · · · a2n

......

...am1 amn · · · amn

x1

x2

...xn

=

a11x1 + a12x2 + · · ·+ a1nxn

a21x1 + a22x2 + · · ·+ a2nxn

...am1x1 + am2x2 + · · ·+ amnxn

= x1

a11

a21

...am1

+ x2

a12

a22

...am2

+ · · ·+ xn

a1n

a2n

...amn

= x1C1 + x2C2 + · · ·+ xnCn

Exercise 117Let A be an m× n matrix. Show that if yA = 0 for all y ∈ IRm then A = 0.

Page 219: Fundamentals of Linear Algebra - Mathematics & Statisticsmelbial2/classes/Linear Algebra... · 2006-01-13 · 1.1 Systems of Linear Equations Many practical problems can be reduced

219

Solution.Suppose the contrary. Then there exist indices i and j such that aij 6= 0. Let ybe the 1 ×m matrix such that the jth column is 1 and 0 elsewhere. Then the(1, j)−th entry of yA is aij 6= 0, a contradiction

Exercise 118An n× n matrix A is said to be idempotent if A2 = A.(a) Show that the matrix

A =12

(1 11 1

)

is idempotent(b) Show that if A is idempotent then the matrix (In −A) is also idempotent.

Solution.(a) Easy calculation shows that A2 = A.(b) Suppose that A2 = A then (In−A)2 = In−2A+A2 = In−2A+A = In−A

Exercise 119The purpose of this exercise is to show that the rule (ab)n = anbn does not holdwith matrix multiplication. Consider the matrices

A =(

2 −41 3

), B =

(3 2−1 5

)

Show that (AB)2 6= A2B2.

Solution.We have

(AB)2 =(

100 −4320 289

)

and

A2B2 =(

160 −460−5 195

)

Exercise 120Show that AB = BA if and only if AT BT = BT AT .

Solution.AB = BA if and only if (AB)T = (BA)T if and only if BT AT = AT BT

Exercise 121Suppose AB = BA and n is a non-negative integer.(a) Use induction to show that ABn = BnA.(b) Use induction and (a) to show that (AB)n = AnBn.

Page 220: Fundamentals of Linear Algebra - Mathematics & Statisticsmelbial2/classes/Linear Algebra... · 2006-01-13 · 1.1 Systems of Linear Equations Many practical problems can be reduced

220 CHAPTER 7. SOLUTIONS TO REVIEW PROBLEMS

Solution.(a) The equality holds for n = 0 and n = 1. Suppose that ABn = BnA. ThenABn+1 = (AB)Bn = (BA)Bn = B(ABn) = B(BnA) = Bn+1A.(b) The equality holds for n = 1. Suppose that (AB)n = AnBn. Then (AB)n+1 =(AB)n(AB) = (AnBn)(AB) = (AnBn)(BA) = An(BnB)A = An(Bn+1A) =An(ABn+1) = An+1Bn+1

Exercise 122Let A and B be symmetric matrices. Show that AB is symmetric if and only ifAB = BA.

Solution.AB is symmetric if and only if (AB)T = AB if and only if BT AT = AB if andonly if AB = BA

Exercise 123Show that tr(AAT ) is the sum of the squares of all the entries of A.

Solution.If A = (aij) then AT = (aji). Hence, AAT = (

∑nk=1 aikajk). Using the definition

of the trace we have

tr(AAT ) =n∑

i=1

(n∑

k=1

aikaik) =n∑

i=1

n∑

k=1

a2ik

Exercise 124(a) Find two 2× 2 singular matrices whose sum in nonsingular.(b) Find two 2× 2 nonsingular matrices whose sum is singular.

Solution.(a)

A =(

1 00 0

), B =

(0 00 1

)

(b)

A =(

1 00 1

), B =

( −1 00 −1

)

Exercise 125Show that the matrix

A =

1 4 02 5 03 6 0

is singular.

Solution.If B is a 3× 3 matrix such that BA = I3 then

b31(0) + b32(0) + b33(0) = 0

But this is equal to the (3, 3) entry of I3 which is 1. A contradiction

Page 221: Fundamentals of Linear Algebra - Mathematics & Statisticsmelbial2/classes/Linear Algebra... · 2006-01-13 · 1.1 Systems of Linear Equations Many practical problems can be reduced

221

Exercise 126Let

A =(

1 21 3

)

Find A−3.

Solution.

A−3 = (A−1)3 =(

3 −2−1 1

)3

=(

41 −30−15 11

)

Exercise 127Let

A−1 =(

2 −13 5

)

Find A.

Solution.Using Exercise 94, we find

A = (A−1)−1 =(

513

113

− 313

213

)

Exercise 128Let A and B be square matrices such that AB = 0. Show that if A is invertiblethen B is the zero matrix.

Solution.If A is invertible then B = InB = (A−1A)B = A−1(AB) = A−10 = 0

Exercise 129Find the inverse of the matrix

A =(

sin θ cos θ− cos θ sin θ

)

Solution.Using Exercise 94 we find

A−1 =(

sin θ − cos θcos θ sin θ

)

Exercise 130Which of the following are elementary matrices?(a)

1 1 00 0 10 1 0

Page 222: Fundamentals of Linear Algebra - Mathematics & Statisticsmelbial2/classes/Linear Algebra... · 2006-01-13 · 1.1 Systems of Linear Equations Many practical problems can be reduced

222 CHAPTER 7. SOLUTIONS TO REVIEW PROBLEMS

(b) (1 0−5 1

)

(c)

1 0 00 1 90 0 1

(d)

2 0 0 20 1 0 00 0 1 00 0 0 1

Solution.(a) No. This matrix is obtained by performing two operations: r2 ↔ r3 andr1 ← r1 + r3.(b) Yes: r2 ← r2 − 5r1.(c) Yes: r2 ← r2 + 9r3.(d) No: r1 ← 2r1 and r1 ← r1 + 2r4

Exercise 131Let A be a 4×3 matrix. Find the elementary matrix E, which as a premultiplierof A, that is, as EA, performs the following elementary row operations on A :(a) Multiplies the second row of A by -2.(b) Adds 3 times the third row of A to the fourth row of A.(c) Interchanges the first and third rows of A.

Solution.(a)

1 0 0 00 −2 0 00 0 1 00 0 0 1

(b)

1 0 0 00 1 0 00 0 1 00 0 3 1

(c)

0 0 1 00 1 0 01 0 0 00 0 0 1

Page 223: Fundamentals of Linear Algebra - Mathematics & Statisticsmelbial2/classes/Linear Algebra... · 2006-01-13 · 1.1 Systems of Linear Equations Many practical problems can be reduced

223

Exercise 132For each of the following elementary matrices, describe the corresponding ele-mentary row operation and write the inverse.(a)

E =

0 0 10 1 01 0 0

(b)

E =

1 0 0−2 1 00 0 1

(c)

E =

1 0 00 1 00 0 5

Solution.(a) r1 ↔ r3, E−1 = E.(b) r2 ← r2 − 2r1

1 0 02 1 00 0 1

(c) r3 ← 5r3

1 0 00 1 00 0 1

5

Exercise 133Consider the matrices

A =

3 4 12 −7 −18 1 5

, B =

8 1 52 −7 −13 4 1

, C =

3 4 12 −7 −12 −7 3

Find elementary matrices E1, E2, E3, and E4 such that(a)E1A = B, (b)E2B = A, (c)E3A = C, (d)E4C = A.

Solution.(a)

E1 =

0 0 10 1 01 0 0

Page 224: Fundamentals of Linear Algebra - Mathematics & Statisticsmelbial2/classes/Linear Algebra... · 2006-01-13 · 1.1 Systems of Linear Equations Many practical problems can be reduced

224 CHAPTER 7. SOLUTIONS TO REVIEW PROBLEMS

(b) E2 = E1.(c)

E3 =

1 0 00 1 0−2 0 1

(d)

E4 =

1 0 00 1 02 0 1

Exercise 134Determine if the following matrix is invertible.

1 6 42 4 −1−1 2 5

Solution.Consider the matrix

1 6 4 | 1 0 02 4 −1 | 0 1 0−1 2 5 | 0 0 1

Step 1: r2 ← r2 − 2r1 and r3 ← r3 + r1

1 6 4 | 1 0 00 −8 −9 | −2 1 00 8 9 | 1 0 1

Step 2: r3 ← r3 + r2

1 6 4 | 1 0 00 −8 −9 | −2 1 00 0 0 | −1 1 1

It follows that the given matrix is row equivalent to a matrix with a row ofzeros. By Theorem 255 this matrix is singular

Exercise 135For what values of a does the following homogeneous system have a nontrivialsolution? {

(a− 1)x1 + 2x2 = 02x1 + (a− 1)x2 = 0

Solution.Let A be the coefficient matrix of the given system, i.e.

A =(

a− 1 22 a− 1

)

Page 225: Fundamentals of Linear Algebra - Mathematics & Statisticsmelbial2/classes/Linear Algebra... · 2006-01-13 · 1.1 Systems of Linear Equations Many practical problems can be reduced

225

By Exercise 94, A is invertible if and only if (a − 1)2 − 4 6= 0 ,i.e. a 6= 3 anda 6= −1. In this case the trivial solution is the only solution. Hence, in order tohave nontrivial solution we must have a = −1 or a = 3

Exercise 136Find the inverse of the matrix

1 1 10 2 35 5 1

Solution.We first construct the matrix

1 1 1 | 1 0 00 2 3 | 0 1 05 5 1 | 0 0 1

Step 1: r3 ← r3 − 5r1

1 1 1 | 1 0 00 2 3 | 0 1 00 0 −4 | −5 0 1

Step 2: r2 ← 12r2 and r3 ← − 1

4r3

1 1 1 | 1 0 00 1 3

2 | 0 12 0

0 0 1 | 54 0 − 1

4

Step 3: r1 ← r1 − r2

1 0 − 12 | 1 − 1

2 00 1 3

2 | 0 12 0

0 0 1 | 54 0 − 1

4

Step 4: r2 ← r2 − 32r3 and r1 ← r1 + 1

2r3

1 0 0 | 138 − 1

2 − 18

0 1 0 | − 158

12

38

0 0 1 | 54 0 − 1

4

It follows that

A−1 =

138 − 1

2 − 18

− 158

12

38

54 0 − 1

4

Exercise 137What conditions must b1, b2, b3 satisfy in order for the following system to beconsistent?

x1 + x2 + 2x3 = b1

x1 + x3 = b2

2x1 + x2 + 3x3 = b3

Page 226: Fundamentals of Linear Algebra - Mathematics & Statisticsmelbial2/classes/Linear Algebra... · 2006-01-13 · 1.1 Systems of Linear Equations Many practical problems can be reduced

226 CHAPTER 7. SOLUTIONS TO REVIEW PROBLEMS

Solution.The augmented matrix of the system is

1 1 2 b1

1 0 1 b2

2 1 3 b3

Step 1: r2 ← r2 − r1 and r3 ← r3 − 2r1

1 1 2 b1

0 −1 −1 b2 − b1

0 −1 −1 b3 − 2b1

Step 2: r3 ← r3 − r2

1 1 2 b1

0 −1 −1 b2 − b1

0 0 0 b3 − b2 − b1

The system is consistent if and only if b3 − b2 − b1 = 0

Exercise 138Prove that if A is symmetric and nonsingular than A−1 is symmetric.

Solution.Let A be an invertible and symmetric n× n matrix. Then (A−1)T = (AT )−1 =A−1. That is, A−1 is symmetric

Exercise 139If

D =

4 0 00 −2 00 0 3

find D−1.

Solution.According to Exercise 100 we have

D−1 =

14 0 00 − 1

2 00 0 1

3

Exercise 140Prove that a square matrix A is nonsingular if and only if A is a product ofelementary matrices.

Solution.Suppose first that A is nonsingular. Then by Theorem 19, A is row equivalentto In. That is, there exist elementary matrices E1, E2, · · · , Ek such that In =

Page 227: Fundamentals of Linear Algebra - Mathematics & Statisticsmelbial2/classes/Linear Algebra... · 2006-01-13 · 1.1 Systems of Linear Equations Many practical problems can be reduced

227

EkEk−1 · · ·E1A. Then A = E−11 E−1

2 · · ·E−1k . But each E−1

i is an elementarymatrix by Theorem 18.Conversely, suppose that A = E1E2 · · ·Ek. Then (E1E2 · · ·Ek)−1A = In. Thatis, A is nonsingular

Exercise 141Prove that two m× n matrices A and B are row equivalent if and only if thereexists a nonsingular matrix P such that B = PA.

Solution.Suppose that A ∼ B. Then there exist elementary matrices E1, E2, · · · , Ek suchthat B = EkEk−1 · · ·E1A. Let P = EkEk−1 · · ·E1. Then by Theorem 18 andTheorem 15 (a), P is nonsingular.Conversely, suppose that B = PA, for some nonsingular matrix P. By Theorem19, P ∼ In. That is, In = EkEk−1 · · ·E1P. Thus, B = E−1

1 E−12 · · ·E−1

k A andthis implies that A ∼ B

Exercise 142Let A and B be two n × n matrices. Suppose A is row equivalent to B. Provethat A is nonsingular if and only if B is nonsingular.

Solution.Suppose that A ∼ B. Then by the previous exercise, B = PA, with P nonsingu-lar. If A is nonsingular then by Theorem 15 (a), B is nonsingular. Conversely,if B is nonsingular then A = P−1B is nonsingular

Exercise 143Show that the product of two lower (resp. upper) triangular matrices is againlower (resp. upper) triangular.

Solution.We prove by induction on n that the product of tow lower triangular matricesis lower triangular. For n = 2 we have

AB =(

a11 0a21 a22

)(b11 0b21 b22

)

=(

a11b11 0a21b11 + a22b21 a22b22

)

Thus, AB is lower triangular. So suppose that the result is true for all lowertriangular matrices of sise ≤ n−1. Let A and B be two lower triangular matricesof size n× n. Then

AB =(

a 0X A1

)(b 0Y B2

)

=(

ab 0bX + A1Y A1B1

)

Page 228: Fundamentals of Linear Algebra - Mathematics & Statisticsmelbial2/classes/Linear Algebra... · 2006-01-13 · 1.1 Systems of Linear Equations Many practical problems can be reduced

228 CHAPTER 7. SOLUTIONS TO REVIEW PROBLEMS

where X and Y are (n− 1)× 1 matrices and A1, B1 are (n− 1)× (n− 1) lowertriangular matrices. By the induction hypothesis, A1B1 is lower triangular sothat AB is lower triangular.The fact that the product of two upper triangular matrices is upper triangularfollows by transposition.

Exercise 144Show that a 2× 2 lower triangular matrix is invertible if and only if a11a22 6= 0and in this case the inverse is also lower triangular.

Solution.By Exercise 94, the lower triangular matrix

A =(

a11 0a21 a22

)

is invertible if and only in a11a22 6= 0 and in this case

A−1 =( 1

a110

− a21a11a22

1a22

)

Exercise 145Let A be an n × n matrix and suppose that the system Ax = 0 has only thetrivial solution. Show that Akx = 0 has only the trivial solution for any positiveinteger k.

Solution.Since Ax = 0 has only the trivial solution then A is invertible. By indiction onk and Theorem 15(a), Ak is invertible and consequently the system Akx = 0has only the trivial solution

Exercise 146Show that if A and B are two n× n matrices then A ∼ B.

Solution.Since A is invertible then A ∼ In. That is, there exist elementary matri-ces E1, E2, · · · , Ek such that In = EkEk−1 · · ·E1A. Similarly, there exist ele-mentary matrices F1, F2, · · · , Fl such that In = FlFl−1 · · ·F1B. Hence, A =E−1

1 E−12 · · ·E−1

k FlFl−1 · · ·F1B. That is, A ∼ B

Exercise 147Show that an n × n matrix invertible matrix A satisfies the property (P): IfAB = AC then B = C.

Solution.Suppose that A is invertible. Let B abd C be two n × n matrices such thatAB = AC. Then A(B − C) = 0. Multiplying both sides by A−1 to obtainB − C = 0 or B = C

Page 229: Fundamentals of Linear Algebra - Mathematics & Statisticsmelbial2/classes/Linear Algebra... · 2006-01-13 · 1.1 Systems of Linear Equations Many practical problems can be reduced

229

Exercise 148Show that an n × n matrix A is invertible if and only if the equation yA = 0has only the trivial solution.

Solution.Suppose that A is invertible and let y be a 1 × n matrix such that yA = 0.Multiplying from the left by A−1 we find y = 0.Conversely, suppose that the equation yA = 0 has only the trivial solution.Then AT x = 0 has only the trivial solution. By Theorem 19, AT is invertibleand consequently A is invertible

Exercise 149Let A be an n× n matrix such that An = 0. Show that In −A is invertible andfind its inverse.

Solution.We have (In−A)(In +A+A2+ · · ·+An−1 = In +A+ · · ·+An−1−A−A2−· · ·−An−1−An = In. Thus, In−A is invertible and (In−A)−1 = In +A+ · · ·+An−1

Exercise 150Let A = (aij(t)) be an m × n matrix whose entries are differentiable functionsof the variable t. We define dA

dt = (daij

dt ). Show that if the entries in A and Bare differentiable functions of t and the sizes of the matrices are such that thestated operations can be performed, then(a) d

dt (kA) = k dAdt .

(b) ddt (A + B) = dA

dt + dBdt .

(c) ddt (AB) = dA

dt B + AdBdt .

Solution.(a) d

dt (kA) = ( ddt (kaij)) = (k daij

dt ) = k(daij

dt ) = k dAdt .

(b) ddt (A + B) = ( d

dt (aij + bij)) = (daij

dt + dbij

dt ) = dAdt + dB

dt .(c) Use the product rule of differentiation

Exercise 151Let A be an n×n invertible matrix with entries being differentiable functions oft. Find a formula for dA−1

dt .

Solution.Since A is invertible then AA−1 = In. Taking derivative of both sides andusing part (c) of the previous exercise we find dA

dt A−1 + AdA−1

dt = 0. Thus,AdA−1

dt = −dAdt A−1. Now premultiply by A−1 to obtain dA−1

dt = −A−1 dAdt A−1

Exercise 152 (Sherman-Morrison formula)Let A be an n × n invertible matrix. Let u, v be n × 1 matrices such thatvT A−1u + 1 6= 0. Show that(a) (A + uvT )(A−1 − A−1uvT A−1

1+vT A−1u) = In.

(b) Deduce from (a) that A is invertible.

Page 230: Fundamentals of Linear Algebra - Mathematics & Statisticsmelbial2/classes/Linear Algebra... · 2006-01-13 · 1.1 Systems of Linear Equations Many practical problems can be reduced

230 CHAPTER 7. SOLUTIONS TO REVIEW PROBLEMS

Solution.(a) Using the properties of matrix multiplication we find

(A + uvT )(A−1 − A−1uvT A−1

1+vT A−1u) = In − uvT A−1

1+vT A−1u+ uvT A−1 − uvT A−1uvT A−1

1+vT A−1u=

In + uvT A−1vT A−1u−uvT A−1uvT A−1

1+vT A−1u= In + vT A−1u(uvT A−1−uvT A−1)

1+vT A−1u= In.

(b) It follows from Theorem 20 and (a)that A + uvT is invertible and

(A + uvT )−1 = A−1 − A−1uvT A−1

1 + vT A−1u

Exercise 153Let x = (x1, x2, · · ·xm)T be an m × 1 matrix and y = (y1, y2, · · · , yn)T be ann× n matrix. Construct the m× n matrix xyT .

Solution.Using matrix multiplication we have

xyT =

x1

x2

...xm

(y1, y2, · · · , yn)

=

x1y1 x1y2 · · · x1yn

x2y1 x2y2 · · · x2yn

......

...xmy1 xmy2 · · · xmyn

Exercise 154Show that a triangular matrix A with the property AT = A−1 is always diagonal.

Solution.Since A is invertible and upper triangular then A−1 is also upper triangular.Since A is upper triangular then AT is lower triangular. But AT = A−1. Thus,AT is both upper and lower triangular so it must be diagonal. Hence, A is alsodiagonal

Chapter 3

Exercise 186(a) Find the number of inversions in the permutation (41352).(b) Is this permutation even or odd?

Solution.(a) There are five inversions: (41), (43), (42), (32), (52).(b) The given permutation is odd

Page 231: Fundamentals of Linear Algebra - Mathematics & Statisticsmelbial2/classes/Linear Algebra... · 2006-01-13 · 1.1 Systems of Linear Equations Many practical problems can be reduced

231

Exercise 187Evaluate the determinant of each of the following matrices(a)

A =(

3 5−2 4

)

(b)

A =

−2 7 65 1 −23 8 4

Solution.(a) |A| = (3)(4)− (−2)(5) = 22.(b) |A| = −2(−14− 6)− 7(20 + 6) + 6(40− 3) = 0

Exercise 188Find all values of t for which the determinant of the following matrix is zero.

A =

t− 4 0 00 t 00 3 t− 1

Solution.|A| = t(t− 1)(t− 4) = 0 implies t = 0, t = 1, or t = 4

Exercise 189Solve for x

∣∣∣∣x −13 1− x

∣∣∣∣ =

∣∣∣∣∣∣

1 0 −32 x −61 3 x− 5

∣∣∣∣∣∣

Solution.Evaluating the determinants on both sides we find

x(1− x) + 3 = x(x− 5) + 18− 3(6− x)

Simplifying to obtain the quadratic equation 2x2 − 3x− 3 = 0 whose roots arex1 = 3−√33

4 and x2 = 3+√

334

Exercise 190Evaluate the determinant of the following matrix

A =

1 2 34 5 60 0 0

Solution.Since the given matrix has a row consisting of 0 then |A| = 0

Page 232: Fundamentals of Linear Algebra - Mathematics & Statisticsmelbial2/classes/Linear Algebra... · 2006-01-13 · 1.1 Systems of Linear Equations Many practical problems can be reduced

232 CHAPTER 7. SOLUTIONS TO REVIEW PROBLEMS

Exercise 191Evaluate the determinant of the following matrix.

∣∣∣∣∣∣∣∣∣∣

2 7 −3 8 30 −3 7 5 10 0 6 7 60 0 0 9 80 0 0 0 4

∣∣∣∣∣∣∣∣∣∣

Solution.The given matrix is upper triangular so that the determinant is the product ofentries on the main diagonal, i.e. equals to −1296

Exercise 192Use the row reduction technique to find the determinant of the following matrix.

A =

2 5 −3 −2−2 −3 2 −51 3 −2 2−1 −6 4 3

Solution.Step 1: r1 ← r1 + r4

B1 =

1 −1 1 1−2 −3 2 −51 3 −2 2−1 −6 4 3

and |B1| = |A|.Step 2: r2 ← r2 + 2r1 and r3 ← r3 + r4

B2 =

1 −1 1 10 −5 4 −30 −3 2 5−1 −6 4 3

and |B2| = |A|.Step 3: r4 ← r4 + r1

B3 =

1 −1 1 10 −5 4 −30 −3 2 50 −7 5 4

and |B3| = |A|.Step 4: r2 ← r2 − 2r3

B4 =

1 −1 1 10 1 0 −130 −3 2 50 −7 5 4

Page 233: Fundamentals of Linear Algebra - Mathematics & Statisticsmelbial2/classes/Linear Algebra... · 2006-01-13 · 1.1 Systems of Linear Equations Many practical problems can be reduced

233

and |B4| = |A|.Step 5: r3 ← r3 + 3r2 and r4 ← r4 + 7r2

B5 =

1 −1 1 10 1 0 −130 0 2 −340 0 5 −87

and |B5| = |A|.Step 6: r3 ← 1

2r3

B6 =

1 −1 1 10 1 0 −130 0 1 −170 0 5 −87

and |B6| = 12 |A|.

Step 7: r4 ← r4 − 5r3

B7 =

1 −1 1 10 1 0 −130 0 1 −170 0 0 − 2

and |B7| = 12 |A|. Thus, |A| = 2|B7| = −4

Exercise 193Given that ∣∣∣∣∣∣

a b cd e fg h i

∣∣∣∣∣∣= −6,

find(a) ∣∣∣∣∣∣

d e fg h ia b c

∣∣∣∣∣∣,

(b) ∣∣∣∣∣∣

3a 3b 3c−d −e −f4g 4h 4i

∣∣∣∣∣∣(c) ∣∣∣∣∣∣

a + g b + h c + id e fg h i

∣∣∣∣∣∣(d) ∣∣∣∣∣∣

−3a −3b −3cd e f

g − 4d h− 4e i− 4f

∣∣∣∣∣∣

Page 234: Fundamentals of Linear Algebra - Mathematics & Statisticsmelbial2/classes/Linear Algebra... · 2006-01-13 · 1.1 Systems of Linear Equations Many practical problems can be reduced

234 CHAPTER 7. SOLUTIONS TO REVIEW PROBLEMS

Solution.(a) The process involves row interchanges twice so

∣∣∣∣∣∣

d e fg h ia b c

∣∣∣∣∣∣= −6

(b) We have∣∣∣∣∣∣

a b cd e fg h i

∣∣∣∣∣∣= 1

3

∣∣∣∣∣∣

3a 3b 3cd e fg h i

∣∣∣∣∣∣

= 112

∣∣∣∣∣∣

3a 3b 3cd e f4g 4h 4i

∣∣∣∣∣∣= − 1

12

∣∣∣∣∣∣

3a 3b 3c−d −e −f4g 4h 4i

∣∣∣∣∣∣

Thus, ∣∣∣∣∣∣

3a 3b 3c−d −e −f4g 4h 4i

∣∣∣∣∣∣= 72

(c) ∣∣∣∣∣∣

a + g b + h c + id e fg h i

∣∣∣∣∣∣=

∣∣∣∣∣∣

a b cd e fg h i

∣∣∣∣∣∣= −6

(d)∣∣∣∣∣∣

a b cd e fg h i

∣∣∣∣∣∣= −1

3

∣∣∣∣∣∣

−3a −3b −3cd e fg h i

∣∣∣∣∣∣= −1

3

∣∣∣∣∣∣

−3a −3b −3cd e f

g − 4d h− 4e i− 4f

∣∣∣∣∣∣

Thus, ∣∣∣∣∣∣

−3a −3b −3cd e f

g − 4d h− 4e i− 4f

∣∣∣∣∣∣= 18

Exercise 194Determine by inspection the determinant of the following matrix.

1 2 3 4 56 7 8 9 1011 12 13 14 1516 17 18 19 202 4 6 8 10

Solution.The determinant is 0 since the first and the fifth rows are proportional

Page 235: Fundamentals of Linear Algebra - Mathematics & Statisticsmelbial2/classes/Linear Algebra... · 2006-01-13 · 1.1 Systems of Linear Equations Many practical problems can be reduced

235

Exercise 195Find the determinant of the 1× 1 matrix A = (3).

Solution.By definition the determinant of a 1× 1 matrix is just the entry of that matrix.Thus, |A| = 3

Exercise 196Let A be a 3× 3 matrix such that |2A| = 6. Find |A|.Solution.We have, 6 = |2A| = 23|A| = 8|A|. Thus, |A| = 3

4

Exercise 197Show that if n is any positive integer then |An| = |A|n.

Solution.The proof is by induction on n ≥ 1. The equality is valid for n = 1. Supposethat it is valid up to n. Then |An+1| = |AnA| = |An||A| = |A|n|A| = |A|n+1

Exercise 198Show that if A is an n× n skew-symmetric and n is odd then |A| = 0.

Solution.Since A is skew-symmetric then AT = −A. Taking the determinant of bothsides we find |AT | = | − A| = (−1)n|A| = −|A| since n is odd. Thus, 2|A| = 0and therefore |A| = 0

Exercise 199Show that if A is orthogonal, i.e. AT A = AAT = In then |A| = ±1. Note thatA−1 = AT .

Solution.Taking to determinant of both sides of the equality AT A = In to obtain|AT ||A| = 1 or |A|2 = 1 since |AT | = |A|. It follows that |A| = ±1

Exercise 200If A is a nonsingular matrix such that A2 = A, what is |A|?Solution.Taking the determinant of both sides to obtain |A2| = |A| or |A|(|A| − 1) = 0.Hence, either A is singular or |A| = 1

Exercise 201True or false: If

A =

1 0 01 2 03 1 0

then rank(A) = 3. Justify your answer.

Page 236: Fundamentals of Linear Algebra - Mathematics & Statisticsmelbial2/classes/Linear Algebra... · 2006-01-13 · 1.1 Systems of Linear Equations Many practical problems can be reduced

236 CHAPTER 7. SOLUTIONS TO REVIEW PROBLEMS

Solution.Since A has a column of zero then |A| = |AT | = 0. In this case, rank(A) < 3

Exercise 202Find out, without solving the system, whether the following system has a non-trivial solution

x1 − 2x2 + x3 = 02x1 + 3x2 + x3 = 03x1 + x2 + 2x3 = 0

Solution.The coefficient matrix

A =

1 −2 12 3 13 1 2

has determinant |A| = 0. By Theorem 32, the system has a nontrivial solution

Exercise 203For which values of c does the matrix

A =

1 0 −c−1 3 10 2c −4

have an inverse.

Solution.Finding the determinant we get |A| = 2(c + 2)(c − 3). The determinant is 0 ifc = −2 or c = 3

Exercise 204If |A| = 2 and |B| = 5, calculate |A3B−1AT B2|.

Solution.|A3B−1AT B2| = |A|3|B|−1|A||B|2 = |A|4|B| = 80

Exercise 205Show that |AB| = |BA|.

Solution.We have |AB| = |A||B| = |B||A| = |BA|

Exercise 206Show that |A + BT | = |AT + B| for any n× n matrices A and B.

Solution.We have |A + BT | = |(A + BT )T | = |AT + B|

Page 237: Fundamentals of Linear Algebra - Mathematics & Statisticsmelbial2/classes/Linear Algebra... · 2006-01-13 · 1.1 Systems of Linear Equations Many practical problems can be reduced

237

Exercise 207Let A = (aij) be a triangular matrix. Show that |A| 6= 0 if and only if aii 6= 0,for 1 ≤ i ≤ n.

Solution.Let A = (aij) be a triangular matrix. By Theorem 32, A is nonsingular if andonly if |A| 6= 0 and this is equivalent to a11a22 · · · ann 6= 0

Exercise 208Express ∣∣∣∣

a1 + b1 c1 + d1

a2 + b2 c2 + d2

∣∣∣∣as a sum of four determinants whose entries contain no sums.

Solution.∣∣∣∣

a1 + b1 c1 + d1

a2 + b2 c2 + d2

∣∣∣∣ =∣∣∣∣

a1 c1

a2 c2

∣∣∣∣ =∣∣∣∣

a1 d1

a2 d2

∣∣∣∣ =∣∣∣∣

b1 c1

b2 c2

∣∣∣∣ =∣∣∣∣

b1 d1

b2 d2

∣∣∣∣

Exercise 209Let

A =

3 −2 15 6 21 0 −3

(a) Find adj(A).(b) Compute |A|.Solution.(a) The matrix of cofactors is

C11 C12 C13

C21 C22 C23

C31 C32 C33

=

−18 17 −6− 6 −10 −2−10 − 1 28

The adjoint of A is the transpose of the cofactors matrix.

adj(A) =

−18 − 6 −1017 −10 −1− 6 − 2 28

(b) |A| = −18(−280 + 2) + 6(476− 6)− 10(−34− 60) = −94

Exercise 210Find the determinant of the matrix

A =

3 0 0 05 1 2 02 6 0 −1−6 3 1 0

Page 238: Fundamentals of Linear Algebra - Mathematics & Statisticsmelbial2/classes/Linear Algebra... · 2006-01-13 · 1.1 Systems of Linear Equations Many practical problems can be reduced

238 CHAPTER 7. SOLUTIONS TO REVIEW PROBLEMS

Solution.Expanding along the first row

|A| = 3C11 + 0C12 + 0C13 + 0C14 = −15

Exercise 211Find the determinant of the following Vandermonde matrix.

A =

1 1 1a b ca2 b2 c2

Solution.We have∣∣∣∣∣∣

1 1 1a b ca2 b2 c2

∣∣∣∣∣∣=

∣∣∣∣∣∣

1 0 1a b− c ca2 b2 − c2 c2

∣∣∣∣∣∣

=

∣∣∣∣∣∣

1 0 0a b− c c− aa2 b2 − c2 c2 − a2

∣∣∣∣∣∣=

∣∣∣∣b− c c− a

b2 − c2 c2 − a2

∣∣∣∣= (b− c)(c2 − a2)− (b2 − c2)(c− a)= (b− c)(c− a)(a− b)

Exercise 212Let A be an n× n matrix. Show that |adj(A)| = |A|n−1.

Solution.Suppose first that A is invertible. Then adj(A) = A−1|A| so that |adj(A)| =||A|A−1| = |A|n|A−1| = |A|n

|A| = |A|n−1. If A is singular then adj(A) is singu-lar. To see this, suppose there exists a square matrix B such that Badj(A) =adj(A)B = In. Then A = AIn = A(adj(A)B) = (Aadj(A))B = 0 and this leadsto adj(A) = 0 a contradiction to the fact that adj(A) is nonsingular. Thus,adj(A) is singular and consequently |adj(A)| = 0 = |A|n−1

Exercise 213If

A−1 =

3 0 10 2 33 1 −1

find adj(A).

Solution.The determinant of A−1 is |A−1| = −21 and this implies that |A| = − 1

21 . Thus,

adj(A) = |A|A−1 =

− 1

7 0 − 121

0 − 221 − 1

7− 1

7 − 121

121

Page 239: Fundamentals of Linear Algebra - Mathematics & Statisticsmelbial2/classes/Linear Algebra... · 2006-01-13 · 1.1 Systems of Linear Equations Many practical problems can be reduced

239

Exercise 214If |A| = 2, find |A−1 + adj(A)|.

Solution.We have |A−1 + adj(A)| = |A−1(In + |A|In)| = |A−1|(1 + |A|)n = 3n

|A| = 3n

2

Exercise 215Show that adj(αA) = αn−1adj(A).

Solution.The equality is valid for α = 0. So suppose that α 6= 0. Then adj(αA) =|αA|(αA)−1 = (α)n|A| 1αA−1 = (α)n−1|A|A−1 = (α)n−1adj(A)

Exercise 216Consider the matrix

A =

1 2 32 3 41 5 7

(a) Find |A|.(b) Find adj(A).(c) Find A−1.

Solution.(a) |A| = 1(21− 20)− 2(14− 4) + 3(10− 3) = 2.(b) The matrix of cofactors of A is

1 −10 71 4 −3−1 2 −1

The adjoint is the transpose of this cofactors matrix

adj(A)

1 1 −1−10 4 2

7 −3 −1

(c)

A−1 =adj(A)|A| =

12

12 − 1

2−5 2 1

72 − 3

2 − 12

Exercise 217Prove that if A is symmetric then adj(A) is also symmetric.

Solution.Suppose that AT = A. Then (adj(A))T = (|A|A−1)T = |A|(A−1)T = |A|(AT )−1 =|A|A−1 = adj(A)

Page 240: Fundamentals of Linear Algebra - Mathematics & Statisticsmelbial2/classes/Linear Algebra... · 2006-01-13 · 1.1 Systems of Linear Equations Many practical problems can be reduced

240 CHAPTER 7. SOLUTIONS TO REVIEW PROBLEMS

Exercise 218Prove that if A is a nonsingular triangular matrix then A−1 is also triangular.

Solution.Suppose that A = (aij) is a lower triangular invertible matrix. Then aij = 0 ifi < j. Thus, Cij = 0 if i > j since in this case Cij is the determinant of a lowertriangular matrix with at least one zero on the diagonal. Hence, adj(A) is lowertriangular and A−1 = adj(A)

|A| is also lower triangular

Exercise 219Let A be an n× n matrix.(a) Show that if A has integer entries and |A| = 1 then A−1 has integer entriesas well.(b) Let Ax = b. Show that if the entries of A and b are integers and |A| = 1then the entries of x are also integers.

Solution.(a) If A has integer entries then adj(A) has integer entries. If |A| = 1 thenA−1 = adj(A) has integer entries.(b) Since |A| = 1 then A is invertible and x = A−1b. By (a), A−1 has integerentries. Since b has integer entries then A−1b has integer entries

Exercise 220Show that if Ak = 0 for some positive integer k then A is singular.

Solution.Suppose the contrary. Then we can multiply the equation Ak = 0 by A−1 k− 1times to obtain A = 0 which is a contradiction

Exercise 221Use Cramer’s Rule to solve

x1 + 2x3 = 6−3x1 + 4x2 + 6x3 = 30−x1 − 2x2 + 3x3 = 8

Solution.

A =

1 0 2−3 4 6−1 −2 3

, |A| = 44.

A1 =

6 0 230 4 68 −2 3

, |A1| = −40.

A2 =

1 6 2−3 30 6−1 8 3

, |A2| = 72.

Page 241: Fundamentals of Linear Algebra - Mathematics & Statisticsmelbial2/classes/Linear Algebra... · 2006-01-13 · 1.1 Systems of Linear Equations Many practical problems can be reduced

241

A3 =

1 0 6−3 4 30−1 −2 8

, |A| = 152.

Thus, x1 = |A1||A| = − 10

11 , x2 = |A2||A| = 18

11 , x3 = |A3||A| = 38

11

Exercise 222Use Cramer’s Rule to solve

5x1 + x2 − x3 = 49x1 + x2 − x3 = 1x1 − x2 + 5x3 = 2

Solution.

A =

5 1 −19 1 −11 −1 5

, |A| = −16.

A1 =

4 1 −11 1 −12 −1 5

, |A1| = 12.

A2 =

5 4 −19 1 −11 2 5

, |A2| = −166.

A3 =

5 1 49 1 11 −1 2

, |A3| = −42.

Thus, x1 = |A1||A| = − 3

4 , x2 = |A2||A| = 83

8 , x3 = |A3||A| = 21

8

Chapter 4

Exercise 283Show that the midpoint of P (x1, y1, z1) and Q(x1, y2, z2) is the point

M(x1 + x2

2,y1 + y2

2,z1 + z2

2).

Solution.Let M = (x, y, z) be the midpoint of the line segment PQ. Then

−−→OM = −−→

OP +−−→PM = −−→

OP + 12

−−→PQ

= (x1, y1, z1) + 12 (x2 − x1, y2 − y1, z2 − z1)

= (x1+x22 , y1+y2

2 , z1+z22 )

Page 242: Fundamentals of Linear Algebra - Mathematics & Statisticsmelbial2/classes/Linear Algebra... · 2006-01-13 · 1.1 Systems of Linear Equations Many practical problems can be reduced

242 CHAPTER 7. SOLUTIONS TO REVIEW PROBLEMS

Exercise 284(a) Let ~n = (a, b, c) be a vector orthogonal to a plane P. Suppose P0(x0, y0, z0)is a point in the plane. Write the equation of the plane.(b) Find an equation of the plane passing through the point (3,−1, 7) and per-pendicular to the vector ~n = (4, 2,−5).

Solution.(a) Let P = (x, y, z) be an arbitrary point in the plane. Then the vector−−→P0P = (x−x0, y− y0, z− z0) is contained in the plane. Since ~n is orthogonal tothe plane then < ~n,

−−→P0P >= 0 and this leads to the equation a(x− x0) + b(y −

y0) + c(z − z0) = 0.(b) Here we have a = 4, b = 2, c = −5, x0 = 3, y0 = −1, and z0 = 7. Substitutingin the equation obtained in (a) to find 4x + 2y − 5z + 25 = 0

Exercise 285Find the equation of the plane through the points P1(1, 2,−1), P2(2, 3, 1) andP3(3,−1, 2).

Solution.First we find the normal vector as follows

~n = −−−→P1P2 ×−−−→P1P3 =

∣∣∣∣∣∣

~i ~j ~k1 1 22 −3 3

∣∣∣∣∣∣= 9~i +~j − 5~k

Using Exercise282 (a) we find 9(x−1)+(y−2)−5(z−1) = 0 or 9x+y−5z = 16

Exercise 286Find the parametric equations of the line passing through a point P0(x0, y0, z0)and parallel to a vector ~v = (a, b, c).

Solution.Let P = (x, y, z) be an arbitrary point on the line. Then −−→

PP0 = t~v; that is,(x− x0, y− y0, z− z0) = (ta, tb, tc). Hence, the parametric equations of the lineare x = x0 + ta, y = y0 + bt, z = z0 + tc

Exercise 287Compute < ~u,~v > when ~u = (2,−1, 3) and ~v = (1, 4,−1).

Solution.< ~u,~v >= (2)(1) + (−1)(4) + (3)(−1) = −5

Exercise 288Compute the angle between the vectors ~u = (−1, 1, 2) and ~v = (2, 1,−1).

Solution.If θ denotes the angle between the vectors then

cos θ =< ~u,~v >

||~u||||~v|| = −12.

Since 0 ≤ θ ≤ π then θ = 2π3

Page 243: Fundamentals of Linear Algebra - Mathematics & Statisticsmelbial2/classes/Linear Algebra... · 2006-01-13 · 1.1 Systems of Linear Equations Many practical problems can be reduced

243

Exercise 289For any vectors ~u and ~v we have

||~u× ~v||2 = ||~u||2||~v||2− < ~u,~v >2 .

Solution.Suppose that ~u = (x1, y1, z1) and ~v = (x2, y2, z2). Then ~u × ~v = (y1z2 −y2z1, x2z1 − x1z2, x1y2 − x2y1). Thus,

||~u× ~v||2 = (y1z2 − y2z1)2 + (x2z1 − x1z2)2 + (x1y2 − x2y1)2

On the other hand,

||~u||2||~v||2− < ~u,~v >2= (x21 + y2

1 + z21)(x2

2 + y22 + z2

2)− (x1x2 + y1y2 + z1z2)2.

Expanding one finds that the given equality holds

Exercise 290Show that ||~u× ~v|| is the area of the parallelogram with sides ~u and ~v.

Solution.Let θ be the angle between the vectors ~u and ~v. Constructing the parallelogramwith sides ~u and ~v one finds that its area is ||~u||||~v|| sin θ. By the previousexercise, we have

||~u× ~v||2 = ||~u||2||~v||2− < ~u,~v >2

= ||~u||2||~v||2 − ||~u||2||~v||2 sin2 θ= ||~u||2||~v||2(1− sin2 θ= ||~u||2||~v||2 cos2 θ

Since θ ∈ [0, π] then sin θ ≥ 0 and therefore ||~u× ~v|| = ||~u||||~v|| sin θ

Exercise 291Let P be the collection of polynomials in the indeterminate x. Let p(x) = a0 +ax +a2x

2 + · · · and q(x) = b0 +b1x+b2x2 +c . . . be two polynomials in P. Define

the operations:(a) Addition: p(x) + q(x) = a0 + b0 + (a1 + b1)x + (a2 + b2)x2 + · · ·(b) Multiplication by a scalar: αp(x) = αa0 + (αa1)x + (αa2)x2 + · · · .Show that P is a vector space.

Solution.Since P is a subset of the vector space of all functions defined on IR then itsuffices to show that P is a subspace. Indeed, the sum of two polynomilasis again a plynomial and the scalar multiplication by a polynomial is also aplynomial

Exercise 292Define on IR2 the following operations:(i) (x, y) + (x′, y′) = (x + x′, y + y, );(ii) α(x, y) = (αy, αx).Show that IR2 with the above operations is not a vector space.

Page 244: Fundamentals of Linear Algebra - Mathematics & Statisticsmelbial2/classes/Linear Algebra... · 2006-01-13 · 1.1 Systems of Linear Equations Many practical problems can be reduced

244 CHAPTER 7. SOLUTIONS TO REVIEW PROBLEMS

Solution.Let x 6= y. Then α(β(x, y)) = α(βy, βx) = (αβx, αβy) 6= (αβ)(x, y) then IR2

with the above operations is not a vector space

Exercise 293Let U = {p(x) ∈ P : p(3) = 0}. Show that U is a subspace of P.

Solution.Let p, q ∈ U and α ∈ IR. Then αp + q is a polynomial such that (αp + q)(3) =αp(3) + q(3) = 0. That is, αp + q ∈ U. This says that U is a subspace of P

Exercise 294Let Pn denote the collection of all polynomials of degree n. Show that Pn is asubspace of P.

Solution.Let p(x) = a0 +a1x+ · · ·+anxn, q(x) = b0 + b1x+ · · ·+ bnxn, and α ∈ R. Then(αp + q)(x)(αa0b0) + (αa1b1)x + · · ·+ (αanbn)xn ∈ Pn. Thus, Pn is a subspaceof P

Exercise 295Show that < f, g >=

∫ b

af(x)g(x)dx is an inner product on the space C([a, b])

of continuous functions.

Solution.(a) < f, f >=

∫ b

af2(x)dx ≥ 0. If f ≡ 0 then < f, f >= 0. Conversely, if

< f, f >= 0 then by the continuity of f , we must have f ≡ 0.

(b) α < f, g >= α∫ b

af(x)g(x)dx =

∫ b

a(αf)(x)g(x)dx =< αf, g > .

(c) < f, g >=< g, f > .(d) < f, g + h >=< f, g > + < f, h >

Exercise 296Show that if u and v are two orthogonal vectors of an inner product space, i.e.< u, v >= 0, then

||u + v||2 = ||u||2 + ||v||2.

Solution.Indeed,||u + v||2 =< u + v, u + v >=< u, u > −2 < u, v > + < v, v >= ||u||2 + ||v||2since < u, v >= 0 by orthogonality

Exercise 297(a) Prove that a line through the origin in IR3 is a subspace of IR3 under thestandard operations.(b) Prove that a line not through the origin in IR3 in not a subspace of IR3.

Page 245: Fundamentals of Linear Algebra - Mathematics & Statisticsmelbial2/classes/Linear Algebra... · 2006-01-13 · 1.1 Systems of Linear Equations Many practical problems can be reduced

245

Solution.(a) Let (L) be a line through the origin with direction vector (a, b, c). Then theparametric equations of (L) are given by

x(t) = ta, y(t) = tb, z(t) = tc.

Let (x1, y1, z1), (x2, y2, z2) ∈ (L) and α ∈ IR. Then

αx1 + x2 = (αt1 + t2)a = t3a

where t3 = αt1 + t2. Similarly, αy1 + y2 = t3b and αz1 + z2 = t3c. This showsthat α(x1, y1, z1) + (x2, y2, z2) ∈ (L).(b) Let (L) be the line through the point (−1, 4, 2) and with direction vector(1, 2, 3). Then the parametric equations of (L) are given by x(t) = −1+t, y(t) =4+2t, z(t) = 2+3t. It is clear that (−1, 4, 2) ∈ (L), (0, 6, 5) ∈ (L) but (−1, 4, 2)+(0, 6, 5) = (−1, 11, 7) 6∈ (L)

Exercise 298Show that the set S = {(x, y) : x ≤ 0} is not a vector space of IR2 under theusual operations of IR2.

Solution.(−1, 0) ∈ S but −2(−1, 0) = (2, 0) 6∈ S so S is not a vector space

Exercise 299Show that the collection C([a, b]) of all continuous functions on [a, b] with theoperations:

(f + g)(x) = f(x) + g(x)(αf)(x) = αf(x)

is a vector space.

Solution.Since for any continuous functions f and g and any scalar α the function αf +gis continuous then C([a, b]) is a subspace of F ([a, b]) and hence a vector space

Exercise 300Let S = {(a, b, a + b) : a, b ∈ IR}. Show that S is a subspace of IR3 under theusual operations.

Solution.Indeed, α(a, b, a+b)+(a′, b′, a′+b′) = (α(a+a′), α(b+b′), α(a+b+a′+b′)) ∈ S

Exercise 301Let V be a vector space. Show that if u, v, w ∈ V are such that u + v = u + wthen v = w.

Page 246: Fundamentals of Linear Algebra - Mathematics & Statisticsmelbial2/classes/Linear Algebra... · 2006-01-13 · 1.1 Systems of Linear Equations Many practical problems can be reduced

246 CHAPTER 7. SOLUTIONS TO REVIEW PROBLEMS

Solution.Using the properties of vector spaces we have v = v + 0 = v + (u + (−u)) =(v + u) + (−u) = (w + u) + (−u) = w + (u + (−u)) = w + 0 = w

Exercise 302Let H and K be subspaces of a vector space V.(a) The intersection of H and K, denoted by H ∩K, is the subset of V thatconsists of elements that belong to both H and K. Show that H∩V is a subspaceof V.(b) The union of H and K, denoted by H ∪K, is the susbet of V that consistsof all elements that belong to either H or K. Give, an example of two subspacesof V such that H ∪K is not a subspace.(c) Show that if H ⊂ K or K ⊂ H then H ∪K is a subspace of V.

Solution.(a) Let u, v ∈ H ∩K and α ∈ R. Then u, v ∈ H and u, v ∈ K. Since H and Kare subspaces then αu + v ∈ H and αu + v ∈ K that is αu + v ∈ H ∩K. Thisshows that H ∩K is a subspace.(b) One can easily check that H = {(x, 0) : x ∈ IR} and K = {(0, y) : y ∈ IR}are subspaces of IR2. The vector (1, 0) belongs to H and the vector (0, 1) belongsto K. But (1, 0)+(0, 1) = (1, 1) 6∈ H∪K. It follows that H∪K is not a subspaceof IR2.(c) If H ⊂ K then H ∪ K = K, a subspace of V. Similarly, if K ⊂ H thenH ∪K = H, again a subspace of V

Exercise 303Let u and v be vectors in an inner product vector space. Show that

||u + v||2 + ||u− v||2 = 2(||u||2 + ||v||2).Solution.Recall that ||u|| = √

< u, u >. Then

||u + v||2 + ||u− v||2 = < u + v, u + v > + < u− v, u− v >= < u, u > +2 < u, v > + < v, v >+ < u, u > −2 < u, v > + < v, v >= 2(< u, u > + < v, v >) = 2(||u||2 + ||v||2)

Exercise 304Let u and v be vectors in an inner product vector space. Show that

||u + v||2 − ||u− v||2 = 4 < u, v > .

Solution.Indeed,

||u + v||2 − ||u− v||2 = < u + v, u + v > − < u− v, u− v >= < u, u > +2 < u, v > + < v, v >− < u, u > +2 < u, v > − < v, v >= 4 < u, v >

Page 247: Fundamentals of Linear Algebra - Mathematics & Statisticsmelbial2/classes/Linear Algebra... · 2006-01-13 · 1.1 Systems of Linear Equations Many practical problems can be reduced

247

Exercise 305(a) Use Cauchy -Schwarz’s inequality to show that if u = (x1, x2, · · · , xn) ∈ IRn

and v = (y1, y2, · · · , yn) ∈ IRn then

(x1y1 + x2y2 + · · ·+ xnyn)2 ≤ (x21 + x2

2 + · · ·+ x2n)(y2

1 + y22 + · · ·+ y2

n).

(b) Use (a) to show

n2 ≤ (a1 + a2 + · · ·+ an)(1a1

+1a2

+ · · ·+ 1an

)

where ai > 0 for 1 ≤ i ≤ n.

Solution.(a) The standard inner product in IRn is given by < u, v >= x1y1 +x2y2 + · · ·+xnyn. By Cauchy - Schwarz inequality we have that < u, v >2≤< u, u >< v, v >that is

(x1y1 + x2y2 + · · ·+ xnyn)2 ≤ (x21 + x2

2 + · · ·+ x2n)(y2

1 + y22 + · · ·+ y2

n).

(a) Let xi =√

ai and yi = 1√ai

in (a)

Exercise 306Let C([0, 1]) be the vector space of continuous functions on [0, 1]. Define

< f, g >=∫ 1

0

f(x)g(x)dx.

(a) Show that < ., . > is an inner product on C([0, 1]).(b) Show that

[∫ 1

0

f(x)g(x)dx

]2

≤[∫ 1

0

f(x)2dx

] [∫ 1

0

g(x)2dx

].

This inequality is known as Holder’s inequality.(c) Show that

[∫ 1

0

(f(x) + g(x))2dx

] 12

≤[∫ 1

0

f(x)2dx

] 12

+[∫ 1

0

g(x)2dx

] 12

.

This inequality is known as Minkowski’s inequality.

Solution.(a) See Exercise 295(b) This is just the Cauchy-Schwarz’s inequality.(c) The inequality is definitely valid if

∫ 1

0(f(x) + g(x))2dx = 0. So we may

assume that this integral is nonzero. In this case, we have∫ 1

0(f(x) + g(x))2dx =

∫ 1

0(f(x) + g(x))(f(x) + g(x))dx

=∫ 1

0f(x)(f(x) + g(x))dx +

∫ 1

0g(x)(f(x) + g(x))dx

≤[∫ 1

0f2(x)dx

] 12

[∫ 1

0(f(x) + g(x))2dx

] 12

+[∫ 1

0g2(x)dx

] 12

[∫ 1

0(f(x) + g(x))2dx

] 12

Page 248: Fundamentals of Linear Algebra - Mathematics & Statisticsmelbial2/classes/Linear Algebra... · 2006-01-13 · 1.1 Systems of Linear Equations Many practical problems can be reduced

248 CHAPTER 7. SOLUTIONS TO REVIEW PROBLEMS

Now divide both sides of this inequality by[∫ 1

0(f(x) + g(x))2dx

] 12

to obtain thedesired inequality

Exercise 307Let W = span{v1, v2, · · · , vn}, where v1, v2, · · · , vn are vectors in V. Show thatany subspace U of V containing the vectors v1, v2, · · · , vn must contain W, i.e.W ⊂ U. That is, W is the smallest subspace of V containing v1, v2, · · · , vn.

Solution.Let U be a subspace of V containing the vectors v1, v2, · · · , vn. Let x ∈ W.Then x = α1v1 + α2v2 + · · ·+ αnvn for some scalars α1, α2, · · · , αn. Since U isa subspace then x ∈ U . This gives x ∈ U and consequently W ⊂ U

Exercise 308Express the vector ~u = (−9,−7,−15) as a linear combination of the vectors~v1 = (2, 1, 4), ~v2 = (1,−1, 3), ~v3 = (3, 2, 5).

Solution.The equation ~u = α~v1 + β ~v2 + γ ~v3 gives the system

2α + β + 3γ = −9α − β + 2γ = −74α + 3β + 5γ = −15

Solving this system (details omitted) we find α = −2, β = 1 and γ = −2

Exercise 309(a) Show that the vectors ~v1 = (2, 2, 2), ~v2 = (0, 0, 3), and ~v3 = (0, 1, 1) span IR3.(b) Show that the vectors ~v1 = (2,−1, 3), ~v2 = (4, 1, 2), and ~v3 = (8,−1, 8) donot span IR3.

Solution.(a) Indeed, this follows because the coefficient matrix

A =

2 2 20 0 30 1 1

of the system Ax = b is invertible for all b ∈ IR3 ( |A| = −6)(b) This follows from the fact that the coefficient matrix with rows the vectors~v1, ~v2, and ~v3 is singular

Exercise 310Show that

M22 = span{(

1 00 0

),

(0 10 0

),

(0 01 0

),

(0 00 1

)}

Page 249: Fundamentals of Linear Algebra - Mathematics & Statisticsmelbial2/classes/Linear Algebra... · 2006-01-13 · 1.1 Systems of Linear Equations Many practical problems can be reduced

249

Solution.

Indeed, every(

a bc d

)can be written as

a

(1 00 0

)+ b

(0 10 0

)+ c

(0 01 0

)+ d

(0 00 1

)}

Exercise 311(a) Show that the vectors ~v1 = (2,−1, 0, 3), ~v2 = (1, 2, 5,−1), and ~v3 = (7,−1, 5, 8)are linearly dependent.(b) Show that the vectors ~v1 = (4,−1, 2) and ~v2 = (−4, 10, 2) are linearly inde-pendent.

Solution.(a) Suppose that α1 ~v1 + α2 ~v2 + α3 ~v3 = ~0. This leads to the system

2α1 + α2 + 7α3 = 0−α1 + 2α2 − α3 = 0

5α2 + 5α3 = 03α1 − α2 + 8α3 = 0

The augmented matrix of this system is

2 −1 0 3 01 2 5 −1 07 −1 5 8 0

The reduction of this matrix to row-echelon form is carried out as follows.

Step 1: r1 ← r1 − 2r2 and r3 ← r3 − 7r2

0 − 5 −10 6 01 2 5 −1 00 −15 −30 15 0

Step 2: r1 ↔ r2

1 2 5 −1 00 − 5 −10 6 00 −15 −30 15 0

Step 2: r3 ← r3 − 3r2

1 2 5 −1 00 −5 −10 6 00 0 0 −3 0

The system has a nontrivial solution so that {~v1, ~v2, ~v3} is linearly dependent.(b) Suppose that α(4,−1, 2) + β(−4, 10, 2) = (0, 0, 0) this leads to the system

4α1 − 4α2 = 0−α1 + 10α2 = 02α2 + 2α2 = 0

Page 250: Fundamentals of Linear Algebra - Mathematics & Statisticsmelbial2/classes/Linear Algebra... · 2006-01-13 · 1.1 Systems of Linear Equations Many practical problems can be reduced

250 CHAPTER 7. SOLUTIONS TO REVIEW PROBLEMS

This system has only the trivial solution so that the given vectors are linearlyindependent

Exercise 312Show that the {u, v} is linearly dependent if and only if one is a scalar multipleof the other.

Solution.Suppose that {u, v} is linearly dependent. Then there exist scalars α and β notboth zero such that αu + βv = 0. If α 6= 0 then u = −β

αv, i.e. u is a scalarmultiple of v. A similar argument if β 6= 0.Conversely, suppose that u = λv then 1u + (−λ)v = 0. This shows that {u, v}is linearly dependent

Exercise 313Let V be the vector of all real-valued functions with domain IR. If f, g, h aretwice differentiable functions then we define w(x) by the determinant

w(x) =

∣∣∣∣∣∣

f(x) g(x) h(x)f ′(x) g′(x) h′(x)f ′′(x) g′′(x) h′′(x)

∣∣∣∣∣∣

We call w(x) the Wronskian of f, g, and h. Prove that f, g, and h are linearlyindependent if and only if w(x) 6= 0.

Solution.Suppose that αf(x) + βg(x) + γh(x) = 0 for all x ∈ IR. Then this leads to thesystem

f(x) g(x) h(x)f ′(x) g′(x) h′(x)f ′′(x) g′′(x) h′′(x)

αβγ

Thus {f(x), g(x), h(x)} is linearly independent if and only if the coefficient ma-trix of the above system is invertible and this is equivalent to w(x) 6= 0

Exercise 314Use the Wronskian to show that the functions ex, xex, x2ex are linearly indepen-dent.

Solution.Indeed,

w(x) =

∣∣∣∣∣∣

ex xex x2ex

ex ex + xex 2xex + x2ex

ex 2ex + xex 2ex + 4xex + x2ex

∣∣∣∣∣∣= 2ex 6= 0

Exercise 315Show that {1 + x, 3x + x2, 2 + x− x2} is linearly independent in P2.

Page 251: Fundamentals of Linear Algebra - Mathematics & Statisticsmelbial2/classes/Linear Algebra... · 2006-01-13 · 1.1 Systems of Linear Equations Many practical problems can be reduced

251

Solution.Suppose that α(1 + x) + β(3x + x2) + γ(2 + x− x2) = 0 for all x ∈ IR. That is,(β− 2γ)x2 + (α + 3β + γ)x + α + 2γ = 0 for all x ∈ IR. This leads to the system

β − 2γ = 0α + 3β + γ = 0α + 2γ = 0

Solving this system by Gauss elimination (details omitted) one finds α = β =γ = 0. That is {1 + x, 3x + x2, 2 + x− x2} is linearly independent

Exercise 316Show that {1 + x, 3x + x2, 2 + x− x2} is a basis for P2.

Solution.Since dim(P2) = 3 and {1 + x, 3x + x2, 2 + x − x2} is linearly independent bythe previous exercise then {1 + x, 3x + x2, 2 + x− x2} is a basis by Theorem 58

Exercise 317Find a basis of P3 containing the linearly independent set {1 + x, 1 + x2}.

Solution.Note first that any polynomial in the span of {1 + x, 1 + x2} is of degree atmost 2. Thus, x3 6∈ span{1 + x, 1 + x2} so that the set {1 + x, 1 + x2, x3} islinearly independent. Since 1 is not in the span {1, 1 + x, 1 + x2, x3} is linearlyindependent. Since dim(P3) = 4 then {1, 1 + x, 1 + x2, x3} is a basis of P3

Exercise 318Let ~v1 = (1, 2, 1), ~v2 = (2, 9, 0), ~v3 = (3, 3, 4). Show that S = {~v1, ~v2, ~v3} is abasis for IR3.

Solution.Since dim(R3) = 3 then by Theorem 58 it suffices to show that the given vectorsare linearly independent, that is the matrix A whose rows consists of the givenvectors is invertible which is the case since |A| = −1 6= 0

Exercise 319Let S be a susbet of IRn with n+1 vectors. Is S linearly independent or linearlydependent?

Solution.By Theorem 53 S is linearly dependent

Exercise 320Find a basis for the vector space M22 of 2× 2 matrices.

Page 252: Fundamentals of Linear Algebra - Mathematics & Statisticsmelbial2/classes/Linear Algebra... · 2006-01-13 · 1.1 Systems of Linear Equations Many practical problems can be reduced

252 CHAPTER 7. SOLUTIONS TO REVIEW PROBLEMS

Solution.We have already shown that

M22 = span

{M1 =

(1 00 0

),M2 =

(0 10 0

),M3 =

(0 01 0

),M4

(0 00 1

)}

Now, if α1M1 + α2M2 + α3M3 + α4M4 = 0 then(

α1 α2

α3 α4

)=

(0 00 0

)

and this shows that α1 = α2 = α3 = α4 = 0. Hence, {M1,M2,M3, M4} is abasis for M22

Exercise 321(a) Let U,W be subspaces of a vector space V. Show that the set U + W ={u + w : u ∈ U and w ∈ W} is a subspace of V.(b) Let M22 be the collection of 2×2 matrices. Let U be the collection of matricesin M22 whose second row is zero, and W be the collection of matrices in M22

whose second column is zero. Find U + W.

Solution.(a) Let v1 = u1 + w1 and v2 = u2 + w2 be two vectors in U + W and α ∈ IR.Then αv1 + v2 = (αu1 + u2) + (αw1 + w2) ∈ U + W since αu1 + u2 ∈ U andαw1 + w2 ∈ W. Hence, U + W is a subspace of V.

(b) U + W ={(

a 00 0

): a ∈ IR

}

Exercise 322Let S = {(1, 1)T , (2, 3)T } and S′ = {(1, 2)T , (0, 1)T } be two bases of IR2. Let~u = (1, 5)T and ~v = (5, 4)T .

(a) Find the coordinate vectors of ~u and ~v with respect to the basis S.(b) What is the transition matrix P from S to S′?(c) Find the coordinate vectors of ~u and ~v with respect to S′ using P.(d) Find the coordinate vectors of ~u and ~v with respect to S′ directly.(e) What is the transition matrix Q from S′ to S?(f) Find the coordinate vectors of ~u and ~v with respect to S using Q.

Solution.

(a) Since ~u = −7(1, 1)T + 4(2, 3)T then [~u]S =( −7

4

). Similarly, [~v]S =

(7−1

).

(b) Since (11

)=

(12

)−

(01

)

Page 253: Fundamentals of Linear Algebra - Mathematics & Statisticsmelbial2/classes/Linear Algebra... · 2006-01-13 · 1.1 Systems of Linear Equations Many practical problems can be reduced

253

and (23

)= 2

(12

)−

(01

)

Thus, the transition matrix from S to S′ is

P =(

1 2−1 −1

)

(c)

[~u]S′ = P [~u]S =(

1 2−1 −1

)( −74

)=

(13

)

and

[~v]S′ = P [~v]S =(

1 2−1 −1

)(7−1

)=

(5−6

)

(d) The equation (15

)= α

(12

)+ β

(01

)

implies α = 5 and β = 3 so

[~u]S′ =(

13

)

Similarly, (54

)= α

(12

)+ β

(01

)

implies α = 4 and β = −6 so that

[~v]S′ =(

5−6

)

(e)

Q = P−1 =( −1 −2

1 1

)

(f)

[~u]S = Q[~u]S′ =( −1 −2

1 1

)(13

)=

( −14

)

and

[~v]S = Q[~v]S′ =( −1 −2

1 1

)(5−6

)=

(7−1

)

Exercise 323Suppose that S′ = { ~u1, ~u2, ~u3} is a basis of IR3, where ~u1 = (1, 0, 1), ~u2 =(1, 1, 0), ~u3 = (0, 0, 1). Let S = {~v1, ~v2, ~v3}. Suppose that the transition matrixfrom S to S′ is

1 1 22 1 1−1 −1 1

Determine S.

Page 254: Fundamentals of Linear Algebra - Mathematics & Statisticsmelbial2/classes/Linear Algebra... · 2006-01-13 · 1.1 Systems of Linear Equations Many practical problems can be reduced

254 CHAPTER 7. SOLUTIONS TO REVIEW PROBLEMS

Solution.We have

~v1 = ~u1 + 2 ~u2 − ~u3 =

320

~v2 = ~u1 + ~u2 − ~u3 =

210

~v3 = 2 ~u1 + ~u2 + ~u3 =

313

Exercise 324Show that if A is not a square matrix then either the row vectors of A or thecolumn vectors of A are linearly dependent.

Solution.Let A be an n×m matrix with n 6= m. Suppose that the row vectors of A arelinearly independent. Then rank(A) = n. If the column vectors of A are linearlyindependent then rank(AT ) = m. But rank(A) = rank(AT ) and this impliesn = m a contradiction. Thus, the column vectors of A are linearly dependent

Exercise 325Prove that the row vectors of an n×n invertible matrix A form a basis for IRn.

Solution.Suppose that A is an n× n invertible matrix. Then rank(A) = n. By Theorem71 the rows of A are linearly independent. Since dim(IRn) = n then by Theorem55 the rows of A form a basis for IRn

Exercise 326Compute the rank of the matrix

A =

1 2 2 −13 6 5 01 2 1 2

and find a basis for the row space of A.

Solution.The reduction of this matrix to row-echelon form is as follows.

Step 1: r2 ← r2 − 3r1 and r3 ← r3 − r1

1 2 2 −10 0 −1 30 0 −1 3

Page 255: Fundamentals of Linear Algebra - Mathematics & Statisticsmelbial2/classes/Linear Algebra... · 2006-01-13 · 1.1 Systems of Linear Equations Many practical problems can be reduced

255

Step 2: r3 ← r3 − r2

1 2 2 −10 0 −1 30 0 0 0

Hence, rank(A) = 2 and {(1, 2, 2,−1), (0, 0, 1,−3)} is a basis for the row spaceof A

Exercise 327Let U and W be subspaces of a vector space V. We say that V is the direct sumof U and W if and only if V = U +W and U ∩W = {0}. We write V = U ⊕W.Show that V is the direct sum of U and W if and only if every vector v in Vcan be written uniquely in the form v = u + w.

Solution.Suppose that V = U ⊕W . Let v ∈ V such that v = u1 + w1 and v = u2 + w2.Then u1 − u2 = w2 − w1. This says that u1 − u2 ∈ U ∩ W = {0}. That is,u1 − u2 = 0 or u1 = u2. Hence, w1 = w2.Conversely, suppose that every vector in v ∈ V can be written uniquely as thesum of a vector in u ∈ U and a vector in w ∈ W. Suppose that v ∈ U ∩W. Thenv ∈ U and v ∈ W. But v = v + 0 so that v = 0

Exercise 328Let V be an inner product space and W is a subspace of V. Let W⊥ = {u ∈V :< u, w >= 0, ∀w ∈ W}.(a) Show that W⊥ is a subspace of V.(b) Show that V = W ⊕W⊥.

Solution.(a) Let u1, u2 ∈ W⊥ and α ∈ IR. Then < αu1 + u2, w >= α < u1, w > + <u2, w >= 0 for all w ∈ W. Hence, αu1 + u2 ∈ W and this shows that W⊥ is asubspace of V.(b) We first show that W ∩W⊥ = {0}. Indeed, if 0 6= w ∈ W ∩W⊥ then w ∈ Wand w ∈ W⊥ and this implies that < w, w >= 0 which contradicts the fact thatw 6= 0. Hence, we must have W ∩W⊥ = {0}. Next we show that V = W +W⊥.Let {v1, v2, · · · , vn} be an orthonormal basis of W. Then by Exercise 266 (d),every vector v ∈ V can be written as a sum of the form v = w1 + w2 wherew1 ∈ W and w2 ∈ W⊥

Exercise 329Let U and W be subspaces of a vector space V . Show that dim(U + W ) =dim(U) + dim(W )− dim(U ∩W ).

Solution.If U∩W = {0} then we are done. So assume that U∩W 6= {0}. Then U∩W hasa basis, say {z1, z2, · · · , zk}. Extend this to a basis {z1, z2, · · · , zk, uk+1, · · · , um}of U and to a basis {z1, z2, · · · , zk, wk+1, · · · , wn} of W. Let v ∈ U + W . Thenv = u + w = (α1z1 + α2z2 + · · · + αkzk + αk+1uk+1 + · · · + αmum) + (β1z1 +

Page 256: Fundamentals of Linear Algebra - Mathematics & Statisticsmelbial2/classes/Linear Algebra... · 2006-01-13 · 1.1 Systems of Linear Equations Many practical problems can be reduced

256 CHAPTER 7. SOLUTIONS TO REVIEW PROBLEMS

β2z2 + · · · + βkzk + βk+1wk+1 + · · · + βnwn) = γ1z1 + γ2z2 + · · · + γkzk +γk+1uk+1 + · · · + γmum + δk+1wk+1 + · · · + δnwn. This says that U + W =span{z1, z2, · · · , zk, uk+1, · · · , um, wk+1, · · · , wn}. Now, suppose that γ1z1+γ2z2+· · · + γkzk + γk+1uk+1 + · · · + γmum + δk+1wk+1 + · · · + δnwn = 0. Thenδk+1wk+1 + · · · + δnwm = −(γ1z1 + · · · + γkzk + γk+1uk+1 + · · · + γmum ∈ U.Hence, δk+1wk+1 + · · · + δnwn ∈ U ∩ W and this implies that δk+1wk+1 +· · · + δnwn = σ1z1 + σ2z2 + · · · + σkzk or δk+1wk+1 + · · · + δnwn − σ1z1 −σ2z2 − · · · − σkzk = 0. Since {z1, z2, · · · , zk, wk+1, · · · , wn} is linearly indepen-dent then δk+1 = · · · = δn = 0. This implies that γ1z1 + γ2z2 + · · · + γkzk +γk+1uk+1 + · · · + γmum = 0. Because {z1, z2, · · · , zk, uk+1, · · · , um} is linearlyindependent then we must have γ1 = · · · = γk = 0. Thus, we have shown that{z1, z2, · · · , zk, uk+1, · · · , um, wk+1, · · · , wn} is a basis of U + W. It follows thatdim(U + W ) = dim(U) + dim(W )− dim(U ∩W )

Exercise 330Show that cos 2x ∈ span{cos2 x, sin2 x}.

Solution.This follows from the trigonometric identity cos (2x) = cos2 x− sin2 x

Exercise 331If X and Y are subsets of a vector space V and if X ⊂ Y , show that spanX ⊂spanY.

Solution.First of all note that X ⊂ Y ⊂ spanY. Let u ∈ spanX. Then u = α1x1 +α2x2 +· · ·+αnxn where x1, x2, · · · , xn ∈ X. But for each i, αixi ∈ spanY. Since spanYis a subspace then u ∈ spanY

Exercise 332Show that the vector space F (IR) of all real-valued functions defined on IR is aninfinite-dimensional vector space.

Solution.We use the proof by contradiction. Suppose that dim(F (IR)) = n. Then any Sof m > n functions must be linearly dependent. But S = {1, x, · · · , xn} ⊂ F (IR)is linearly independent set with n + 1 functions, a contradiction. Thus, F (IR)must be an infinite-dimensional vector space

Exercise 333Show that {sin x, cos x} is linearly independent in the vector space F ([0, 2π]).

Solution.Suppose that α and β are scalars such that α sin x+β cosx = 0 for all x ∈ [0, 2π].In particular, taking x = 0 yields β = 0. By taking, x = π

2 we find α = 0

Exercise 334Find a basis for the vector space V of all 2× 2 symmetric matrices.

Page 257: Fundamentals of Linear Algebra - Mathematics & Statisticsmelbial2/classes/Linear Algebra... · 2006-01-13 · 1.1 Systems of Linear Equations Many practical problems can be reduced

257

Solution.

If A =(

a bc d

)∈ V then we must have b = c. Hence

A =(

a bc d

)= a

(1 00 0

)+ c

(0 11 0

)+ d

(0 00 1

)

That is,

V = span

{(1 00 0

),

(0 11 0

),

(0 00 1

)}

It remains to show that the set S ={(

1 00 0

),

(0 11 0

),

(0 00 1

)}is

linearly independent. Indeed, if

α

(1 00 0

)+ β

(0 11 0

)+ γ

(0 00 1

)=

(0 00 0

)

Then α = β = γ = 0. Thus, S is a basis and dim(V ) = 3

Exercise 335Find the dimension of the vector space Mmn of all m× n matrices.

Solution.For 1 ≤ i ≤ m and 1 ≤ j ≤ n let Aij be the matrix whose (i, j)th entry is 1and 0 otherwise. Let S = {Aij : 1 ≤ i ≤ m and 1 ≤ j ≤ n}. We will show thatS is a basis of Mmn. Indeed, if A = (aij) ∈ Mmn then A =

∑mi=1

∑nj=1 aijAij .

That is, Mmn = spanS. To show that S is linearly independent, we supposethat

∑mi=1

∑nj=1 αijAij = 0. Then the left-hand side is an m × n matrix with

entries αij . By the equality of matrices we conclude that αij = 0 that is S islinearly indepndent and hence a basis for Mmn. Also, dim(Mmn) = mn

Exercise 336Let A be an n × n matrix. Show that there exist scalars a0, a1, a2, · · · , an2 notall 0 such that

a0In + a1A + a2A2 + · · ·+ an2An2

= 0. (7.1)

Solution.Since dim(Mnn) = n2 then the n2 + 1 matrices In, A, A2, · · · , An2

are linearlydependent. So there exist scalars a0, a1, · · · , an2 not all 0 such that (7.1) holds

Exercise 337Show that if A is an n × n invertible skew-symmetric matrix then n must beeven.

Page 258: Fundamentals of Linear Algebra - Mathematics & Statisticsmelbial2/classes/Linear Algebra... · 2006-01-13 · 1.1 Systems of Linear Equations Many practical problems can be reduced

258 CHAPTER 7. SOLUTIONS TO REVIEW PROBLEMS

Solution.Since A is skew-symmetric then AT = −A. Taking the determinant of bothsides to obtain |A| = |AT | = (−1)n|A|. Since A is invertible then |A| 6= 0 andconsequently (−1)n = 1. That is, n is even

Chapter 5

Exercise 363Show that λ = −3 is an eigenvalue of the matrix

A =

5 8 164 1 8−4 −4 −11

and then find the corresponding eigenspace V−3.

Solution.The characteristic equation of the matrix A is

λ− 5 −8 −16−4 λ− 1 −8−4 −4 λ + 11

Expanding the determinant and simplifying we obtain

(λ + 3)2(λ− 1) = 0.

Thus, λ = −3 is an eigenvalue of A.A vector x = (x1, x2, x3)T is an eigenvector corresponding to an eigenvalue λ ifand only if x is a solution to the homogeneous system

(λ− 5)x1 − 8x2 − 16x3 = 0−4x1 + (λ− 1)x2 − 8x3 = 04x1 + 4x2 + (λ + 11)x3 = 0

(7.2)

If λ = −3, then (7.2) becomes−8x1 − 8x2 − 16x3 = 0−4x1 − 4x2 − 8x3 = 04x1 + 4x2 + 8x3 = 0

(7.3)

Solving this system yields

x1 = −s− 2t, x2 = s, x3 = t

The eigenspace corresponding to λ = −3 is

V−3 =

−2t− s

st

: s, t ∈ IR

= span

−201

,

−110

Page 259: Fundamentals of Linear Algebra - Mathematics & Statisticsmelbial2/classes/Linear Algebra... · 2006-01-13 · 1.1 Systems of Linear Equations Many practical problems can be reduced

259

Exercise 364Find the eigenspaces of the matrix

A =(

3 08 −1

)

Solution.The characteristic equation of the matrix A is

(λ− 3 0−8 λ + 1

)

Expanding the determinant and simplifying we obtain

(λ− 3)2(λ + 1) = 0.

Thus, λ = 3 and λ = −1 are the eigenvalues of A.A vector x = (x1, x2)T is an eigenvector corresponding to an eigenvalue λ if andonly if x is a solution to the homogeneous system

{(λ− 3)x1 = 0−8x1 + (λ + 1)x2 = 0 (7.4)

If λ = 3, then (7.4) becomes{ −8x1 + 4x2 = 0 (7.5)

Solving this system yields

x1 =12s, x2 = s

The eigenspace corresponding to λ = 3 is

V3 ={(

12ss

): s ∈ IR

}= span

{(121

)}

If λ = −1, then (7.4) becomes{ −4x1 = 0−8x1 = 0 (7.6)

Solving this system yieldsx1 = 0, x2 = s

The eigenspace corresponding to λ = −1 is

V−1 ={(

0s

): s ∈ IR

}= span

{(01

)}

Page 260: Fundamentals of Linear Algebra - Mathematics & Statisticsmelbial2/classes/Linear Algebra... · 2006-01-13 · 1.1 Systems of Linear Equations Many practical problems can be reduced

260 CHAPTER 7. SOLUTIONS TO REVIEW PROBLEMS

Exercise 365Find the characteristic polynomial, eigenvalues, and eigenspaces of the matrix

A =

2 1 12 1 −2−1 0 −2

Solution.The characteristic equation of the matrix A is

λ− 2 −1 −1−2 λ− 1 21 0 λ + 2

Expanding the determinant and simplifying we obtain

(λ− 3)2(λ + 1)2 = 0.

Thus, λ = 3 and λ = −1 are the eigenvalues of A.A vector x = (x1, x2, x3)T is an eigenvector corresponding to an eigenvalue λ ifand only if x is a solution to the homogeneous system

(λ− 2)x1 − x2 − x3 = 0−2x1 + (λ− 1)x2 + 2x3 = 0x1 + (λ + 2)x3 = 0

(7.7)

If λ = 3, then (7.7) becomes

x1 − x2 − x3 = 0−2x1 + 2x2 + 2x3 = 0x1 + 5x3 = 0

(7.8)

Solving this system yields

x1 = −5s, x2 = −6s, x3 = s

The eigenspace corresponding to λ = 3 is

V3 =

−5s−6ss

: s ∈ IR

= span

−5−61

If λ = −1, then (7.7) becomes−3x1 − x2 − x3 = 0−2x1 − 2x2 + 2x3 = 0x1 + x3 = 0

(7.9)

Solving this system yields

x1 = −s, x2 = 2s, x3 = s

Page 261: Fundamentals of Linear Algebra - Mathematics & Statisticsmelbial2/classes/Linear Algebra... · 2006-01-13 · 1.1 Systems of Linear Equations Many practical problems can be reduced

261

The eigenspace corresponding to λ = −1 is

V−1 =

−s2ss

: s ∈ IR

= span

−121

Exercise 366Find the bases of the eigenspaces of the matrix

A =

−2 0 1−6 −2 019 5 −4

Solution.The characteristic equation of the matrix A is

λ + 2 −16 λ + 2 0−19 −5 λ + 4

Expanding the determinant and simplifying we obtain

(λ + 8)(λ2 + 1) = 0.

Thus, λ = −8 is the only eigenvalue of A.A vector x = (x1, x2, x3)T is an eigenvector corresponding to an eigenvalue λ ifand only if x is a solution to the homogeneous system

(λ + 2)x1 − x3 = 06x1 + (λ + 2)x2 = 0−19x1 − 5x2 + (λ + 4)x3 = 0

(7.10)

If λ = −8, then (7.10) becomes

−6x1 − x3 = 06x1 − 6x2 = 0−19x1 − 5x2 − 4x3 = 0

(7.11)

Solving this system yields

x1 = −16s, x2 = −1

6s, x3 = s

The eigenspace corresponding to λ = −8 is

V−8 =

− 1

6s− 1

6ss

: s ∈ IR

= span

− 1

6− 1

61

Exercise 367Show that if λ is a nonzero eigenvalue of an invertible matrix A then 1

λ is aneigenvalue of A−1.

Page 262: Fundamentals of Linear Algebra - Mathematics & Statisticsmelbial2/classes/Linear Algebra... · 2006-01-13 · 1.1 Systems of Linear Equations Many practical problems can be reduced

262 CHAPTER 7. SOLUTIONS TO REVIEW PROBLEMS

Solution.Let x be an eigenvector of A corresponding to the nonzero eigenvalue λ. ThenAx = λx. Multiplying both sides of this equality by A−1 and then dividing theresulting equality by λ to obtain A−1x = 1

λx. That is, x is an eigenvector ofA−1 corresponding to the eigenvalue 1

λ

Exercise 368Show that if λ is an eigenvalue of a matrix A then λm is an eigenvalue of Am

for any positive integer m.

Solution.Let x be an eigenvector of A corresponding to the eigenvalue λ. Then Ax = λx.Multiplying both sides by A to obtain A2x = λAx = λ2x. Now, multiplyingthis equality by A to obtain A3x = λ3x. Continuing in this manner, we findAmx = λmx

Exercise 369(a) Show that if D is a diagonal matrix then Dk, where k is a positive integer,is a diagonal matrix whose entries are the entries of D raised to the power k.(b) Show that if A is similar to a diagonal matrix D then Ak is similar to Dk.

Solution.(a) We will show by induction on k that if

D =

d11 0 · · · 00 d22 · · · 0...

......

0 0 · · · dnn

then

Dk =

dk11 0 · · · 00 dk

22 · · · 0...

......

0 0 · · · dknn

Indeed, the result is true for k = 1. Suppose true up to k − 1 then

Dk = Dk−1D =

dk−111 0 · · · 00 dk−1

22 · · · 0...

......

0 0 · · · dk−1nn

d11 0 · · · 00 d22 · · · 0...

......

0 0 · · · dnn

=

dk11 0 · · · 00 dk

22 · · · 0...

......

0 0 · · · dknn

(b)Suppose that D = P−1AP. Then D2 = (P−1AP )(P−1AP ) = P−1AP 2.Thus, by induction on k one finds that Dk = P−1AkP

Page 263: Fundamentals of Linear Algebra - Mathematics & Statisticsmelbial2/classes/Linear Algebra... · 2006-01-13 · 1.1 Systems of Linear Equations Many practical problems can be reduced

263

Exercise 370Show that the identity matrix In has exactly one eigenvalue. Find the corre-sponding eigenspace.

Solution.The characteristic equation of In is (λ − 1)n = 0. Hence, λ = 1 is the onlyeigenvalue of In. The corresponding eigenspace is V1 = span{e1, e2, · · · , en} =IRn

Exercise 371Show that if A ∼ B then(a) AT ∼ BT .(b) A−1 ∼ B−1.

Solution.(a) Suppose that A ∼ B and let P be an invertible matrix such that B =P−1AP. Taking the transpose of both sides we obtain BT = (PT )−1AT PT ;that is, AT ∼ BT .(b) Suppose that A and B are invertible and B = P−1AP. Taking the inverseof both sides we obtain B−1 = P−1A−1P. Hence A−1 ∼ B−1

Exercise 372If A is invertible show that AB ∼ BA for all B.

Solution.Suppose that A is an n× n invertible matrix. Then BA = A−1(AB)A. That isAB ∼ BA

Exercise 373Let A be an n× n nilpotent matrix, i.e. Ak = 0 for some positive ineteger k.

(a) Show that λ = 0 is the only eigenvalue of A.(b) Show that p(λ) = λn.

Solution.(a) If λ is an eigenvalue of A then there is a nozero vector x such that Ax = λx.By Exercise 368, λk is an eigenvalue of Ak and Akx = λkx. But Ak = 0 soλkx = 0 and since x 6= 0 we must have λ = 0.(b) Since p(λ) is of degree n and 0 is the only eigenvalue of A then p(λ) = λn

Exercise 374Suppose that A and B are n×n similar matrices and B = P−1AP. Show that ifλ is an eigenvalue of A with corresponding eigenvector x then λ is an eigenvalueof B with corresponding eigenvector P−1x.

Solution.Since λ is an eigenvalue of A with corresponding eigenvector x then Ax = λx.Postmultiply B by P−1 to obtain BP−1 = P−1A. Hence, BP−1x = P−1Ax =λP−1x. This says that λ is an eigenvalue of B with corresponding eigenvectorP−1x

Page 264: Fundamentals of Linear Algebra - Mathematics & Statisticsmelbial2/classes/Linear Algebra... · 2006-01-13 · 1.1 Systems of Linear Equations Many practical problems can be reduced

264 CHAPTER 7. SOLUTIONS TO REVIEW PROBLEMS

Exercise 375Let A be an n×n matrix with n odd. Show that A has at least one real eigenvalue.

Solution.The characteristic polynomial is of degree n. The Fundamental Theorem ofAlgebra asserts that such a polynomial has exactly n roots. A root in this casecan be either a complex number or a real number. But if a root is complex thenits conjugate is also a root. Since n is odd then there must be at least one realroot

Exercise 376Consider the following n× n matrix

A =

0 1 0 00 0 1 00 0 0 1−a0 −a1 −a2 −a3

Show that the characterisitc polynomial of A is given by p(λ) = λ4 + a3λ3 +

a2λ2 + a1λ + a0. Hence, every monic polynomial (i.e. the coefficient of the

highest power of λ is 1) is the characteristic polynomial of some matrix. A iscalled the companion matrix of p(λ).

Solution.The characterisitc polynomial of A is

p(λ) =

∣∣∣∣∣∣∣∣

λ −1 0 00 λ −1 00 0 λ −1a0 a1 a2 λ + a3

∣∣∣∣∣∣∣∣Expanding this determinant along the first row we find

p(λ) = λ

∣∣∣∣∣∣

λ −1 00 λ −1a1 a2 λ + a3

∣∣∣∣∣∣+

∣∣∣∣∣∣

0 −1 00 λ −1a0 a2 λ + a3

∣∣∣∣∣∣= λ[λ(λ2 + a3λ + a2) + a1] + a0

= λ4 + a3λ3 + a2λ

2 + a1λ + a0

Exercise 377Find a matrix P that diagonalizes

A =

0 0 −21 2 11 0 3

Solution.From Exercise 346 the eigenspaces corresponding to the eigenvalues λ = 1 andλ = 2 are

V1 = span

−211

Page 265: Fundamentals of Linear Algebra - Mathematics & Statisticsmelbial2/classes/Linear Algebra... · 2006-01-13 · 1.1 Systems of Linear Equations Many practical problems can be reduced

265

and

V2 = span

−101

,

010

Let ~v1 = (−2, 1, 1)T , ~v2 = (−1, 0, 1), and ~v3 = (0, 1, 0)T . It is easy to verify thatthese vectors are linearly independent. The matrices

P =

−2 −1 01 0 11 1 0

and

D =

1 0 00 2 00 0 2

satisfy AP = PD or D = P−1AP

Exercise 378Show that the matrix A is diagonalizable.

A =

0 1 00 0 14 −17 8

Solution.From Exercise 5.13, the eigenvalues of A are λ = 4, λ = 2+

√3 and λ = 2−√3.

Hence, by Theorem 80 A is diagonalizable

Exercise 379Show that the matrix A is not diagonalizable.

A =

2 1 12 1 −2−1 0 −2

Solution.By Exercise 365, the eigenspaces of A are

V−1 =

−s2ss

: s ∈ IR

= span

−121

and

V3 =

−5s−6ss

: s ∈ IR

= span

−5−61

Since there are only two eigenvectors of A then A is not diagonalizable

Page 266: Fundamentals of Linear Algebra - Mathematics & Statisticsmelbial2/classes/Linear Algebra... · 2006-01-13 · 1.1 Systems of Linear Equations Many practical problems can be reduced

266 CHAPTER 7. SOLUTIONS TO REVIEW PROBLEMS

Exercise 380Show that if A is diagonalizable then the rank of A is the number of nonzeroeigenvalues of A.

Solution.Suppose that A is diagonalizable. Then there exist matrices P and D suchthat D = P−1AP, where D is diagonal with the diagonal entries being theeigenvalues of A. Hence, rank(D) = rank(P−1AP ) = rank(AP ) = rank(A) byTheorem 70. But rank(D) is the number of nonzero eigenvalues of A

Exercise 381Show that A is diagonalizable if and only if AT is diagonalizable.

Solution.Suppose that A is diagonalizable. Then there exist matrices P and D such thatD = P−1AP, with D diagonal. Taking the transpose of both sides to obtainD = DT = PT AT (P−1)T = Q−1AT Q with Q = (P−1)T = (PT )−1. Hence, AT

is diagonalizable. Similar argument for the converse

Exercise 382Show that if A and B are similar then A is diagonalizable if and only if B isdiagonalizable.

Solution.Suppose that A ∼ B. Then there exists an invertible matrix P such thatB = P−1AP. Suppose first that A is diagonalizable. Then there exist an in-vertible matrix Q and a diagonal matrix D such that D = Q−1AQ. Hence,B = P−1QDQ−1 and this implies D = (P−1Q)−1B(P−1Q). That is, B is diago-nalizable. For the converse, repeat the same argument using A = (P−1)−1BP−1

Exercise 383Give an example of two diagonalizable matrices A and B such that A+B is notdiagonalizable.

Solution.Consider the matrices

A =(

2 10 −1

), B =

( −1 00 2

)

The matrix A has the eigenvalues λ = 2 and λ = −1 so by Theorem 80, A isdiagonalizable. Similar argument for the matrix B. Let C = A + B then

C =(

1 10 1

)

This matrix has only one eigenvalue λ = 1 with corresponding eigenspace (de-tails omitted)

V1 = span{(1, 0)T }

Page 267: Fundamentals of Linear Algebra - Mathematics & Statisticsmelbial2/classes/Linear Algebra... · 2006-01-13 · 1.1 Systems of Linear Equations Many practical problems can be reduced

267

Hence, there is only one eigenvector of C and by Theorem 78, C is not diago-nalizable

Exercise 384Show that the following are equivalent for a symmetric matrix A.(a) A is orthogonal.(b) A2 = In.(c) All eigenvalues of A are ±1.

Solution.(a) ⇒ (b): Suppose that A is orthogonal. Then A−1 = AT . This impliesIn = A−1A = AT A = AA = A2.(b) ⇒ (c): Suppose that A2 = In. By Theorem 81, there exist an orthogonalmatrix P and a diagonal matrix D such that D = PT AP. This implies thatPDPT = A. Hence, In = A2 = PDPT PDPT = PD2PT or D2 = In since Pis orthogonal. If λi is an eigenvalue of A then λi is on the main diagonal of Dand since D2 = In then λ2

i = 1. It follows that λi = ±1.(c) ⇒ (a): Suppose that all the eigenvalues of A are ±1. Since A is symmetricthen D = PT AP. Thus, In = D2 = PT APPT AP = PT A2P. Thus, In = A2 =AT A and by Theorem 20, A is invertible with A−1 = AT . This means that A isorthogonal

Exercise 385A matrix that we obtain from the identity matrix by writing its rows in a differentorder is called permutation matrix. Show that every permutation matrix isorthogonal.

Solution.‘The columns of a permutation matrix is a rearrangement of the orthonormal set{e1, e2, · · · , en}. Thus, by Theorem 63 a permutation matrix is always orthogonal

Exercise 386Let A be an n× n skew symmetric matrix. Show that(a) In + A is nonsingular.(b) P = (In −A)(In + A)−1 is orthogonal.

Solution.(a) We will show that the homogeneous system (In + A)x = 0 has only thetrivial solution. So let x ∈ IRn such that (In + A)x = 0. Then 0 =< x, x +Ax >=< x, x > + < x, Ax >=< x, x > +xT Ax. But xT Ax ∈ IR so that(xT Ax)T = xT Ax, i.e. xT AT x = xT Ax and since AT = −A then xT Ax = 0.Hence, < x, x >= 0. This leads to x = 0.

(b) It is easy to check that (In−A)(In+A) = (In+A)(In−A). Then, PPT =(In−A)(In +A)−1(In−A)−1(In +A) = (In−A)[(In−A)(In +A)]−1(In +A) =(In−A)([(In+A)(In−A)]−1(In+A) = (In−A)(In−A)−1(In+A)−1(In+A) = In.Thus, P is orthogonal

Page 268: Fundamentals of Linear Algebra - Mathematics & Statisticsmelbial2/classes/Linear Algebra... · 2006-01-13 · 1.1 Systems of Linear Equations Many practical problems can be reduced

268 CHAPTER 7. SOLUTIONS TO REVIEW PROBLEMS

Exercise 387We call square matrix E a projection matrix if E2 = E = ET .

(a) If E is a projection matrix, show that P = In − 2E is orthogonal andsymmetric.(b) If P is orthogonal and symmetric, show that E = 1

2 (In − P ) is a projectionmatrix.

Solution.(a) PT = In − 2ET = In − 2E so that P is symmetric. Moreover, PPT =(In − 2E)(In − 2E) = In − 4E + 4E2 = In − 4E + 4E = In. Hence, P isorthogonal.(b) Suppose P is a symmetric and orthogonal n × n matrix. Then ET =12 (In−PT ) = 1

2 (In−P ) = E and E2 = 14 (In−P )(In−P ) = 1

4 (In−2P +P 2) =14 (2In − 2P ) = E

Chapter 6

Exercise 416Show that the function T : IR2 → IR3 defined by T (x, y) = (x + y, x− 2y, 3x) isa linear transformation.

Solution.Let (x1, y1), (x2, y2) ∈ IR2 and α ∈ IR. Then

T (α(x1, y1) + (x2, y2)) = T (αx1 + x2, αy1 + y2)= (αx1 + x2 + αy1 + y2, αx1 + x2 − 2αy1 − 2y2, 3αx1 + 3x2)= α(x1 + y1, x1 − 2y1, 3x1) + (x2 + y2, x2 − 2y2, 3x2)= αT (x1, y1) + T (x2, y2)

Hence, T is a linear transformation

Exercise 417(a) Show that D : Pn −→ Pn−1 given by D(p) = p′ is a linear transformation.(b) Show that I : Pn −→ Pn+1 given by I(p) =

∫ x

0p(t)dt is a linear transforma-

tion.

Solution.(a) Let p, q ∈ Pn and α ∈ IR then

D[αp(x) + q(x)] = (αp(x) + q(x))′

= αp′(x) + q′(x) = αD[p(x)] + D[q(x)]

Thus, D is a linear transformation.

(b) Let p, q ∈ Pn and α ∈ IR then

I[αp(x) + q(x)] =∫ x

0(αp(t) + q(t))dt

= α∫ x

0p(t)dt +

∫ x

0q(t)dt = αI[p(x)] + I[q(x)]

Page 269: Fundamentals of Linear Algebra - Mathematics & Statisticsmelbial2/classes/Linear Algebra... · 2006-01-13 · 1.1 Systems of Linear Equations Many practical problems can be reduced

269

Hence, I is a linear transformation.

Exercise 418If T : IR3 −→ IR is a linear transformation with T (3,−1, 2) = 5 and T (1, 0, 1) =2. Find T (−1, 1, 0).

Solution.Suppose that (−1, 1, 0) = α(3,−1, 2) + β(1, 0, 1). This leads to a linear systemin the unknowns α and β. Solving this system we find α = −1 and β = 2. SinceT is linear then

T (−1, 1, 0) = −T (3,−1, 2) + 2T (1, 0, 1) = −5 + 4 = −1

Exercise 419Let T : IR3 −→ IR2 be the transformation T (x, y, z) = (x, y). Show that T islinear. This transformation is called a projection.

Solution.Let (x1, y1, z1 ∈ IR3, (x2, y2, z2) ∈ IR3 and α ∈ IR. Then

T (α(x1, y1, z1) + (x2, y2, z2)) = T (αx1 + x2, αy1 + y2, αz1 + z2)= (αx1 + x2, αy1 + y2) = α(x1, y1) + (x2, y2)= αT (x1, y1, z1) + T (x2, y2, z2)

Hence, T is a linear transformation

Exercise 420Let θ be a given angle. Define T : IR2 −→ IR2 by

T

(xy

)=

(cos θ − sin θsin θ cos θ

)(xy

)

Show that T is a linear transformation. Geometrically, Tv is the vector thatresults if v is rotated counterclockwise by an angle θ. We call this transformationthe rotation of IR2 through the angle θ

Solution.

Let(

x1

y1

)and

(x2

y2

)be two vectors in IR2 and α ∈ IR. Then

T

(x1

y1

)+

(x2

y2

))= T

((αx1 + x2

αy1 + y2

))

=(

cos θ − sin θsin θ cos θ

)(αx1 + x2

αy1 + y2

)

=(

(αx1 + x2) cos θ − (αy1 + y2) sin θ(αx1 + x2) sin θ + (αy1 + y2) cos θ

)

= α

(cos θ − sin θsin θ cos θ

)(x1

y1

)+

(x2

y2

)

= αT

(x1

y1

))+ T

(x2

y2

))

This shows that T is linear

Page 270: Fundamentals of Linear Algebra - Mathematics & Statisticsmelbial2/classes/Linear Algebra... · 2006-01-13 · 1.1 Systems of Linear Equations Many practical problems can be reduced

270 CHAPTER 7. SOLUTIONS TO REVIEW PROBLEMS

Exercise 421Show that the following transformations are not linear.

(a) T : Mnn −→ IR given by T (A) = |A|.(b) T : Mmn −→ IR given by T (A) = rank(A).

Solution.(a) Since |A + B| 6= |A| + |B| in general (See Exercise 173), then the giventransformation is not linear.(b) Suppose m > n. let A be the m × n matrix such that aii = 1 and aij = 0for i 6= j. Let B ∈ Mmn such that bii = −1 and bij = 0. Then rank(A) =rank(B) = n. Also, A + B = 0 so that rank(A + B) = 0. Hence, T is not linear

Exercise 422If T1 : U −→ V and T2 : V −→ W are linear transformations, then T2 ◦ T1 :U −→ W is also a linear transformation.

Solution.Let u1, u1 ∈ U and α ∈ IR. Then

(T2 ◦ T1)(αu1 + u2) = T2(T1(αu1 + u2))= T2(αT1(u1) + T1(u2))= αT2(T1(u1)) + T2(T1(u2))= α(T2 ◦ T1)(u1) + (T2 ◦ T1)(u2)

Exercise 423Let {v1, v2, · · · , vn} be a basis for a vector space V, and let T : V −→ V be alinear transformation. Show that if T (vi) = vi, for 1 ≤ i ≤ n then T = idV , i.e.T is the identity transformation on V.

Solution.Let v ∈ V. Then there exist scalars α1, α2, · · · , αn such that v = α1v1 + α2v2 +· · ·+ αnvn. Since T is linear then T (v) = αTv1 + αTv2 + · · ·+ αnTvn = α1v1 +α2v2 + · · ·+ αnvn = v

Exercise 424Let T be a linear transformation on a vector space V such that T (v − 3v1) = wand T (2v − v1) = w1. Find T (v) and T (v1) in terms of w and w1.

Solution.Consider the system in the unknowns T (v) and T (v1)

{T (v) − 3T (v1) = w2T (v) − 2T (v1) = w1

Solving this system to find T (v) = 15 (3w1 − w) and T (v1) = 1

5 (w1 − 2w)

Page 271: Fundamentals of Linear Algebra - Mathematics & Statisticsmelbial2/classes/Linear Algebra... · 2006-01-13 · 1.1 Systems of Linear Equations Many practical problems can be reduced

271

Exercise 425Let T : Mmn → Mmn be given by T (X) = AX for all X ∈ Mmn, where A is anm×m invertible matrix. Show that T is both one-one and onto.

Solution.We first show that T is linear. Indeed, let X, Y ∈ Mmn and α ∈ IR. ThenT (αX + Y ) = A(αX + Y ) = αAX + AY = αT (X) + T (Y ). Thus, T is linear.Next, we show that T is one-one. Let X ∈ ker(T ). Then AX = 0. Since A isinvertible then X = 0. This shows that ker(T ) = {0} and thus T is one-one.Finally, we show that T is onto. Indeed, if B ∈ R(T ) then T (A−1B) = B. Thisshows that T is onto

Exercise 426Consider the transformation T : IRn −→ IRm defined by T (x) = Ax, whereA ∈ Mmn.

(a) Show that R(T ) = span{c1, c2, · · · , cn}, where c1, c2, · · · , cn are the columnsof A.(b) Show that T is onto if and only if rank(A) = m(i.e. the rows of A arelinearly independent).(c) Show that T is one-one if and only if rank(A) = n (i.e. the columns of Aare linearly independent).

Solution.(a) We have

R(T ) = {Ax : x ∈ IRn} =

[ c1 c2 · · · cn ]

x1

x2

...xn

= {x1c1 + x2c2 + · · ·+ xncn : xi ∈ IR}= span{c1, c2, · · · , cn}

(b) Suppose that T is onto. Then R(T ) = span{c1, c2, · · · , cn} = IRm. Thus,rank(A) = dim(span{c1, c2, · · · , cn}) = dim(IRm) = m. Since dim(span{c1, c2, · · · , cn}) =dim(span{r1, r2, · · · , rm}) then the vectors {r1, r2, · · · , rm} are linearly indepen-dent.Conversely, suppose that rank(A) = m. Then dim(span{c1, c2, · · · , cn}) =dim(IRm). By Exercise 265, R(T ) = IRm. That is, T is onto.(c) Suppose that T is one-one. Then ker(T ) = {0}. By Theorem 91, dim(R(T )) =rank(A) = dim(IRn) = n. An argument similar to (b) shows that the columnsof A are linearly independent. Conversely, if rank(A) = n then by Theorem 91,nullity(T ) = 0 and this implies that ker(T ) = {0}. Hence, T is one-one

Exercise 427Let T : V −→ W be a linear transformation. Show that if the vectors

T (v1), T (v2), · · · , T (vn)

Page 272: Fundamentals of Linear Algebra - Mathematics & Statisticsmelbial2/classes/Linear Algebra... · 2006-01-13 · 1.1 Systems of Linear Equations Many practical problems can be reduced

272 CHAPTER 7. SOLUTIONS TO REVIEW PROBLEMS

are linearly independent then the vectors v1, v2, · · · , vn are also linearly indepen-dent.

Solution.Suppose that α1v1 + α2v2 + · · · + αnvn = 0. Then α1T (v1) + α2T (v2) + · · · +αnT (vn) = T (0) = 0. Since the vectors T (v1), T (v2), · · · , T (vn) are linearly inde-pendent then α1 = α2 = · · · = αn = 0. This shows that the vectors v1, v2, · · · , vn

are linearly independent

Exercise 428Show that the projection transformation T : IR3 → IR2 defined by T (x, y, z) =(x, y) is not one-one.

Solution.Since (0, 0, 1) ∈ ker(T ) then by Theorem 89, T is not one-one

Exercise 429Let Mnn be the vector space of all n×n matrices. Let T : Mnn → Mnn be givenby T (A) = A−AT .(a) Show that T is linear.(b) Find ker(T ) and R(T ).

Solution.(a) Let A,B ∈ Mnn and α ∈ IR. Then T (αA + B) = (αA + B − (αA + B)T =α(A−AT ) + (B −BT ) = αT (A) + T (B). Thus, T is linear.(b) Let A ∈ ker(T ). Then T (A) = 0. That is AT = A. This shows that Ais symmetric. Conversely, if A is symmetric then T (A) = 0. It hollows thatker(T ) = {A ∈ Mnn : A is symmetric}. Now, if B ∈ R(T ) and A is such thatT (A) = B then A − AT = B. But then AT − A = BT . Hence, BT = −B,i.e.B is skew-symmetric. Conversely, if B is skew-symmetric then B ∈ R(T ) sinceT ( 1

2B) = 12 (B −BT ) = B. We conclude that R(T ) = {B ∈ Mnn : B is skew −

symmetric}Exercise 430Let T : V → W. Prove that T is one-one if and only if dim(R(T )) = dim(V ).

Solution.Suppose that T is one-one. Then ker(T ) = {0} and therefore dim(ker(T )) = 0.By Theorem 91, dim(R(T )) = dimV. The converse is similar

Exercise 431Show that the linear transformation T : Mnn → Mnn given by T (A) = AT is anisomorphism.

Solution.If A ∈ ker(T ) then T (A) = 0 = AT . This implies that A = 0 and consequentlyker(T ) = {0}. So T is one-one. Now suppose that A ∈ Mmn. Then T (AT ) = Aand AT ∈ Mnn. This shows that T is onto. It follows that T is an isomorphism

Page 273: Fundamentals of Linear Algebra - Mathematics & Statisticsmelbial2/classes/Linear Algebra... · 2006-01-13 · 1.1 Systems of Linear Equations Many practical problems can be reduced

273

Exercise 432Let T : P2 → P1 be the linear transformation Tp = p′. Consider the standardordered bases S = {1, x, x2} and S′ = {1, x}. Find the matrix representation ofT with respect to the basis S and S′.

Solution.One can easily check that p(1) = 0(1)+0x, p(x) = 1+0x, and p(x2) = 0(1)+2x.Hence, the matrix representation of T with respect to S and S′ is

(0 1 00 0 2

)

Exercise 433Let T : IR2 → IR2 be defined by

T

(xy

)=

(x−y

)

(a) Find the matrix representation of T with respect to the standard basis S ofIR2.(b) Let

S′ = {(

11

),

( −11

)}

Find the matrix representation of T with repspect to the bases S and S′.

Solution.(a) We have

T

(10

)=

(10

)

T

(01

)=

(0−1

)

So the matrix representation of T with respect to the standard basis is(

1 00 −1

)

(b) One can easily check that

T

(10

)=

12

(11

)− 1

2

( −11

)

and

T

(01

)=

12

(11

)+

12

( −11

)

So the matrix representation of T with respect to the standard basis is(

12

12

− 12

12

)

Page 274: Fundamentals of Linear Algebra - Mathematics & Statisticsmelbial2/classes/Linear Algebra... · 2006-01-13 · 1.1 Systems of Linear Equations Many practical problems can be reduced

274 CHAPTER 7. SOLUTIONS TO REVIEW PROBLEMS

Exercise 434Let V be the vector space of continuous functions on IR with the ordered basisS = {sin t, cos t}. Find the matrix representation of the linear transformationT : V → V defined by T (f) = f ′ with respect to S.

Solution.Since T (sin t) = cos t and T (cos t) = − sin t then the matrix representation of Twith respect to S is (

0 −11 0

)

Exercise 435Let T : IR3 → IR3 be the linear transformation whose matrix representation withthe respect to the standard basis of IR3 is given by

A =

1 3 11 2 00 1 1

Find

T

123

.

Solution.We have

T

123

=

1 3 11 2 00 1 1

123

=

955